Anda di halaman 1dari 424

U S M L E S T E P 2 C K N OT E S

U W & Qb a n k
By: hranksh
DISEASES OF THE ESOPHAGUS
Dysphagia for both solids & liquids suggests an underlying motility disorder.
Dysphagia for solids followed by dysphagia for liquids suggests organic causes.
Barium swallow is the initial test of choice for all patients with dysphagia.
Subsequent endoscopy is dependent on the barium swallow findings (i.e., the outline of the patient's current
esophageal anatomy).
When a motility disorder of the esophagus is suggested by contrast studies, the next step in evaluation is
usually esophagoscopy, to exclude mechanical causes of dysphagia, such as stricture or esophageal cancer.
Esophagoscopy is then followed by manometry, which confirms the diagnosis.
If Zenker's diverticulum is found on esophagography, endoscopy should be avoided.

ACHALASIA
Achalasia has a lower esophageal sphincter that does not relax (high tone).
Histopathology reveals hypertrophied inner circular muscles with absent or degenerating neurons (ganglion
cells) in myenteric plexuses.
The cause is not known, but a similar condition in South America is caused by the parasite, Trypanosoma
cruzi [Chagas disease; cardiomyopathy, megacolon and achalasia].
It presents with progressive dysphagia, chest pain, food regurgitation & aspiration.
An esophagogram typically reveals a dilated esophagus with a bird's beak narrowing of the distal esophagus.
Manometry is the gold standard for diagnosis of achalasia [significant decrease or absence of peristaltic
waves and increased LES tone].
Botulinum toxin injection is the treatment of choice for achalasia in the elderly, and in those patients who
cannot tolerate more invasive procedures, such as pneumatic dilation or surgical myotomy.

ESOPHAGEAL CANCER
The two major histological types are squamous cell carcinoma (SCC) and adenocarcinoma.
The risk factors for adenocarcinoma include Barrett's esophagus, chronic GERD, obesity, high dietary
calorie and fat intake, smoking, medications that promote GERD, etc. Patients with Barrett's esophagus have a
1% per year risk of developing adenocarcinoma of the esophagus. Adenocarcinoma generally occurs in
patients who had GERD symptoms more than 20 years and presents with weight loss, anorexia and
asymmetric narrowing of the esophageal lumen.
The major risk factors for squamous cell cancer of the esophagus are smoking, alcohol, dietary deficiency of
beta-carotene, vitamin B-1, zinc, selenium, environmental viral infections, toxin producing fungi, hot food and
beverages, pickled vegetables and food rich in N-nitroso compounds, etc.
Esophageal cancer mimic achalasia. Features that favor esophageal cancer over achalasia are: short history,
rapid weight loss, asymmetric narrowing of the esophageal lumen, and inability of endoscope to pass
through the lower esophageal sphincter.

1
SCLERODERMA
It is a collagen vascular disorder which can present with loss of distal peristalsis in the lower two-thirds of the
esophagus and significant decrease in the lower esophageal sphincter tone [becomes incompetent (low tone)
with time, leading to reflux esophagitis and a stricture].
There is fibrosis and complete atrophy of the esophageal smooth muscle.
Characteristic symptoms; "sticking sensation in the throat" or dysphagia accompanied by heartburn.
This condition is progressive and difficult to treat (give PPIs).

DIFFUSE ESOPHAGEAL SPASM


It is usually seen in young females, manifests with intermittent episodes of chest pain and dysphagia.
In many patients, it is associated with emotional factors and functional gastrointestinal disorders.
Manometric studies demonstrate high amplitude peristaltic contractions
LES usually has a normal relaxation response.
Esophagogram is frequently normal, although the classic corkscrew esophagus is seen occasionally.
Treatment includes: antispasmodics, dietary modulation, psychiatric counseling and Ca++ channel blocker.

INFECTIOUS ESOPHAGITIS
It is mostly due to Candida, Herpes, or Cytomegalovirus.
This generally occurs in immunocompromised individuals (e.g. those with AIDS with CD4 < 200/mm3,
malignancy, diabetes) and may present with dysphagia, oral thrush or odynophagia (pain on swallowing).
If the primary complaint is odynophagia rather than dysphagia, Candida is less likely.
AIDS patients with dysphagia usually require 1-2 weeks of initial empirical oral fluconazole therapy since
Candida esophagitis is the most common etiology.
If fluconazole does not work after 2 weeks of therapy, then endoscopy should be performed. Endoscopy with
washings, culture and biopsy may reveal the cause.

ULCERATIVE ESOPHAGITIS
Patients with severe odynophagia without dysphagia or thrush are more likely to have ulcerative esophagitis.
Cytomegalovirus (CMV) is a frequent cause of ulcerative esophagitis in HIV patients.
Consider pill esophagitis in a young patient who is taking medications for acne with the acute onset of
odynophagia.
The triad of focal substernal burning pain with odynophagia, evidence of large shallow superficial
ulcerations, and presence of intracellular inclusions are diagnostic of CMV esophagitis.
Ganciclovir is used to treat CMV esophagitis. IV Foscarnet is an alternative to Ganciclovir and is used when
Ganciclovir is not effective or not tolerated by the patient.
Herpes simplex virus (HSV) esophagitis ulcers are usually multiple, well circumscribed and have a
"volcano" (small and deep) like appearance (treatment is Acyclovir).

2
ZENKER'S DIVERTICULUM
Zenker's diverticulum is defined as a posterior herniation of mucosa of the proximal esophagus immediately
above the upper esophageal sphincter, through the fibers of the cricopharyngeal muscle.
Motor dysfunction & incoordination are responsible for the problem and is generally seen in females.
The pathophysiology involves retention of food material, regurgitation, subsequent aspiration and resulting
pneumonia.
The diverticulum may vary in size and is generally asymptomatic; however, the occasional patient may present
with complaints of food sticking in the throat, halitosis, regurgitation, and pneumonia.
There is no pain associated with the diverticulum.
Barium esophagography is the confirmatory test of choice. Esophagoscopy should be avoided due to the
associated risk of perforation.
The treatment is surgery [includes excision and frequently cricopharyngeal myotomy].

MALLORY-WEISS TEARS
Caused by forced repeated vomiting.
Increased intra-gastric pressure during vomiting could cause tear in the mucosa of the cardia and distal
esophagus.

GERD
GERD predisposes to Barrett's esophagus, adenocarcinoma of the esophagus, erosive esophagitis, & peptic
stricture formation.
Chronic GERD and Barrett's esophagus are risk factors for adenocarcinoma of the esophagus. Patients with
Barrett's esophagus have a 1% per year risk of developing adenocarcinoma of the esophagus. On the other
hand, the major risk factors for squamous cell cancer of the esophagus are smoking and alcohol.
When patient presents with typical symptoms of GERD start empiric treatment with PPI.
If the patients fails to respond to empiric PPI or disease is complicated e.g. dysphagia, odynophagia, weight
loss, occult bleeding, iron deficiency anemia upper GI endoscopy.
If upper GI endoscopy is negative Esophageal pH monitoring.
Recognize when to order upper endoscopy in the management of gastro-esophageal reflux disease. The
following are some alarm signals:
1) Nausea/vomiting
2) Weight loss, anemia or melena/blood in the stool
3) Long duration of symptoms (>1-2 years), especially in Caucasian males >45 years old
4) Failure to respond to proton pump inhibitors

3
PEPTIC STRICTURE
Typically causes progressive dysphagia to solid food without weight loss or anorexia, and symmetric &
circumferential narrowing of the lower esophageal lumen.
As stricture progresses, it blocks reflux leading to improvement of heartburn symptoms.
Other causes of peptic stricture include radiation, scleroderma, and causative agents.

PEPTIC ULCER DISEASE


Dyspepsia is defined as pain or discomfort centered in the upper abdomen.
American Gastroenterological Association recommended that patients with dyspepsia, age younger than 45
years, and presentation without any alarming symptoms (e.g., bleeding, anemia, dysphagia, and weight loss)
should have a noninvasive test for H. pylori (i.e., serologic or breath test). If H. pylori positive Eradication
antibiotic therapy against H.pylori is indicated. If H. pylori negative, the recommendation is an empirical trial
for one month of anti-secretory therapy, such as H2 blockers or proton pump inhibitors, or a prokinetic agent.
Patients with dyspepsia, age older than 45 years, or presentation with any alarming symptoms at any age
Gastroenterology referral is required (due to the small risk of gastric cancer). Such patients should be
considered for endoscopy and H. pylori testing.
Hemorrhage is the most common complication of peptic ulcer disease. Upper GI bleeding commonly
presents as hematemesis with or without melena. Massive upper GI bleeding may present as hematochezia,
which is usually a sign of lower GI bleeding. Blood or coffee-ground-like material on nasogastric tube lavage
can confirm this clinical diagnosis. Most bleeding peptic ulcers will stop bleeding spontaneously, and will not
rebleed during hospitalization.
Recurrent peptic ulcers with hypercalcemia are best explained by MEN I.
MEN I: tumors of 3Ps; anterior pituitary, parathyroid and pancreatic islets (e.g. Zollinger-Ellison).
MEN IIa: pheochromocytoma, medullary thyroid carcinoma, and parathyroid adenoma.
MEN IIb: pheochromocytoma, medullary cancer of thyroid, neuromas and marfanoid status.

DUODENAL ULCER
Duodenal ulcer disease is typically presents with epigastric pain that improves with eating [empty stomach
unopposed acidic fluid that empties into the duodenum, and ingestion of food causes alkali solutions to be
released into the duodenum & relieves pain].
Over 90% of patients with duodenal ulcer disease are infected by H. pylori. Antibiotic therapy is the most
accepted and recommended management for the eradication of H. pylori. Give a combination of omeprazole,
clarithromycin, and amoxicillin.
Duodenal ulcer NEVER causes malignancy.

GASTRIC ULCER
Aspirin decreases the protective prostaglandins production and causes direct mucosal injury of the stomach.

4
ZOLLINGER-ELLISON SYNDROME
It is a gastrin-producing non-beta cell pancreatic tumor.
As a result of uncontrolled gastrin secretion, parietal cell hyperplasia develops and stomach acid production is
significantly increased.
Increased production of stomach acid Inactivation of pancreatic enzymes malabsorption Steatorrhea.
Abdominal pain and diarrhea are the usual presenting symptoms.
Patients may have a personal or family history of multiple endocrine neoplasia type 1 (MEN I), which
consists of primary hyperparathyroidism, pituitary tumors, and pancreatic tumors (e.g., insulinoma,
gastrinoma, VIP-oma).
Endoscopy shows prominent gastric folds, Multiple duodenal ulcers are typical, and a jejunal ulcer (ulcer
located beyond the duodenal bulb) is almost pathognomic for this condition.
All patients suspected of having this syndrome should have fasting serum gastrin levels done. A serum gastrin
value greater than 1000 pg/mL is diagnostic of the disorder.
Patients with non-diagnostic fasting serum gastrin levels should have a secretin stimulation test done.
In addition, measurement of gastric pH levels on a single specimen is important to exclude the possibility of
secondary hypergastrinemia due to achlorhydria.

GASTROPARESIS
Neuropathy is seen in approximately 50% of patients with long-standing DM. Diabetic autonomic neuropathy
is related to the duration of disease and glycemic control. Any part of the gastrointestinal tract can be affected
in diabetic autonomic neuropathy. Involvement of the stomach causes gastroparesis, which commonly presents
as anorexia, nausea, vomiting, abdominal bloating and early satiety.
Due to delayed gastric emptying, glycemic control is difficult to achieve. Patients typically have post meal
hypoglycemia after insulin injection.
The management of diabetic gastroparesis includes:
1) Improved glycemic control
2) Small, frequent meals
3) A dopamine antagonist (e.g., metoclopramide drug of choice, domperidone) before meals
4) Bethanechol
5) Erythromycin: This drug interacts with motilin receptors and can promote gastric emptying.
6) Cisapride is effective in diabetic gastroparesis, but it is currently available only through the manufacturer. It
is issued only after providing adequate documentation of the need for the drug, and after a thorough
assessment of the individual's risk factors for cardiac arrhythmias.

5
CANCER STOMACH
The only malignancy that has decreased universally. The reason is not known.
Once a histologic diagnosis of gastric cancer has been made, evaluation of the extent of the disease is the most
appropriate, timely, & individualized next step in patient care.
CT scan is a standard diagnostic tool employed in patients with newly diagnosed gastric cancer to evaluate the
extent of the tumor.
Surgical resection of the tumor remains as the mainstay of the treatment.

ULCERATIVE COLITIS
P-ANCA is present in 60-80% of patients of Ulcerative colitis. Erythema nodosum, arthralgias, diarrhea, and
positive P-ANCA in a young patient are highly suggestive of inflammatory bowel disease.
Ulcerative colitis can also present as bloody diarrhea.
Bloody diarrhea in a young patient should make you think about inflammatory bowel disease.
UC is limited to the colon and nearly always involves the rectal mucosa.
Proctosigmoidoscopy and biopsy can quickly make the diagnosis of UC; one of its advantages that this
diagnostic test can be performed in the acute setting; however, physicians should not wait for these studies
before starting treatment when the patient is acutely ill.] Barium enema is contraindicated in acutely ill and
toxic patients.
Fulminant colitis is a serious complication of ulcerative colitis; plain radiographs can usually reveal the
distended colon.
Primary sclerosing cholangitis is more commonly seen in patients with ulcerative colitis and presents with
jaundice, pruritus, and right upper quadrant pain.
Patients with UC that have pancolitis, should begin surveillance colonoscopy after eight years of disease.
Ulcerative colitis may present with a toxic megacolon (x-ray shows distended colon filled with gas). It is an
emergency and prompt admin of IV steroids, nasogastric decompression & fluid management is required.

CROHN'S DISEASE
Always suspect Crohn's disease in a young patient with chronic bloody diarrhea.
Crohn's disease can involve the GI tract anywhere, from the esophagus to anus. It is frequently associated
with intestinal fistula, strictures and anal disease.
enterohepatic recycling bile acids cholesterol gall bladder stone formation.
Laboratory findings of every chronic inflammatory disease can include anemia and reactive thrombocytosis.
Always suspect Crohn's disease in a young patient with chronic bloody diarrhea.

6
PRIMARY SCLEROSING CHOLANGITIS (PSC)
It is an inflammatory destruction of both intra and extra hepatic bile ducts. Hay jaundice, pruritus, RUQ
pain, or acute cholangitis. Cholestatic liver disease in the context of ulcerative colitis is highly suggestive of
primary sclerosing cholangitis (PSC).
ERCP is the investigation of choice. Cholangiography shows characteristic 'Beading' due to stricture and
dilations of intra and extra hepatic ducts.
LFT shows very high Alkaline phosphatase, mild ALT & ALT (< 300). Other findings are increased serum
and IgM, P-ANCA.
Ursodeoxycholic acid as a bile acid-binding medication to lower liver enzymes. UV light to treat pruritus.
Liver transplantation is indicated in late stage PSC.
Cholangiocarcinoma can complicate primary sclerosing cholangitis, especially in patients who smoke and
have ulcerative colitis. One of the early manifestations is severe stricturing within the biliary tree, which
causes a cholangitis characterized by fever, jaundice, and leukocytosis. The most appropriate diagnostic
procedure is to biopsy the prominent stricture. cholangiocarcinoma is a relative contraindication for liver
transplantation because of concerns about recurrence of the malignancy.

7
DIARRHEA
CHRONIC DIARRHEA

Patients with chronic diarrhea have metabolic acidosis and hypokalemia. Metabolic acidosis occurs due to the
loss of bicarbonate in the stool.
Chronic diarrhea can lead to malabsorption of fatty acids and bile salts; this in turn predisposes to the
formation calcium oxalate stones.
Bacterial overgrowth, pancreatic insufficiency, Whipple's and Crohn's disease all result in chronic
diarrhea with increased fecal fat content.

8
WHIPPLE'S DISEASE
Whipple's disease is caused by the bacteria Tropheryma whippelii and typically affects the bowel, causing
malabsorption.
It should be suspected in all patients with fever of unknown origin, chills, generalized lymphadenopathy,
hyperpigmentation, migratory polyarthralgia, weight loss, abdominal pain and diarrhea. Other organs like
CNS, heart and skin may be involved.
The diagnosis of Whipple's disease can be confirmed with upper gastrointestinal endoscopy and biopsy of the
small intestine and shows villous atrophy and Periodic Acid-Schiff positive deposits in lamina propria and
these biopsy findings are pathognomonic of Whipple's disease.
In doubtful cases, electron microscopy may be required to demonstrate the Whipple bacillus for confirmation
of the diagnosis.

Lactose intolerance
Lactose intolerance may result in chronic diarrhea but fecal fat will not be high in such causes.
Lactase is a brush border enzyme that hydrolyses lactose. Its concentration declines steadily as one ages into
adulthood, especially in people of non-European ancestry. When there is an inability to absorb lactose found in
milk and dairy products, the condition is called lactose intolerance.
Patients typically manifest with osmotic diarrhea (high osmotic gap, stool pH is acidic), abdominal cramps,
bloating and flatulence after ingestion of such products.
Currently, the lactose hydrogen breath test has largely replaced the lactose tolerance test. A positive hydrogen
breath test is characterized by a rise in the measured breath hydrogen level after the ingestion of lactose, thus
indicating bacterial carbohydrate metabolism. Lactase deficiency is characterized by a positive Clinitest of
stool for reducing substances (undigested lactose).

TROPICAL SPRUE
Always suspect tropical sprue in patients with chronic diarrhea and a history of living in endemic areas (e.g.,
Puerto Rico) for more than one month.
Tropical sprue is a chronic diarrheal disease characterized by malabsorption of nutrients, especially vitamin
B12 and folic acid, thus leading to megaloblastic anemia.
The disease is characterized by signs and symptoms of malabsorption, such as glossitis, cheilosis, protuberant
abdomen, pallor, and pedal edema. Other associated symptoms include fatty diarrhea, cramps, gas, fatigue
and progressive weight loss. Hyperactive bowel sounds and borborygmi are commonly found on examination.
The diagnosis can be established with small intestinal mucosal biopsy, which typically shows blunting of villi
and infiltration of chronic inflammatory cells, including lymphocytes, plasma cells and eosinophils.

9
CHRONIC INFLAMMATORY DIARRHEA
Chronic inflammatory diarrhea (> 4 weeks) is typically associated with inflammatory changes in the blood
(anemia, elevated ESR, acute phase reactants, reactive thrombocytosis).
Blood/leukocyte-positive stool is another important finding.

DIARRHEA IN AIDS
Diarrhea, in HIV-infected patients, can be due to multiple etiological agents therefore; an etiologic diagnosis
must be made before starting antibiotic therapy.
Cryptosporidium parvum is a major cause of chronic diarrhea in HIV-infected patients with CD4 < 180
cells/mm3. Cryptosporidiosis usually causes a profuse watery diarrhea. A modified acid-fast stain showing
oocysts in the stool is very suggestive of its infection.
Isospora belli is also suggested by the presence of acid-fast oocysts in the stool.
Patients with CMV colitis generally have a CD4 < 50 cells/L. The typical presentation consists of chronic
bloody diarrhea with abdominal pain.

TYPES OF DIARRHEA
Motor diarrhea is exemplified by hyperthyroidism.
Factitial diarrhea is typically associated with psychiatric disturbances.
Secretory diarrhea is usually the result of some medication use or hormonal disturbances.
Osmotic diarrhea is caused by the ingestion of osmotically active, poorly absorbable substances. Lactose
intolerance is a classic example (chronic diarrhea).

DIARRHEA IN INFANCY

The most common cause of diarrhea in infancy (outbreaks) is Rotavirus or Norwalk.


Antidiarrheal compounds almost never used in children.

INFECTIOUS DIARRHEA
Infectious diarrhea is classified into bloody/inflammatory and non-bloody.
Common causes of bloody diarrhea; CASES
o Campylobacter
o Amoeba (E. histolytica)
o Shigella
o E. coli (E. coli 0157:H7, enteroinvasive E. coli)
o Salmonella

10
Campylobacter jejuni is the most common cause of bloody diarrhea in USA and the most common source of
infection is undercooked infected poultry. Presence of severe abdominal pain along with diarrhea is a
helpful clue. Treat with erythromycin.

E. Histolytica can cause bloody diarrhea, it would be diagnosed on stool examination. The colonoscopy
would also show the presence of 'flask shaped' colonic ulcers. Serologic testing may still be required to
detect infection with Entamoeba histolytica.
Amebiasis is caused by the protozoa Entamoeba histolytica, which exists in the cyst stage (the infective form)
and the trophozoite stage (the invasive form). Infection with this parasite is most common in developing
countries that have poor sanitation, as it primarily occurs through contact with contaminated food or water.
Amebic abscesses within the liver contain debris characterized as "anchovy paste" in appearance.
Supplemental laboratory findings that support the diagnosis include leukocytosis (>10,000/mm3) or an
elevated alkaline phosphatase. Cyst aspiration is not typically recommended because of the associated risks.
Treatment requires both tissue and luminal agents. Commonly used tissue agents include oral metronidazole
and tinidazole, while commonly used luminal agents include paromomycin, iodoquinol, or diloxanide furoate.
Surgical intervention may be indicated, however, when the abscess has ruptured, eroded into adjacent
structures, or caused extrahepatic complications such as small bowel obstruction

Shigella is a very common cause of dysentery in USA. Dysentery due to Shigella usually occurs in daycare
centers and other institutional settings. Shigella can also cause hemolytic uremic syndrome. Treat with
trimethoprim/ sulfamethoxazole.

Salmonella is commonly associated with contaminated eggs, milk & chicken.

Salmonella, Shigella, Yersinia, Campylobacter and Chlamydia (SSYCC) are involved in the pathogenesis
of reactive arthritis.

Entero-hemorrhagic E. coli, especially Escherichia coli 0157:H7 are important causes of colitis and are
usually transmitted by eating improperly cooked beef (hamburgers) and other foods. E.coli 0157:H7 can also
cause hemolytic uremic syndrome, which is characterized by microangiopathic hemolytic anemia,
thrombocytopenia, and renal failure; therefore antibiotics are contraindicated and platelet transfusion is also
contraindicated.
Enterotoxigenic E. coli is the most frequent offender causing travelers diarrhea. Travelers with abdominal
cramps, diarrhea and malaise should be suspected for infection by this organism.

Sporadic yersiniosis develops with Eating undercooked pork leading to bloody diarrhea. Granulomas may
also be seen in patients with Yersinia infection.

11
C. difficile toxin is responsible for pseudomembranous colitis. Patients usually have bloody diarrhea, and
most cases are associated with prior antibiotic use. Pathogenesis: secretion of toxin, which is toxic to
intestinal epitheliocytes. Cytotoxin assay can be detected in stool. Treat with oral or IV metronidazole or oral
vancomycin and discontinuation of the causative antibiotic.

Campylobacter jejuni is the most frequent precipitant of Guillain-Barre Syndrome (GBS).

Vibrio parahaemolyticus is typically associated with ingestion of contaminated seafood and subsequent
non-bloody diarrhea.
Vibrio vulnificus is associated with ingestion raw shellfish, causes severe disease in those who have
underlying liver disease. Vibrio vulnificus is iron-loving bacteria, associated with iron overload and the
development of bullous skin lesions. Treat with doxycycline.

Untreated water can be associated with several diarrheal infections such as Vibrio cholerae, and
enterotoxigenic E. coli; the resulting is typically watery, non-bloody diarrhea.

Bacillus cereus is classically associated with ingestion of reheated rice and subsequent food poisoning
predominantly present with vomiting (within 1 6 hours) because of performed toxin, watery, non-bloody
diarrhea develops later.

Staphylococcus food poisoning predominantly present with nausea & vomiting (within 1 6 hours) because
of performed toxin.

Nepal is hyperendemic for Giardia and Cyclospora (contaminated water).

GIARDIASIS
Causative organism is Giardia lamblia trophozite.
Certain mountainous areas of north & west USA and Nepal are hyperendemic for Giardia.
Any patient who returns from developing country, South America (e.g. Nepal), or Rocky mountain and has
symptoms suggestive of malabsorption (i.e. foul-smelling stools, fatty stools, bloody diarrhea, flatulence,
nausea, malaise, abdominal cramps, and may be weight loss) should be considered for empirical treatment
with metronidazole for giardiasis.
Giardia lamblia adheres to mucosal surfaces by adhesive disk and produce malabsorption.
Giardia causes fat and vitamin malabsorption if not eradicated.

Scombroid is a type of poisoning that occurs after ingesting scombroid fish (tuna, mackerel, mahi mahi),
which may contain a large amount of histamine. When ingested, scombroid can give symptoms within a few
minutes: rash, diarrhea, vomiting, and wheezing, along with a burning sensation in the mouth, dizziness, and
paresthesias. Treat with Antihistamines such as diphenhydramine.

12
C. botulinum is usually transmitted by the ingestion of contaminated food, especially honey, resulting infant
botulism. The causative organism produces the toxin in the intestinal tract, which subsequently triggers the
symptoms. It is a protease that blocks acetylcholine release. Infants typically present between two weeks and
nine months with constipation and poor feeding. This is followed by progressive hypotonia, weakness, loss of
deep tendon reflexes, cranial nerve abnormalities (impaired gag reflex), and respiratory difficulties. Signs of
autonomic dysfunction such as hypotension and neurogenic bladder can occur early in the course of the
disease. Adult form of botulism, the toxin ingested per se produces the symptoms.

Mycobacterium avium complex causes disseminated disease with bowel infiltration and malabsorption in
patients with severe immune compromise. Furthermore, it is not characterized by oocysts in the stool.

Small bowel disease, surgical resection or chronic diarrhea can lead to malabsorption of fatty acids and
bile salts; this predisposes to the formation of calcium oxalate stones.

CELIAC DISEASE
Suspect celiac disease in any patient who presents with malabsorption and iron deficiency anemia.
Celiac disease presents with symptoms and signs of malabsorption, which include:
1) Characteristically bulky, foul-smelling, floating stool
2) Loss of muscle mass or subcutaneous fat
3) Pallor - due to iron deficiency anemia (MCV < 80)
4) Bone pain - due to osteomalacia
5) Easy bruising - due to vitamin K deficiency
6) Hyperkeratosis - due to vitamin A deficiency
Fatigue and weight loss are common complaints.
Up to 24% of cases of celiac disease is associated with dermatitis herpetiformis. Dermatitis herpetiformis is a
rare, chronic, intensely burning, and pruritic papulovesicular skin disease. It is commonly associated with
subclinical gluten-sensitive enteropathy. It is characterized by granular IgA deposits in the upper dermis;
therefore, testing for IgA anti-endomysial antibodies (best test) and IgG/IgA antigliadin antibodies can be
performed when this condition is being suspected.
The diagnosis is confirmed with serological studies such as ELISA for IgA antibodies to gliadin and the
immunofluorescence test for IgA antibodies to endomysium (anti-endomysial antibodies), the presence of
which is virtually pathognomonic for celiac disease. In addition, antibodies against tissue transglutaminase
are highly sensitive and specific.

13
LAXATIVE ABUSE
Laxative abuse is characterized by very frequent (10 20) watery, nocturnal diarrhea.
Diagnosis can be confirmed with the characteristic biopsy findings of dark brown discoloration of the colon
with lymph follicles shining through as pale patches (melanosis coli); typically seen in those abusing
anthraquinone-containing laxatives e.g. bisacodyl.
Hypokalemia due to alkalemia from the laxative abuse. The greatest concern from hypokalemia is
development of life threatening cardiac arrhythmias.

BACTERIAL OVERGROWTH
It is a malabsorption syndrome which can be associated with a history of abdominal surgery.
The clinical manifestations of bacterial overgrowth depend on the underlying cause and severity of the disease.
Symptoms may be nonspecific and include abdominal pain, watery diarrhea, dyspepsia, and weight loss. In
severe and advanced cases, patients may present with tetany (hypocalcemia due to vitamin D deficiency),
night blindness (due to vitamin A deficiency), neuropathy (due to vitamin B-12 deficiency), dermatitis,
arthritis, and hepatic injury.
Physical examination may reveal abdominal distention with identifiable succussion splash (due to palpable
soft, fluid-filled loops of bowel).
Macrocytic anemia (due to vitamin B-12 malabsorption) can be seen on laboratory examination.

IRRITABLE BOWEL SYNDROME (IBS)


IBS is a diagnosis of exclusion.
IBS is a functional disorder of the gastrointestinal tract, characterized by abdominal pain with diarrhea and/or
constipation.
There is no nocturnal diarrhea.
There is no laboratory or pathologic hallmark of IBS (i.e. normal colonic mucosa).

14
CARCINOID SYNDROME
Carcinoid syndrome is a constellation of symptoms that are exhibited by patients with carcinoid tumors. The
most common location of such tumors is the appendix; however, a patient who presents with systemic signs of
carcinoid (i.e., carcinoid syndrome) usually has these tumors in the small bowel. These tumors secrete
excessive amounts of the hormone serotonin, as well as other chemicals that cause the blood vessels to dilate.
When carcinoid spreads to the liver, it may typically manifests with the classic triad of flushing, secretory
diarrhea (often with abdominal cramps), and right sided valvular heart lesions (wheezing and dyspnea).
Episodic flushing (20-30 seconds) is the clinical hallmark of carcinoid syndrome.
Pathognomonic plaque-like deposits of fibrous tissue occur most commonly on the endocardium on the right
side of the heart. Patients with carcinoid syndrome are at risk of developing niacin deficiency, owing to the
increased formation of serotonin from tryptophan.
The presence of 5-HIAA in the urine is indicative of a carcinoid tumor. Serotonin levels are increased.
Carcinoids do not have the tendency to spread and have a good prognosis.
Surgery with complete removal of the tumor tissue is usually the first-line treatment. It can result in a
permanent cure if it is possible to remove the tumor entirely. For carcinoid tumors locate at the tip of the
appendix, appendectomy is sufficient treatment. Small bowel resection is required for carcinoids located in the
small bowel. The mainstay of treatment for advanced carcinoid tumors that cannot be removed surgically is
octreotide injection.

DIVERTICULOSIS
The two most common causes of painless GI bleeding in an elderly patient over 65 years of age are
diverticulosis and angiodysplasia.
Asymptomatic diverticulosis needs only dietary modification in the form of high fiber diet intake.

VASCULAR ECTASIA (ANGIODYSPLASIA)


This condition is the second most common cause of lower GI bleeding in elderly patients.
Such patients typically present with painless bleeding.
There is well-defined associations have been established between vascular ectasia and aortic stenosis (ejection
systolic murmur at the right 2nd intercostal space). Another well-recognized association is renal failure.
Ectasias can present throughout the colon; however, the most often site of bleeding is the cecum or ascending
colon. The colonoscopic finding of a cherry red fern-like pattern of blood vessels that appear to radiate from
a central feeding vessel is diagnostic.

15
DIVERTICULITIS
The diagnosis of uncomplicated acute diverticulitis of the colon can usually be made clinically.
Diverticulitis is associated with a history of constipation (rather than diarrhea) and little fiber in their diet, and
usually produces left lower quadrant pain and fever. It is usually seen in elderly patients.
The best treatment includes IV antibiotics until the symptoms resolve. When patients fail to respond to
antibiotics, a complication must be suspected, such as formation of an abscess, fistula, or frank perforation.
The best diagnostic test to evaluate such complications is via CT scan.
Sigmoidoscopy and colonoscopy can be disastrous in the presence of acutely inflamed bowel and are not
recommended during the acute episode because the risk of perforating the colon is very high.
CT scan is the best test for diagnosing and evaluating the abdomen of patients during an acute episode of
diverticulitis.

PERSISTENT CONSTIPATION
The combination of back pain, anemia, renal dysfunction, and elevated ESR is very typical for multiple
myeloma. Another very characteristic finding is hypercalcemia. Hypercalcemia may manifest as severe
constipation, anorexia, weakness, increased urination (renal tubular dysfunction), or neurologic
abnormalities (e.g., confusion or lethargy). Sometimes, hypercalcemia-induced symptoms may even dominate
the clinical picture of multiple myeloma. Always look for hypercalcemia as a cause of constipation.
Other common causes of persistent constipation may include mechanical obstruction (e.g., secondary to colon
cancer), medications (e.g., anti-cholinergic agents), hormonal disturbances (e.g., hypothyroidism), and some
neurologic abnormalities.
Office-based anoscopy/proctoscopy should be the initial procedure in patients < 50 years who present with
minimal bright red blood per rectum (BRBPR) and do not have any risk factors for colon cancer. Blood
intermixed with stool is not including in this category.
Anoscopy has higher sensitivity for diagnosis hemorrhoids than flexible endoscopy.

CHRONIC MESENTERIC ISCHEMIA


The diagnosis of chronic mesenteric ischemia is suspected in patients with unexplained chronic abdominal
pain [out of proportion to the examination], post-prandial pain that leads to food avoidance (food aversion),
and weight loss.
Evidence of associated atherosclerotic disease is usually present.
Physical findings are usually nonspecific. Abdominal examination may reveal a bruit (50 % of patients).

16
ISCHEMIC COLITIS
Ischemic colitis is seen in approximately 1-7% of patients following aortoiliac surgery. Patients with
extensive atherosclerotic vascular disease are at particularly high risk. Ischemic colitis can also develop after
embolization of atheromatous particles.
In general, the initial presentation of patients with ischemic colitis includes the acute onset of lower
abdominal pain, followed by bloody diarrhea within 12-24 hours.
It is important to recognize that diarrhea with blood is one of the first signs of ischemic colitis.
The most commonly involved segment of the colon is the splenic flexure, which is supplied by narrow
terminal branches of the superior mesenteric artery followed by the recto-sigmoid junction, which is supplied
by narrow terminal branches of the inferior mesenteric artery.
Abdominal x-ray and CT scan of the abdomen may show changes such as thumb-printing (indicating
submucosal edema) and hemorrhage. However, in the early stages, there are often no signs.
If the CT scan is inconclusive, Immediate proctosigmoidoscopy or colonoscopy is important in the initial
evaluation of such a patient in order to assess colonic viability.
Increased levels of lactic acid.
Management depends on the degree of ischemia, abdominal pain, fever, WBC count and peritoneal signs.
Bacterial overgrowth, pancreatic insufficiency, Whipple's and Crohn's disease all result in chronic diarrhea
with increased fecal fat content.
D-xylose is a simple sugar. It does not need to undergo any digestive process before it can be absorbed. Its
absorption requires an intact mucosa only. D-xylose absorption is abnormal both in bacterial overgrowth and
Whipple's disease. However, with bacterial overgrowth, the test becomes normal after antibiotic treatment.
Whipple's disease damages small intestinal mucosa, thus impairing D-xylose absorption

Colon cancer
Colon cancer rarely causes pain, unless it is very advanced. It is also unlikely to occur in a young patient.
The most common site of colon cancer metastasis is the liver. It manifests as right upper quadrant pain,
mildly elevated liver enzymes, & firm hepatomegaly.
Most colon cancers develop from polyps. The risk factors for a polyp progressing into malignancy are villous
adenoma, sessile adenoma, and size >2.5 cm. Only adenomatous polyps are clearly premalignant, but <1% of
such lesions progress to malignancy.
Hyperplastic polyps are non-neoplastic and do not require further work-up.

ACUTE APPENDICITIS
The chronology of abdominal pain in acute appendicitis is typically visceral, followed by somatic pain.
The initial peri-umbilical pain is referred pain and visceral in nature; however, pain shifts to the right lower
quadrant with involvement of the parietal peritoneum and becomes somatic in nature.

17
GASTROINTESTINAL BLEEDING

The most common cause of iron deficiency anemia in elderly is gastrointestinal blood loss. The next step in
evaluation is colonoscopy. [Note that single negative occult blood test is not sufficient to diagnose occult
blood loss].
PUD is the most common cause of upper GI bleeding. Nearly all patients with upper GI tract bleeding should
undergo upper GI endoscopy, usually within 12 hours of admission.
A BUN level > 40 in the presence of a normal serum creatinine level is very suggestive of an upper GI bleed.
The elevated blood urea level is due to the bacterial breakdown of hemoglobin in the GI tract and the resulting
absorption of urea.
The other common scenario where you can see elevated BUN without an increase in creatinine level is the
administration of steroids.
Patients with prerenal azotemia (dehydration) can also have elevated BUN with very mild elevation of
creatinine levels.

18
ACUTE PANCREATITIS
Acute pancreatitis is most commonly secondary to gallstones or alcohol abuse.
However, it may be also secondary to drug toxicity, idiopathic etiology, hypertriglyceridemia, infection,
severe trauma or burns, hypercalcemia, pancreatic abnormality, tumors, toxins or due to post operative or post
ERCP etiology.
Hypertriglyceridemia > 1000mg/dl can cause acute pancreatitis and during the attack the triglyceride levels
can be as high as 3000-5000 mg/dl. Acute pancreatitis secondary to hypertriglyceridemia often has normal
levels of amylase.
Drug-induced pancreatitis is mild and usually resolves with supportive care.
Remember the following scenarios for drug-induced pancreatitis:
o Patient on diuretics ?furosemide, thiazides.
o Patient with inflammatory bowel disease ?sulphasalazine, 5-ASA.
o Patient on immunosuppressive agents ?azathioprine, L-asparaginase.
o Patient with a history of seizures or bipolar disorder ?valproic acid.
o AIDS patient ?didanosine, pentamidine.
o Patient on antibiotics ?metronidazole, tetracycline
Acute onset of pain that radiates to the back and is accompanied with nausea & vomiting, and tenderness is
strongly suggestive of acute pancreatitis.
Complications: exudative left pleural effusion [when amylase concentration is high], abdominal compartment
syndrome, secondary paralytic ileus, intraabdominal hemorrhage, shock and pancreatic pseudocyst.
Hypotension that can complicate acute pancreatitis is thought to arise from intravascular volume loss
secondary to local and systemic vascular endothelial injury (vascular damage) which increases vascular
permeability and transudation of plasma. Systemic vasodilatation may also contribute.
CT scan is diagnostic for pancreatitis.

ERCP is the investigation of choice for patients with recurrent attacks of pancreatitis without obvious cause.
MI is one of the differential diagnosis of acute abdomen, and should be ruled out in patients with risk factors.

19
RETROPERITONEAL HEMORRHAGE
History of anticoagulation + weakness + dizziness + anemia + tachycardia raise concern of internal
hemorrhage.
Physical examination findings may include a bluish discoloration of the flanks (Grey Turner sign) or
periumbilical region (Cullen sign) resulting from blood accumulation in abdominal fascial planes.

Anticoagulation with warfarin places patients at risk for hemorrhage. Retroperitoneal hemorrhage may occur
even without a supratherapeutic INR.
Back pain, signs & symptoms of hemodynamic compromise should raise the suspicious for retroperitoneal
hemorrhage.

20
CHRONIC PANCREATITIS
An alcoholic patient with chronic abdominal pain and diarrhea is classic for chronic pancreatitis.
Chronic pancreatitis is an inflammatory condition characterized by chronic abdominal pain and pancreatic
insufficiency.
Pancreatic exocrine failure presents as malabsorption Steatorrhea weight loss. Fecal elestase study is
the most specific and sensitive test to diagnose pancreatic exocrine failure.
Pancreatic exocrine failure presents as Diabetes mellitus.
Currently, quantitative estimation of stool (72-hour fecal fat collection) is the gold standard for diagnosis of
Steatorrhea [normal < 6 grams fat/day].
Acid steatocrit is a test for fat malabsorption with a positive predictive value of 90% as compared to the 72-
hour fecal fat collection as the gold standard.

Pancreatic cancer
The known risk factors for the development of pancreatic cancer include: family history, Increasing age (50
years), chronic pancreatitis, smoking, diabetes mellitus, obesity, and a diet high in fat.
Cigarette smoking is the most consistent reversible risk factor for pancreatic cancer.
Alcoholism, Gall stones, and Coffee intake are NOT a risk factor for pancreatic cancer.
Pancreatic carcinoma is the primary differential diagnosis in patients with chronic pancreatitis.
Pancreatic carcinoma most commonly presents with a dull upper abdominal pain that radiates to the back,
weight loss, or jaundice. Classic findings include a nontender but palpable gallbladder at the right costal
margin in a jaundiced patient (Courvoisier sign) or left supraclavicular adenopathy (Virchow node) in a
patient with metastatic disease.
Laboratory evaluation of those with pancreatic cancer typically demonstrates increased serum bilirubin and
alkaline phosphatase in conjunction with a mild anemia.
Abdominal ultrasound is usually the initial imaging performed on patients with jaundice. When the
ultrasound is nondiagnostic, the next step is to obtain an abdominal CT scan. Abdominal CT scan is a very
sensitive and specific tool used in the diagnosis of pancreatic carcinoma.
Endoscopic retrograde pancreatography (ERCP) is an excellent tool in the diagnosis of pancreatic cancer, with
a sensitivity and specificity of 90-95%. Because it is an invasive procedure, however, it is most commonly
reserved for those patients who have already undergone a nondiagnostic ultrasound and a nondiagnostic CT
scan.
The most commonly used tumor marker for pancreatic cancer is the cancer associated antigen (CA) 19-9,
which has a sensitivity and specificity of 80-90%. CA 19-9 can be elevated in patients with jaundice but no
pancreatic cancer, however, which reduces its utility as a screening tool. Postoperative monitoring of
pancreatic cancer with CA 19-9 may be helpful in evaluating the tumor response to chemotherapy.

21
VIPoma
VIPomas are cancerous pancreatic tumors that affect cells in the pancreas that produce vasoactive intestinal
peptide (VIP). The cause is not known. Remember the pancreatic cholera - VIPoma
VIPomas are diagnosed most commonly at age 50 or so. Women are more likely to be affected than men.
VIPomas cause severe watery diarrhea, hypokalemia resulting in leg cramps, and a decrease in the amount
of acid in the stomach. Other symptoms include dehydration, abdominal pain and cramping, weight loss,
facial flushing and redness.
High level of VIP (vasoactive intestinal peptide) in the blood is diagnostic.
A CT scan or MRI is ordered to determine the location of the tumor.
The first goal of treatment is to correct dehydration. Intravenous (IV) fluids are often required to replace
fluids lost in diarrhea. The next goal is to slow the diarrhea. Some medications can help control the diarrhea,
such as octreotide. If the tumor has not metastasized, surgery can often cure it.

GLUCAGONOMA presents with narcotizing dermatitis, weight loss, anemia, and persistent hyperglycemia.

INSULINOMA is a rare beta-cell tumor that releases Insulin and causes hypoglycemia.

PANCREATIC PSEUDOCYST
The development of palpable mass in the epigastrum 4 weeks after the onset of acute pancreatitis is highly
suggestive of pancreatic pseudocyst.
Pseudopancreatic cysts are collections of amylase rich fluid, debris, and tissue in the lesser sac of peritoneum
(or bursa omentalis minor) and it's not a true cyst as it lacks an epithelial lining.
A complication of both acute and chronic pancreatitis, the pseudocyst is best diagnosed by ultrasound.
Pseudopancreatic cysts can resolve on their own over 6 weeks. Therefore all the uncomplicated
pseudopancreatic cysts smaller than 5cm should be observed for 6 weeks before any therapeutic intervention is
done. A pseudocyst that doesn't resolve spontaneously can lead to serious complications and should be treated
with either percutaneous drainage or a gastrostomy (an opening in the posterior stomach wall which would
drain the pseudocyst into the stomach).

22
HEPATITIS
Laboratory tests used in the evaluation of liver disease either assess liver functionality (eg, prothrombin time,
bilirubin, albumin, cholesterol) or structural integrity and cellular intactness (eg, transaminases, gamma
glutaryl transferase, alkaline phosphatase).
Elevated levels of serum transaminases are indicative of hepatocyte damage. A progressive decrease in
transaminase levels signals either recovery from liver injury or that very few hepatocytes are functional.
The diagnosis of hepatocellular carcinoma is established through liver biopsy or the combination of
confirmatory imaging studies and elevated alpha-fetoprotein levels.

HAV
Hepatitis A virus is an picornavirus with incubation period of 30 days.
Acute hepatitis A presents with mild influenza-like symptoms that progress to jaundice, abdominal pain,
nausea, vomiting, dark urine and pale stools.
Evaluation of liver damage in patients with acute hepatitis is based upon liver function tests & viral serology.
Acute hepatitis: ALT > AST
Significant prolonged PT correlates with increased mortality.
The two available forms of prophylaxis include a vaccine which provides active immunity, and serum
immune globulin, which provides passive immunity.
Active immunity with the hepatitis A vaccine is preferred but requires four weeks to take effect.
Hepatitis A immune globulin will provide passive immunity for approximately 4 to 6 months.
Close contacts with individuals with hepatitis A should be given serum immune globulin.
Hepatitis A vaccine or serum immune globulin should be given to all non-immunized travelers to endemic
countries.
If travel will occur in < 4 weeks, serum immune globulin should be given.
If travel will occur in > 4 weeks, hepatitis A vaccine should be given instead as it offers long-term protection.
A booster vaccination after 6 to 12 months is then necessary to ensure long-term protection ( 10 years).

23
HBV
In general, individuals who should be screened for hepatitis B virus include:
1. Individuals born in hyperendemic areas (Africa, Southeast Asia, Middle East, Pacific islands, Amazon
River basin, Haiti, and the Dominican Republic), and those who plan to remain abroad for longer than 6
months.
2. IV drug abusers
3. Persons who received blood transfusions before 1986.
4. Dialysis patients
5. Men who have sex with men.
6. HIV-positive individuals
7. Pregnant women
8. Contacts (family, household, or sexual) of HBV-infected individuals.

Acute hepatitis B infection has an incubation period that varies from one to four months.
In the United States, sexual contact and percutaneous transmission (eg, intravenous drug abuse) are the
primary means of acquiring hepatitis B infection.
Most patients are asymptomatic and have subclinical or anicteric hepatitis. More than 90% of adults with acute
hepatitis B infection will recover completely. A minority will proceed to develop chronic hepatitis B infection,
and approximately 0.1-0.5% will develop fulminant hepatic failure (FHF).
Some develop a serum sickness-like syndrome that is followed by fatigue, anorexia, nausea, jaundice, and
right upper quadrant pain.

24
ALT and AST levels may reach 1000-2000 IU/L, with the ALT typically higher than the AST. In most
patients, symptoms gradually disappear and aminotransferase levels normalize over a few months.

Testing for both HBsAg and IgM anti-HBc offers the best screening for acute hepatitis B infection.

HBsAg: 1st marker to be detected in serum, remains detectable during the entire symptomatic phase of Acute
hepatitis B and suggests infectivity.
Anti-HBsAg: appears after successful vaccination or clearance of HBsAg. Remains detectable for life.
Indicator of noninfectious and immunity. There is latent period between clearance of HBsAg and appearance
of Anti-HBs called window period.
HBcAg: normally sequestrated within HBsAg (not detectable in serum).
Anti-HBc: appears shortly after HBsAg, remains detectable long time after patients recovers. IgM fraction
signals for acute phase of the disease. IgG fraction signals for recovery of the disease. IgM anti-HBc presents
in the window period.
HBeAg: appears after HBsAg. Active viral replication and infectivity. If presents > 3 months then increased
likelihood of chronic hepatitis B.

A combination of hepatitis B virus immune globulin and lamivudine is the most effective measure to prevent
recurrent HBV infection before, at the time of or after liver transplantation.

HBV vaccine is a recombinant vaccine containing HBsAg, which stimulated Anti HBsAg, person who
receives it will have immunity and thus be positive for HBsAb and negative for HBsAg.
If vaccined but no response to vaccine must be vaccined again.
If vaccined many times and still no response, give HBIG on exposure.
If exposed to virus and never been vaccinated before, give hepatitis B immunoglobulin (HBIg) and hepatitis B
vaccine.

ACUTE HBV will have HBsAg, HBeAg and Anti-HBc.


Chronic HBV will have HBsAg for > 6months, HBeAg positive and high titers of HBV DNA.
Recovery phase of HBV will have HBsAb, HBsAg, Anti-HBe and ABSENT HBeAg.

Treatment of HBV: Interferon or Lamivudine.


Degree of elevation of ALT is important in deciding the Treatment.
Chronic Hepatitis B patients with persistently elevated ALT levels (more than 2 times of the upper limit),
detectable serum HBsAg, HBeAg, and heavy viral load of HBV DNA should be treated with interferon or
lamivudine.
Generally, Lamivudine has less side effects, ORAL, and preferred in children, immunocompromised patients
& psychic patients [Interferon is contraindicated in psychic patients].

25
Of all acute Hepatitis B cases 90% recover, minority go to chronic, out of those 1% goes to Fulminant
hepatic failure, defined as hepatic encephalopathy that develops within 8 weeks of the onset of acute liver
failure and evidence by marked increase in ALT and signs of Hepatic encephalopathy. Liver transplantation is
the only effective method for treatment, so initial step in treatment at this point is to put the patients name
on the list.

Newborn of mothers with active Hepatitis B should be passively immunized at birth with hepatitis B
immunoglobulin (HBIg) followed by hepatitis B vaccine Vaccination. Transmission from mother with chronic
disease & positive HBsAg and HBeAg to the fetus is 90%.

Post-exposure prophylaxis of hepatitis B infection includes administration of HBIG (Hepatitis B immune


globulin) and three shots of Hepatitis B vaccine given at set intervals.

FULMINANT HEPATIC FAILURE (FHF)


Fulminant hepatic failure is defined as hepatic encephalopathy that develops within eight weeks of the onset of
acute liver failure. It develops more commonly in patients who are heavier users of acetaminophen, alcohol,
or methamphetamines and in those who are co-infected with hepatitis B and D viruses.
Hepatic encephalopathy, in conjunction with a markedly increased ALT, an increased PT, and
coagulopathy.
Because FHF has a mortality rate that exceeds 80%, patients with this condition are considered high-priority
candidates for liver transplantation if a suitable donor is available.
Orthotopic liver transplantation remains the only effective mode of treatment of fulminant hepatic failure
and should be considered in any patient presenting with this condition, regardless of etiology.
General contraindications to liver transplantation include irreversible cardiopulmonary disease that poses a
prohibitive risk, incurable or recent (< 5 years) malignancy external to the liver, and active alcohol and drug
abuse.
Fresh frozen plasma in conjunction with intravenous vitamin K administration can be employed to manage the
coagulopathy (active bleeding) associated with fulminant hepatic failure. However, the prophylactic
administration of FFP is not recommended since it has not been proven to change the mortality.

26
HCV
Acute hepatitis C is typically asymptomatic.
Risk factors for rapid progression of liver fibrosis in chronic Hepatitis C are:
Male sex
Acquire infection after age of 40
Co-infection with HBV or HIV
Alcohol intake

HCV RNA is the single most sensitive serological marker used in screening for HCV infection. As the first
marker of HCV infection to appear in the serum, HCV RNA may be detected with PCR within days to weeks
after infection, it is considered the gold standard.
Anti-HCV antibodies can be detected with the ELISA technique as early as eight weeks after exposure.
However, some patients may not be positive for several months after exposure or may never test positive. In
addition, this marker does not distinguish between acute and chronic infections.

There is some evidence that shows that sexual transmission of HCV may occur, but the incidence is extremely
low. In monogamous, heterosexual partners, the risk is less than 0.1% annually. With such a low risk, both the
United States Public Health Service and the National Institutes of Health have not recommended barrier
precautions between stable monogamous sexual partners.

HCV-RNA is detectable in colostrum (early form of breast milk); however, studies have shown that
breastfeeding does not increase the risk of transmission to the neonate. For this reason, both the American
College of Obstetricians and Gynecologists and the American Academy of Pediatrics support breastfeeding by
HCV-infected mothers.

Perinatal transmission of HCV is a much less frequent cause of infection, and the overall incidence of vertical
transmission is around 2-5 %. Factors which seem to increase the overall risk of vertical transmission are HIV
co-infection and a high HCV viral load. A caesarean section does not seem to significantly decrease the risk of
vertical transmission. Currently, pregnant women with HCV are not advised to have cesarean sections, other
than for the usual obstetric indications.

In general, individuals who should be screened for hepatitis C virus include:


IV drug abusers
Persons receiving blood transfusions or organ transplants before July 1992 or from an individual who later
sero-converted to HCV-positive.
Hemodialysis patients
Persons with chronically elevated ALT values
Children born to HCV-positive mothers
Health care workers who have suffered needle stick injury or mucosal exposure to HCV-positive blood.
All patients with vasculitis.

27
The decision to treat a patient of chronic hepatitis C depends on several factors like:
1) Natural history of disease
2) Stage of the disease
3) Findings of liver biopsy
4) The levels of liver enzymes
5) Presence of HCV RNA
6) Efficacy and adverse effects of the drugs in the patient.

Anti viral treatment in patients with normal aminotransferases can lead to persistently elevated ALT in about
20% of patients. Also the natural history of chronic hepatitis C with normal aminotransferase level is relatively
benign and antiviral treatment can lead to liver injury.
Studies have shown that patients with chronic hepatitis C who show persistently normal liver enzymes on
multiple occasions have minimal histological abnormalities, therefore they do not need to be treated with
interferon or antiviral drugs at this stage. Do nothing and only follow up.
All chronic hepatitis C patients with elevated ALT, detectable HCV RNA and histologic evidence of chronic
hepatitis of at least moderate grade [compensated liver disease] are candidates for antiviral therapy with
interferon and ribavirin. Combination therapy has shown to be beneficial than monotherapy with either
interferon or ribavirin in the above setting [contraindicated in pregnancy]. Ribavirin is particularly highly
teratogenic. Monotherapy with interferon alpha only would have been sufficient in the case of patients with a
mild grade of chronic hepatitis C.
Liver transplantation is not indicated unless there are signs of progressive disease and/or decompensated
cirrhosis (when PT and Albumin levels are very affected).
Current recommendations state that all patients with chronic hepatitis C should receive vaccinations to
Hepatitis A and B if not already immune.
The overall incidence of vertical transmission of HCV is approximately 2-5%.
All patients, including pregnant patients, with chronic hepatitis C should receive vaccinations against
Hepatitis A and B if not already immune. Both vaccinations are safe for use during pregnancy.
American College of Obstetricians and Gynecologists and the American Academy of Pediatrics support
breastfeeding by HCV-infected mothers.
Currently, pregnant women with HCV are not advised to have cesarean sections, other than for the usual
obstetric indications.

Chronic hepatitis C classically presents with "waxing and waning" transaminases levels but few symptoms.
Only 20% of patients develop cirrhosis. Laboratory values and hepatic function often worsen in the setting of
physiological stress. Patients may complain of arthralgias & myalgia (with false positive Rheumatoid factor
[RF] or Anti-nuclear Antibody [ANA]).

28
Chronic hepatitis C is associated with number of extra hepatic complications like: CIB PLAP
1) Cryoglobulinemia membranous glomerulonephritis (characterized by proteinuria).
2) Idiopathic thrombocytopenic purpura.
3) B cell lymphomas.
4) Plasmacytomas.
5) Lichen planus.
6) Autoimmune diseases like Sjogren's syndrome and thyroiditis.
7) Porphyria cutanea tarda.

Immune complex mediated extrahepatic complications are more common with hepatitis B than with hepatitis
C except for cryoglobulinemia.

HEV
It is rare in USA, but outbreaks in Mexico, India, Afghanistan and Burma.
Hepatitis E is transmitted via feco-oral route (usually infected water).
It is generally self-limited, does not lead to chronic hepatitis, cirrhosis or carrier state.
Hepatitis E virus has a high rate of progression to fulminant hepatitis in pregnant women especially in the 3rd
trimester.
IgM antibodies appear with the appearance of clinical symptoms.
IgG antibodies appear after the acute phase.

HGV
HGV is a flavivirus transmitted by parenteral route.
Infection followed by viremia lasting for at least 10 years.
The pathogenic role of HGV is still uncertain.
Anti-HGV antibodies is found in 50% of IV drug abusers and 30% of patients receiving hemodialysis.

29
NON-ALCOHOLIC FATTY LIVER DISEASE OR NON-ALCOHOLIC HEPATIC STEATOSIS
Which refers to a spectrum of liver damage ranging from steatosis, steato-hepatitis, and advanced fibrosis in
non-alcoholic patient. Risk factors are obesity, type II DM, Hyperlipidemia, TPN, drugs (e.g. corticosteroids,
amiodarone, diltiazem, tamoxifen, HAART), Cushings syndrome and bypass surgery.
Insulin resistance plays a central role in the Pathophysiology of non-alcoholic fatty liver disease by increasing
rate of lipolysis and elevating the circulating insulin levels. Impaired responsiveness of fat cells to insulin
accumulation of fat in the liver. Steatosis can then progress to steatohepatitis and fibrosis likely secondary to
lipid peroxidation and oxidative stress.
Non-alcoholic steatohepatitis (NASH) is often asymptomatic and manifests as hepatomegaly and elevated
transaminases.
Imaging will reveal fatty infiltration of the liver. Liver biopsy shows macrovascular fat deposition with
displacement of the nucleus to the periphery. Looks like alcoholic steatosis but ALT/AST is NOT 2:1.
Treatment; control the underlying comorbid condition and also Urosdeoxycholic acid might help reduce
transaminases levels and improve liver histological findings.

ALCOHOLIC HEPATITIS
There are 3 major pathological stages of alcoholic liver disease:
o Fatty liver (steatosis)
o Alcoholic hepatitis
o Alcoholic fibrosis/cirrhosis
Acute alcoholic hepatitis is characterized by acute onset right upper quadrant pain with fever, hepatomegaly
(normal liver span is 612 in the midclavicular line), elevated MCV, and the absolute values of serum AST
and ALT are almost always less than 500 IU/L and AST:ALT > 2.
Balloon degeneration with polymorphic cellular infiltrates is seen in the livers of those with acute alcoholic
hepatitis.
Fatty liver (steatosis), alcoholic hepatitis and even early alcoholic fibrosis can be reversible with the cessation
of alcohol intake.
True cirrhosis (with regenerative nodules) is irreversible, regardless of alcohol abstinence.

30
CHRONIC LIVER DISEASE
Chronic liver disease causes hyperestrogenemia as a result of decreased clearance of estrogen from the blood
by the liver due to increased portosystemic shunting. Additionally, decreased sex hormones binding globulin
synthesize by the diseased liver increases the ability of circulating estrogen to exert its biologic effects.
Symptoms of hyperestrogenemia include; telangiectasia, palmar erythema, testicular atrophy, impotence and
gynecomastia.
Hypothalamic-pituitary dysfunction can occur in patients with chronic liver disease TSH, T4, T3.
Patients with chronic liver disease most commonly have respiratory alkalosis.
Liver functions can be classified into 3 categories:
1) Synthetic: clotting factors, cholesterol, proteins.
2) Metabolic: metabolize drugs, steroids, including detoxification.
3) Excretory: bile.
Laboratory tests used to evaluate liver disease is classified into two classes:
A- Tests to asses the function of liver which includes:
1- PT 2- Albumin 3- Cholesterol 4-Billirubin.
B-Tests that assess structural integrity and cellular damage:
1-Transaminases (AST & ALT) 2- Gamma glutamyl transferase (GGT) 3- Alkaline phosphatase.
Now, PT (prothrombin time) is considered the single most important test to assess the function of the liver
because all clotting factors (except factor VIII) are synthesized in the liver.
Elevated Transaminases are indicative of liver cell damages since this intracellular enzyme leaks out of
damaged cells. A marked increase in AST or ALT is an indication of an ongoing tissue destruction.
A progressive decrease in AST or ALT could mean either recovery from liver injury or that there is little tissue
is left (as in Fulminant hepatitis), so this interpretation is dependant on functional test like PT.
If PT is OK and Transaminases decrease means recovery, if PT is increased, Transaminases decrease means
fulminant hepatitis.
Evaluation of liver damage in patients with chronic hepatitis is based upon liver biopsy.
All patients with chronic liver disease (e.g. chronic hepatitis C) should receive vaccinations to Hepatitis A and
B if not already immune. Both vaccinations are safe for use during pregnancy.
Chronic liver disease or cirrhosis (e.g. hemochromatosis) of any cause is a risk factor for hepatocellular
carcinoma.
In cirrhotic liver; live is unable to metabolize circulating estrogen hyperestrogenemia testicular
atrophy, gynecomastia, decrease body hair in males, palmer erythema, spider angiomas.
Caput medusae; dilatation of superficial veins of the anterior abdominal wall due to portal hypertension.
Asterixis; is a neurologic sign associated with poor hepatic function and increase in ammonia [ healthy liver
detoxify ammonia to urea which excreted by urine].
Hypoalbuminemia decrease oncotic pressure fluids move to extravascular spaces edema.
Alcoholism is the most common cause of cirrhosis in USA. Infection with HCV is the second most common
cause of cirrhosis in USA.

31
Alcoholism inflammation and parenchymal necrosis of the liver, AST: ALT > 2:1, markedly PT.
Rx: stop alcohol, nutritional support, & corticosteroids (e.g. methylprednisolone).
All patients with cirrhosis should be screened for esophageal varices by upper GI endoscopy.
Non-selective beta-blockers (e.g. propranolol) are used for the primary and secondary prevention of variceal
bleeding in cirrhotic patients who have portal hypertension with esophageal varices. To ensure appropriate
effect, the drug should be dosed to decrease the resting pulse by 25%.
The first step in treatment of acute variceal bleeding is to establish vascular access with 2 large bore
intravenous needles or a central line. The 2nd step is to control the bleeding itself.
Somatostatin and its long acting octreotide acts by reducing splanchnic blood flow, inhibiting gastric acid
secretion & exerting gastric cytoprotective states.

Progression of liver disease in patients with chronic hepatitis is relatively more rapid in the following
conditions:
Male gender
Acquiring infection after age of 40
Longer duration of infection
Co-infection with HBV or HIV
Immunosuppression
Liver co-morbidities like alcoholic liver disease, hemochromatosis, alpha-1 antitrypsin deficiency

Factors independently associated with high rates of liver fibrosis in chronic hepatitis:
Male gender
Acquiring infection after age of 40
Alcoholic intake in any amount can hasten the progression of fibrosis in patients with chronic hepatitis C

HEPATORENAL SYNDROME
Suspect it in a patient with chronic (severe) liver disease, hypotension, hyponatremia, oliguria, azotemia and
normal urine analysis.
No available treatment of hepatorenal syndrome. Take measures to withhold all possible precipitating factors.
Careful intravenous volume load and discontinuation of spironolactone & other diuretics.

CHRONIC AUTOIMMUNE HEPATITIS


It is characterized by the presence of anti-smooth muscle antibodies or ANA in the serum.
Histologically: prominent infiltration of the liver by lymphocytes and plasma cells.

32
DRUG INDUCED LIVER INJURY
Beside the liver injury there is rash, arthralgias, fever, leukocytosis, and eosinophilia [these findings are absent
in case of isoniazid-induced hepatic cell injury].
There are two kinds of drug induced liver injury:
Direct toxic effects which is dose dependant e.g. acetaminophen, tetracycline
Idiosyncratic liver injury which is NOT dose dependant e.g. isoniazid with histological features similar to
those seen in patients with viral hepatitis (panlobular mononuclear infiltration & hepatic cell necrosis.
10 20% of patients on ISONIAZID (INH) will develop mild elevation of aminotransferases (< 100 IU/L)
within first weeks of the treatment. This hepatic injury is typically self-limited and will resolve without
intervention (just close follow-up).
If signs or symptoms of ISONIAZID hepatitis are observed, the drug should be discontinued immediately.

ACETAMINOPHEN-INDUCED HEPATIC INJURY


In the United States, acetaminophen toxicity is the most common cause of acute hepatic failure, and fulminant
hepatitis responsible for as many as 40% of cases.
Remember that acute hepatic failure is defined as the development of liver failure within eight weeks of
hepatocellular injury onset. If hepatic encephalopathy is also seen, then the syndrome is described as
fulminant hepatic failure.
Acetaminophen toxicity occurs more commonly in the context of heavy alcohol intake and fasting. Remember
the chronic alcohol intake increases the risk of hepatic injury by stimulating P450 system and decreasing the
amount of glutathione (used for the metabolism of acetaminophen).
Acute alcohol intake can reduce the risk of hepatic injury by the acetaminophen because it competes with the
CYP2E1 which results in decrease production of toxic metabolite.
Grossly elevated AST levels (often > 10,000 IU/L) will be seen.
Adverse prognostic factors: acidosis (pH < 7.3), INR > 6.5, azotemia (serum creatinine > 3.5),
hyperphosphatemia (> 1.2 mmol/L), and increased blood lactate levels (> 3.5 mmol/L).
Treatment include; correct acid-base disturbances, electrolyte disturbance, coagulation defects, hypoglycemia,
encephalopathy and renal failure.
Gut emptying procedures & activated charcoal are best effective if carried out in the first hour.
N-acetylcysteine should be administered promptly in cases of acetaminophen toxicity and will improve
cerebral blood flow and oxygenation and it helps to reduce the chance of hepatic injury. Patients with
acetaminophen above certain level on the nomogram are the best candidates for the treatment. For this
purpose, 4-hour post-ingestion acetaminophen levels are determined to decide whether the patient will
benefit from N acetyl cysteine, or not. On the other hand, if the patient (adult) has ingested > 7.5gm of
acetaminophen and levels will not be available within 8hrs after ingestion, he should be given the antidote. If
the dose is below the amount, which is usually toxic to liver. We can wait for the acetaminophen levels, and
can decide once we interpret the levels on the nomogram.

33
REYE'S SYNDROME
Reye syndrome, a condition also known as fatty liver with encephalopathy.
It is seen exclusively in children younger than 15 years of age, and is preceded by an upper respiratory tract
infection with influenza or varicella in 60-80% of all cases.
Treatment with salicylates significantly increases the likelihood of developing Reye syndrome. Aspirin is
usually not used in children for the fear of Reye's syndrome. There are only 2 indications for aspirin in
children- 1) Juvenile Rheumatoid Arthritis and 2) Kawasaki's disease.
Pathophysiologically, Reye syndrome is characterized by diffuse mitochondrial injury that leads to nausea,
vomiting, headache, excitability, delirium, combativeness, hypoglycemia, liver failure, and progressive CNS
damage.
Histologically, extensive fatty vacuolization of the liver without inflammation is present. The liver is enlarged
but there is no icterus. Important laboratory findings include elevated serum aminotransferases, elevated
ammonia levels, PT prolongation, hypoglycemia and metabolic acidosis.
Treatment consists of administration of glucose with FFP and mannitol to decrease cerebral edema.

Ischemic hepatic injury occurs in the setting of hypotension and manifests as acute massive increase in
AST & ALT with milder associated increase in total bilirubin and alkaline phosphatase. In patients who
survive, liver enzymes return to normal levels in few weeks.

ASCITES
Know the stepwise approach of the treatment for ascites:
1) Sodium and water restriction (2 L/day)
2) Spironolactone
3) Loop diuretic (not more than 1 L/day of diuresis due to the risk of hepato-renal syndrome)
4) Frequent abdominal paracentesis with or without albumin infusion (2 - 4 L/day, as long as the renal function
is okay), less aggressive paracentesis is recommended in patients with borderline renal function.
Spironolactone is potassium-sparing diuretics and aldosterone receptor antagonist. Spironolactone is the
diuretic of choice to treat cirrhotic ascites (amiloride & triamterene are alternatives).
In cases of massive ascites that respiratory distress; the best next step is paracentesis, which can be
diagnostic and therapeutic. LFTs will be of no use unless the ascetic fluid turns out to be a transudate.
Paracentesis provides the most useful diagnostic information in patients with ascites.

Serum ascites albumin gradient (SAAG) is used to distinguish between portal hypertension and other causes of
ascites. [SAAG = serum albumin concentration ascites albumin concentration]. SAAG 1.1 g/dl is accurate
in diagnosis a transudative process consistent with portal hypertension or increased capillary hydrostatic
pressure within the liver capillary beds.

34
SPONTANEOUS BACTERIAL PERITONITIS
Always suspect SBP in cirrhotic patient with ascites who develops fever or abdominal pain.
SBP is peritonitis in the absence of an apparent source of infection. SBP is almost always seen in adults
secondary to severe cirrhosis especially alcoholic cirrhosis. It almost always occurs in patients with ascites.
Its characterized by growing a single bacteria in ascites fluid, usually E. Coli.
Ascites fluid containing PMN >250 cells/mm3, SAAG (Serum to Ascites Albumin Gradient) >1.1.
Suspected SBP should be treated empirically with Cefotaxime or an Ampicillin with an aminoglycoside.

Some of the common indications of transjagular intrahepatic portosystemic shunt (TIPS):


1) Refractory cirrhotic hydrothorax.
2) Refractory ascites (defined as diuretic resistant or diuretic refractory ascites).
3) Recurrent variceal bleeding not controlled by other minimal invasive measures.
4) Patients waiting for liver transplantation and needing portocaval shunts.

HEPATIC ENCEPHALOPATHY
Hepatic encephalopathy characterized by reverse of sleep cycle, asterixis, & delta waves in EEG.
Asterixis is a rhythmic flapping of the hands when extended at the wrist.
Precipitating factors of hepatic encephalopathy:
High protein diet
Alkalosis
Diuretic therapy
GI bleeding
Narcotics
High volume paracentesis

Pathogenic factors involved in development of hepatic encephalopathy:


Accumulation of ammonia in the blood
Production of false neurotransmitters
sensitivity of CNS to inhibitory transmitters like GABA
Zinc deficiency
Hepatic encephalopathy treatment includes: lactulose (non absorbable disaccharide syrup), reduced-protein
diet, neomycin, rifaximin & laxatives.
Patients with encephalopathy who did not respond to lactulose should be placed on an ornithine-aspartate
infusion or oral sodium benzoate.

35
WILSON'S DISEASE OR HEPATOLENTICULAR DEGENERATION
Autosomal recessive.
Results in the excessive accumulation of copper in tissue, which can cause
hepatic injury.
May cause an asymptomatic elevation of the transaminases, typically
associated with a decreased ceruloplasmin level, Kayser-Fleischer rings
[greenish brown deposits around cornea], or neuropsychiatric symptoms
(such as Parkinsonian-like tremor or depression)
[Atrophy of the lenticular nucleus is most marked in patients with Wilson's
disease].
Wilson is often associated with Fanconis syndrome, hemolytic anemia,
neuropathy.
Wilson's disease can cause fulminant hepatitis, RTA type 1 (distal tubular acidosis), & RTA type 2 (proximal
tubular acidosis).
Liver biopsy is the gold standard for diagnosis; Mallory's hyaline.
Decreased serum ceruloplasmin, increased urinary copper.
Disease starts in infancy, but since all infants have increased level of copper for the first 3 months, it is best to
diagnose it after the first year.
Rx: copper chelation e.g. d- Penicillamine or trientine and Zinc. Fulminant Hepatitis needs liver
transplantation.

HEMOCHROMATOSIS
It is a common autosomal recessive disease with the classic gene mutation C282Y on chromosome 6.
This mutation indirectly causes increased intestinal iron absorption, resulting in an accumulation of iron in
the liver, pancreas, heart, adrenals, testes, pituitary, kidneys, joints, and skin.
Hemochromatosis is characterized by increased skin pigmentation, symptoms of Bronze diabetes (polyuria &
polydipsia), arthralgias, loss of libido (Impotence), weight loss, cirrhosis and heart failure in later stages.
Transferrin saturation ( 50%) and Ferritin (> 1000) is a simple reliable screen test.
Hepatoma (Hepatocellular Carcinoma) is the most serious complications.
Patients are at increased risk for Listeria Monocytogenes, maybe due to impaired phagocytosis.
Iron overload is also a risk for Yersinia Enterocolitica and sepsis from Vibrio Vulnificus which are iron
loving bacteria.
If patient presents with joint pain and hepatomegally and no other symptoms suspect it and FIRST thing you
do is CBC to check Fe level.

36
ALPHA-1 ANTITRYPSIN DEFICIENCY
It is an autosomal recessive disorder leading to deficient levels of alpha-1 antitrypsin (A1AT)
Produced primarily in the liver, alpha-1 antitrypsin (A1AT) is a serum protein that inhibits several different
proteolytic enzymes (eg, pancreatic trypsin, chymotrypsin, neutrophil elastase). It is also an acute-phase
reactant that plays a significant role in inflammatory states. Alpha-1 antitrypsin is coded for by a single, highly
pleomorphic gene on chromosome 14.
Histologically, the granules evident in the hepatocytes of afflicted individuals are thought to represent the
unsecreted A1AT. They stain with the periodic acid-Schiff (PAS) reaction and resist digestion by diastase.
Approximately 75-85% of individuals suffering from A1AT deficiency will prematurely develop severe
panacinar emphysema. Other common pulmonary complications include chronic bronchitis and
bronchiectasis, but emphysema remains the most common cause of death.
Liver disease is of the greatest concern in the first two decades of life. Those affected typically demonstrate
hepatomegaly or hepatosplenomegaly, cholestasis, and elevation of the hepatocellular enzymes. The most
severe consequences of liver involvement include cirrhosis (which is the second most common cause of death
in this population) and hepatocellular carcinoma.
The diagnosis of A1AT deficiency is established by measurement of the serum A1AT level, followed by
confirmatory genetic testing.
Treatment options include augmentation therapy with purified human A1AT for those with severe
deficiencies. Individuals suffering from severe pulmonary impairment are candidates for lung transplants,
while those in hepatic failure can be treated with liver transplantation.

HYDATID CYSTS
It is most commonly seen in areas where sheep are raised. The patient will usually be 'sheep breeders'.
Echinococcosis is a parasitic disease caused by tapeworm echinococcus. E. granulosus, causing cystic
echinococcosis, and E. multilocularis, causing alveolar echinococcosis.
Dogs and other canids are the definitive hosts and sheep are the intermediate hosts.
The infectious eggs excreted by dogs in the feces are passed on to other animals and humans. After ingestion
of eggs by humans, the oncospheres are hatched and they penetrate the bowel wall disseminating
hematogenously to various visceral organs, leading to formation of hydatid cysts.
The liver, followed by the lung, is the most common viscus involved; however, any viscera can be involved.
Hydatid cyst is a fluid-filled cyst with an inner germinal layer and an outer acellular laminated membrane.
The majority of human infections with E. granulosus are asymptomatic. The initial infection is always
asymptomatic, with subsequent signs and symptoms dependent upon the site of the cysts and their size. The
liver is affected in most patients, and hydatid cysts in this location may ultimately cause hepatomegaly, right
upper quadrant pain, nausea, and vomiting.
Microscopic examination of the tissue demonstrates a characteristic encapsulated and calcified cyst that
contains fluid and budding cells that will eventually become daughter cysts.

37
AMEBIC LIVER ABSCESS
Right upper quadrant pain, leukocytosis (>10,000/mm3), elevated alkaline phosphatase, and CT findings
(Typically, a solitary lesion is found.).

In general, this diagnosis should be strongly suspected in any young male with a tender solitary abscess in the
right lobe of the liver who is from or has traveled to an endemic area (e.g. Mexico) within the past six months.
Amebiasis is caused by the protozoa Entamoeba histolytica, which exists in the cyst stage (the infective form)
and the trophozoite stage (the invasive form).
Infection with this parasite is through contact with contaminated food or water.
Most patients with amebiasis are asymptomatic, but dysentery (bloody diarrhea), liver abscess, and other
clinical manifestations (eg, pulmonary, cardiac, or brain involvement) have been observed.
Amebic abscesses within the liver contain debris characterized as "anchovy paste" in appearance, this fluid
within the cyst is usually sterile and odorless unless secondarily infected. Though cyst aspiration is not
typically recommended because of the associated risks (eg, bleeding, amebic peritonitis, puncture of an
echinococcal cyst, secondary bacterial infection).
Treatment requires both tissue and luminal agents. Commonly used tissue agents include oral metronidazole
and tinidazole, while commonly used luminal agents include paromomycin, iodoquinol, or diloxanide furoate.
Surgical intervention may be indicated, when the abscess has ruptured, eroded into adjacent structures, or
caused extrahepatic complications such as small bowel obstruction.

38
LIVER TUMORS
Metastases are the most common malignant neoplasms of the adult liver and are twenty times more
common than hepatocellular carcinoma. Typically, multiple hepatic nodules of varying sizes are seen. The
liver is the second most common site of metastatic spread (after the lymph nodes) because of its large size,
dual blood supply, high perfusion rate, and the filtration function of Kupffer cells. The most common primary
tumors to metastasize to the liver include malignancies of the gastrointestinal tract, lung, breast, and skin
(melanoma).
Hepatocellular carcinoma (HCC) is a far less common cause of hepatic malignancy than is metastasis
from another source. When HCC is present, it is typically accompanied by an elevation of alpha-fetoprotein.
High serum alpha fetoprotein (> 500 mg/ml) in an adult with liver disease and no obvious gastrointestinal
malignancy is strongly suggestive of hepatocellular carcinoma. Imaging of HCC usually reveals one lesion
with poorly defined margins instead of several masses of varying sizes..
Hepatic adenomas are benign epithelial tumors of the liver that usually arise as a solitary mass in the right
hepatic lobe. They are found predominantly in young and middle-aged women who have a lengthy history of
oral contraceptive usage. Physical findings include a palpable abdominal mass, hepatomegaly, or jaundice.
Liver function tests are elevated on occasion. Alpha-fetoprotein is normal unless malignant transformation of
the tumor has occurred. Severe intra-tumor hemorrhage (obtaining biopsy of these tumors are normally
contraindicated) and malignant transformation are the most dreaded complications. All symptomatic
adenomas should be resected.
Focal nodular hyperplasia (FNH) is a very common non-malignant hepatic tumor. It arises as hyperplasic
response to hyper-perfusion by anomalies arteries that are present in the center of the nodule. Biopsy shows
sinusoids & Kupffer cells.

39
HYPERBILIRUBINEMIA
If a patient has jaundice as he has elevated serum bilirubin (normal total bilirubin < 1mg/dl).
The next step is to determine whether he has predominantly conjugated or unconjugated hyperbilirubinemia.
Use urine dipstick, if positive for the presence of bilirubin in urine conjugated hyperbilirubinemia. OR
Conjugated hyperbilirubinemia is present when direct or conjugated bilirubin constitutes >50% of total
bilirubin. Unconjugated hyperbilirubinemia is present when indirect or unconjugated bilirubin constitutes
>90% of the total bilirubin.

CONJUGATED HYPERBILIRUBINEMIA
Typically, elevated conjugated bilirubin levels are suggestive of hepatobiliary disease (eg, cirrhosis or
hepatitis) because the bilirubin conjugates will reflux back into the plasma when the secretion of conjugated
bilirubin into the bile is slowed.
Predominantly conjugated hyperbilirubinemia occurs in following conditions:
A) Intrahepatic causes:
1) Intrahepatic obstruction, which can occur in, conditions like viral or autoimmune hepatitis, alcoholic
hepatitis, drug reactions, third trimester of pregnancy or postoperative states.
2) Congenital defects in biliary excretion, i.e. Dubin Johnson and Rotor's syndrome.
B) Extrahepatic biliary obstruction.

The next step in the evaluation of patients with conjugated hyperbilirubinemia is to study the liver enzymes.
Patients with dominant aminotransferase elevation usually have hepatocellular disease whereas patients with
dominant elevation of alkaline phosphatase have predominant intra or extrahepatic biliary obstruction.
In the second group of patients it is very important to rule out an extra hepatic obstruction with an
ultrasonogram or a CT scan of abdomen. If these imaging techniques fail to show an extra hepatic biliary
dilatation then the next step would be to go for an ERCP or PTC.
Rotor's syndrome is a benign condition in which there is a defect of hepatic storage of conjugated bilirubin
resulting in its leakage into plasma. Liver function tests are normal. Treatment is unnecessary.
Dubin-Johnson is a familial disorder of hepatic bile secretion. Leads to conjugated hyperbilirubinemia. May
be aggravated by women taking OCPs. Liver biopsy reveals cells with DARK granular pigments in the
hepatocytes caused by epinephrine metabolites. Grossly liver is strikingly black. Treatment is unnecessary.

40
UNCONJUGATED HYPERBILIRUBINEMIA
Elevated unconjugated bilirubin levels typically indicate an increased level of bilirubin formation (such as
that seen in hemolysis) or a slowing in bilirubin conjugation (such as Gilbert's syndrome).
Gilbert's syndrome, Crigler-Najjar type 1, and Crigler-Najjar type 2 are three familial disorders characterized
by varying degrees of severity and unconjugated hyperbilirubinemia.
Gilbert's syndrome is a familial disorder of bilirubin glucuronidation in which the production of UDP
glucuronyl transferases (enzymes that mediate glucuronidation of various substances) is reduced. Clinical
manifestations of Gilbert's syndrome include icterus secondary to a mild, predominantly unconjugated
hyperbilirubinemia (< 3 mg/dL). Those patients who are symptomatic tend to have nonspecific complaints,
including malaise, fatigue, or abdominal discomfort. Certain events, such as hemolysis, fasting or consuming
a fat-free diet, physical exertion, febrile illness, stress, or fatigue are thought to be triggers for
hyperbilirubinemia in patients with Gilbert's syndrome. The diagnosis is suggested in those patients with no
apparent liver disease who have mild unconjugated hyperbilirubinemia thought to be provoked by one of the
classic triggers. Presumptive diagnosis can be made when the unconjugated hyperbilirubinemia persists with
repeat testing, but liver function tests, complete blood count, blood smear, and reticulocyte count are normal.
Treatment is generally considered to be unnecessary in patients with Gilbert's syndrome.
Crigler-Najjar syndrome type 1 is an autosomal recessive disorder of bilirubin metabolism characterized
by severe jaundice and neurologic impairment due to kernicterus (bilirubin encephalopathy). In these infants,
the indirect bilirubin levels are typically > 20-25 mg/dL but can rise to as high as 50 mg/dL. Liver enzymes
and histology are normal. If intravenous phenobarbital is administered, the serum bilirubin remains
unchanged. Phototherapy or plasmapheresis are typically helpful in the short term, though liver transplant is
the only curative option.
Crigler-Najjar syndrome type 2 is a milder autosomal recessive disorder of bilirubin metabolism
characterized by lower serum bilirubin levels (< 20 mg/dL) and survival into adulthood with no kernicterus
or neurologic impairment. Liver enzymes and histology are normal. If intravenous phenobarbital is
administered, the serum bilirubin is reduced. Treatment is often unnecessary in milder cases, though periodic
administration of phenobarbital or clofibrate can reduce serum bilirubin levels if necessary.
Chronic hemolysis can result in an unconjugated hyperbilirubinemia with anemia.

PAINLESS OBSTRUCTIVE JAUNDICE


Painless jaundice in an elderly patient should make you think about pancreatic head carcinoma.
Adenocarcinoma of the head of pancreas causes obstruction of extra hepatic bile duct leading to elevation of
direct bilirubin (conjugated) and marked icterus. It may be associated with weight loss, dull back pain, and
anorexia. There is astronomical elevation of alkaline phosphatase.
Abdominal examination is usually negative except for mild tenderness in epigastric region.
CT scan may help in diagnosis.

41
Obstructive jaundice (cholestatic)
direct bilirubin and alkaline phosphatase.
Abdominal U/S done 1st.
Abdominal CT is the next diagnostic step if U/S does not explain the obstructive jaundice.

N.B.
Elevated alkaline phosphatase levels in the setting of normal levels of serum alanine aminotransferase (ALT),
serum aspartate aminotransferase (AST) and gamma glutamyl transferase (GGT), suggesting that the cause is
probably bone rather than liver pathology. The most common cause of asymptomatic elevation of alkaline
phosphatase in an elderly patient is Paget's disease.

GALLSTONES (CHOLELITHIASIS)
There are 3 types: 1- Cholestrol, 2- Pigment stones (mostly calcium bilirubinate, 20%) and 3- Mixed stones.
Water insoluble cholesterol is secreted in bile where its converted into soluble miscles by bile acids and
phospholipids. If too much cholesterol and too little bile then cholesterol crystals precipitate.
Predisposing factors are Fat, Femlae, Forty, Fertile (OCP), cloFibrate.
Remember 80% of stones are radiolucent so x-ray cant see them.
TPN did not pass through GI decrease activity and decrease CCK impaired gall bladder contraction
gall bladder stasis gall bladder stone formation.
Estrogen promotes formation of cholesterol stones by stimulation the activity of HMG-CoA reductase.
Abdominal ultrasound is the best tool for the initial investigation of gall bladder pathology.
Abdominal ultrasound should be used to search for gall stones in all patients experiencing a first attack of
acute pancreatitis (CT scan of limited use in the evaluation of gallstones).
Abdominal CT is used to confirm the diagnosis of acute pancreatitis.
Asymptomatic gallstones should not be treated. The only exception to this rule would be patients who are at
increased risk of developing gallbladder carcinoma or gallstone complications (e.g. morbid obese patient
undergoing gastric bypass surgery & porcelain gallbladder).
Laparoscopic cholecystectomy is indicated in all patients with symptomatic gall stones who are medically
stable enough to undergo surgery.
Choledocholithiasis symptomatic of biliary colic and without any systemic toxicity is treated with analgesic
and spasmolytics and elective surgery is done at a later date.
Ursodeoxycholic acid is a medication used to reduce biliary secretion of cholesterol and decreases the
cholesterol saturation of bile, resulting in gradual dissolution of cholesterol-containing (small radiolucent)
gallstones in 30 to 40% of patients with normal, functional gallbladders who are poor surgical candidates.
However, this medication is very costly and associated with a high risk of relapse when therapy is halted.

42
TPN
TPN did not pass through GI decrease activity and decrease CCK impaired gall bladder contraction
gall bladder stasis gall bladder stone formation.
TPN causes gall bladder stasis and predisposes to gall bladder stone formation and bile sludging
cholecystitis.

POST-OPERATIVE CHOLESTASIS
It is a benign condition that often develops after a major surgery (characterized by hypotension, extensive
blood loss into tissues & massive blood replacement).
The jaundice is thought to develop secondary to 3 factors: 1- increased pigment load (caused by transfusion),
2- decreased liver functionally (caused by hypotension), 3- reduced renal bilirubin excretion (caused by
tubular necrosis due to hypotension).
Generally jaundice is elevated by the 2nd or 3rd day post-operatively.
Alkaline phosphatase levels are markedly elevated.
AST & ALT levels are typically normal or mildly elevated.

POSTCHOLECYSTECTOMY PAIN
It is usually due to one of three reasons: sphincter of Oddi dysfunction, a common bile duct stone, or
functional pain.
Normal liver function tests and no evidence of dilatation of biliary tree, both sphincter of Oddi dysfunction
and common bile duct stone can be ruled out functional pain with no obvious organic basis and should be
treated symptomatically with analgesics and reassurance.
If abnormal alkaline phosphatase or dilatation of the biliary tree observed on abdominal ultrasound, then the
presence of either a common bile duct stone or sphincter of Oddi dysfunction would be far more likely. ERCP
would then be indicated to confirm and treat the condition, either through stone removal or sphincterotomy.

ACUTE ASCENDING CHOLANGITIS


Charcot's triad of fever, severe jaundice, and right upper quadrant abdominal pain. Confusion and
hypotension may also be observed in those with suppurative cholangitis, producing Reynold's pentad.
Most commonly arises secondary to obstruction of the common bile duct by stone or stricture.
Immediate provision of supportive care and broad-spectrum antibiotics is required. In 80% of patients, this
course of treatment is sufficient. However, some patients continue to have persistent abdominal pain,
hypotension despite adequate fluids, high fever, or mental confusion. These signs and symptoms are
indications for urgent biliary decompression, preferably by endoscopic retrograde
cholangiopancreatography (ERCP). Therapeutic intervention typically includes sphincterotomy with stone
removal and/or stent insertion.

43
ACUTE CHOLECYSTITIS
Acute cholecystitis is secondary to gallstone formation in over 90% of cases, most commonly arising when a
gallstone impacts in the cystic duct. Ingestion of fatty foods then stimulates the contraction of the gallbladder
against the obstructed cystic duct, causing severe colicky pain. The tissue behind the duct obstruction becomes
inflamed, typically as a result of bacterial overgrowth secondary to stasis. The subsequent ischemic changes
can cause gangrene and perforation, with generalized peritonitis or a well-circumscribed abscess the more
common outcomes. Other potential complications include cholangitis and chronic cholecystitis.
Acute cholecystitis typically develops after a large or fatty meal and is characterized by the sudden
appearance of steady pain in the epigastrium or right upper quadrant. The pain may radiate to the right scapula
or be accompanied by Murphy's sign, which is the sharp arrest of inspiration with gallbladder palpation. Other
classical findings include fever, vomiting, and leukocytosis.
Total serum bilirubin may range from 1-4 mg/dL, and serum aminotransferases and alkaline phosphatase are
often mildly elevated.
Treatment for acute cholecystitis includes supportive care (nothing by mouth, with intravenous administration
of antibiotics, alimentation, and analgesics).
Laparoscopic cholecystectomy is recommended shortly after hospitalization and should be performed
immediately in cases of perforation or gangrene.

EMPHYSEMATOUS CHOLECYSTITIS
Emphysematous cholecystitis is a common form of acute cholecystitis in elderly diabetic males.
It is a form of acute cholecystitis that arises due to secondary infection of the gallbladder wall with gas-
forming bacteria (eg, Clostridium, Escherichia, Staphylococcus, Streptococcus, Pseudomonas, and
Klebsiella). Predisposing factors include vascular compromise (by obstruction or stenosis of the cystic artery,
which is the sole arterial supply to the gallbladder), immunosuppression (as seen with diabetes mellitus),
gallstones, and infection with gas-forming bacteria.
Crepitus in the abdominal wall adjacent to the gallbladder is occasionally detectable, but peritoneal signs are
absent. Complications include gangrene and perforation. If perforation occurs, the abdominal pain is relieved
but peritonitis results.
Diagnosis is confirmed with an abdominal radiograph demonstrating air fluid levels in the gallbladder or an
ultrasound showing curvilinear gas shadowing in the gallbladder. Laboratory findings may include a mild to
moderate unconjugated hyperbilirubinemia or a small elevation in aminotransferases.
Treatment includes immediate fluid and electrolyte resuscitation, early cholecystectomy, and parenteral
antibiotic therapy that is effective against the gram-positive anaerobic Clostridium sp. (ampicillin-sulbactam,
piperacillin-tazobactam, or the combination of an aminoglycoside or quinolone with clindamycin or
metronidazole).

44
ENDOSCOPIC RETROGRADE CHOLANGIOPANCREATOGRAPHY (ERCP)
One of the known complications of ERCP is an iatrogenic biliary enteric fistula characterized by the
presence of air in the biliary tree. Other complications include pancreatitis, biliary peritonitis, sepsis,
hemorrhage, and adverse effects from the contrast, sedative, or anticholinergic agents.
Certain procedures (eg, the air insufflation test for infertility) can result in the finding of free air under the
diaphragm.

ACALCULOUS CHOLECYSTITIS
Acute acalculous cholecystitis is an acute inflammation of the gallbladder in the absence of gallstones.
It is most commonly seen in hospitalized and severely ill patients in the following conditions:
1) Extensive burns
2) Severe trauma
3) Prolonged TPN
4) Prolonged fasting
5) Mechanical ventilation
The initial investigation of choice is an ultrasonogram, which shows signs of acute cholecystitis and no
gallstones; however, CT scan and HIDA scans are more sensitive and specific for the diagnosis.

PRIMARY BILIARY CIRRHOSIS (PBC)


A chronic liver disease characterized by autoimmune destruction of the intrahepatic bile ducts and
cholestasis.
Pruritus is usually the first symptom and may be very severe, especially at night. Fatigue is also often
reported. Physical findings typically include hepatosplenomegaly and xanthomatous lesions in the eyelids or
in the skin and tendons. As the disease progresses, jaundice, steatorrhea, portal hypertension, and osteopenia
may develop.
Abnormalities on laboratory testing are: alkaline phosphatase, cholesterol, and serum IgM.
Diagnosis is confirmed with demonstration of anti-mitochondrial antibodies in the serum.
Ursodeoxycholic acid is the drug of choice in treating PBC as it slows disease progression.
Advanced disease is an indication for liver transplantation.

45
INTRAHEPATIC CHOLESTASIS OF PREGNANCY (ICP)
It is a functional disorder of bile formation that develops in the second and third trimesters of pregnancy. The
etiology is unclear, but it is thought that genetic and hormonal factors (eg, higher levels of estrogen or
progesterone) influence the development of ICP.
The condition is characterized by intense pruritus especially significant on the palms and soles and worsens
at night. Evidence of skin excorations may be present. Jaundice exists in 10-20% of patients.
The diagnosis of ICP is indeed one of exclusion.
On laboratory evaluation, increased serum bile acid concentrations and may be the only finding.
Liver function studies are sometimes suggestive of cholestasis, with alkaline phosphatase and the total and
direct bilirubin increased. GGTP is usually either normal or only mildly elevated. Serum aminotransferases
may be quite high (sometimes > 1000U/L), which requires the ruling out of viral hepatitis.
Treatment is based on symptom relief and preventing complications in the mother and fetus. Ursodeoxycholic
acid is most promising, as it increases bile flow and can relieve pruritus. Hydroxyzine and cholestyramine are
alternative therapies, though they appear to be less effective and can have concerning side effects.
Affected women are also at increased risk of developing gallstones.
ICP may recur in subsequent pregnancies and poses more danger to the fetus than to the mother, as fetal
prematurity, meconium-stained amniotic fluid, and intrauterine demise are all known complications.
Therefore, the preferred approach in managing the pregnancies of women with ICP includes an early delivery
once fetal lung maturity is established.
Jaundice in the third trimester of pregnancy should be evaluated specifically for those hepatic disorders
associated with pregnancy. Marked pruritus and elevated of total bile acids are suggestive of intrahepatic
cholestasis of pregnancy.

URSODEOXYCHOLIC ACID
Ursodeoxycholic acid decreases the cholesterol content of the bile by reducing the hepatic secretion and
intestinal reabsorption of cholesterol. It is a used to dissolve small radiolucent gallstones in patients with
normal, functional gallbladders who are poor surgical candidates.
However, this medication is very costly and associated with a high risk of relapse when therapy is halted.
Ursodeoxycholic acid is the most commonly used drug in the treatment of primary biliary cirrhosis as it
relieves symptoms, slows disease progression and lengthens transplant-free survival time.

DUCTOPENIA
Vanishing bile duct syndrome is a rare disease involving progressive destruction of the intrahepatic bile
ducts. Liver biopsy (histologically) shows markedly decreased quantity of bile ducts (ductopenia).
Primary biliary cirrhosis ductopenia.
Failing liver transplantation, graft-versus-host disease, Hodgkins ductopenia.

46
CHOLEDOCHAL CYST
A choledochal cyst is a congenital abnormality of the biliary ducts characterized by the dilatation of intra or
extra-hepatic biliary ducts or both. It has a multifactorial origin, but most of the cases are related to an
anomalous pancreaticobiliary junction, which leads to weakness and dilatation of the biliary wall due to the
reflux of alkaline pancreatic secretions into the biliary tree.
The clinical presentation varies with the age. In infants it may present with jaundice and the passage of acholic
stools. In children it causes abdominal pain, jaundice, and attacks of recurrent pancreatitis, which may be
evident by increases in the amylase and lipase levels. Adults with choledochal cysts commonly present with
vague epigastric or right upper quadrant abdominal pain or cholangitis.
Choledochal cysts can degenerate into cholangiocarcinoma.
The initial investigation of choice is an ultrasonogram followed by CT scan or magnetic resonance imaging
(MRI) as needed.
Isolated (single) liver mass is much more likely to be the result of metastatic disease.

GALL BLADDER CARCINOMA


It is a rare tumor, that most often arises in Hispanic & southwestern native American females who have a
history of gall stones (chronic Choledocholithiasis).
It is typically during or after cholecystectomy.
Cholecystectomy is sufficient to treat carcinoma of the gall bladder confined to the mucosa (lamina propria).

SPHINCTER OF ODDI DYSFUNCTION


Endoscopic retrograde cholangiopancreatography (ERCP) is one of the most sensitive and specific tools for
imaging the biliary system, and is particularly useful if therapeutic intervention is planned.
ERCP with sphincterotomy (incising the musculature) is the treatment of choice for sphincter of Oddi
dysfunction
HIDA scan is a very sensitive tool for diagnosis of cholecystitis or sphincter of Oddi dysfunction.

SPLENIC ABSCESS
Fever, chills, left upper quadrant pain, and splenic fluid collection are findings consistent with infective
endocarditis with septic emboli to the spleen.
Left sided endocarditis can send septic emboli to regions rich in blood supply, such as brain, kidneys, liver and
spleen.
Right sided endocarditis is more likely to cause septic pulmonary emboli.
Septic emboli predisposes to abscess formation.

47
PORCELAIN GALLBLADDER

This abdominal radiograph contains the classic finding of a "porcelain gallbladder," a term used to describe the
bluish, brittle, calcium-laden gallbladder wall that can develop in some patients with chronic cholecystitis.
The porcelain gallbladder is typically diagnosed on radiograph, sometimes incidentally, by a rim of calcium
deposits that outline the gallbladder.
Porcelain gallbladder results from the intramural deposition of calcium salts, secondary to either chronic
irritation from gallstones or as a component of the naturalprogression of chronic inflammation.
The condition is associated with an increased risk of gallbladder carcinoma and requires surgical resection
(cholecystectomy).

CHRONIC DIC
Migratory thrombophlebitis and atypical venous thromboses are suggestive of chronic DIC, which is most
likely due to cancer.
The typical laboratory findings of chronic DIC include mild prolongation of PT, low fibrinogen levels and
positive Fibrin split products (FSP).
The most common causes are malignancies of the lung, pancreas, stomach and prostate.
CT of the chest, abdomen and pelvis are thus indicated to identify and further evaluate the underlying
malignancy, along with age-appropriate cancer screening (e.g., digital rectal exam, mammography,
colonoscopy).

48
IgA deficiency significantly increases the risk of developing anaphylactic reaction to transfused blood
products. The risk of anaphylaxis may be reduced in susceptible patients by providing IgA-deficient blood
products or by performing additional washes on red blood cells or platelet products. Anaphylaxis typically
presents within minutes of starting transfusion and manifests as wheezing, respiratory distress, tachycardia
and hypotension. Similar mechanism seen in patients with ahaptoglobinemia or anti-haptoglobin antibodies.

REACTION TO CYTOKINES
It is the most common transfusion reaction leads to febrile non hemolytic reactions (fever, chills, malaise).
During blood storage, leukocytes release cytokines, which when transfused cause transient fever, chills, and
malaise.
Therefore, leukocyte depletion techniques (although not commonly employed due to high cost) can reduce the
probability of febrile transfusion reaction. These techniques are cell washing, use of frozen deglycerolized red
cells, use of leukocyte depletion RBC filters, etc.

ACUTE HEMOLYTIC TRANSFUSION REACTION


In which donor erythrocytes are rapidly destroyed by preformed recipient antibodies. This is typically due to
ABO mismatching.
It is classically presents as fever, chills, flank pain, and hemoglobinuria.
Diagnosis is made by positive direct antiglobulin test (Coombs), plasma free hemoglobin showing pink
plasma with hemoglobin concentration > 25 mg/dl, urine analysis will also show the presence of hemoglobin,
and repeated typing & cross matching revealing a mismatch.
Complications: DIC, ARF, cardiovascular collapse (shock) and even death.
Treatment: stop the transfusion, & supportive treatment (e.g. NSAIDs, acetaminophen, IV fluids ).

FRESH FROZEN PLASMA


FFP is the therapeutic agent of choice for coagulopathy in patients with liver failure.
Warfarin treated patients should be given fresh frozen plasma instead of vitamin K when emergency surgical
procedure is to be performed.
Patients who develop serious bleeding (e.g., intracerebral hemorrhage) due to excess anticoagulation with
warfarin should be given fresh frozen plasma (FFP) for the rapid reversal of anticoagulation.
Vitamin K reverses the action of warfarin, but takes 8-12 hours to be effective.
Patients with prosthetic valves usually tolerate cessation of oral anticoagulant therapy for about a week
without significant increase of the risk of thrombosis. So, reversing with FFP is a very good option. It is
difficult to anticoagulate after the operation if you give vitamin K in these people and it takes several days to
achieve therapeutic INR.

49
Calcium gluconate infusion is employed in rare cases of severe hypocalcaemia following massive blood
transfusion.
Warming the blood is recommended only during rapid massive transfusion to prevent hypothermia.

Individuals who received blood transfusions before 1992 should be screened for hepatitis C.
Those who received blood transfusions before 1986 should be screened for hepatitis B.

HERPES ZOSTER (SHINGLES)


It develops when there is reactivation of VZV (chickenpox) in the sensory dorsal root ganglia.
Elderly & immunosuppressed patients (HIV, DIM, chemotherapy, malignancy,) are at high risk.
Pain is the most common symptom [light touch to the skin elicits intense pain].
Vesicular eruption (rash) appears several days after the pain along the dermatomal distribution.
It may lead chronic pain syndrome (post-herpetic neuralgia).
Treatment: analgesics and antiviral like acyclovir.

DIGOXIN
Digoxin is a drug of narrow therapeutic window.
Several drugs e.g. verapamil ( renal clearance of digoxin) digoxin concentration toxicity.
The most common side effects of digoxin toxicity is GI disturbance (e.g. anorexia, nausea & vomiting).

FOLIC ACID & VITAMIN B12 DEFICIENCY


Patients who eat heated food all the time, such as toast and tea are more likely to develop folic acid deficiency
[Folic acid is heat sensitive] macrocytic anemia. Folic acid stores in the body can be depleted in 45
months.
Strict vegetarians (vegans) may become deficient in vitamin B12 macrocytic anemia and neurologic
deficits. The body has B12 stores sufficient to last at least 34 years.

ANAL FISSURE
Anal fissures are slit-like tears of the anal canal. These are mostly located on the posterior or anterior anal
verge, and are most commonly caused by the passage of large, hard constipated stools.
The most common symptoms are severe pain and bright red rectal bleeding during defecation.
The initial treatment of both acute and chronic anal fissures includes dietary modification (e.g., high-fiber diet
and large amounts of fluids), a stool softener and a local anesthetic.
Surgical repair of a fissure is indicated in chronic or refractory acute fissures to provide a wider aperture for
stool to pass and to interrupt the internal sphincter spasm. Lateral sphincterotomy is the standard treatment for
chronic fissures.

50
*** AS A ROLE IN ENDOCRINOLOGY, IMAGING IS ALWAYS PERFORMED AFTER
BIOCHEMICAL DIAGNOSIS OF DISORDER IS MADE.

HYPERPROLACTINEMIA
Approximately 10% of pituitary tumors do not produce any hormone (non-functioning).
The most common pituitary adenoma is prolactin producing lactotroph adenomas.
Dopamine agonists (e.g. cabergoline or bromocriptine) are the mainstay of the treatment for
most patients with prolactinomas (micro- as well as macro-).
Surgery is reserved for adenomas not responsive to cabergoline or bromocriptine.

ACROMEGALY
The gold standard test for making a diagnosis of acromegaly is measurement of growth
hormone levels following an oral glucose load. Most patients with acromegaly are unable
to suppress growth hormone levels following an oral glucose load.
The most common cause of death in patients with acromegaly is cardiovascular (congestive
cardiac failure, diastolic dysfunction), accounting for 38-62% of deaths. Normalization of
growth hormone level following successful treatment reduces cardiovascular mortalities.

HYPOPITUITARISM
The two most common causes of hypopituitarism in the postpartum period are Sheehan's
syndrome and lymphocytic hypophysitis.
Sheehan's syndrome develops due to ischemic necrosis of the pituitary gland (sometimes
even the hypothalamic nuclei) because of peri-partum bleeding. First symptom is failure to
lactate.

51
DIABETES INSIPIDUS (DI)
DI is characterized by defective ADH production due to hypothalamic-pituitary disorders
(central diabetes insipidus) or resistance to ADH action on the renal tubules (nephrogenic
diabetes insipidus) as in Lithium-induced DI, Demeclocycline, clochicine.
ADH excessive diluted urine (polyuria), increased thirst, hypernatremia Patients
are typically polydipsic and prefer cold beverages Serum osmolality and urine
osmolality.

Water deprivation test is used to differentiate between psychogenic polydipsia and DI. Fluid
restriction produces plasma hyperosmolality. The normal reaction to plasma
hyperosmolality is the production of maximally concentrated urine (urine hyperosmolality);
this is seen in patients with psychogenic polydipsia. In patients with DI, the urine continues
to be dilute (low osmolality) despite high plasma osmolality.

Administration of DDAVP after water deprivation can distinguish between central and
nephrogenic diabetic insipidus. Patients with central DI will have an increase of urine
osmolality by at least 50% following the administration of AVP or DDAVP, whereas
patients with nephrogenic DI will not have such an increase. The treatment of choice for
central DI is desmopressin, which is usually administered intranasally.

Water deprivation should not be performed when a patient is hypovolemic.

Normal saline is the initial fluid of choice in hypotensive, dehydrated patients with DI.
Hypotonic fluids can be started when the intravascular volume improves.

In lithium-induced DI, amiloride is the preferred therapeutic agent because it prevents


further lithium accumulation in the renal tubules.

***Diabetes insipidus presents as polyuria, polydipsia and excretion of dilute urine in the
presence of elevated serum osmolality.
***Primary polydipsia is due to excessive water drinking; both plasma and urine are diluted.
***SIADH results in hyponatremia, low serum osmolality and inappropriately high urine
osmolality.

52
SYNDROME OF INAPPROPRIATE SECRETION OF ADH (SIADH)
SIADH is commonly seen in patients with intracranial hemorrhage & lung carcinoma.
SIADH is characterized by hypotonic hyponatremia with euvolemia (without hypertension
or edema). Low plasma osmolality (<280 mOsm/Kg) with high urine osmolality (>100-150
mOsm/Kg) in suspected patients is diagnostic.
Water restriction is the initial treatment of choice.
Summary of the management of hyponatremia due to SIADH:
Mild (asymptomatic with sodium 120-130 meq/L) = Fluid restriction
Moderate (asymptomatic with sodium 110-120 meq/L) = Loop diuretic + Normal saline
Severe (sodium <120 or if the patient is seizing) = Hypertonic saline (3% sodium
chloride) with or without furosemide to raise the serum sodium to 125.
Rapid correction of serum sodium levels can lead to central pontine myelinosis.

"Cerebral salt-wasting syndrome"


It may occur in patients with subarachnoid hemorrhage. The pathology involves: (1) an
inappropriate secretion of vasopressin, which causes water retention, and (2) an increased
secretion of atrial/brain natriuretic peptide, which causes cerebral salt-wasting.
These changes result in hyponatremia, which usually resolves within 1-2 weeks.

53
HYPERTHYROIDISM
Thyrotoxicosis: T3, T4 level and TSH levels.
Radioactive iodine uptake (RAIU) used to differentiae between primary and secondary
hyperthyroidism.
RAIU primary hyperthyroidism (de novo synthesis)
RAIU secondary hyperthyroidism.
The most important causes of thyrotoxicosis with low radioactive iodine uptake include:
1) Subacute lymphocytic (painless) thyroiditis
2) Subacute granulomatous thyroiditis
3) Iodine-induced thyroid toxicosis
4) Levothyroxine overdose
5) Struma ovarii

In Graves' disease; there is sympathetic nervous system-mediated lid retraction.


Graves' ophthalmopathy is the most common cause of exophthalmous. It results from
lymphocytes infiltrate the EOMs and orbital fat causes edema, proliferation of local
interstitial fibroblasts and deposition glycosaminoglycans (hyaluronic acid) in the retro-
orbital muscles and tissue leads to proptosis. It may be unilateral or bilateral. Eye
grittiness or sandiness or excessive tearing may be due to keratitis.

Excess thyroid hormones target organ sensitivity to endogenous catecholamines.


Hypertension in patients with thyrotoxicosis is predominantly systolic (with increase pulse
pressure) and caused by hyperdynamic circulation.

Untreated hyperthyroid patients are also at risk for cardiac tachyarrhythmias, including
atrial fibrillation.

Untreated hyperthyroid patients are at risk for rapid bone loss resulting from increased
osteoclastic activity in the bone cells.
Thyrotoxicosis T3 and T4 direct effect on the bone cells osteoclastic
activity rapid bone loss (bone resorption) and hypercalcemia.

54
Thyroid-stimulating immunoglobulins (TSI) are present in patients with Graves' disease.
TSI stimulate TSH receptors on the thyroid follicular cells, thereby resulting in thyroid
hormone overproduction.

Radioactive iodine therapy is the preferred treatment for most patients with Graves'
disease. The contraindications of radioactive iodine treatment are pregnancy and very
severe ophthalmopathy. The incidence of thyroid cancer is unchanged in patients who
receive radioactive iodine therapy. Hypothyroidism is the most common side effect of
Radioactive iodine therapy, but it is easy to be treated with levothyroxine therapy.
Radioiodine therapy is more likely to cause permanent hypothyroidism in patients with
Graves' disease than those with multinodular goiter or toxic adenoma.
Exacerbation of ophthalmopathy may occur at the beginning of Radioactive iodine
therapy. It can be prevented by high doses of glucocorticoids before & after therapy.
Glucocorticoids (e.g. hydrocortisone & prednisone) decrease peripheral conversion of
T4 to T3 (biologically active thyroid hormone) decrease level of circulating T3
improvement of hyperthyroid state.
Radioactive iodine therapy death of thyroid cells release of thyroid hormones from
dead cells excess thyroid hormones in the circulation exacerbation of the hyperthyroid
state (e.g. atrial fibrillation and hand tremors).
It is recommended in the elderly patients and patients with significant cardiovascular
disease to be treated with Antithyroid drugs to deplete thyroid hormone stores before the
beginning of Radioactive iodine therapy.

Antithyroid drugs e.g. propylthiouracil (PTU) & Methimazole are primarily used in the
treatment of hyperthyroidism during pregnancy, and in preparation for surgery or
radioactive iodine therapy. Antithyroid drugs may be used in selected cases of Graves'
disease with mild hyperthyroidism. The most common side effect is allergic reaction. The
most serious side effect is agranulocytosis.
Fever and sore throat in any patient taking anti-thyroid drugs suggests agranulocytosis.
Anti-thyroid drugs should be stopped, WBC count checked, and IV administration of broad
spectrum antibiotics (especially for pseudomonas). If WBC count < 1000/mm3
permanent discontinuation of the drug. If WBC count > 1500/mm3 anti-thyroid drug
toxicity is unlikely to be the cause of fever and sore throat.

55
In the management of patients with hyperthyroidism, propanolol is generally used for
symptomatic relief until the underlying cause is identified and definitively treated.

Factitious thyrotoxicosis results from ingestion of exogenous thyroid hormones. Patients


present with signs and symptoms of hyperthyroidism (e.g. palpitations, sweating, weight
loss, hyperactivity & diarrhea), may be lid lag, BUT NO GOITER OR EXOPHTHALMOUS.
There is T3, T4, TSH and RAIU shows diffusely iodine uptake by the thyroid.
Biopsy shows follicular atrophy due to suppression of endogenous thyroid hormone
production.

TSH-secreting pituitary adenoma (mostly macroadenomas) excessive TSH levels


stimulate the thyroid gland hyperthyroid state (elevated T3, T4). It is biochemically
characterized by elevated circulating thyroid hormone levels with inappropriately normal or
elevated TSH levels. The pathophysiology generally involves overproduction of the alpha-
subunit of TSH.

HYPOTHYROIDISM
Hyperlipidemia, unexplained hyponatremia, anemia, & elevation of serum muscle enzymes
(myopathy) are indicators for thyroid function tests.
Hyperlipidemia occurs with increased frequency in hypothyroid patients. Caution must be
exercised in patients with poorly-controlled hypothyroidism because these patients have an
increased risk of myopathy with statin use.
Hyponatremia may occur in hypothyroid patients due to inappropriate ADH secretion.
Patients with hypothyroidism may have asymptomatic elevation of serum muscle enzymes.
The most common anemia in hypothyroid patients is normocytic, normochromic.
Patients with chronic autoimmune thyroiditis may have pernicious anemia.
Women of reproductive age may develop iron deficiency anemia secondary to menorrhagia.

56
Generalized resistance to thyroid hormones have elevated T3, T4 and TSH levels;
however, these patients are clinically hypothyroid (not hyperthyroid). Results from
receptors defect on the peripheral tissues.

SICK EUTHYROID SYNDROME low T3 syndrome


Any patient with an acute, severe disease may have an abnormal thyroid function test (i.e.,
abnormal thyroid hormone and TSH levels).
This condition is thought to be due to caloric deprivation and an increase in cytokine levels
(e.g., interleukin-1 and interleukin-6).
For these reasons, thyroid function testing is usually not performed in patients with an acute,
severe disease if there is no clinical suspicion of an underlying thyroid disease.
The most common thyroid hormone pattern is a fall in total and free T3 levels, with normal
levels of T4 and TSH (often referred to as 'low T3 syndrome').
On recovery from the non-thyroidal illness, patients usually experience a modest, transient
increase in the serum TSH level; therefore, thyroid function tests are also not performed in
patients recovering from major systemic illness, unless there is a strong indication.

THYROIDITIS
Subacute granulomatous (De Quervain's) thyroiditis is associated with intense pain
in the thyroid region.

Hashimoto's thyroiditis: painless diffuse rubbery enlargement of the thyroid. TSH


level and T3, T4 levels. Anti-thyroid peroxidase antibodies are present in more than
90% of patients with Hashimoto's thyroiditis. Increased risk of thyroid lymphoma (CT of
the neck shows enlargement of the thyroid gland around the trachea core biopsy).
Biopsy of Hashimotos thyroiditis thyroid shows dense lymphocytic infiltrates.
Biopsy of Graves disease thyroid shows follicular hyperplasia.
Biopsy of papillary thyroid cancer shows orphan Annie nuclei, psammoma bodies.

57
NEOPLASIA OF THE THYROID
Thyroid cancer of epithelial cell origin FAP: Follicular, Anaplastic & Papillary.
Thyroid cancer of C-cell origin: medullary.

Papillary thyroid cancer (PTC) is the most common thyroid malignancy. It is characterized
by a slow, infiltrative local spread affecting other parts of the thyroid gland and regional
lymph nodes. The presence of psammoma bodies are a characteristic histopathological
feature. PTC is unencapsulated. The prognosis of patients with PTC is excellent, even in
the presence of metastasis.

Follicular thyroid cancer is encapsulated. Histopathologically, demonstration of invasion


of the capsule and blood vessels is required for differentiating follicular cancers from
follicular adenomas. Follicular thyroid cancers have the propensity to invade blood vessels
and metastasize to distal organs.

Medullary cancer of the thyroid secretion of calcitonin.


MEN I: pituitary tumors, pancreatic tumors , and primary hyperparathyroidism.

MEN 2A MEN 2 B
Medullary thyroid cancer Medullary thyroid cancer
Pheochromocytoma Pheochromocytoma
Primary parathyroidism hyperplasia Mucosal neuromas (tongue, lips, eyelids, GIT)
Marfnoid habitus

Normal levels of calcium and PTH rules out MEN 2A.

THYROID NODULES
Most thyroid nodules are benign colloid nodules.
The second most common cause of thyroid nodules is follicular adenoma.
The first step in the evaluation of a patient with a thyroid nodule is measurement of the
thyroid stimulating hormone (TSH) level.
If TSH level levels of thyroid antibodies and thyroxine (T4) are measured.
If TSH level radioiodine uptake (RAIU)and scan are typically performed.

58
If the scan shows a hot nodule (increased uptake of the tracer in the nodule with
decreased uptake in rest of the thyroid gland) NO FNAB because the chances of
malignancy in a hot nodule are extremely low.
If normal TSH level fine needle aspiration biopsy (FNAB) is the next step.

Ultrasound used for guiding the FNAB (especially if the nodule is non-palpable) and for
following nodule growth.

Hashimoto's thyroiditis can have thyroid nodule formation.

HYPERCALCEMIA
PRIMARY HYPERPARATHYROIDISM
Primary hyperparathyroidism: elevated PTH levels, hypercalcemia and hypophosphatemia.
Primary hyperparathyroidism is the most common cause of hypercalcemia in ambulatory
patients.
Surgical intervention (i.e., parathyroidectomy) is needed for all patients with symptomatic
primary hyperparathyroidism.
Not all asymptomatic primary hyperparathyroidism patients require such treatment.
The indication for surgery in asymptomatic patients is the presence of at least one of the
following features:
1) Serum calcium level at least 1 mg/dL above the upper normal limit with urinary calcium
excretion greater than 50mg/24hr
2) Urinary calcium excretion greater than 400mg/24hr
3) Young patients (< 50 years of age)
4) Bone mineral density lower than T-2.5 at any site
5) Difficulty in follow-up of the patient

If the abovementioned criteria for surgery are not met, conservative treatment is advocated.
Patients are monitored with serum calcium levels every six months, and serum creatinine
levels and bone mineral density every year.

59
Bisphosphonate therapy and loop diuretics are used for the treatment of symptomatic
hypercalcemia. Long-term bisphosphonate therapy in patients with asymptomatic
hyperparathyroidism leads to an increased bone mineral density, and may be used in some
patients who refuse surgery.

Secondary hyperparathyroidism is almost a universal finding in patients with end stage


renal disease (ESRD). Phosphate retention, which begins to occur as the glomerular
filtration rate falls, plays a central role in increasing parathyroid hormone levels.
Hypocalcemia, hyperphosphatemia, and increased parathyroid hormone levels are
characteristic biochemical abnormalities of secondary hyperparathyroidism in chronic renal
failure.

Malignancy is one of the most frequent causes of hypercalcemia. There are various
mechanisms by which malignancy produces hypercalcemia: the production of cytokines,
parathyroid hormone related peptide, calcitriol and ectopic PTH.

Metastatic solid tumors (e.g. lung cancer and breast cancer) metastasize to bone local
production of cytokines such as IL-1 & TNF in the bones by the malignant cells
metastatic osteolysis (Plain x-ray shows lytic lesions) hypercalcemia. Serum phosphate
levels are generally normal in local bone osteolysis.

Non-metastatic solid tumors (e.g. lung and breast) production of parathyroid hormone
related peptide (PTHrP) hypercalcemia and serum PTH is typically low (Normal PTH
10-55 pg/ml).

Hodgkin's disease production of calcitriol hypercalcemia.

Certain types of lymphoma Increased vitamin D production hypercalcemia.

Bisphosphonates are the drugs of choice for mild to moderate hypercalcemia due to
malignancy.

60
All women who have metastatic breast cancer and radiographic lytic bone disease, and are
receiving either hormone therapy or chemotherapy.

Corticosteroids are used in patients with hypercalcemia due to extra renal production of
1,25 dihydroxy vitamin D (calcitriol) such as sarcoidosis.

IV fluids and furosemide are used in the treatment of hypercalcemic crisis.

Primary hyperparathyroidism and familial hypocalciuric hypercalcemia (FHH): increased


PTH levels, increased serum calcium, and decreased serum phosphorus. Measurements of
24hr urinary calcium excretion and creatinine clearance can confirm the diagnosis of
FHH and distinguish it from primary hyperparathyroidism. In primary hyperparathyroidism
urinary calcium excretion is normal or elevated while in familial hypocalciuric
hypercalcemia urinary calcium excretion typically is below 200 mg/day (5 mmol/day) when
creatinine clearance is normal.

Prolonged bed rest (immobilization) of an individual with a high bone turnover (e.g., young
children and adults, patients with Paget's disease) increased osteoclastic activation
hypercalcemia occurs within days of immobilization severe osteopenia.
Bisphosphonate therapy in patients who are immobilized is helpful in reducing
hypercalcemia and preventing osteopenia.

Fat-soluble vitamins e.g. A, D, E, K are stored in the body for long period of time, and
hence the potential to accumulate toxic levels (especially with obese persons) when
supplements are taken.
Vitamin D toxicity hypercalcemia constipation, abdominal pain, polydipsia, polyuria
and weight loss. In case of vitamin D toxicity, both vitamin D and calcium should be
withheld until symptoms improve and levels return to normal.
Vitamin A toxicity dry skin, blurred vision, headache and abdominal pain.

61
HYPOCALCEMIA
PRIMARY HYPOPARATHYROIDISM
The most important causes of hypoparathyroidism are surgical and autoimmune.
Autoimmune hypoparathyroidism is usually seen in patients with mucocutaneous
candidiasis.
Hypoparathyroidism PTH hypocalcemia, elevated levels of serum phosphorus.

In patients with Pseudohypoparathyroidism, labs show low serum calcium, high serum
phosphate and high serum PTH. Serum alkaline phosphatase and vitamin D levels are
within normal limits.

Renal failure 1,25 dihydroxy vitamin D intestinal calcium absorption from the
gut hypocalcemia high serum PTH and hyperplasia of parathyroid glands (secondary
hyperparathyroidism) hyperphosphatemia.

Patients with X-linked hypophosphatemic rickets have low serum phosphate due to renal
phosphate wasting. They have normal serum calcium, normal serum PTH and normal
serum alkaline phosphatase and normal levels of serum 25-hydroxyvitamin D. There may
also be a functional defect in the activity of 1-hydroxylase and as a result serum levels of
calcitriol may be low.

RICKETS
Rickets is characterized by defective mineralization of both bone and growth plate
cartilage. It is also caused by vitamin D deficiency, but is seen only in children.
Deformities in rickets include; bowed legs, costochondral beading, craniotabes, and bossing
of the skull.
In type II vitamin D dependent rickets, there is mutation of vitamin D receptor. Therefore,
these patients have normal serum levels of calcitriol but it is ineffective and as a result
osteomalacia occurs.

62
OSTEOMALACIA
Vitamin D is either formed in the skin when exposed to sun or absorbed from the small
intestine (deficiency can occur in a patient who does not go out of the house).
Osteomalacia is characterized by defective mineralization of the bones.
Osteomalacia is caused by vitamin D deficiency in adults formation of 1,25 dihydroxy
vitamin D (calcitriol) intestinal calcium (& phosphorus) absorption from the gut
calcium (hypocalcemia) high serum PTH and hyperplasia of parathyroid glands
secondary hyperparathyroidism calcium level to low-normal serum calcium and
hypophosphatemia.
Labs show low or low-normal serum calcium, low serum phosphate, increased PTH, low
plasma 25-OH vitamin D and normal, low or elevated levels of 1,25 dihydroxy vitamin D
(calcitriol).
Therefore, vitamin D deficiency causes more marked hypophosphatemia than
hypocalcemia, especially in early stages.
Radiologic features (i.e., symmetrical 'looser zones' or pseudofractures, blurring of the
spine) are classic for osteomalacia.
Hypocalcemia with concordant changes of serum calcium and phosphate levels are
usually caused by vitamin D deficiency (e.g., due to malabsorption).

OSTEOPOROSIS
The risk factors for osteoporoses include: Thin body habitus, smoking, alcohol intake,
steroid use, menopause, malnutrition, family history of osteoporosis, anorexic patients
and Asian or Caucasian race.
Osteoporosis is a bone disease characterized by decreased bone mass and a deterioration in
bone structure but the bone that is present is normally mineralized per unit volume, which
leads to increased susceptibility to fractures, especially in the hip, the spine, and wrist.
A compression fracture of the vertebrae is a common complication of advanced
osteoporosis. It usually manifests as acute back pain without an obvious preceding trauma in
a predisposed patient. Neurologic examination will be normal.
Normal serum calcium, normal serum phosphate and normal serum parathyroid hormone.

63
Dual energy x-ray absorptiometry (DEXA) is currently the gold standard and most widely
used investigation for detecting osteoporosis and osteopenia.
Calcium intake should be 1,000 mg/day for persons between 31 and 50 years old and
should reach 1,200 mg in persons over 50 years. Vitamin D is responsible for intestinal
absorption of calcium and is a necessary supplement for optimal calcium assimilation.
Patients should be encouraged to do lifestyle modifications, such as weight-bearing
exercise, smoking cessation, and alcohol abstention.

PAGET'S DISEASE OF THE BONE


The exact cause of the disorder is unclear, but several workers have shown evidence of viral
etiology in Paget's disease.
The single most common cause of asymptomatic isolated elevation of alkaline phosphatase
in an elderly patient is Paget's disease.
Pathophysiologic mechanism is impaired bone remodeling in focal areas of the bone.
Paget's disease of the bone is characterized by excessive bone destruction and repair.
Patients are often asymptomatic; however, those with symptoms usually first note an
increase in hat size (asymmetric enlargement of the cranium) and occasional headaches. In
patients with cranial enlargement, hearing loss is a common complication.
Normal serum calcium, normal serum phosphate and normal serum parathyroid hormone
levels are seen in patients with osteoporosis and Paget's disease.

Markedly elevated serum alkaline phosphate level is seen in Paget's disease.


Radiologic and histologic evidence of increased bone turnover (resorption and formation)
can be readily assessed by measuring the biochemical markers of bone turnover:
1) Alkaline phosphatase - marker of bone formation; it is elevated in patients with Paget's
disease, and is most commonly used to assess the activity of disease, as well as response
to treatment.
2) Urinary n telopeptide - most commonly used to as a marker of bone resorption.

In evaluation of a patient of Pagets disease, the initial goal is to establish which bone have
been affected. Full extend of the disease is best ascertained by full body scan (which shows
increase uptake by the affected bones) followed by radiographic confirmation.

64
Asymptomatic patients of Pagets disease do not require treatment.
Symptomatic patients are best treated with oral or IV Bisphosphonates e.g. alendronate.
Indications of treatment of Pagets disease:
1) Bone pain.
2) Hypercalcemia of immobilization.
3) Neurological deficit.
4) High-output cardiac failure.
5) Preparation of orthopedic surgery.
6) Involvement of weight-bearing bones (to prevent deformity).

A small number of patients of Pagets disease will develop sarcomatous changes over 10
years and third will present as a new lytic lesion and a sudden increase in alkaline
phosphatase.

Plasma calcium exists in three forms: ionized calcium (is the only physiologically active
form), albumin-bound calcium, and calcium bound to inorganic and organic anions.
Patients with hypoalbuminemia can have a low level of total plasma calcium; however,
they may not manifest with clinical hypocalcemia because their levels of ionized calcium
(the physiologically active form) have remained normal.
The serum albumin level should always be measured simultaneously with the serum
calcium level in order to calculate the correct total serum calcium value.
With every 1 g/dL change in serum albumin, serum calcium changes by 0.8 mg/dL.

An increased extracellular pH level (e.g., respiratory alkalosis secondary to


hyperventilation) an increase in the affinity of serum albumin to calcium increasing
the levels of albumin-bound calcium decreasing the levels of ionized calcium clinical
manifestations of hypocalcemia (e.g., crampy pain, paresthesias and carpopedal spasm).

Hypocalcemia can occur during or immediately after the surgery, especially in patients
undergoing major surgery and requiring extensive transfusions. Hyperactive deep tendon
reflexes may be the initial manifestations. Hypermagnesemia, on the other hand, results in
loss of deep tendon reflexes.

65
DIABETES MELLITUS (DM)
An increased risk for developing type 2 DM is associated with the following: Overweight
(BMI over 25 Kg/sqmet), Family history of type 2 DM, African- Americans, Previous
impaired glucose tolerance, History of gestational diabetes mellitus).

Fasting blood glucose measurement is now the recommended test for screening diabetes
mellitus (DM). If fasting blood glucose level over 126 mg/dL patient's fasting plasma
glucose level should be measured again before the diagnosis of DM can be established
Repeat fasting blood glucose measurement, if the measured value is still elevated, the
diagnosis is confirmed, and treatment is started. If fasting blood glucose level over 100-125
mg/dL impaired fasting blood glucose or pre-diabetes.

Other diagnostic tests for DM include:


1) A random blood glucose level of 200 mg/dL or greater in a symptomatic patient is
diagnostic for DM.
2) 75 g oral glucose tolerance test: A two hour value greater than or equal to 200 mg/dL is
diagnostic.

Oral hypoglycemic agents are used for the treatment of type 2 DM when lifestyle
modification and dietary modification fail to maintain euglycemia. Insulin therapy is the
mainstay of treatment in type 2 DM when lifestyle modification and oral hypoglycemic
agents fail. However, such treatments are better justified once the diagnosis of type 2 DM
has been firmly established.
Most patients who are initially controlled with one antidiabetic medication eventually
require the addition of more antidiabetic drugs to achieve optimal glycemic control.
Combining antidiabetic agents with different mechanisms of action is typically done to
achieve better glycemic control.
Metformin (a biguanide) is the only antidiabetic drug that causes some weight loss. It is
very important to assess the patient's renal status before starting metformin, since this drug
can produce lactic acidosis in patients with compromised renal status.
Thiazolidinedione (TZDs) drug (pioglitazone or rosiglitazone), Sulfonylurea, and insulin
lead to weight gain with long-term use.

66
Measurement of glycosylated hemoglobin (HbA1c) is an excellent way to monitor chronic
glycemic control. It is reflective of the patient's average glucose levels over the preceding
100-120 days (which correlates with RBC survival time). Generally, every 1% increase in
HbA1c corresponds with a 35 mg/dL increase in the mean plasma glucose level. The goal of
therapy is to have HbA1c < 7%.

Neuropathy is seen in approximately 50% of patients with long-standing DM. Patients may
manifest with distal symmetrical sensorimotor polyneuropathy, proximal motor neuropathy,
mononeuropathy and/or autonomic neuropathy. Risk of diabetic neuropathy increases
significantly in patients with risk factors for atherosclerosis (i.e., dyslipidemia,
hypertension, smoking, obesity).

Symmetric distal sensorimotor polyneuropathy is the most common type of diabetic


neuropathy, and is characterized by the classic "stocking glove" pattern of sensory loss.
Aside from symmetric distal sensorimotor polyneuropathy, diabetes can also cause
mononeuropathies of cranial and peripheral nerves.

Diabetic autonomic neuropathy involves the cardiovascular (postural hypotension,


abnormal sweating, syncope), genitourinary (cystopathy, erectile dysfunction and female
sexual dysfunction [reduced sexual desire, dyspareunia, reduced vaginal lubrication]) and
gastrointestinal (gastroparesis, enteropathy).

Long-standing DM autonomic neuropathy weak detrusor contraction cystopathy.


Diabetic cystopathy begins with weak urinary stream, dribbling, hesitancy, frequency, and
sense a full bladder and failure to void completely. The diagnosis can be made with
cystometry and urodynamic studies. The initial management involves strict voluntary
urinary voiding schedule coupled with oral bethanechol. If there is no response,
intermittent catheterization is recommended. Internal sphincter resection at the bladder
neck is used in extreme cases of diabetic cystopathy.

Mononeuropathies may be cranial (most commonly involving CN 3, 4 and 6) or peripheral


(most commonly involving the radial, peroneal and median nerves). The etiology of
mononeuropathy is mainly vascular, and recovery is usually seen in a few months time. For
unknown reasons, patients with diabetes are predisposed to pressure palsies.

67
Diabetic neuropathy pain is often a difficult condition to treat. The current treatment options
include tricyclic antidepressants (TCAs such as), gabapentin or NSAIDs. Agents such as
mexiletine, phenytoin, topiramate, carbamazepine and topical capsaicin cream have also
been used. Referral to a pain management center may be necessary in some cases.

Tricyclic antidepressants (e.g. amitriptyline, desipramine, nortriptyline) are the drugs of


choice for diabetic neuropathy. TCAs can worsen urinary symptoms (due to cystopathy)
and orthostatic hypotension (due to cardiovascular autonomic neuropathy). Gabapentin is
an alternative for these patients.

Spot urine collection and timed urine collection for the measurement of urine
microalbumin/creatinine ratio is accepted as a good screening test for microalbuminuria
in diabetic patients.
24-hour urine collection is the most accurate screening test for microalbuminuria in
diabetic patients. But, its inconvenience to patients makes it less preferred by physicians.

Glomerular hyperfiltration is the earliest renal abnormality seen in diabetic nephropathy. It


can be detected as early as several days after the diagnosis of diabetes was made. It is also
the major pathophysiologic mechanism of glomerular injury in these patients. Thickening
of the glomerular basement membrane is the first change that can be quantitated.

Diabetic nephropathy is the leading cause of end stage renal disease in the United States.
Increased extracellular matrix, basement membrane thickening, mesangial expansion,
and fibrosis characterize diabetes mellitus nephropathy. The sequence of pathological
changes in the kidneys of a patient with diabetes mellitus is as follows:
1) Within the first year of diabetes mellitus - Glomerular hyperperfusion and renal
hypertrophy with increase in glomerular filtration rate.
2) First five years of diabetes mellitus - Glomerular basement membrane thickening,
glomerular hypertrophy, and mesangial volume expansion with glomerular filtration rate
returning to normal.
3) Within 5-10 years of diabetes mellitus - Microalbuminuria, which later progresses to
overt nephropathy (macroproteinuria > 300 mg/day).

68
You should remember that effectiveness of ACE inhibitors in diabetic nephropathy is
related to their ability to reduce intraglomerular hypertension and, thereby, decrease
glomerular damage.

Tight blood pressure control delay development of cardiovascular diseases & renal failure
ACE inhibitors (usually given with hydrochlorothiazide) are the drug of choice in diabetic
patients for lowering blood pressure to target levels (< 130/80 mmHg).
Indications of ACE inhibitors: 1- Blood pressure higher than target value 130/80 mmHg.
2- Diabetic nephropathy (microalbumin/creatinine ratio > 30)
Intensive blood pressure control (< 130/80 mmHg) is the only intervention which has been
conclusively shown to reduce decline in GFR once azotemia (i.e. elevated creatinine)
develops.

Even though ACE inhibitors/ARBs retard the progression of diabetic nephropathy, they are
strictly contraindicated in pregnant women. Labetalol (an alpha and beta blocker) is a
perfect substitute for enalapril in pregnant women with diabetic nephropathy. The goal is to
bring the protein excretion to less than 500 to 1000 mg/day and blood pressure to less than
130/80 mmHg.
Although hydralazine is indicated to control HTN in pregnancy, it is ineffective as
monotherapy and does not prevent proteinuria or diabetic nephropathy. It can, however, be
used to treat HTN in combination with methyl-dopa.

Risk factors for the development of diabetic foot ulcers include diabetic neuropathy,
peripheral vascular disease, poor glycemic control, bony abnormalities of the foot, male sex,
smoking, chronic diabetes (>10 years), and a history of previous ulcer or amputation.
Neuropathy is found in approximately 80% of diabetics with foot ulcers.

Peripheral vascular disease (PVD) is a risk factor for diabetic ulcers. The poor blood
supply prevents adequate healing process. Ulcers of arterial insufficiency usually occur on
the tips of the fingers, rather than at the pressure points. The presence of non-healing
ulcers with poor pulsations in the lower extremities is an indication for angiography.

69
Foot ulcers can be classified as follows:
Grade 0: High-risk foot without an ulcer.
Grade 1: Superficial ulcer with full skin thickness involvement, but no involvement of
underlying tissue.
Grade 2: Deep ulcer penetrating to ligament or muscle, but no bone involvement or
abscess formation.
Grade 3: Deep ulcer with cellulitis, abscess formation or osteomyelitis.
Grade 4: Localized gangrene
Grade 5: Extensive gangrene involving the whole foot.

Theses six interventions have been shown to be useful in the management of diabetic foot:
1) Off-loading
2) Debridement
3) Wound dressings
4) Antibiotics
5) Revascularization
6) Amputation

Proper wound care and debridement are key steps in the management of diabetic ulcers.
Antibiotics alone do not cure diabetic ulcers.
The key steps in the management of grade 1 and 2 diabetic ulcers are proper wound care
and debridement. Debridement and removal of all the infected and necrotic tissue is
essential in such patients.
Patients with grade 3 ulcers require a short period of hospitalization, surgical debridement,
culture of material obtained from deep in the ulcer, bone biopsy, and intravenous
antibiotic therapy.
Patients with grade 4 and 5 ulcers need urgent hospitalization and surgical consultation
for possible amputation.

Diabetic foot infection can be classified as follows:


1) Non-limb-threatening infections are superficial, lack systemic toxicity, and have minimal
cellulitis extending less than 2 cm from the portal of entry. Ulceration, if present, does not
fully extend through the skin, lacking significant ischemia. S. aureus is the major

70
pathogen followed by facultative streptococci; however, facultative gram-negative bacilli
and anaerobes are not uncommon.

2) Limb-threatening infections have more extensive cellulitis, lymphangitis, ulcers


penetrating through the skin into subcutaneous tissue, and prominent ischemia. Such
infections are commonly polymicrobial with S. aureus, group B streptococci,
Enterococcus and facultative gram-negative bacilli being major pathogens along with
anaerobic gram-positive cocci and Bacteroides species.
Intravenous cefotetan, ampicillin/sulbactam, or the combination of clindamycin and a
fluoroquinolone is the appropriate empirical treatment for limb-threatening infections in
diabetics, whereas mild, or non-limb-threatening, infections can be treated with oral
antibiotics like cephalosporin, clindamycin, amoxicillin/clavulanate and
fluoroquinolones.
Osteomyelitis in diabetic patients that involves the bone adjacent to the foot ulcers is
explained by the contiguous spread of infection.

Erectile impotence in diabetic patients may be due to multiple reasons, including


autonomic neuropathy, medications, functional hypogonadism, and problems with penile
circulation.
Phosphodiesterase inhibitors (e.g., sildenafil) are the first-line drugs in the treatment of
diabetic patients with erectile dysfunction. Remember the following when treatment with
phosphodiesterase inhibitors is being considered:
1) Sildenafil is contraindicated in patients being treated with nitrates, and in those who are
hypersensitive to sildenafil.
2) Sildenafil is used with precaution in conditions predisposing to priapism.
3) Concurrent use of drugs which interfere with the metabolism of sildenafil (e.g.,
erythromycin, cimetidine) may predispose to adverse reactions by prolonging its plasma
half life.
4) While combining with an alpha-blocker (e.g. doxazosin), it is important to give the drugs
with at least a 4-hour interval to reduce the risk of hypotension.

71
HYPOGLYCEMIA
In normal individuals, a blood glucose level below 60 mg/dL results in near-complete
suppression of insulin secretion.
There are two important causes of hypoglycemia in non-diabetic patients with elevated
insulin levels:
1) Insulinoma (beta cell tumor)
2) Surreptitious use of insulin or sulfonylurea

Elevated C-peptide levels and proinsulin levels are seen in patient with beta cell tumors
and sulfonylurea-induced hypoglycemia.

Sulfonylurea causes an increased output of endogenous insulin from the beta cells. The
diagnosis is confirmed by measuring the plasma sulfonylurea level.

Exogenous insulin-induced hypoglycemia is associated with very high serum insulin levels
combined with low c-peptide levels. The low c-peptide levels result from the suppression of
endogenous insulin production.

The liver maintains glucose levels in the blood via either glycogenolysis or
gluconeogenesis. In fasting, glycogen reserves drop dramatically in the first 12 hours,
during which gluconeogenesis starts to play an important role. After 24 hours, it represents
the sole source of glucose. The main substrates for gluconeogenesis are: Alanine, lactate
and glycerol 3-phosphate. Alanine is the major gluconeogenic amino acid, and it is
converted into pyruvate in the liver by alanine aminotransferase (ALT). Pyruvate is
eventually converted to glucose though a series of reactions in the liver, and glucose is then
released into the bloodstream.

72
METABOLIC SYNDROME
Insulin resistance plays a central role in its pathogenesis. Metabolic syndrome is diagnosed
when at least 3 of the following criteria are met:
1) Abdominal obesity (Men: waist circumference > 40 inches, Women > 35 inches).
2) Blood pressure > 130/80 mmHg.
3) Fasting blood glucose > 100110mg/dl.
4) Triglycerides > 150 mg/dl.
5) HDL cholesterol (Men < 40 mg/dl; Women < 50 mg/dl).

Insulin resistance, typical for patients with central obesity is the key pathogenetic factor in
the development of type-2 diabetes mellitus and associated abnormalities (hypertension,
Hyperlipidemia).

NON-KETOTIC HYPEROSMOLAR SYNDROME (NKHS)


It occurs almost exclusively in type 2 diabetes, because insulin in these patients is sufficient
to prevent ketosis, but not hyperglycemia.
NKHS is characterized by hyperglycemia (often > 600 mg/dl), plasma hyperosmolality >
320 mOsmol/L, dehydration and NO KETOSIS.
Clinical features include: weakness, polyuria, polydipsia, lethargy, confusion & coma.
Focal neurological deficits are common with NKHS.
Severe hyperosmolality is responsible for weakness, lethargy, confusion, altered mental
status, neurological focal deficits & eventual coma.

Alcoholic ketoacidosis = ketoacidosis + high anion gap + near normal blood glucose level

73
DIABETIC KETOACIDOSIS (DKA)
For making a diagnosis of DKA, three things are necessary: blood glucose level >250
mg/dL, pH < 7.3 or low serum bicarbonate (< 15 - 20 mmol/L), and detection of plasma
ketones.
Metabolic acidosis in DKA is typically associated with hyperkalemia. It is called
paradoxical hyperkalemia because body potassium reserves are actually deleted ( total
body potassium) due to increased gastrointestinal loss and osmotic diuresis.
The mechanism of hyperkalemia associated with metabolic acidosis in DKA:
1) Extracellular shift of potassium in exchange with resultant intracellular potassium
deficit.
2) Impaired insuli-dependant cell entry of the potassium ion.

Essential measures in the management of DKA include the following:


1) Restoration of intravascular volume: using 0.9% saline (normal saline)
2) Correction of hyperglycemia: using intravenous regular insulin
3) Correction of electrolyte abnormalities and acidosis: Potassium correction is very crucial.
4) Treatment of precipitating factors such as infections: using antibiotics.

Bicarbonate administration is reserved for DKA patients with severe acidosis (pH less than
or equal to 7.1), plasma bicarbonate < 5 mEq/L, or severe hyperkalemia.
All hyperkalemic patients should start receiving potassium once serum potassium goes < 4.5
mEq/L.
In patients with normal or low serum potassium, potassium replacement should be started
with the initial fluid therapy.
Arterial pH or anion gap is the most reliable indicator of metabolic recovery in patients with
diabetic ketoacidosis.

THIAZIDE DIURETICS have some unfavorable metabolic side effects including


hyperglycemia (decreased tolerance to glucose), increased LDL cholesterol, and plasma
triglycerides. Electrolyte abnormalities that can be induced by thiazide diuretics include
hyponatremia, hypokalemia, and hypercalcemia. Risk of acute gout arthritis due to uric
acid retention induced by hydrochlorothiazide.

74
METABOLIC ALKALOSIS
This patient is having alkaline pH with increased bicarbonate suggesting metabolic
alkalosis. Next step would be to look for urinary chloride, which is less than 20 mg/dl
suggestive of chloride sensitive metabolic alkalosis.
Metabolic alkalosis can be classified into two broad categories based on urinary chloride
level, which are chloride sensitive metabolic alkalosis and chloride resistant metabolic
alkalosis:
1) Chloride sensitive metabolic alkalosis has urinary chloride < 20 meq/l. It can be corrected
with saline infusion. Some causes are thiazide or loop diuretics, surreptitious vomiting,
loss of gastric secretion, ingestion of large dose of nonabsorbable antacids, cystic fibrosis,
congenital chlorhydrorrheea, villous adenoma etc.
2) Chloride resistant metabolic alkalosis has urinary chloride >20 meq/l. They are generally
characterized by ECF expansion and HTN and thus are not corrected by saline infusion.
Some conditions are primary hyperaldosteronism, Cushing's syndrome etc. As well as
Bartter's syndrome and Gitelman's syndrome.

CUSHING SYNDROME
Cushing's syndrome presents with central obesity, wasting of the limbs, purple striae, easy
bruising and hypertension.

24-hr urinary free cortisol is a useful screening test for patients with suspected Cushing's
syndrome.
Low dose dexamethasone suppression test can also be used as a screening test.
Once the diagnosis has been established, the plasma ACTH level is measured to determine
the etiology of the syndrome.
Elevated plasma ACTH levels can occur in Cushing's disease and ectopic ACTH
production.
To distinguish between these two conditions, the high-dose dexamethasone suppression
test is used. Failure to suppress 24-hr urinary cortisol or serum cortisol levels by 50% of
the basal cortisol level makes ectopic ACTH syndrome more likely than Cushing'sdisease.

75
Cushing's syndrome due to ectopic ACTH production is characterized by the rapid
development of symptoms, hypokalemic alkalosis, pigmentation and hypertension. Serum
cortisol and 24-hour urine cortisol levels are not suppressed following the administration of
high dose dexamethasone (high dose dexamethasone suppression test) in most patients with
ectopic ACTH syndrome.

HYPERALDOSTERONISM
Normal function of aldosterone: reabsorb Na+ & excrete K+ and acid (H+).
When a patient presents with HTN and spontaneous always suspect:
1) Primary hyperaldosteronism
2) Renovascular disease
3) Bilateral adrenal hyperplasia
4) Renin secreting tumor
5) Cushing's syndrome

PRIMARY HYPERALDOSTERONISM
The most common cause is unilateral aldosterone-producing adenoma.
Aldosterone Na+ (hypernatremia), K+ (hypokalemia) and H+ (metabolic
alkalosis). Hypernatremia intravascular volume hypertension.
Suspect primary hyperaldosteronism in a young patient with hypertension, muscle
weakness and numbness.
Presence of hypokalemia and hypertension warrants investigations for secondary cause.
The most preferred screening test for hyperaldosteronism is to measure plasma aldosterone
concentration to plasma renin activity (PA:PRA). If the ratio > 30 ( PA : PRA)
primary hyperaldosteronism [high aldosterone/renin ratio is characteristic].
Aldosterone suppression test is used to confirm diagnosis of primary hyperaldosteronism;
give oral or IV NaCl then measure 24-hour urinary or plasma aldosterone levels. If urine
aldosterone > 14mg/24hours primary hyperaldosteronism
Once diagnosis of primary hyperaldosteronism is confirmed, CT scan of adrenal is done to
look for aldosterone producing tumor.

76
Adrenal adenomas are removed surgically.
Bilateral adrenal hyperplasia treated with spironolactone (blocks aldosterone).

In renovascular disease there is high PRA.

BARTTER'S SYNDROME
The differential diagnoses of normotensive patients with hypokalemia and metabolic
alkalosis (alkaline pH with increased bicarbonate) include:
1) Diuretic use
2) Surreptitious vomiting
3) Bartter's syndrome
4) Gitelman's syndrome

Classic Bartter's syndrome is like having furosemide-secreting tumor.


Bartter's syndrome is caused by defect in the thick ascending limb of the loop of Henle, in
which it losses NaCl polyuria hypovolemia activation of the renin-angiotensin
aldosterone system (RAAS) potassium and hydrogen ion secretion hypokalemia and
metabolic alkalosis (alkaline pH with increased bicarbonate).
Patients with Bartter's syndrome have hypokalemia, high urinary chloride levels,
metabolic alkalosis, and normal blood pressure.
Measurement of plasma aldosterone concentration to plasma renin activity (PA:PRA)
shows PA : PRA.

Patients with DIURETIC ABUSE may also present with hypokalemia, alkalosis and
normotension, urine chloride concentrations are high. If diuretic abuse is strongly suspected,
measurement of the urine diuretic level can be performed. Alkalosis can be corrected with
saline infusion.

77
SURREPTITIOUS VOMITING
Surreptitious vomiting chloride sensitive metabolic alkalosis.
Patients have low urine chloride concentration due to hypovolemia and hypochloremia.
Physical findings that are characteristic of surreptitious vomiting are scars/calluses on the
dorsum of the hands, and dental erosions.

CONGENITAL ADRENAL HYPERPLASIA (CAH)


Autosomal disorder deficiency of one of the 5 enzymes responsible for cortisol synthesis
in the adrenal gland plasma cortisol level stimulation of the pituitary gland
secretion of ACTH.
The most common cause of congenital adrenal hyperplasia is C-21 hydroxylase deficiency
accumulation of 17-hydroxyprogesterone (precursor of adrenal androgen)
hyperandrogenism.
Adolescent onset of hirsutism and virilism, particularly in the presence of normal
menstruations, along with elevated 17-hydroxyprogesterone, is diagnostic of congenital
adrenal hyperplasia.
Treatment is glucocorticoid (hydrocortisone) replacement.

McCune-Albright syndrome is a rare condition characterized by precocious puberty,


cafe au lait spots (large, have irregular borders and are not associated with axillary or
genital freckles) and multiple bone defects (polyostotic fibrous dysplasia). It is responsible
for 5% of the cases of female precocious puberty, and may be associated with other
endocrine disorders, such as hyperthyroidism, prolactin- or GH-secreting pituitary
adenomas, and adrenal hypercortisolism. McCune-Albright syndrome is sporadic and has
been recently attributed to a defect in the G-protein cAMP-kinase function in the affected
tissue, thereby resulting in autonomous activity of that tissue. Remember the 3 P's of
McCune-Albright syndrome: precocious puberty, pigmentation (cafe au lait spots) and
polyostotic fibrous dysplasia.

78
Idiopathic hirsutism is due to excessive conversion of testosterone to dihydrotestosterone
in the hair follicle. In such cases, there is usually a positive family history and occurs in
females of Mediterranean origin. Levels of 17-OH progesterone will be normal in such
cases and there will be no evidence of virilization.

ANDROGEN-SECRETING NEOPLASM
Rapidly developing hyperandrogenism symptoms with virilization (excessive muscular
development and clitoral enlargement) is highly suggestive of an androgen-secreting
neoplasm of the ovary or adrenal.
The best screening test for virilizing neoplasm is the measurement of serum testosterone
and DHEAS levels.
DHEA is secreted both from the ovaries and adrenals.
DHEAS, a sulfated form of DHEA, is specifically secreted from the adrenals.
Elevated testosterone levels with normal DHEAS levels indicate an ovarian source.
Elevated DHEAS levels with relatively normal testosterone levels indicate an adrenal
source.
Serum LH and FSH levels are likely to be suppressed due to excessive androgen levels.

PRIMARY ADRENOCORTICAL DEFICIENCY (ADDISONS DISEASE)


Addison's disease is caused by destruction of the adrenal cortex.
Destruction of the adrenal cortex production of cortisol, aldosterone & androgen
ACTH level.
Patients present with hypotension, hyponatremia, hyperkalemia, weakness
(mineralocorticoid deficiency), paresthesias, weakness, fatigue, depression, irritability,
hypotension, lymphocytosis, eosinophilia (glucocorticoid deficiency) and
hyperpigmentation of the skin and mucous membranes (characteristic of primary
adrenocortical deficiency, and is due to the increased levels of ACTH. This clinical feature
is not seen in patients with secondary adrenal insufficiency, which is due to hypothalamo-
pituitary failure).

79
In developed countries, the most common cause of primary adrenal cortical insufficiency is
autoimmune adrenalitis. In autoimmune adrenalitis, patients usually have other
autoimmune diseases in other endocrine glands such as thyroid, parathyroid and ovaries. In
developing countries, the prominent cause of primary adrenal insufficiency continues to be
adrenal tuberculosis (calcifications in the adrenal glands). Treatment of tuberculosis
usually does not generally result in normalization of adrenal functions. Most patients require
lifelong replacement with glucocorticoids and mineralocorticoids.
Other important causes include infections (viral, fungal), surgical removal, adrenal
hemorrhage and metastasis.
The best screening test for patients with suspected primary adrenal insufficiency is the
cosyntropin (analogue of ACTH) stimulation test. An increase in serum cortisol levels
above 20 mcg/dL 30-60 min after the administration of 250 mcg of cosyntropin virtually
rules out primary adrenocortical insufficiency (Addison's disease).
Measurement of plasma ACTH levels is performed to distinguish between primary and
secondary adrenocortical deficiency once the diagnosis of Addison's disease has been made.
Plasma ACTH levels greater than 50 pg/mL indicate primary adrenocortical deficiency,
whereas levels less than 50 pg/mL indicate secondary adrenocortical deficiency.
Hydrocortisone is used for the treatment of Addison's disease. It is usually started once the
diagnosis has been confirmed using the above mentioned biochemical tests; however, in
very sick patients with a highly suggestive clinical presentation of adrenocortical
insufficiency, treatment with a glucocorticoid may be started without waiting for the
laboratory results.

CENTRAL (TERTIARY) ADRENAL INSUFFICIENCY


Chronic supraphysiological doses of glucocorticoids suppression of the hypothalamic
pituitary (HP) adrenal axis suppress corticotropin-releasing hormone (CRH) secretion
from the hypothalamus blocking ACTH release from the anterior pituitary decrease
secretion of cortisol and androgen from the adrenal gland [central adrenal insufficiency].
Central adrenal insufficiency is characterized by low ACTH and cortisol levels.
Aldosterone secretion from the adrenal gland is independent of CRH, and is thus
aldosterone levels remain unchanged in patients with central adrenal insufficiency.

80
PHEOCHROMOCYTOMA
Pheochromocytoma is due to catecholamine-secreting tumors of the adrenals.
Paroxysms of very high blood pressure, headache, palpitations, abdominal pain, tremor,
and excessive sweating is classic, and suggests sympathetic hyperactivity induced by
catecholamine release.
Elevated urinary catecholamines (vanillylmandelic acid) are diagnostic.
Although initial treatment with a beta-blocker to counteract the sympathetic overactivity
seems reasonable, this approach is actually very dangerous. Blocking beta-receptors leads to
unopposed stimulation of vascular alpha-receptors by circulating catecholamines, which
may result in a rapid, catastrophic increase in blood pressure. For these reasons, Always
give an alpha-blocker first, followed by a beta-blocker; doing this in the wrong order can
precipitate a very dangerous increase in blood pressure.
Some agents have both alpha- and beta-blocking activity (e.g., labetalol); these can be used
safely as the initial treatment.

MEN 2A MEN 2 B
Medullary thyroid cancer Medullary thyroid cancer
Pheochromocytoma Pheochromocytoma
Primary parathyroidism hyperplasia Mucosal neuromas (tongue, lips, eyelids, GIT)
Marfnoid habitus

Normal levels of calcium and PTH rules out MEN 2A.

MEN IIa is an autosomal disorder resulting from germline mutation in the RET proto-
oncogene localized on chromosome 10. In patients with suspected MEN IIa syndrome,
DNA (genetic) testing has replaced biochemical measurement of serum calcitonin as the
recommended screening test. If genetic analysis is positive for a RET proto-oncogene
mutation, total thyroidectomy is indicated.

81
HYPOGONADISM
Primary (testicular) hypogonadism is characterized by elevated serum gonadotrophin
(LH and FSH) levels. Supranormal serum FSH and LH concentrations stimulate testicular
aromatase activity, thereby leading to increased estradiol production.

Functional hypogonadism is characterized by low testosterone and low gonadotrophin


(LH and FSH) levels in the presence of a significant systemic illness (e.g., uncontrolled
diabetes); the underlying pathology is defective gonadotrophin-releasing hormone
(GnRH) secretion.

Secondary (central) hypogonadism, which is characterized by hypogonadism, low


testosterone levels and inappropriately normal gonadotrophin (LH and FSH) levels.
Measurement of serum prolactin levels is the most important biochemical test to perform
in patients with suspected central hypogonadism. Regardless of the cause, high serum
prolactin levels inhibit the release of GnRH, thereby resulting in hypogonadism. Prolactin-
secreting pituitary tumor is one of the most important causes of elevated prolactin levels.

TESTICULAR CANCERS
AFP, beta-hCG, and PLAP are tumor markers useful in diagnosis, staging, and monitoring
patients with testicular cancers.

Leydig cell tumors are the most common type of testicular sex cord stromal tumors
principal source of testosterone and increased aromatase expression increasing estrogen
production secondary inhibition of LH and FSH levels. The most common endocrine
manifestation is gynecomastia, however in prepubertal cases, precocious puberty is
common.

Seminoma: PLAP

Embryonal cell carcinoma: AFP + beta-hCG (50%)

Choriocarcinoma: beta-HCG

82
ERECTILE DYSFUNCTION (ED)
A history of nocturnal or early morning erections should be asked routinely at first in any
patient with ED.
Penile erections normally occur during REM sleep and on waking up. To achieve an
erection, intact nerves and blood supplies are essential. Failure to achieve a spontaneous
erection during the night and/or early morning is pathognomic of organic erectile
dysfunction (ED).

Nocturnal penile tumescence helps to differentiate psychogenic from organic causes of ED.
It is positive in psychogenic causes & negative in organic causes.

A pelvic fracture with urethral injury is commonly accompanied by ED. The causes of ED
in this case are nerve injury (injury of the parasympathetic nerve fibers)and altered arterial
supply.
Venogenic ED may develop after disruption of the tunica albuginea (e.g., penile fracture).
Endocrinologic causes of ED include hyperprolactinemia and testosterone deficiency.
Situational ED is a variant of psychogenic ED, occurring only during certain situations
which cause anxiety. Nighttime and morning erections are preserved.

Intermittent bloody discharge from one nipple is the classic presentation of intraductal
papilloma. It is a benign tumor of major lactiferous ducts, that is most common in
perimenopausal women. Mostly found beneath the areola and difficult to be palpated during
physical examination due to its small size (not > 2 mm) and soft. Mammogram does not
show papilloma, as they're too small. Galactogram guided resection is the treatment of
choice for intraductal papilloma which presents with serous or bloody nipple discharge.
Baseline mammogram at age 35 is recommended for any woman who has an increased risk
for breast cancer.
Mammograms in women less than 35 years of age are not useful since the dense breast
tissue at this age does not allow adequate visualization of any masses, if present.
Fibroadenoma is a painless, firm and mobile solitary breast lesion, its size 2 cm. It occurs
most often in women 1525 years. The condition is benign. Fibroadenomas do not change
with the menstrual cycle.

83
Fibrocystic disease: it is benign condition. Very common in pre-menopausal females.
Patients present with bilateral painful, rubbery, firm, mobile masses; who experiences more
tenderness during her menses. Fibrocystic disease is treated with aspiration of the cyst,
which should yield clear fluid and result in the disappearance of the mass. Afterwards,
patients are typically observed for 4 to 6 weeks.

Tamsulosin is an alpha-1 blocker that is used for benign prostatic hypertrophy.

84
RHEUMATOID ARTHRITIS (RA)
Immunologic tissue injury occurs in rheumatoid arthritis.
Rheumatoid arthritis is an erosive arthritis involves the MCP and PIP joints.
When RA affects the axial skeleton, the cervical spine is the most commonly affected,
resulting in C1-C2 instability sublaxial sublaxation, resulting in neck pain, stiffness and
hyperreflexia.
Joint disease is symmetrical and non-migratory.
Joint stiffness lasts for > 30 minutes and affected joint show signs of inflammation.
The symptoms should last at least 6 weeks.
Splenomegaly is a common finding in patients with RA.
Narrowing of joint space and juxta articular bony erosions is the characteristic x-ray
finding in rheumatoid arthritis (RA).
Due to the autoimmune nature of the disease, systemic symptoms such as fatigue, fever,
malaise and anemia often occur.
ESR is always elevated.
Rheumatoid factor is present in most cases (80%) of rheumatoid arthritis.
Disease modifying anti-rheumatic drugs (DMARD) slow down the progression of bony
erosions and cartilage loss and therefore these agents are now, recommended to be used
earlier in the course of disease.
Erosive joint disease in rheumatoid arthritis is a clear-cut indication for the use of diseases
modifying anti-rheumatic drugs (DMARD) and methotrexate is the initial drug of choice
for this purpose.

Methotrexate is anti-metabolite agent, it acts by interfering with the cellular utilization of


folic acid & folic acid depletion; via inhibiting dihydrofolate reductase decrease folic
macrocytic anemia (low hematocrit and high MCV).
Side effects of methotrexate: macrocytic anemia, pancytopenia, stomatitis (painful oral
ulcers), hepatotoxicity (increase AST & ALT levels), interstitial lung disease and alopecia.
Side effects of methotrexate can be reduced by using concomitant folic acid supplements.
Recommendations for patients on methotrexate: CBC and liver enzymes every 3 months.

85
Etanercept and infliximab are the new generation TNF inhibitors, and highly effective in
patients who have refractory disease with methotrexate. They are very expensive and are not
indicated as first line of treatment.

Baker's cyst is a complication of rheumatoid arthritis (it is extension of the inflamed


synoviam into the popliteal space, it may rupture and leads to swollen painful calf).
Patients with RA are at increased risk of developing osteopenia and osteoprosis.
Long standing RA is the most common cause of systemic amyloidosis.
Patients with rheumatoid arthritis and those who have joint damage are at increased risk of
developing septic arthritis, particularly with Staphylococcus aureus. Inflammatory
monoarthritis presents acutely with a red, swollen joint and motion limited by pain; the
differential diagnosis includes septic arthritis, crystal-induced arthritis and trauma.
Treatment with IV antibiotics should be initiated promptly for septic arthritis to lessen the
likelihood of joint destruction.

ADULT STILL'S DISEASE


Adult still's disease which is a variant of rheumatoid arthritis.
Patient presents with high spiking fevers and characteristic evanescent salmon colored
maculopapular or macular rash that involves trunk and extremities. Rash typically
develops along with fever. Temperature may show variation up to 4C.
Other important findings include arthritis or arthralgias and significant leukocytosis.
To make the diagnosis of adult still's disease, conditions like systemic lupus erythematosus
(SLE), rheumatoid arthritis, malignancy, infectious mononucleosis and Parvovirus infection
should not be present.
Rheumatoid factor and ANA are usually negative.

Young patients with high spiking fevers associated with characteristic salmon colored
evanescent rash, arthralgias, and leukocytosis most likely have adult stills disease.

86
JUVENILE RHEUMATOID ARTHRITIS (STILL'S DISEASE)
Systemic features such as high-grade fever, fleeting maculopapular rashes,
hepatosplenomegaly, lymphadenopathy, pleuropericarditis and myocarditis are
characteristic.
The mainstays of treatment are NSAIDs and monitoring of the liver enzymes.
Corticosteroids are used if the patient does not respond to NSAIDs, or in the presence of
myocarditis or anterior uveitis.

SYSTEMIC LUPUS ERYTHEMATOSUS (SLE)


The diagnosis of Systemic lupus erythematosus (SLE) is established when four or more of
the diagnostic criteria of SLE are met. These criteria include DOPAMINE RASH:
Discoid rash, Oral ulcers, Photosensitivity, Arthritis (migratory and asymmetrical), Malar
rash, Immunological abnormality, Neurological abnormality (lupus cerebritis, seizures),
ESR elevated, Renal abnormality, Antinuclear antibodies, Serositis, and Hematological
abnormality.
More than 90% of patients have arthritis most commonly affecting MCP and PIP joints.
Arthritis in SLE is non-deforming (NO permanent joint deformities.
There is decrease in serum C3 level.
Anti-DsDNA antibodies are highly specific for and confirms the diagnosis of SLE.
Anti-Smith antibodies are highly specific for systemic lupus erythematosus but are found in
only 30-50% of SLE cases.

Renal involvement in SLE is due to immune complex mediated glomerular injury. These
immune complexes are primarily composed of anti-DsDNA antibodies.
Diffuse proliferative glomerulonephritis is the severe form of glomerulonephritis seen in
patients with systemic lupus erythematosus.
Kidney biopsy is indicated to guide the treatment in all systemic lupus erythematosus
patients with renal involvement.

87
Hydroxychloroquine (DMRAD) is the safest drug for systemic lupus erythematosus (SLE).
The most common side effects of hydroxychloroquine are allergic skin reactions and nausea
but they are not serious ones. However, rarely, it may cause serious eye disease including
macular degeneration; therefore eye examinations at 6 months to 1 year intervals should be
performed in all patients who are taking hydroxychloroquine.
Serial measurement of anti-dsDNA and C3 levels is used to monitor response to treatment.

Prognosis of SLE depends on renal involvement (chronic renal failure).


Chronic renal failure is the most common cause of death in patients with SLE.

SCLERODERMA
Scleroderma is a disorder characterized by diffuse fibrosis of skin and internal organs.
Raynaud' s phenomena are seen in most patients with scleroderma.
Scleroderma of the esophagus: characteristic symptoms (i.e., "sticking sensation in the
throat" or dysphagia accompanied by heartburn) and manometric findings (i.e., absence of
peristaltic waves in the lower two-thirds of the esophagus, and significant decrease in the
lower esophageal sphincter tone).
Scleroderma patients have positive ANA and Anti-Scl-70 antibodies.
Scleroderma is associated with anti-centromere antibodies in up to 90% of patients.
There is no available treatment for scleroderma.
Steroids for acute flares; penicillamine can be used for skin changes.
Calcium channel blockers for Raynauds.
ACEIs for renal disease and malignant hypertension.

88
SYSTEMIC SCLEROSIS
Systemic sclerosis is a form of scleroderma with widespread organ involvement.
Pathogenesis: connective tissue thickening, production of myofibroblast proliferation and
increase collagen and ground substance production.
Thickening of the skin begins in acral sites (hands and feet) with edema that transitions to
dermal sclerosis with obliteration of skin appendages (hair follicles and sweat glands) and
flexion contractures.
The face may take on a mask-like appearance due to dermal thickening.
Calcinosis cutis and Raynaud' s phenomena are also commonly observed.
Involvement of the kidneys produces hypertension (treated with ACEIs).
Pulmonary arterial hypertension ultimately results in right heart failure.
Esophageal and gastric dysmotility cause these patients to suffer from GERD.
Antinuclear antibodies and anti-topoisomerase-I antibodies would most likely present.

CREST SYNDROME
CREST syndrome is a localized form of scleroderma.
CREST is an acronym for the cardinal features of this syndrome Calcinosis, Raynaud' s
phenomena, Esophageal dysfunction, Sclerodactyly, and Telangiectasias.
In CREST syndrome hardening of the skin is limited to the face and hands whereas in the
diffuse form of scleroderma it extends to involve the trunk and proximal extremities.
Calcinosis presents as nodular swellings, which may be symptomatic or asymptomatic. If
symptomatic it presents with pain and tenderness, sometimes it may ulcerate and exude a
white chalky fluid. X-ray of the involved area shows calcifications.
Sclerodactyly: fibrosis of the skin of the fingers results in shinny appearance and thickening
of the skin of the fingers and flexion contractions and joint pain.
Telangiectasias is a mat like patches found on the face and palms.
Pulmonary arterial HTN (loud P2) may occur in 25-50% of patients.
Anti-centromeric antibodies are associated with CREST syndrome.

89
SJOGREN'S SYNDROME
It is an autoimmune disorder resulting in chronic dysfunction of the exocrine glands.
It is characterized by dry mouth (Xerostomia dental caries and difficulty speaking and
swallowing), dry eyes (keratoconjunctivitis, with burning, itching, and foreign body
sensation), nasal dryness, vaginal dryness, chronic bronchitis and reflux esophagitis.
Parotid enlargement may be chronic or relapsing.
Anti SS-A (Ro) and anti SS-B (La) are more frequently positive in patients with Sjogren' s
syndrome.
Sjogren' s patient with positive anti SS-A usually have extra glandular manifestations.
Lip biopsy is the most specific test for Sjogren' s syndrome. The diagnosis is confirmed
when the biopsy shows lymphoid foci in accessory salivary glands.

Sjogren's syndrome can cause RTA type 1 (distal tubular acidosis): which is characterized
by non-AG metabolic acidosis, urine pH greater than 5.5 and low serum potassium.
Non-Hodgkin lymphoma is a complication of Sjogren's syndrome.

BEHCET'S DISEASE
Behcet's disease is a rare multi-systemic inflammatory disease with an autoimmune etiology
This syndrome is more common in the Turkish, Asian and Middle Eastern population.
Diagnosis is based on the recurrence of the symptoms and exclusion of other conditions
such as Crohn's disease and syphilis.
The international criteria for diagnosis includes: Recurrent oral ulcers plus two of the
following features
Recurrent genital ulcers;
Eye lesions, including anterior uveitis and posterior uveitis;
Retinal vascularization;
Skin lesions including erythema nodosum (painful and nodular with areas of
hyperpigmentation suggested by the old, healed nodules), acneiform nodules and
papulopustular lesions;

90
Positive pathergy test.
Some patients have involvement of the gastrointestinal, skeletal and vascular systems.
Combined environmental and hereditary factors are assumed to play a role in the
pathophysiology of this condition.
Corticosteroids or cytotoxic drugs offer relief from most of the symptoms but do not protect
from progression to dementia or blindness.

ANKYLOSING SPONDYLITIS (AS)


Also called Apophyseal joint arthritis.
Back pain worse in the morning with stiffness which improves with exercise for more than
three months.
Anterior uveitis is the most frequent extra-articular manifestation of ankylosing spondylitis
and it presents with unilateral painful red eye, blurred vision, photophobia and presence of
pus in the anterior chamber on slit lamp eye examination.
Abnormal spine movements: reduced range of forward flexion of the lumber spine on
Schober testing and reduced chest expansion.
Enthesopathy (heel pain) is quite specific for spondyloarthropathy.
Fusion of the costovertebral joints resulting in chest wall motion restriction and restrictive
pattern on pulmonary function testing.
Rheumatoid factor levels are usually negative.
HLA-B27 is frequently (>90% of the patients) present in patients of ankylosing spondylitis
and other spondyloarthropathies.
Plain X-ray of the Sacroiliac joint is the next best step in a suspected patient of ankylosing
spondylitis, sacroiliitis is the earliest change to be seen.
Regular exercise/physiotherapy is the only beneficial treatment that halts the disease
progression in ankylosing spondylitis.

AS and IBD are both associated with HLA-B27 and may occur in association with each
other. Both conditions may also associated with positive p-ANCA despite the absence of
vasculitis in both conditions.

91
Patients suffering from ankylosing spondylitis for 20 years or more are at increased risk of
vertebral fractures due to decreased bone mineral density. Vertebral fractures in those
patients may occur with minimal trauma.

REACTIVE ARTHRITIS SSYCC


Reactive arthritis is the term used when arthritis develops after infection with Salmonella,
Shigella, Yersinia, Campylobacter and Chlamydia.
To make a diagnosis of reactive arthritis, time interval between gastrointestinal or
genitourinary infection and joint disease should be less than 6 weeks.
Rheumatoid factor levels are usually negative.
The treatment of choice for reactive arthritis or Reiter's syndrome is with NSAIDs.
Tetracycline may be added if the disease is due to genitourinary infection with Chlamydia
and this will result in early resolution of symptoms.
Patients who are refractory to the above treatment may benefit from sulfasalazine,
infliximab and methotrexate.

REITER'S SYNDROME
Chlamydia causes Reiter's syndrome characterized by urethritis, conjunctivitis,
mucocutaneous lesions, and arthritis.
Mucocutaneous lesions: Keratoderma blennorrhagicum (found on palms and soles) and
circinate balanitis (painless shallow ulcers present on the glans penis or urethral meatus).
Rheumatoid factor is usually negative in Reiter's syndrome.
HIV should be considered in patients with Reiter's syndrome.

92
PSORIATIC ARTHRITIS
Marginal bony erosions and irregular joint destruction.
Asymmetric oligoarticular arthritis, affecting two to three joints simultaneously.
DIP involvement and the presence of nail changes may be the only clue in these patients.
The proximal joints of the hands and feet are frequently affected.
Patients usually have typical features (silvery scales on erythematous plaques over flexural
surfaces) of psoriasis present for years.
Sausage-shaped digits may be present. Morning stiffness more than hour.
Think of psoriatic arthritis when pitting nails is present in the history.
Ibuprofen (which may be taken for psoriatic arthritis) increases lithium blood levels, thereby
increasing the chances for psoriasis exacerbation.
NSAIDs, anti-TNF agents and methotrexate are used for treatment of psoriatic arthritis;
systemic corticosteroids are relatively contraindicated.

ENTHESITIS ENTHESOPATHY
It is an inflammation and pain at the sites of where tendons and ligaments attach to bone.
It is most commonly seen in cases of recurrent tendon or ligament stress.
It is a common finding in HLA-B27 associated arthropathies such as ankylosing spondylitis,
psoriatic arthritis and reactive arthritis.
It is classically manifests with heel pain due to tenderness at the site of insertion of the
Achilles tendon.
The tibial tuberosities and the iliac crests are also common sites of symptomatic enthesitis.

93
OSTEOARTHRITIS WEAR AND TEAR ARTHRITIS
Osteoarthritis is non inflammatory disease.
Osteoarthritis characterized by progressive deterioration and loss of articular cartilage.
Osteoarthritis is the end result of any joint disorder. It is a degenerative joint disease.
Obesity is a major risk factor for osteoarthritis.
Osteoarthritis predominantly involves the DIP joints.
Bilateral knee joint pain with transient morning stiffness (lasting < 15 minutes) and
improved with rest, is characteristic symptom of osteoarthritis (OA).
Progressive joint pain in weight bearing joints, brief morning stiffness, and reduced range of
movement.
Osteoarthritis of the cervical spine usually presents with diffuse neck pain.
Affected joints are tender to touch, signs of inflammation are usually minimal or absent.
Crepitus is a popping sound occurs in OA as a of cartilage erosion & incongruous joint
surfaces.
There are 6 criteria used to establish the diagnosis of OA in the setting of painful knee: age
> 50, crepitus, bony enlargement, bony tenderness, and lack of warmth/inflammation, and
minimal or no morning stiffness. If three or more criteria are met, the specificity for OA is
69%.
X-ray reveals narrowing of joint space, osteophyte formation, subchondral sclerosis, and
subchondral cysts.
Heberden's nodes are seen at the DIP joints. Boucher's nodes are seen at the PIP joints.
Weight loss is considered as the best management in the patients with knee osteoarthritis.
Exercise program is meant to improve functional status of the patient.
Always acetaminophen is the initial drug of choice for the pain.
If acetaminophen fails to control pain NSAIDs, like indomethacin, are indicated.
Indomethacin is contraindicated in patients with renal failure and history of GI bleeding.

94
GOUT
Gout results from deposition of monosodium urate crystals into the joint space.
Sudden onset of pain and swelling in the metatarsophalangeal joint is suggestive of gout.
Gout can present with nodular swelling of the digits resulting in significant deforming
arthritis. Rheumatoid nodules predominantly occur over pressure points such as the elbow
and extensor surface of the proximal ulna. All of these patients will have positive
rheumatoid factor levels.
This is more common in patients with chronic renal disease, myeloproliferative disorders
(e.g. polycythemia vera), psoriasis, chronic alcoholism and using medications such as
thiazides (diuretics or water pills), niacin etc.
Induction of chemotherapy results in rapid tumor cell lysis and release uric acid in the
circulation. For prevention, allopurinol and probenecid are used.

You should never leave a tender inflamed joint without aspiration.

Joint fluid aspiration reveals negatively birefringent needle shaped urate crystals.
X-ray shows punched out erosion with a rim of cortical bone (in patients with chronic gouty
arthritis).
The goal of treatment is to maintain serum uric acid levels to less than 6 mg/dL.
Colchicine is used for the acute treatment of gout.
In patients with frequent attacks of acute gouty arthritis not controlled by colchicine, a 24-
hour uric acid levels in urine is determined. This evaluates whether hyperuricemia is due to
over production or under secretion of uric acid.
A value of < 800 mg/day suggests under secretion give Uricosuric agent e.g.
Probenecid.
A value of > 800 mg/day suggests over production give xanthine oxidase inhibitor e.g.
Allopurinol.

Cessation of alcohol and staying on low purine diet are important measures in the
prevention of future attacks in patients with gouty arthritis.

In chronic tophaceous gout, monosodium urate crystals deposit into the skin, resulting in
formation of tumors (tophi) with a chalky white appearance.

95
Myeloproliferative Disease (MPD) e.g. Polycythemia Vera (PCV) catabolism and
turnover of purines over-production of uric acid serum uric acid precipitation of
gouty arthritis.
Polycythemia Vera (PCV): A typical patient is an old plethoric male who may complain of
pruritus after hot bathing (due to histamine release from an increased number of
circulating basophils). Up to 40% of patients with PCV suffer from gout. Symptoms such as
headache, dizziness, and paresthesias are frequently related to hyperviscosity. They also
have granulocytosis and thrombocytosis on peripheral blood smear, and splenomegaly.
Bone marrow is virtually always hypercellular. There is an elevated leukocyte alkaline
phosphatase, normal oxygen saturation, and low erythropoetin level.

Lesch-Nyhan syndrome is characterized by a deficiency in hypoxanthine-guanine


phosphoribosyl transferase, an enzyme involved in purine metabolism. It is characterized
by self-mutilation, mental retardation, hyperuricemia and neurologic signs (e.g., dystonia,
choreoathetosis and cerebral palsy). It is generally diagnosed in childhood.

PSEUDO-GOUT
Hyperparathyroidism predisposes to the development of pseudo-gout.
In pseudo-gout, joint is acutely red and swollen.
Knee is the most frequent site of involvement.
Precipitating factors may include pre-existing joint damage (e.g. trauma, surgery).
Remember the 4 Hs: Hyperparathyroidism, Hemochromatosis, Hyperphosphatemia,
Hypomagnesaemia, (Hypothyroidism).
Joint fluid aspiration reveals positively rhomboid shaped calcium pyrophosphate dehydrate
(CPPD) crystals.
Radiographics show evidence of chondrocalcinosis (calcified articular cartilage).

96
SEPTIC ARTHRITIS
Staphylococcus aureus causes septic arthritis in previously damaged joints, prosthetic
joints, IV drug abusers & elderly.
The most common organisms in children are Staphylococcus aureus and streptococcus.
E. Coli is a common cause of septic arthritis in intravenous drug abusers.
Neisseria gonorrhoeae is the most common cause of septic arthritis in young sexually
active patients.
Septic arthritis is more likely to develop in disseminated gonococcemia when gonococcal
urethritis is untreated.
Joint will be red and swollen and patient will be febrile. There is usually evidence of
infection elsewhere like that of urinary tract, skin or respiratory tract.
In septic arthritis, synovial fluid WBC count is usually above 100,000/cmm and more than
75% of them are PMNs. Gram stain and culture are usually positive.
Urethral cultures have higher yield than synovial or blood culture in cases of suspected
gonococcal purulent arthritis
IV nafcillin is the treatment of choice for septic arthritis.
In infants a combination of nafcillin and third generation cephalosporin and nafcillin alone
for children over age 5 years is a reasonable empiric antibiotic therapy.
Septic joint in a child is a true surgical emergency and needs immediate surgical drainage.
A delay of even 4-6 hours can lead to avascular necrosis of the femoral head.

97
DISSEMINATED GONOCOCCAL INFECTION SYNDROME
Disseminated gonococcal infection is a syndrome of rash, tenosynovitis and polyarthralgia.
Mucosal gonococcal infection (if untreated) gonococcal bacteremia Disseminated
gonococcal infection. Women are at more likely to develop disseminated gonococcal
infection especially during menses and pregnancy.
Rash is pustular or vesiculopustular and is usually present on trunk and extensor surfaces of
the distal extremities and it may become hemorrhagic.
Tenosynovitis of multiple tendons at wrist, hands and ankles may occur.
Arthralgia is migratory and joint examination does not show redness, swelling or tenderness
Skin and joint manifestation are thought to be immune-mediated caused by immune-
complex formation from gonococcemia.
Blood cultures and cultures from the cutaneous pustules are typically negative due to the
growth requirements of the organism.

VERTEBRAL OSTEOMYELITIS
Vertebral osteomyelitis is usually hematogenous, and the lumbar area is most commonly
affected. Risk factors: IV drug abusers and immunosuppression states (e.g. uncontrolled
DM, patient with sickle cell anemia, HIV patient, patients use corticosteroids, etc.).
S. aureus is the most common pathogen in IV drug abusers, but infection with other gram
negative organisms (e.g. E. coli) is also common.
Back pain accompanied by low-grade fever (may not present) and elevated ESR (>
100mm/h) is the typical presentation. High-grade fever and chills are uncommon.
Physical examination may demonstrate that gentle percussion over the affected vertebrae
elicits pain and paravertebral muscular spasm.
Early diagnosis is very important in patients with this condition, because epidural abscess
and spinal cord compression may develop if the treatment is delayed.
WBCs count may be normal or elevated.
MRI is currently the imaging modality of choice in patients with suspected vertebral
osteomyelitis. Compared to a bone scan, a MRI has the same sensitivity for detecting
osteomyelitis and is less time-consuming.

98
VIRAL ARTHRITIS
Parovirus B19 infection is the most common cause of viral arthritis, and most probably in
adults who have frequent contact with children such as teachers and day-care workers.
Patient presents with arthritis/arthralgias involving the MCP, PIP and wrist with morning
stiffness less than 30 minutes. Slapped check appearance is unusual in adults.
Viral arthritis resolves with 2 months.
No joint swelling and no systemic symptoms (no increase in WBCs count or ESR).
Positive inflammatory markers such as ANA and rheumatoid factor may occur.
Anti-B19 IgM is the diagnostic study of choice when Parovirus infection is suspected.
Treatment involves use of NSAIDs for resolution of symptoms. Antiviral therapy is
unnecessary as the symptoms are self-limited.

WEGENER'S GRANULOMATOSIS
The triad of upper respiratory tract disease, lower respiratory tract disease, and
glomerulonephritis defines Wegener's granulomatosis.
Wegener's granulomatosis is vasculitis of the small vessels and necrotizing granulomatous
lesions affecting upper and lower respiratory tracts as well as causing focal necrotizing
glomerulonephritis.
Respiratory involvement presents as nasal ulcers, nasal congestion, rhinitis, sinusitis, cough,
hemoptysis, epistaxis, dyspnea, and chest discomfort.
These patients usually undergo several treatments for sinusitis and nasal congestion
unsuccessfully before Wegener's granulomatosis is suspected.
Patient may develop renal failure due to rapidly progressive glomerulonephritis.
Renal involvement may be evident by hematuria, proteinuria and cylindruria on routine
urine examination.
Chest x-ray reveals infiltrates, nodules, masses, and cavities.
The demonstration of anti neutrophil cytoplasmic antibodies (ANCA) confirms the
diagnosis of Wegener's granulomatosis.
Positive C-ANCA is highly specific for Wegener's granulomatosis.

99
Renal biopsy may show immune deposits with crescent formations.
Wegener's granulomatosis is treated with a combination of cyclophosphamide and steroids.

Long term use of cyclophosphamide is associated with increase incidence of acute


hemorrhagic cystitis and bladder carcinoma. Drinking plenty of fluids, voiding frequently
and taking MENSA are all helpful in preventing these complications.

CHURG STRAUSS SYNDROME (CSS)


CSS is a multisystem vasculitic disorder of unknown etiology that affects the skin, kidney,
nervous system, lungs, gastrointestinal tract and heart.
Leukotreine antagnoists e.g. zafirlukast are known to cause CSS.
It is characterized by allergic rhinitis, asthma and prominent peripheral blood
eosinophilia. Up to 75% of the patients with CSS have evidence of peripheral neuropathy.
P-ANCA is associated with Churg-Strauss syndrome.
The disease requires treatment with glucocorticoids and sometimes with
immunosuppressants e.g. cyclophosphamide.

GOODPASTURE'S SYNDROME
Goodpasture's syndrome involves the lungs and kidneys (Pulmonary-renal syndrome).
Goodpasture's syndrome is caused by circulating anti-glomerular basement membrane
antibodies.
Linear deposition of IgG on the basement membrane is seen in Goodpasture's syndrome.
Sputum stain may show iron in the form of hemosiderin.
Early removal of these antibodies by emergency plasmapheresis is imperative in order to
minimize the extent of kidney damage.

100
POLYARTERITIS NODOSA (PAN)
Polyarteritis nodosa involves medium sized arteries and most frequently organs include
kidney, liver, gastrointestinal tract, skin, heart and peripheral nerves with the exception of
the lungs.
It has been found to be associated with hepatitis B and C.
Labs shows elevated P-ANCA.
Patients with PAN typically present with clinical features of multisystem disease. These
features include fatigue, weakness, fever, arthralgias, hypertension, renal failure,
neurological abnormalities and abdominal pain.

TEMPORAL ARTERITIS GIANT CELL ARTERITIS


Giant cell arteritis should always be suspected in a patient older than 50 years who presents
with headache and pain in one or both temples, visual problems (decreased vision or
blurry vision) and polymyalgia rheumatica (proximal stiffness in neck, arms, hips).
Other common symptoms: scalp tenderness, and jaw claudication.
Giant cell arteritis chronic aortitis disruption of collagen and elastin Thoracic
aortic aneurysm (such patient need to have continuous monitoring).
Blood samples can be drawn for ESR before the start of glucocorticoid therapy.
Giant cell arteritis must be treated immediately with high dose glucocorticoids once the
clinical diagnosis is made.
One should not wait for temporal artery biopsy before starting glucocorticoid therapy as any
delay in the treatment may result in visual loss.
Temporal artery biopsy can be done within several days after treatment, for confirmation of
the disease.

101
POLYMYALGIA RHEUMATICA
Diagnosis of polymyalgia rheumatica is suggested in this patient because of the following
findings: age of patient greater than 50, morning stiffness for 30 minutes or more and
present for more than a month and it involves shoulder and hips.
Patients complain of stiffness rather than weakness or pain.
When the patient is asked to identify location of his pain; patient typically indicate that the
pain is in the soft tissue not the joints.
No signs of inflammation of the joints.
Polymyalgia rheumatica may be present alone or in 40-50% cases, it is associated with giant
cell arteritis.
It is a diagnosis of exclusion; ESR is greater than 40 mm/hour.
There is no need of temporal artery biopsy or high dose prednisone if there are no symptoms
or signs suggestive of temporal arteritis.
The treatment of choice for polymyalgia rheumatica when it is not associated with giant cell
arteritis is low-dose prednisolone.

INFLAMMATORY MYOPATHIES
DERMATOMYOSITIS
Autoimmunee disorders characterized by proximal muscle weakness, and ultimately
wasting. Facial or ocular muscles are never involved.
Dermatomyositis involve typical skin changes: Heliotrope dusky rash around the eyes
associated with periorbital edema, and Gottron's papules, which are red scaly patches over
the metacarpophalangeal joints.
Occult malignancies are often associated with dermatomyositis or Polymyositis such as:
breast, lung or colon cancer. So a careful metastatic workup should be done in this regard.
Suspected patients should have measurement of serum creatine kinase and aldolase levels.
Anti Jo-1 antibodies are positive in polymyositis and dermatomyositis.
Specific diagnosis is usually made by muscle biopsy.
Inflammatory myopathies are best treated with corticosteroids e.g. prednisone.

102
POLYMYOSITIS
Autoimmunee disorders characterized by proximal muscle weakness, and ultimately
wasting.
Polymyositis presents as difficulty ascending or descending stairs, combing hair, kneeling
down, difficulty in rising from a seated position etc.
In polymyositis there is no skin findings, which helps to distinguish it from
dermatomyositis.
Facial muscles, ocular muscles, and muscles of mastication are never involved.
However, dysphagia may be found due to the involvement of striated muscles of the upper
pharynx.
Occult malignancies are often associated with dermatomyositis or Polymyositis such as:
breast, lung or colon cancer. So a careful metastatic workup should be done in this regard.
Suspected patients should have measurement of serum creatine kinase and aldolase levels.
Anti Jo-1 antibodies are positive in polymyositis and dermatomyositis.
Muscle biopsy is the best diagnostic test and shows mononuclear infiltrates surround
necrotic & regenerating muscles fibers.
Inflammatory myopathies are best treated with corticosteroids e.g. prednisone.
In case the patient fails to improve on or cannot tolerate the side effects of corticosteroid
therapy, immunosuppressive agents such as methotrexate may be effective.

Hypothyroidism should always be considered in patients with an unexplained elevation of


serum CK concentration and myopathy (muscle weakness).

Hyperthyroid myopathy presents as progressive proximal muscle weakness.

103
MIXED CONNECTIVE TISSUE DISEASE
Mixed connective tissue disease represents the over lapping symptoms of systemic lupus
erythematosus (SLE), scleroderma and myositis.
It is associated with autoantibody to ribonuclear protein (Anti-RNP antibodies).
Raynaud' s phenomena in the form of finger ulcers, arthralgias, and esophageal
hypomotility, swollen hands, myositis, rash, leukopenia, and sclerodactyly are the common
initial presentations.

FIBROMYALGIA
The cause is unknown but may be due to sleep disorder, depression, and viral infections.
Fibromyalgia is a chronic widespread pain disorder associated with fatigue, poor sleep,
irritable bowel syndrome and depression for more than 3 months.
Fibromyalgia pain worsens with exercise.
Normal physical exam except excessive muscular tenderness in 9 or more "tender points".
Always consider thyroid abnormalities and fibromyalgia in a patient with diffuse muscle
aches and excessive fatigue with non-restorative sleep.
Inflammation, joint swelling and muscle weakness are absent.
Initial evaluation should include a Complete blood count (CBC), ESR, Thyroid function
tests (TFTs) and enzymes (CK).
Radiographic, laboratory, and histologic studies typically show no abnormalities.
Patents improve with tricyclic antidepressants e.g. amitriptyline and usually given in low
doses at nighttime (increase the amount of restorative phase 4 sleep).
To relieve the pain, non-narcotic analgesics like acetaminophen can given during daytime.
The only other drug other than tricyclic antidepressants studied extensively in fibromyalgia
is cyclobenzaprine.
So either amitriptyline or cyclobenzaprine are the initial drugs of choice.
Selective serotonin reuptake inhibitors (SSRIs) are added when disease is refractory to the
above medicine.
When patient feels better with the pharmacological treatment exercise program is initiated.

104
PAGET'S DISEASE OF THE BONE (OSTEITIS DEFORMANS)
The exact cause of the disorder is unclear, but several workers have shown evidence of viral
etiology in Paget's disease.
The single most common cause of asymptomatic isolated elevation of alkaline phosphatase
in an elderly patient is Paget's disease.
Pathophysiologic mechanism is impaired bone remodeling in focal areas of the bone.
Paget's disease of the bone is characterized by excessive bone destruction and repair.
Increase bone turnover, in which osteoclastic dysfunction (increased numbers of larger than
normal osteoclasts at the involved sites) progression of osteoclastic activity increase
bone breakdown activation of the osteoblasts immature bone deposition occurs
hypertrophy of bones "mosaic" pattern of lamellar bone.

Patients are often asymptomatic; however, those with symptoms usually first note an
increase in hat size (asymmetric enlargement of the cranium) and occasional headaches.
Cranial bone enlargement entrapment of cranial nerve VIII hearing loss.
Normal serum calcium, normal serum phosphate and normal serum parathyroid hormone
levels are seen in patients with osteoporosis and Paget's disease.

Markedly elevated serum alkaline phosphate level as a result of increase bone turnover and
high hydroxyproline level.
Radiologic and histologic evidence of increased bone turnover (resorption and formation)
can be readily assessed by measuring the biochemical markers of bone turnover:
1) Alkaline phosphatase - marker of bone formation; it is elevated in patients with Paget's
disease, and is most commonly used to assess the activity of disease, as well as response
to treatment.
2) Urinary n telopeptide - most commonly used to as a marker of bone resorption.

In evaluation of a patient of Pagets disease, the initial goal is to establish which bone have
been affected. Full extend of the disease is best ascertained by full body scan (which shows
increase uptake by the affected bones) followed by radiographic confirmation.

Asymptomatic patients of Pagets disease do not require treatment.


Symptomatic patients are best treated with oral or IV Bisphosphonates e.g. alendronate.

105
Indications of treatment of Pagets disease:
1) Bone pain.
2) Hypercalcemia of immobilization.
3) Neurological deficit.
4) High-output cardiac failure.
5) Preparation of orthopedic surgery.
6) Involvement of weight-bearing bones (to prevent deformity).

A small number of patients of Pagets disease will develop sarcomatous changes over 10
years and third will present as a new lytic lesion and a sudden increase in alkaline
phosphatase.

In osteoporosis, there is osteoblastic dysfunction.

"BLUE TOE SYNDROME"


The problem can occur spontaneously, and may occur secondary to surgical manipulations.
Cardiac catheterization can cause atheromatous embolism (dislodged plaques) from the
aortic root may embolize to the cerebral, gastrointestinal, renal and cutaneous circulations
localized ischemia and organ dysfunction (e.g. elevation of creatinine and blue toe synd.)
Blue toe syndrome results from emboli to small pedal vessels and characterized by intact
pulses, painful cyanotic toe along with features of ischemic lesions in distal limbs, such as
livedo reticularis (mottled discoloration of the extremities).
Labs are significant for increased ESR, hypocomplementemia and eosinophilia.

Reflex sympathetic dystrophy refers to burning pain in an area with previous history of
trauma. Symptoms refer to vasomotor instability with changes in temperature, color, and
texture of skin. This patient had no such symptoms.

106
BUERGER'S DISEASE
Buerger's Disease usually occurs in young smokers.
It is an episodic, and segmental inflammatory process of small and medium sized arteries
(thromboangiitis obliterans). It is characterized by occlusive disease of the arteries,
migratory superficial thrombophlebitis, and Raynaud's phenomenon.
The disease has an intermittent course with episodes of ischemia. Peripheral pulses are
usually diminished or absent. Intermittent claudication is common.
Cessation of smoking is an important component of management.

Erythromelalgia is a paroxysmal disorder of peripheral blood vessel dilation with


bilateral burning pain on palms and soles. It then progresses to the entire extremity. The
area usually becomes red and warm.

SPINAL CORD COMPRESSION


It is an acute syndrome of acute back pain associated with compression of the spinal cord.
It is considered a neurologic emergency.
Commonly caused by cancer, herniated disk, epidural abscess, or hematoma.
Pain is the earliest symptom in the majority of patients.
Cauda equina is an acute compression of lumbosacral nerve roots
by infection or tumor, presenting with acute motor and sensory loss,
loss of rectal tone, urinary retention or overflow incontinence.
Important physical findings include saddle anesthesia around the
anus or perineum and decreased tone of anal sphincter. This is a
surgical emergency, and a neurosurgeon should be notified as soon
as possible.
High-dose of dexamethasone should be started immediately once the diagnosis is
suspected.
An MRI should be performed to identify the site of compression/fracture, followed by
surgery.

107
LUMBAR SPINAL STENOSIS (NEUROPATHIC CLAUDICATION)
It is a common cause of low back pain & legs pain that radiate to the buttocks & thighs.
Lumbar spinal stenosis is associated with aging, especially after 60 years age.
The most common cause is degenerative joint disease.
Degenerative central canal narrowing of the lumbar spinal canal that can compression
of the nerve roots weakness, sensory loss, numbness/tingling and low back pain
radiates to the buttocks and legs associated with calf pain.
Narrowing of the spinal canal results from encroaching osteophytes at the facet joints,
hypertrophy of the ligamentum flavum and protrusion of intervertebral disks.

Flexion of the lumbar spine widening of the spinal canal relieves pain (e.g. sitting,
leaning forward, flexion of the hip and uphill walking).
Extension of the lumbar spine narrowing of the spinal canal worsens pain (e.g.
standing, leaning backward and downhill walking)

Leg cramping is worse at rest, with standing, and with walking (pseudo-or neurogenic
claudication).
In some patients gait disturbance is so prominent that they complain of having "spaghetti
legs" or walking "like a drunken sailor".

Positive Romberg sign are specific but not sensitive for lumbar spinal stenosis.
The preservation of pedal pulses helps distinguish from vascular claudication.

MRI is the investigation of choice for suspected lumbar spinal stenosis.


Mild to moderate: NSAIDs and abdominal muscle strengthening.
Advanced: Epidural steroid injections can provide relief.
Refractory: Surgical laminectomy may achieve significant short-term success, but many
patients will have a recurrence of symptoms.

108
DISK HERNIATION
Causes include degenerative changes, trauma, or neck/back strain or sprain.
Most common in the lumbar region, especially at L4L5 and L5S1.
Common among middle-aged and older men.
Disc herniation presents as acute onset of electricity-like low back pain radiating down the
buttock and below the knee usually preceded by several months of aching, discogenic
pain. The pain is caused due to impinged nerve.
Herniated disk manifests with sciatica-type pain that radiates along the thigh and typically
below the knee. Exacerbated by straining (e.g., coughing).
Pain of herniated disc syndrome becomes worsened with sitting and lumbar flexion and
therefore is different from pain of spinal stenosis.
Associated with sciatica, paresthesias, muscle weakness, atrophy, contractions, or spasms.
Positive straight leg suggests nerve root irritation due to impingement.
Crossed straight leg test is very specific for disc herniation.
At the same time, no neurologic deficit is present and the perianal area is intact (it is always
important to rule out cauda equina syndrome).

Obtain a plain radiograph if other causes of back pain are suspected (e.g., infection,
fracture).
In patients with acute 'mechanical' back pain without significant neurologic deficit,
conservative approach is preferred for a period of 4-6 weeks. This includes early
mobilization, muscle relaxants, and NSAIDs. Bed rest and physical therapy has not been
shown to be helpful.
If the pain persists after 4-6 weeks of conservative treatment or progressive neurologic
deficit evolves, high-resolution diagnostic modalities are usually employed: MRI or CT
with or without contrast myelography.

Emergency surgical decompression is indicated in case of significant or rapidly progressive


neurologic deficit (foot drop, weakness of the legs) or cauda equina syndrome.
Severe or rapidly evolving neurologic deficits and are indications for discectomy.

109
EPIDURAL ABSCESS
It is an enclosed infection in the epidural space spinal cord compression.
Infections usually occur via hematogenous spread or from direct extension.
Predisposing factors include diabetes, an immunocompromised host, renal disorders,
intravenous drug abuse, and recent spinal procedure.
Patient presents with fever, chills, back pain, paresthesias, and urinary retention. All the
above are signs of a neurologic compressive disorder.
The condition can rapidly lead to rapid paralysis.
MRI is the investigation of choice.
When patients present with symptoms of neurological compromise, an urgent surgical
decompression is required and antibiotics.
In mild infections with little or no spinal cord compression, management is usually medical
with frequent neurologic evaluation and repeat imaging studies. If there is any evidence of
progression or new neurologic symptoms, a surgical consultation should be obtained.

LERICHE SYNDROME
Leriche syndrome occurs as a result of atherosclerotic vascular disease and is characterized
by impotence and intermittent claudication.
This patient does have
Intermittent claudication does not occur at rest or standing and occurs only after walking
and gets relieved by rest.
Physical findings of patient with intermittent claudication may show absent or diminished
pulses below the level of stenosis, low temperature of the involved extremity, bruit,
impaired wound healing, increased venous filling time, shiny skin, hair loss, and atrophy of
skin.

110
METASTASIS FROM CARCINOMA
It is the most common malignant tumor of the skeletal system.
Malignancy is an infrequent but common cause of low back pain.
Important risk factors include old age, history of malignancy,
weight loss, and failure to improve with conservative therapy.
In patients with history of malignancy, back pain raises suspicion
for bone metastasis.
Progressive low back pain not relieved by rest, worse at night with failure to improve with
conservative therapy suggests vertebral body metastasis.
Bone scans identify areas of high bone turnover, and used to evaluate metastatic disease.
Technetium 99 scinti scanning is the most effective method to assess bone metastasis.

Chronic back pain is characteristic for multiple myeloma.


Punched-out lytic lesions and monoclonal immunoglobin proteins in serum & urine

LUMBAR STRAIN
Patient presents with an episode of acute back pain (relation to lifting a heavy object, no
radicular signs, and good response to conservative therapy).
There is usually no point tenderness. Para spinal muscles are involved.
Proper patient education is very helpful in protecting the back from recurrent injury. The
education should emphasize the importance of strengthening the supporting muscles
(including abdominal muscles) by regular exercise, choosing an appropriate sleeping
posture (e.g. avoiding sleeping on the stomach), and learning proper techniques for bending
and lifting objects (Keep the back straight while lifting an object). Exercises with repetitive
twisting and bending should be avoided. While lifting an object, one should also bend the
knees, keeping the back straight; this technique is very useful in preventing strains and back
injuries. Warm-up exercises should be done before any sporting activities.

111
LUMBAGO
Usually related to physical strain and has less dramatic onset.
Usually paravertebral muscle tenderness rather than spinal tenderness is the feature.

COMPRESSION FRACTURE OF THE VERTEBRAE


It is a common complication of advanced osteoporosis (vertebral body demineralization).
It usually manifests as acute back pain without any history of trauma in a predisposed
patient to osteoporosis e.g. old age, postmenopausal women and glucocorticoid therapy.
Neurologic examination will be normal. Local tenderness usually present.
Malignancy presents with similar picture. X-ray spine and bone scan is indicated to
differentiate these two.

Note: Absent ankle reflex can be seen in elderly patients as part of the normal aging
process.
Absent Babinski sign suggests no upper motor neuron-type lesion.
Lower extremity weakness or numbness suggests associated nerve injury.

THIRD TRIMESTER OF PREGNANCY


Low back pain is a very common complaint in the third trimester of pregnancy.
It is a very common problem that is mechanical in nature.
It is believed to be caused by the increase in lumbar lordosis and the relaxation of the
ligaments supporting sacroiliac and other joints of the pelvic girdle.
In addition, hormonal factors may contribute to the problem.

112
SPONDYLOLISTHESIS
Back pain in children requires a careful search into the cause because back pain in children
is frequently due to organic causes.
Patient presents with slowly developing back pain and neurologic dysfunction (e.g., urinary
incontinence). Such clinical manifestations in combination with a palpable "step-off" at the
lumbosacral area are typical for high-grade spondylolisthesis.
Spondylolisthesis is a developmental disorder characterized by a forward slip of vertebrae
(usually L5 over S1). Such a displacement results in chronic back pain and in neurologic
dysfunction, if significant. The area of vertebral displacement (a palpable "step-off") is
detected on physical examination if the problem is severe.

TENNIS ELBOW
Tennis elbow is lateral epicondylitis about the origin of the extensor muscle from lateral
epicondyle of humerus due to repeated forceful wrist extension.
Patient presents with pain near the lateral epicondyle that is worsened by usage.
Examination reveals point tenderness just distal to the lateral epicondyle of the humerus
and exacerbation of pain by extension of wrist against resistance.
Pathophysiology: degeneration of the carpi radialis brevis near the lateral epicondyle.

FROZEN SHOULDER
This is a result of pericapsulitis. Majority of the cases are idiopathic.
There is limited range of motion due to stiffness of glenohumeral joint.
Chronic pain may be present but the most prominent symptom is joint stiffness.
On examination, range of motion is restricted both on active and passive movement in all
cardinal movements.
Arthroscopy establishes the diagnosis by showing decreased joint space volume, and loss of
normal axillary pouch.
Treatment: NSAIDs, corticosteroids injection into the joint space, and physical therapy.

113
ROTATOR CUFF TEAR OR ROTATOR CUFF TENDINITIS
It presents with severe pain and weakness of the shoulder abduction.
It results from impingement of the supraspinatous tendon.
Movements of shoulder like positioning the arm above the shoulder aggravate pain.
Range of motion is limited only on active movement but is normal on passive flexion.
A positive drop arm sign, with inability to actively maintain 90 degree of passive abduction,
may be present in large tears.
Rotator cuff tendinitis can be distinguished from rotator cuff tear by injecting lidocaine that
will result in improvement in range of motion in cases of rotator cuff tendinitis but no
effect in range of motion in cases of rotator cuff tear.
MRI is used for definite diagnosis.

SUBACROMIAL BURSITIS
The subacromial bursa lies between the acromion and the tendon of the supraspinatus
muscle. Inflammation of the subacromial bursa typically occurs in the setting of chronic
micortrauma to the supraspinatus tendon repetitive overhead activity of the arm as in tennis,
swimming, pitching or golf. The supraspinatus tendon is traumatized by compression
between the acromion and the humeral head (impingement syndrome), and its vascular
supply may be temporarily compromised during such episode of compression.
Subacromial bursitis is characterized by shoulder pain, which is absent at rest but present
on overhead activity.
Physical examination reveals pain with active range of motion of the shoulder, Neer's
impingement sign (tederness in the shoulderon passive internal rotation and forward flexion
at shoulder) is present. No atrophy of deltoid muscle is present.
Ultrasonogram or MRI generally confirms the diagnosis.
Treatment is conservative with NSAIDs, physical therapy and activity modification.

114
ANSERINE BURSITIS
Inflammation of the anserine bursa can be the result of abnormal gait, overuse or trauma
Anserine bursitis presents with sharply localized pain over the antero-medial part of the
tibial plateau just below the joint line of the knee; the pain often presents overnight as
pressure from the knees making contact with one another while the patients lie on their
sides can exacerbate the pain.
On examination, there is a well-defined area of tenderness over the medial tibial plateau
just below the joint line.
Valgus stress test does not aggravate the pain, thereby ruling out damage to the medial
collateral ligament.
X-ray of the tibia is normal.
Treatment is with rest, ice and maneuvers to reduce pressure on the bursa.
Corticosteroids injection into the bursa is also helpful.

MEDIAL COLLATERAL LIGAMENT INJURY


Medial collateral ligament injury is associated with abduction injury to the knee.
Medial collateral ligament injury presents with pain along the medial joint line and is
aggravated by walking.
It is caused by Valgus stress applied on the lateral aspect of the knee when it is partially
flexed.
Medial knee pain is aggravated by, Valgus stress testing.

CERVICAL SPONDYLOSIS
Cervical spondylosis affects 10% of patients older than 50 years of age.
History of chronic neck pain is typical.
Limited neck rotation and lateral bending is due to osteoarthritis and secondary muscle
spasm.
Sensory deficit is due to osteophytes-induced radiculopathy and isolated sensory
abnormalities are associated with good prognosis.
Typical radiographic findings include bony spurs and sclerotic facet joints.

115
DE QUERVAIN'S TENOSYNOVITIS
De Quervain's disease is characterized by tendonitis of abductor pollicis longus and
extensor pollicis brevis as they pass through a fibrous sheath at the radial styloid process.
It was originally described in washerwomen in whom these tendons were chronically
irritated by wringing the clothes. However, in present times it is most commonly seen in
post partum females due to repetitive lifting of the infant.
Tenderness is noted over the radial side of wrist and first dorsal compartment.
Clinical diagnosis can be made by positive Finkelstein test (grasping the flexed thumb into
the palm with the fingers aggravates pain).
Treatment is best with local injection of long-acting steroids.

AMYLOIDOSIS
Amyloidosis may be primary or may be secondary to an inflammatory process. Amyloidosis
represents a group of greater than 20 diseases in which complex proteins are deposited in
the bodys tissues and organs as beta-pleated sheets. The disease can be hereditary or
acquired and has varied manifestations depending on the type of amyloid being deposited.

Primary amyloidosis may produce joint disease and gastrointestinal complaints of


diarrhea.

Secondary (AA) amyloidosis results from the deposition of acute phase reactants,
particularly serum amyloid A, in the setting of chronic inflammatory disease.
Potential causes of chronic inflammation associated with secondary amyloidosis include
chronic infection, psoriasis, inflammatory bowel disease, IV drug abuse and autoimmune
diseases such as rheumatoid arthritis.
AA amyloidosis is a systemic disease, and the kidneys and GI are prominently affected.
Nephrotic syndrome is common in these patients and may manifest with hypertension,
proteinuria and edema of the face and legs.
Hepatomegaly is also a common finding in secondary amyloidosis.
Amyloidosis also affects the heart (cardiomyopathy), musculoskeletal system
(pseudohypertrophy) and nervous system (peripheral neuropathy).

116
The appearance of 'speckled pattern' suggests the diagnosis of cardiac amyloidosis.
When a patient with Amyloidosis develops heart failure, the median survival is less than
one year. Treatment is only symptomatic and it focuses on congestive heart failure
symptoms. Chemotherapy and bone marrow transplantation may improve survival in
specific cases, but overall survival rate is very low. In heart is the end stage and next step is
Transplantation.

ANTIPHOSPHOLIPID ANTIBODY SYNDROME


It is characterized by recurrent arterial or venous thrombosis, or recurrent fetal losses in
the presence of antiphospholipid antibodies.
There are three types of antiphospholipid antibodies. The first type is responsible for false-
positive syphilis serology. The second type is lupus anticoagulant, and it falsely elevates
the APTT level. The third type is anticardiolipin antibody.
Antiphospholipid antibody syndrome may occur as a primary condition, or it may be
associated with other autoimmune disorders such as SLE.
Antiphospholipid antibody syndrome places a pregnant patient at increased risk of first or
second trimester abortion. The use of heparin and aspirin reduces this risk. Warfarin is not
used due to its teratogenicity.
Acute thrombosis is treated with heparin and factor anti-X-a activity is measured, since
APTT is not reliable in such cases.

SERUM SICKNESS-LIKE REACTION


It is an immune-complex mediated hypersensitivity reaction to non-human proteins.
It may occur one to two weeks after administration of certain drugs, such as penicillin,
amoxicillin or cefaclor in the setting of a viral disease.
Prominent symptoms include fever, urticarial rash, polyarthralgia and lymphadenopathy.
Serum sickness-like reaction is a clinical diagnosis, and should resolve with withdrawal of
the offending agent, it does not represent a true drug allergy.

117
NEUROGENIC ARTHROPATHY
It is also known as Charcots joint, due to diabetic neuropathy and repeated joint trauma.
The process starts with decreased pain, proprioception, and temperature perception, which
can occur due to DM, peripheral nerve damage, syrniomyelia, spinal cord injury, Vitamin
B12 deficiency, or tabes dorsalis.
As normal neurologic input is lost, patients unknowingly traumatize their weight bearing
joints. This causes secondary degenerative disease, joint deformities, and functional
limitation.
X-rays will reveal loss of cartilage, osteophytes formation and loose bodies.
Associated pain is typically mild.
Management of Charcots joint involving treating the underlying disease and providing
mechanical devices (e.g. special shoes) to assist in weight bearing and to decrease further
trauma.

OSTEONECROSIS
Osteonecrosis is also referred to as aseptic necrosis, avascular necrosis, ischemic necrosis
and osteochondritis dessicans.
Chronic corticosteroid use and chronic excessive ingestion of alcohol account for over
than 90% of cases of avascular necrosis of bone (osteonecrosis). Other precipitating factors
include; trauma and antiphospholipid syndrome.
In all cases, the vasculature to the affected bone is disrupted causing bone and bone
marrow infarction. Subsequently, the bone is unable to remodel and trabecular thinning
occurs ultimately leading to collapse of the affected bone. This process may take months to
years, on occasion to occurs. Clinically, the patient experience progressive pain during this
time. Progression of the disease will ultimately cause joint instability and pain at rest.
In the hip, patients presents with slowly progressive anterior hip pain with limitation of
range of motion.
MRI is the most sensitive test for this condition

118
CARPAL TUNNEL SYNDROME (CTS)
It occurs following entrapment of the median nerve within the carpal tunnel.
It is commonly seen in individuals with history of repetitive hand movements.

It is seen in association with rheumatoid arthritis, leukemia, and hyperparathyroidism.


Tenosynovial inflammation may cause CTS in the setting of rheumatoid arthritis.
Synovial tendon hyperplasia may cause CTS in patients with acromegaly.
Incidence increases in pregnancy secondary to an estrogen-mediated depolymerization of
ground substance which causes interstitial edema in the hands and face; that results in
increase pressure in the carpal tunnel resulting in median nerve compression.
Accumulation of matrix substances throughout the body causes a variety of the systemic
manifestations of hypothyroidism. Matrix accumulation in the median nerve and tendons of
the carpal tunnel may cause carpal tunnel syndrome.

Patients usually complain of numbness, pain, and paresthesias along the median nerve
distribution. There is diminished two-point discrimination over the affected areas. They
have weakness in the affected limb and frequently drop things.
Pain worsened by manual activity and after wake the patient from sleep.
Nerve conduction studies are very useful in diagnosing the carpal tunnel syndrome.
Sensory conduction delay is usually more prominent than motor weakness.
The initial treatment of CTS involves neutral wrist position splinting and NSAIDs.
Local steroid injection is indicated in cases where wrist splinting is insufficient to relieve
pain. Surgical decompression is reserved for cases when conservative management fails.

Tinel's sign is elicited by tapping over the median nerve. This causes a tingling sensation
along the median nerve distribution. It has very low sensitivity and specificity.
Phalen's sign - flexion of both wrists to 90 degrees with the dorsal aspect of hands held in
opposition for 60 seconds leads to pain or paresthesia along the median nerve. This is a
positive Phalen's sign. This test has a good specificity but lower sensitivity.
Carpal compression test is performed by, applying pressure over the carpal tunnel. This
reproduces the patient's symptoms. This test is more sensitive than Tinel's and Phalen's
signs but not diagnostic.

119
ERYTHEMA NODOSUM (EN)
It is characterized by reddish painful, subcutaneous, nodules that usually develop in the
anterior surface of both legs (with areas of hyperpigmentation suggested by the old, healed
nodules). EN occurs most often in women ages 15-40 years.
Histologically, this is a panniculitis involving inflammation of septa in the subcutaneous fat
tissue.
EN tends to be a relatively benign condition which heals in its own within 2-6week period
without scarring. However; EN can be an early sign of more serious disease processes and
rapid identification of the underlying cause may prevent disease-specific mortality.
Recent streptococcal infection is the most common cause of EN
While other important causes include; sarcoidosis, tuberculosis (TB), histoplasmosis and
inflammatory bowel disease.
In Europe and North America, sarcoidosis constitutes a dominant cause of EN while
tuberculosis (TB) remains an important underlying condition in other parts of the world.
Thus, the initial work-up in excluding these diseases include antistreptolysin O (ASO) titer,
a tuberculin skin test, and a chest radiograph.
The association of EN and Sarcoidosis is particularly strong in young, African-American
woman. Cough, arthritis, uveitis, and bilateral hilar lymphadenopathy Sarcoidosis.

Cyclosporine is an immunosuppressor (cytotoxic drug) most commonly used after


transplantation to prevent organ rejection. It predisposes to viral infection, lymphoma and
can be nephrotoxic.

120
Patent ductus arteriosus
Patent ductus arteriosus is an abnormal communication between the descending aorta and
pulmonary artery.
It is associated with birth at high altitudes and maternal rubella infection.
Exam will reveal a hyperdynamic left ventricle and a machinery type of murmur at the left upper
sternal border.
Chest-x ray will show increased vascular markings. The main pulmonary artery will be enlarged.
ECHO is diagnostic.

Congenital cyanotic heart defect


Infants with a congenital heart defect that is causing cyanosis such as pulmonary atresia, tricuspid
atresia, or Tetralogy of Fallot, they require ductus arteriosus patency in order to provide
pulmonary flow. Prostaglandin E1 is a vasodilator used to prevent closure of ductus arteriosus in
such infants, to increase or maintain pulmonary blood flow and improve oxygenation until
definitive surgery can be performed.

Indomethacin is a potent prostaglandin inhibitor that would close the infants ductus arteriosus,
decreasing the pulmonary perfusion causing progressive hypoxia and metabolic acidosis.

Aortic rupture
The most common vascular injury following blunt thoracic trauma is aortic rupture.
The possibility of aortic disruption must be considered in every victim of severe deceleration injury.
The aortic rupture is usually partial and the most common site are the ascending aorta just above the
aortic valve and just distal to the proximal left subclavian artery.
Clinical signs can include hypertension in the upper extremities due to pseudocoartication, and
normo- or hypotension in the lower extremities and weak, hoarse voice due to pressure on the
recurrent laryngeal nerve by periaortic hematoma (unilateral vocal cord paralysis).
Chest x-ray, which may show mediastinal widening, blurring of the aortic knob, depressed left
main stem bronchus, left pleural effusion, deviation of nasogastric tube, and an apical cap.
Suspicion must be high for this injury and the next test it to obtain a spiral CT scan of the chest.
Another option in an unstable patient is to obtain an ECHO.
Once diagnosed, surgery is the treatment. Paraplegia is the most feared complication of surgery.

121
Pleuritic chest pain
One common cause of pleuritic chest pain is pulmonary embolism with subsequent minor or
massive pulmonary infarction.
Pulmonary embolism is a common cause of pleuritic chest pain. It should be high in the differential
diagnosis of a young person presenting with pleuritic chest pain, tachycardia, and dyspnea
particularly if the patient is taking an OCP. The diagnostic test of choice is helical CT of the chest
with IV contrast.
Other causes of pleuritic chest pain includes; pneumonia, pneumothorax, collagen vascular disease,
pericarditis (relived by leaning forward), viral pleuritis and radiation pleuritis.

Dilated cardiomyopathy
A history of recent upper respiratory tract infection followed by sudden onset of cardiac failure in an
otherwise healthy patient is suggestive of dilated cardiomyopathy, most likely secondary to acute
viral myocarditis (most commonly Coxsackie B infection).
ECHO shows dilated ventricles with diffuse hypokinesia resulting in low ejection fraction (systolic
dysfunction).

Concentric vs. Eccentric hypertrophy of the heart


Concentric hypertrophy of the heart is seen following chronic pressure overload, as seen in valvular
aortic stenosis and untreated hypertension.
Eccentric hypertrophy of the heart is seen following chronic volume overload, as seen in valvular
regurgitation.

Atrial flutter
Atrial flutter is the result of a re-entrant rhythm within the atria.
Classic symptoms include palpitations, chest pain, shortness of breath and lightheadedness.
EKG feature: 'saw tooth' pattern of atrial impulses.
In atrial flutter, the ventricular rate is determined by AV node conduction. A 2:1 (most common),
3:1, or 4:1 heart block is often seen. Variable conduction of atrial impulses (e.g. from 2:1 to 6:1),
suggesting variable AV conduction.

122
Atrial flutter with unstable hemodynamics is best treated with cardioversion. Acute atrial stable
hemodynamics can be treated with cardioversion or can be managed with rate control. Chronic
stable atrial flutter is best treated with rate control, which is best achieved with either calcium
channel blockers or beta-blockers.

Aortic dissection
Aortic dissection presents as sudden, tearing chest pain that radiates to the back.
Leg weakness can occur if the dissection progresses to involve the arteries feeding the spinal cord.
Check blood pressure in both arms and auscultate for aortic regurge (early decrescendo diastolic
murmur at the left sternal border).
Mediastinal widening is often, though NOT always, seen on chest X-ray.
Normal EKG.
Transesophageal echocardiography is the preferred diagnostic tool.
Before performing TEE, hypertension should be controlled by intravenous labetalol.
Type A dissections involve the ascending aorta and are treated with medical therapy and surgery.
Type B dissections involve the descending aorta and are treated with medical therapy only.
Intravenous labetalol (beta-blocker) is the most appropriate intervention for acute aortic dissection.

Chest pain that is reproducible with palpitation suggests a musculoskeletal etiology.

Restrictive cardiomyopathy
It may be caused by infiltrative disease (sarcoidosis, amyloidosis), storage disease
(hemochromatosis), Endomyocardial fibrosis or idiopathic.
Echocardiography usually shows a symmetrically thickened ventricle wall, normal or slightly
reduced left ventricle size and normal or near normal systolic function, the appearance of 'speckled
pattern' suggests the diagnosis of cardiac amyloidosis.
Treatment for most causes of restrictive cardiomyopathy is ineffective and discouraging except for
Hemochromatosis. Early identification of hemochromatosis and treatment can reserve the cardiac
function. Phlebotomy and iron chelation therapy with subcutaneous deferoxamine may result in
substantial improvement in most cases of Hemochromatosis if these measures are implemented
early, before cellular damage becomes irreversible.

123
Prolonged, tachysystolic atrial fibrillation
ECG does not reveal P waves.
Prolonged, tachysystolic atrial fibrillation causes significant left ventricular (LV) dilation and a
depressed ejection fraction.
LV dysfunction results from tachycardia, neurohumoral activation, absence of an atrial 'kick' (that
accounts for up to 25% of LV end-diastolic volume), and atrial-ventricular desynchronization.
Controlling the rhythm or rate in patients with prolonged tachysystolic atrial fibrillation usually
improves the LV function significantly, sometimes even dramatically.

Hypertension
Lifestyle modification with weight loss, diet change and exercise can be used as a first line therapy
for hypertension.
A thiazides such as hydrochlorothiazide is the first-line pharmacologic agent for management of
hypertension in a patient without significant co-morbidities.
ACE inhibitors are the first line agent for management of hypertension in patients with Diabetes,
chronic kidney disease, and CHF.
Beta blockers such as atenolol are the first line agents for management of hypertension in patients
with angina, low ejection fraction, or post MI.
Calcium channel blockers such as Amlodipine (peripherally acting Ca channel blocker) are
preferred as the first line agent for management of hypertension in patients with significant
peripheral vascular disease (e.g. intermittent claudication, lower limb pain while walking,
diminished pulses in the lower limb, etc.).
Hydralazine should not be used as a first-line therapy for hypertension because it requires up to four
times daily dosing (may be used in pregnancy, but labetelol or alpha methyldopa is better)

CK-MB vs. Troponin T


Troponin T is the most sensitive marker for cardiac injury, it takes up to 10 days to return to normal
after myocardial infarction.
CK-MB is the most useful lab test in assessing for coronary re-occlusion after an MI because it
typically returns to normal levels within 1-2days.

124
Dipyridamole stress test
Dipyridamole (persantine) and adenosine are coronary vasodilators.
Infusion of these substances in patients without coronary artery disease, increases coronary blood
flow three to five times above the baseline levels.
However, in patients with coronary artery disease, the diseased vessels distal to the obstruction are
already maximally dilated, and their ability to increase myocardial perfusion is limited; therefore,
redistribution of coronary blood flow to 'non-diseased' areas occurs, and the perfusion of 'diseased'
segments diminishes.
This phenomenon demonstrated by dipyridamole is called coronary steal and is used to diagnose
ischemic heart disease.

Note: Dipyridamole is a potent antiplatelet aggregate.

Hypertrophic cardiomyopathy
Hypertrophic cardiomyopathy is inherited in an autosomal dominant pattern (chromosome 14).
It is more common in younger patients especially African-Americans.
Ejection-type systolic murmur at the left lower sternal border that decreases with an increase in
preload (e.g. squatting) is classic for hypertrophic cardiomyopathy [this is in contrast to most
murmurs, which are typically are more severe with increased preload].
Preload (valsalva) ventricular volume degree of obstruction murmur (& vice versa)
The first line of medical therapy is either -blockers or a cardiac acting calcium channel blocker
such as diltiazem since they promote diastolic relaxation.

Mortality in patients with CHF


ACE inhibitors have been shown in multiple studies to improve survival in patients with CHF.
Other commonly used medications with a mortality benefit in CHF include -blockers, Angiotensin
II receptors blockers, and spironolactone.

125
Vascular ring
In vascular ring anomaly, the trachea and esophagus are completely encircled by the aortic arch and
its branches.
Noisy breathing is noted in the first few weeks of life.
Vascular rings can compress the trachea and leads to stridor, wheezing, and shortness of breath
shortly after birth. They are often misdiagnosed as reactive airway disease.
Esophageal symptoms such as choking, vomiting or dysphagia occur in older infants and children
(not in young infants).
Treatment of significant compression is surgery.

Ventricular fibrillation
In the case of witnessed arrests of < 5 minutes duration, defibrillation should be performed
immediately.
In un-witnessed arrest or witnessed arrest occurring > 5 minutes before arrival of the defibrillator, a
cycle of CPR should precede defibrillation.
Lidocaine may be used in patients who remain in cardiac arrest after 3 shocks.

Situational syncope
Situational syncope should be considered in the differential diagnosis of syncopal episodes. The
typical scenario would include a middle age or older male, who loses his consciousness immediately
after urination, or a man who loses his consciousness during coughing fits.
The pathophysiologic mechanism underlying the situational syncope includes autonomic
dysregulation, which can be partially explained by straining and rapid bladder emptying.
Cardioinhibitory and/or vasodepressor mechanisms may be involved.

Jervell-Lange-Nielson syndrome
Jervell-Lange-Nielson syndrome is an autosomal recessive disease characterized by congenital
deafness and QT interval prolongation on EKG caused by molecular defects in ion channels.
QT interval prolongation predisposes patients with this syndrome to a particular type of ventricular
tachycardia called torsades de pointes.

126
Torsades de pointes cause syncopal episodes (without following disorientation) and sudden death.
If the patient is asymptomatic or does not have a clearly documented syncope, beta-blocker like
propranolol alone is sufficient.
If the patient is symptomatic or has a documented episode of syncope, the treatment will be beta-
blocker plus a DDD pacemaker.

Heat stroke ***Extremely important for USMLE exam


Heat stroke is a severe, life threatening and catastrophic medical emergency due to failure of
thermoregulatory center leading to severe hyperthermia with body temperature generally greater
than 40.5 C (105 F).
Heat stroke is of two varieties:
Non exertional heat stoke that occurs in individuals with chronic medical condition that can impair
the thermoregulation like cardiovascular disease, neurological or psychiatric disorders, certain
drugs like Anticholinergic, antihistaminic etc.
Exertional heat stroke, which is generally seen in young healthy individuals who engage in heavy
exercise in presence of high external temperature and humidity.

Dehydration is common in heat stroke, and is manifested by hot, dry skin and hypotension.
Tachycardia, tachypnea, and hemoconcentration are all signs consistent with dehydration.
Hyperthermia in heat stroke disrupts enzymatic activity is producing multisystem tissue damage and
organ dysfunction.
Seizures, altered mental status, acute respiratory distress syndrome (ARDS) [scattered rales on
exam], disseminated intravascular coagulation (DIC) [low platelets and prolonged PT/PTT],
hepatic/renal failure and psychiatric symptoms may occur.

Rapid cooling of the patent is the cornerstone of therapy of heat stroke. It should be done in
adjunction of stabilizing treatment. Evaporation cooling is the modality of choice as it is safe,
quick, effective and non invasive. Patient's naked body is sprinkled with mist of cold water, while
air is circulated with fans.

127
Heat exhaustion
Heat exhaustion, also known as heat prostration is a syndrome characterized by volume
depletion under the conditions of heat stress.
It can be distinguished from heat stroke by T < 40 C and lack of severe CNS symptoms.
Heat exhaustion is characterized by variable and nonspecific clinical features like weakness, fatigue,
impaired judgment, vertigo, nausea, vomiting, frontal headache and muscle cramps.
Physical examination is characteristic of orthostatic hypotension, profuse sweating and moderately
high body temperature (usually less than 40 C).
If untreated heat exhaustion can progress to heat stroke.

ACLS
Any patient in cardiac arrest with a non-shockable rhythm (anything other than VF/VT) should
immediately receive CPR including chest compressions, establishments of a definitive airway and
ventilation with 100% oxygen. IV access should also be established.

Neurocardiogenic syncope
Neurocardiogenic syncope is the most common cause of syncope (fainting).
This is also known as neurally mediated or vasovagal syncope.
It is frequently occurs in response to stress, pain, and certain bodily actions (e.g. urination).
It is particularly common in young women.
It occurs due to excessive vagal tone and is characterized by nausea, diaphoresis, tachycardia,
pallor and felling of warmth spreading over the body immediately before passing out.
Upright tilt table testing may be indicated to confirm the diagnosis if the syncope is recurrent
syncope.

Ventricular free wall rupture


Rupture of the ventricular free wall can occur during remodeling of the transmural infarcts with a
peak incidence 37 days after anterior wall infarction.
Acute decompansation and PEA secondary to rapid development of tamponade is typical.
Rapid diagnosis and Pericardiocentesis is needed to save the patients life.

128
Interventricular wall rupture causes a VSD (development of a new holosystolic murmur).

Papillary muscle rupture can cause hypotension secondary to severe acute mitral regurgitation
(new onset of holosystolic murmur at the apex).

Ventricular aneurysm presents with akinesis of the involved portion of the left ventricular wall,
ventricular arrhythmias, and systemic embolization.

Limb ischemia from AF


Classically patients with limb ischemia presents with some or all of the five Ps: pain, pulslesness,
pallor, paresthesias and paralysis.
Anticoagulation with warfarin is the most effective way to decrease the risk of embolic phenomena
in patients with AF.

Constrictive pericarditis
Constrictive pericarditis results from obliteration of pericardial space and fibrosis of pericardium
following an acute fibrinous or serofibrinous pericarditis or chronic pericardial effusion.
Tuberculosis is the most common cause of constrictive pericarditis, in immigrant population.
Some other causes of constrictive pericarditis are idiopathic (42% in USA), post radiotherapy (31%
in USA), post surgical (11%), connective tissue disorders, neoplasm, uremia, sarcoidosis, etc.

Constrictive pericarditis will lead to inability of ventricle to fill properly during diastole and would
further cause the signs and symptoms of decreased cardiac output (fatigue, muscle wasting etc) and
/or signs and symptoms of venous overload like elevated JVP, dyspnea, ascites, positive Kussmaul's
sign (inspiratory increase in the jugular venous pressure), pedal edema, tender hepatomegaly etc.
Constrictive pericarditis should be considered in patients with unexplained elevation of JVP and
history of predisposing condition.

The apical impulse is not easily palpable in constrictive pericarditis.


Sharp 'x' and 'y' descent on central venous tracing is characteristic of constrictive pericarditis as is
the presence of pericardial knock (early third heart sound heard after S2).
Finding of pericardial calcifications on the chest X-ray is an important clue.
CT scan usually shows increased thickness of the pericardium.

129
Cardiogenic pulmonary edema
Nitroglycerin (NTG), either IV, sublingual, or topical relives dyspnea and tachycardia associated
with cardiogenic pulmonary edema by rapidly reducing preload (quicker than morphine and loop
diuretics). NTG should be used cautiously in patients with hypotension.
Pulmonary capillary wedge pressure is the most important differentiating point between ARDS and
cardiogenic pulmonary edema.
PCWP in cardiogenic pulmonary edema is greater than 18 mmHg.
PCWP in ARDS is normal (usually less than 15 mmHg).

ARDS Non-cardiogenic pulmonary edema


ARDS results from diffuse inflammatory injury to the alveoli of lungs. Inflammatory mediators are
released as a result of local or distant tissue injury. Neutrophils recruited to the lungs become
activated and release proteases and free oxygen radicals which damage alveolar endothelial and
epithelial cells. Alveolar capillary permeability increases and protein is released into alveoli
increasing osmotic pressure in alveoli and as a result, free water is also dragged into the alveoli.
Accumulation of protein rich fluid in alveoli as a result of diffuse alveolar damage makes the lungs
stiff and they become less distensible or compliant.
Alveolar-arterial PO2 difference is widened in cases of ARDS and hypoxemia occurs as a result. It
occurs largely due to ventilation-perfusion mismatch.
Pulmonary hypertension is very common in ARDS and it occurs as a result of hypoxemic
vasoconstriction of pulmonary vasculature.
Criteria for ARDS diagnosis is:
1) A pulmonary capillary wedge pressure less than 18 mmHg.
2) PaO2 to FiO2 ratio of 200 mmHg or less, regardless of the level of PEEP.
3) Diffuse, bilateral infiltrates on chest-X ray.

130
Ventricular tachycardia
Ventricular tachycardia (regular, wide complex tachycardia) in the presence of a stable blood
pressure does not warrant cardioversion. The best treatment is loading with lidocaine or amiodarone
(drug of choice). Either of these 2 agents will aid in the conversion to normal sinus rhythm.
Cardioversion is an excellent option for sustained ventricular tachycardia with hemodynamic
compromise.
When someone is having recurrent VT, first thing to do after stabilizing the patient is to search for
underlying cause. Most likely, an electrolyte imbalance.

Supraventricular tachycardia (SVT)


Classic supraventricular tachycardia (look at the narrow complex QRS).
SVT is usually recognized by a HR of >140/min, regular, loss of 'P' waves and narrow QRS.
If the patient is hemodynamically unstable, electrical cardioversion should be performed
immediately.
However, if the patient is stable, vagal maneuvers (carotid massage) should be attempted initially.
If these maneuvers fail to covert him to normal sinus rhythm, IV adenosine push is the drug of
choice.

Torsades de pointes
Torsades de pointes is also termed polymorphic ventricular tachycardia.
It is a very rapid VT, characterized by a gradually changing QRS morphology.
It is often self-terminating but may degenerate into frank ventricular tachycardia.
The majority of these patients have prolonged QT interval during sinus rhythm.
It is important to recognize this arrhythmia, both because of its life threatening potential and because
of specific modalities of therapy, which it requires.
The most common offending agent, which causes Torsades, is Quinidine. Other drugs include
procainamide, tricyclics and disopyramide.
In the acute setting magnesium replacement is the cornerstone of the treatment.

131
Atrial premature beat
An atrial premature beat results from the premature activation of the atria that originates from a site
other than the SA node. EKG shows an early P wave.
Atrial premature beats may be present with underlying heart disease or may even be present in the
absence of any underlying heart disease (e.g. anxiety, hypoxia, caffeine, or electrolyte abnormalities)
They are usually asymptomatic or can cause palpitations.
They may be single or may be seen in the pattern of bigeminy.
Occasionally they may result in supraventricular or less commonly ventricular arrhythmias.

Premature beats are benign and neither require any follow up nor treatment.

Treatment is required only when the patient with atrial premature contraction has disturbing
palpitations or supraventricular tachycardia with beta-blockers or calcium channel blockers.

Tobacco and alcohol are reversible risk factors for the development of atrial premature beats.
Precipitating factors like tobacco, alcohol, or adrenergic agonists need to be identified and corrected
before opting for pharmacological therapy.

Pulmonary embolism
Suspect pulmonary embolism in any patient who presents with sudden onset of shortness of breath,
pleuritic chest pain, normal lung exam, hypoxic, and have tachypnea, tachycardia and hypotension.
Pulmonary vascular disease includes PE.
Patients with suspected pulmonary embolism should have a chest-x ray and ABG, followed by EKG
and V/Q scan.
While there are suggestive X-ray findings, the chest x-ray is normal in most cases.
The most typical findings of a pulmonary embolism on arterial blood gas sampling are hypoxemia
and hypocarbia.
The most common EKG finding is sinus tachycardia (classic ECG pattern of S1Q3T3 is not a very
common feature of pulmonary embolism).
If ventilation/perfusion scan is inconclusive in patients undergoing diagnostic work-up for
pulmonary thromboembolism, order venous ultrasonography to reveal DVT or CT angiogram.
Although considered the 'gold standard,' pulmonary angiography, an invasive procedure is
employed if venous ultrasonography or CT angiogram is negative. Chest CT angiogram is also
frequently used in indeterminate cases of pulmonary embolism, but the problem with CT angiogram

132
is that it can diagnose only large emboli. Small emboli can be easily missed.

Deep venous thrombosis is not a clinical diagnosis and therefore all suspected cases of DVT should
be evaluated with noninvasive testing and the test of choice for this purpose is compression
ultrasonography.

When an acute pulmonary embolus occurs with hemodynamic compromise, the best treatment is
fibrinolytic therapy. But, if the patient has contraindications to the use of fibrinolytic therapy, an
embolectomy is the treatment of choice.
If there is hemodynamic compromise and the clot is situated in the main pulmonary artery, the best
treatment option is to perform a pulmonary embolectomy.
Even after an embolectomy, the patient may require warfarin (coumadin) for a few months.

Right ventricular infarction management


Right ventricular infarction occurs in more than 30% of cases of inferior wall MI (ST elevation in
II, III, and aVF leads).
RV infarction presents with clear lung fields, jugular venous distension, and hypotension.
The first step in the treatment of right ventricular MI is generally IV fluids resuscitation to
increase the RV stroke volume and enhance left ventricular filling.
Nitrates are contraindicated in the setting of right ventricular wall infarction, aortic stenosis, or
recent phosphodiesterase inhibitor use (sildenafil Viagra) as nitrates will reduce preload and
exacerbate rather than improve symptoms.
In cases of RV MI complicated by bradycardia, temporary pacing is used to optimize RV output.

Abdominal aortic aneurysm (AAA)


AAA classically presents with a pulsatile abdominal mass.
Smoking cessation is the intervention with the greatest likelihood of slowing AAA progression.
Hypertension in patients with AAA should be treated with beta-blockers.
The study of choice for diagnosis and follow-up of abdominal aortic aneurysms is an abdominal
ultrasound.
Indications of surgery in patients with AAA include: diameter > 5 cm, presence of symptoms, or
rapid rate of growth.

133
After an AAA repair, diarrhea with blood in the stools should raise the suspicion of ischemic colitis.
If the CT scan is inconclusive a sigmoidoscopy/colonoscopy is recommended.

Evaluation of Angina
Coronary angiography is considered as the gold standard for diagnosis; however, it is invasive and
expensive procedure.
Exercise EKG testing is the most commonly test for evaluating patients with stable angina.
Medications that should be withheld prior to testing include anti-ischemic medications, digoxin,
and medications that slow the heart (e.g. beta-blocker).

Septic shock ***Please try to understand these concepts. They are heavily tested in board exams.

Septic shock is a form of distributive shock that results from decreased systemic vascular resistance
due to significant vasodilatation.
The hyperdynamic type of septic shock is characterized by:
1) An elevated cardiac output
2) Low systemic vascular resistance, right atrial pressure and pulmonary capillary wedge pressure,
3) A frequently normal mixed venous oxygen concentration.
Hyperdynamic circulation can be observed during septic shock, leading to an elevated cardiac
output.
Normal Mixed venous oxygen concentration (MVo2) can be observed in patients with septic shock,
and results from hyperdynamic circulation and improper distribution of the cardiac output. It is not
indicative of normal tissue perfusion in this case!
Decreased PCWP N = 15 mmHg (that reflects decreased left atrial pressure) indicates low left
ventricular preload, and therefore helps to rule out cardiogenic shock.

Other forms of distributive shock, such as neurogenic shock and hypovolemic shock, are typically
accompanied by a low MVo2 due to increased oxygen extraction by hypoperfused tissues. Low left
ventricular preload is also characteristic for volume depletion or intravascular fluid redistribution.

134
Cardiogenic shock ***Extremely high yield question for USMLE!!!

A depressed CO combined with an elevated PCWP (an indicator of left atrial pressure, and most of
the times left ventricular end diastolic pressure) is indicative of left ventricular failure.
SVR is elevated as a result of neurohumoral activation (sympathetic hyperactivity, renin-angiotensin
system activation) that is intended to preserve cardiac output and maintain tissue perfusion at an
acceptable level.

Elevated PCWP is also observed in patients with volume overload, but CO is not decreased (it will
be increased) in these patients, and SVR is not usually increased.
Depressed PCWP is characteristic for patients with absolute or relative hypovolemia that can result
from blood loss or intravascular volume redistribution; decreased left ventricular end diastolic
pressure (decreased preload) is the cause of hypotension and shock in these patients.

Occlusion of the right coronary artery ***Extremely high yield question for USMLE!!!

ST elevation in the inferior leads (II, III, and aVF) suggesting inferior wall myocardial infarction.
Inferior wall myocardial infarction results from the occlusion of either the right coronary artery or
the left circumflex artery.
In most studies with acute inferior myocardial infarction, the culprit lesion was in the right coronary
artery than in the left circumflex artery.
Also, remember that in patients with inferior myocardial infarction who have right ventricular
infarction, the culprit artery virtually always is the right coronary artery.
Bradycardia and hypotension suggesting involvement of SA node and right ventricle.

Cocaine abuse
Cocaine inhibits catecholamines reuptake from neuronal synapses, and therefore sympathomimetic.
In some, cocaine-induced vasospasm can be so severe as to cause myocardial ischemia and /or
infarction.
Cocaine and amphetamine intoxication present in a similar manner, but psychosis is more
commonly associated with amphetamine use. Common symptoms of stimulant intoxication include
dilated pupils, hypertension (autonomic instability), and tachycardia.
Pupillary dilatation and blood at the external nares in patients with chest pain can be a clue to
cocaine-induced vasospasm.

135
Cocaine induced myocardial ischemia (non ST elevation and negative CK-MB & troponin) are
treated with benzodiazepines (e.g. diazepam), nitrates, and aspirin

Cocaine induced ST elevation myocardial infarction (STEMIs) are treated the same as classic
STEMIs, with PTCA or thrombolysis.

Beta blockers should be avoided as they may aggravate the cocaine mediated vasoconstriction.

Peri- and Post-operative mortality


Administration of a beta-adrenergic receptor antagonist before, during and after surgery in at-risk
patients significantly lowers the incidence of perioperative myocardial ischemic events.
Beta-blockers exert this cardio-protective effect by decreasing cardiac work and causing a resultant
decrease in myocardial oxygen demand.

Total abstinence from alcohol is the mainstay of alcoholic dilated cardiomyopathy management
and it may reverse this condition if it is employed earlier in the course of the disease.

Pulsus paradoxus
Pulsus paradoxus is defined as greater than 10 mmHg fall of the systolic blood pressure during
inspiration.

This physiologic drop is the result of increased venous return to the right heart during inspiration,
which impedes left ventricular filling and thus causing drop in systolic pressure.
It is seen in some forms of cardiac disease (tamponade, pericardial effusion) and pulmonary disease
(tension pneumothorax, severe asthma).

136
Widened mediastinum
Causes of widened mediastinum on chest x-ray include aortic dissection, mediastinal mass,
mediastinal hemorrhage, esophageal rupture, SVC syndrome, and anthrax exposure.
SVC syndrome typically presents with dyspnea and facial swelling

Cushings syndrome
Cushings syndrome is an important cause of secondary hypertension.
Cushings syndrome may result from adrenal cortical hyperplasia, ACTH-producing pituitary
adenoma (Cushings disease), ectopic ACTH, or from administration of exogenous steroids.
In addition to hypertension, hypokalemia, high systemic cortisol concentrations cause
hyperglycemia, proximal muscle weakness, central adiposity, thinning of the skin, weight gain,
and psychiatric problems (e.g. sleep disturbances, depression, and psychosis).

Hyperparathyroidism
Hyperparathyroidism is an uncommon cause of secondary hypertension.
Asymptomatic hypercalcemia in a hypertensive patient should raise suspicion for this condition.
The presence of stones (kidney), bones (hypercalcemia), groans (constipation), and psychiatric
overtones (depression/fatigue/psychosis) should raise suspicion for hyperparathyroidism.

Peripheral vascular disease


Patients with peripheral vascular disease typically have the risk factors to those for heart disease and
present with claudication, impotence, and skin changes.
The degree of vascular insufficiency and the severity of the disease is best assessed with ankle-
brachial pressure index (ABI or ABPI). This test is done by determining the ratio of ankle to
brachial systolic blood pressure. The normal value is 1; numbers that approach 0 are associated with
more advanced disease.

137
Pathophysiology and treatment of hypertension
Rennin-angiotensin-aldosterone system is of critical importance in understanding hypertension.
Rennin is produced in the juxtaglomerular cells of the kidney in response to hypoperfusion.
Rennin cleaves angiotensinogen into angiotensin I.
Angiotensin I is converted into angiotensin II by angiotensin converting enzyme (ACE) in the lung.
Angiotensin II is a potent vasoconistrictor and promotes vasopressin (ADH) release from the
pituitary and aldosterone production in the adrenal cortex.
Aldosterone acts on the distal convoluted tubules and collecting ducts to promote sodium and water
resorption from the kidneys.
The net result of activation of this system is an increase in blood pressure, total body sodium, total
body water, and blood volume.

Direct rennin inhibitor increases natriuresis, decreases serum angiotensin II, and decreases
aldosterone production.

Angiotensin receptor blockers (ARBs) prevent angiotensin II from acting on angiotensin receptors.

Loop diuretics induce natriuresis only.

Alpha-adrenergic blockers are direct vasodilators used to treat hypertension.

Renal artery stenosis


The presence of a systolic-diastolic abdominal bruit in a patient with hypertension and
atherosclerosis is strongly suggestive of renal artery stenosis.
Abdominal bruits are quite common in young people affecting up to 30% of patients without HTN.

MI with pulmonary edema


Diuretics are necessary adjunct to the treatment of MI if pulmonary edema is present.
Beta-blockers are a standard therapy in MI, but should be avoided in this scenario as they can
worsen acute heart failure.

138
Subacute bacterial infective endocarditis (SABE)
Subacute bacterial infective endocarditis is usually seen in patients with damaged valves.
SABE presents with progressive fatigue and a chronic waxing and waning low-grade fever.
Strep viridans is the most frequent case of infective endocarditis in patients with damaged heart
valves (highly susceptible to penicillin and well treated with IV penicillin G or IV ceftriaxone).

Staph aureus is major cause of acute infective endocarditis in IV drug abusers.


Staph. Epidermidis is the most frequent cause of infective endocarditis in patients with prosthetic
valves.

Empirical intravenous broad-spectrum antibiotic (e.g. vancomycin & gentamicin) should be given
till blood culture results appear.
Once the specific causative bacterium responsible for endocarditis is identified, the antibiotic
coverage can be narrowed from empiric to specific.

Aortic regurgitation (AR)


In USA, aortic root dilatation and bicuspid aortic valve are the most common causes of AR.
AR causes widening of the pulse pressure (systolic diastolic blood pressure).
A wide pulse pressure causes a water hammer pulse, which many patients experience as
pounding heartbeat.
Lying flat or turning to the left brings the heart closer to the chest wall and make the patient more
aware of the forceful heartbeat.

Sustained handgrip
Handgrip is typically used to differentiate between the murmurs of aortic stenosis and mitral regurge
Sustained handgrip systemic vascular resistance afterload ventricular volume
outflow obstruction.
Thus, sustained handgrip mitral regurge murmur and aortic stenosis murmur.

139
Dialysis (renal replacement therapy) in end stage renal disease
Absolute indications
1) Fluid overload not responsive to medical treatment.
2) Hyperkalemia not responsive to medical management.
3) Uremic pericarditis.
4) Refractory metabolic acidosis.
5) Uremic seizure and coma.

Relative indications
1) Glomerular filtration rate < 10 ml/min (< 15 ml/min in diabetics).
2) Serum creatinine > 8 mg/dL (> 6 mg/dL in diabetics).
3) Severe uremic symptoms (Uremic seizure and coma is an indication of immediate dialysis).

Contraindications
1) Debilitating chronic disease.
2) Severe irreversible dementia.

Note: Indication of renal replacement therapy is not strictly based on serum creatinine or BUN levels.

Viral infection
In an otherwise young healthy patient who develops CHF, myocarditis should be considered high on
the differential diagnosis.
Viral infection, especially with Coxsackie B virus, is the most common cause.
Patients often, though not always, have preceding viral-type symptoms.

Marfans syndrome
Aortic dissection is the most dangerous complication of Marfans syndrome.
A common physical exam finding is an early diastolic murmur as the dissection causes AR.

140
Compartment syndrome
Extremities subjected to at least 46 hours of ischemia can suffer from both intracellular and
interstitial edema upon reperfusion.
When edema causes the pressure within a muscular fascial compartment to rise above 30 mmHg,
compartment syndrome occurs leading to further ischemic injury to the confined tissues.
The most reliable indicators of early compartment syndrome include pain in the affected extremity
that is out of proportion to the physical examination findings and pain that is exacerbated by
passive stretch of the muscles in the affected compartment.
The clinical findings of compartment syndrome include 5 Ps: Pain (earliest symptom),
Paresthesias (burning or tingling sensation), Pallor (as a result of tense swelling), Pulselessness
(late finding) and Paresis/Paralysis (late finding).
Treatment of compartment syndrome is by emergent fasciotomy.

Aortic stenosis (AS)


Exertional syncope is explained by aortic stenosis.
Physical findings characteristic for AS: increased intensity of the apical impulse, narrow pulse
pressure, and ejection-type systolic murmur at the base of the heart with radiation to the carotids.
Age-dependant idiopathic sclerocalcic changes are the most frequent cause of isolated aortic
stenosis in elderly patients. These changes are common and usually have minimal hemodynamic
significance, but sometimes may be severe.

Bicuspid aortic valve is a congenital anomaly that can lead to severe calcification and aortic
stenosis, but usually manifests earlier in life.

Anasarca
Anasarca or generalized edema, characterized by pulmonary edema and ascites in addition to
swollen extremities and face.
Anasarca results from organ failure or hypoalbuminemia.
In glomerulonephritis, edema results from decreased glomerular filtration rate as well as proteinuria
and hypoalbuminemia in some cases.

141
Atrial myxoma
A mid-diastolic rumbling murmur heard best at the apex is often suggestive of mitral stenosis, but
can be due to an intra-cardiac tumor obstructing the mitral valve as well.
Atria myxoma is the most common primary intracardiac tumor, and is usually located in the left
atrium. It can cause systemic symptoms such as fever and weight loss, neurologic symptoms due to
tumor embolization (e.g. left-sided weakness), and presents as a mass on echocardiography.

Mitral valve prolapse (MVP)


Mitral valve prolapse is the most common cause of isolated mitral regurgitation in North America.
Myxomatous valve degeneration is the pathologic cause of mitral valve prolapse.
Mitral valve prolapse causes mid-systolic click over the apex with systolic murmur at the apex after
the click if mitral regurgitation is present. Squatting decreases the murmur.
MVP sometimes associated with chest pain, anxiety, palpitations and/or hyperventilation.

Collagen abnormalities in Marfans syndrome predispose patients to aortic dissection and mitral
valve prolapse.

Isolated systolic hypertension (ISH)


ISH is an important cause of hypertension in elderly patients.
Pathophysiologic mechanism is believed to be decreased elasticity of the arterial walls, which leads
to an increased systolic blood pressure, without concurrent increase (and even decrease) in diastolic
pressure. As a result, patients with ISH have a widened pulse pressure.
Widened pulse pressure was recently recognized as an important cardiovascular risk factor.
Therefore, ISH should be treated appropriately, in spite of the fact that diastolic blood pressure is
not elevated. Hydrochlorothiazide is considered the drug of choice for this condition.

Hypertension management
The goal of therapy to maintain blood pressure below 140/90 mmHg in patients with uncomplicated
hypertension and below 130/80 mmHg in patients with diabetes mellitus or renal disease.
BP ranging from 120-139/80-89 mmHg (pre-hypertension), life-style modification is recommended.
BP ranging from 140-159/90-99 mmHg (stage I hypertension), single-drug therapy should attempted

142
BP exceeds 160/100 mmHg (stage II hypertension), two-drug combination are appropriate.

Hydrochlorothiazide is the initial anti-hypertensive of choice in the absence of specific indications


for other anti-hypertensive. Thiazides diuretics are particularly beneficial in older patients, African-
Americans, and those affected with osteoprosis.

Nitroglycerin
The main mechanism responsible for pain relief in patients with anginal pain treated with
nitroglycerin is dilatation of veins (capacitance vessels) venous pooling of the blood
significant decrease in ventricular preload decrease in heart size decrease of oxygen
requirements of the heart.
Nitroglycerin also causes arterial vasodilatation (but not as venous) and therefore, decreases
ventricular afterload, but this effect has less significance in relieving anginal pain.

Acute mitral regurgitation


Acute mitral regurgitation can occur due to papillary muscle dysfunction in patients with acute MI.
Acute mitral regurgitation characteristically causes a rise in left atrial pressure without significance
changes in left atrial size, left ventricular size or left ventricular ejection fraction.
Acute mitral regurgitation can also occurs due to papillary muscle rupture (as a mechanical
complication of MI that occurs in the first week after an acute MI).
MR due to papillary muscle ischemia is sometimes reversed when the damaged myocardium is
reperfused, but papillary muscle rupture requires emergent surgery.

Chronic mitral regurgitation results in increased left atrial size.

Aortic dissection
Tearing chest pain that radiates to the back and difference in BP greater than 30 mmHg between the
arms are important clues for aortic dissection.
Transesophageal echocardiography is the preferred diagnostic tool or CT with contrast (invasive).

143
Renal artery stenosis
Renal artery stenosis is a common cause of resistant hypertension in a patient with atherosclerosis.
Carefully auscultate the periumbilical area of such a patient to reveal continuous (or systolic with
diastolic component) murmur.

The presence of a systolic-diastolic abdominal bruit in a patient with hypertension and


atherosclerosis is strongly suggestive of renal artery stenosis.

Increased pulsations of intercostal arteries is typical for coarctation of aorta.

Oral contraceptives
Chronic use of oral contraceptives is a common cause of secondary hypertension. It is believed to
be caused by an estrogen-mediated increase in the synthesis of angiotensinogen in the liver.
Simply discontinuing its use can correct the problem.

Niacin
High-dose niacin therapy is used to treat lipid abnormalities and to raise HDL level.
Cutaneous flushing and intensive generalized pruritus are well-known side effects of high-dose
niacin. This is explained by histamine and prostaglandin-induced peripheral vasodilatation and
can be reduced by low-dose aspirin.

Cardiac tamponade
It is a life-threatening condition in which a pericardial effusion develops so rapidly or becomes so
large that it compresses the heart.
Suspect cardiac tamponade in an adult patient who has sustained blunt chest trauma if he has
jugular venous distension (with a prominent 'y' descent), tachycardia, distant heart sounds, pulsus
paradoxus, and hypotension (despite aggressive fluid resuscitation).
Chest x-ray findings typically reveal a normal cardiac silhouette without tension pneumothorax.
Electrical alternans is an important EKG finding for the diagnosis of pericardial tamponade. Other
findings include sinus tachycardia and low voltage QRS complexes.

144
Right ventricular failure ***try to understand this scenario

Chronic obstructive lung disease COPD chronic hypoxemia constriction of the pulmonary
arterial system and with time pulmonary hypertension right ventricular hypertrophy and right
ventricular failure.
Right ventricular failure causes elevated jugular venous pressure, congestive hepatosplenomegaly,
hepatojugular reflex, ascites and lower extremity edema.
Right ventricular failure is NOT a common cause of pulmonary edema.

Marfans syndrome
Marfans syndrome is an autosomal dominant disorder with variable penetrance.
The majority of patients with Marfans syndrome will present with aneurysms of the ascending
aorta. Associated findings may be aortic annular dilatation that will cause aortic regurgitation
(early diastolic murmur).
Surgery is required to replace both the aortic valve and entire ascending aorta.
Aortic dissection is the most dangerous complication of Marfans syndrome.

Aneurysm of the descending aorta in a young male is usually due to blunt trauma to the chest.

Atherosclerosis is the most common cause of aneurysm of the descending aorta in elder patients

Heart murmurs
Diastolic murmurs, continuous murmurs and loud systolic murmurs revealed on cardiac
auscultation should always be investigated using transthoracic Doppler echocardiography.
Mid-systolic soft murmur (grade I-II/IV) in an asymptomatic young patient are usually benign and
need no further work-up.

Digoxin intoxication
Digoxin is a cardiac glycoside used to treat atrial fibrillation and heart failure.
Potential side effects of digoxin include nausea, vomiting, diarrhea, blurry yellow vision, and
arrhythmias presented as palpitations (with almost every type of arrhythmia but not torsades).
Because of these myriad adverse effects patients digoxin level should be monitored closely.

145
Hypertrophic cardiomyopathy (HCMP)
Crescendo-decrescendo systolic murmur along the left sternal border without carotid radiation
(due to abnormal mitral leaflet motion) is the description of murmur present in Hypertrophic
cardiomyopathy.
Outflow obstruction in HCMP results from septal hypertrophy and systolic anterior motion of the
mitral valve.
Common symptoms of HCMP include syncope, dyspnea and chest pain.
Syncope in HCMP is often multifactorial and can be due to outflow obstruction, arrhythmia,
ischemia, and a ventricular baroreceptor response that inappropriately causes vasodilatation.

Systolic heart failure


It is important to know the basic pathophysiologic changes associated with heart failure.
First look at CI (a measure of cardiac output), it is always decreased in systolic heart failure.
Next, look at LVEDV it should be elevated in systolic heart failure and may normal in pure
diastolic heart failure.
TPR is elevated in systolic heart failure due to neurohumoral activation that includes sympathetic
hyperactivity and activation of rennin-angiotensin-aldosterone system.

Normal CI, elevated LVEDV, elevated LVEDP, and decreased TPR = high-output heart failure.

Mitral stenosis
Common due to rheumatic fever.
Cardiac auscultation reveals a loud S1 and a mid diastolic rumbling murmur at the apex.
Mitral stenosis increase pressure in the left atrium left atrial dilatation atrial fibrillation
(ECG shows irregularly irregular rhythm and loss of P waves) lack of an atrial kick
worsening flow through the stenotic mitral valve increased congestion in the lungs acute onset
of dyspnea.

146
Seizure vs. Syncope
A seizure results in loss of consciousness will often followed by a post-ictal state of cloud sensorium
(disorientation, sleepy, etc.)
People who experience syncope return to their baseline mental status immediately after regaining
consciousness.

Autosomal dominant polycystic kidney disease (ADPKD)


It typically presents with hypertension, palpable bilateral abdominal masses and microhematuria.
Ultrasound is diagnostic.

Intracranial berry aneurysm is a common complication, and is seen in 5 to 10% of the cases.
Although such aneurysms are common and dangerous when coupled with hypertension, routine
screening for intracranial aneurysms is not recommended.
The other major extra-renal complications of ADPKD are:
1) Hepatic cysts - most common extrarenal manifestations of ADPKD
2) Valvular heart disease - most often mitral valve prolapse and aortic regurgitation
3) Colonic diverticula
4) Abdominal wall and inguinal hernia

Autosomal recessive polycystic kidney disease presents in neonates or children as abdominal mass,
renal failure, or failure to thrive.

Pericardial effusion
On chest x-ray, a large pericardial effusion may appear as an enlarged, globular-appearing cardiac
shadow water bottle heart shape.
Due to the increased fluid in the pericardial space, physical exam will reveal distant heart sounds
and non-palpable pulse at the cardiac apex or point of maximal impulse.

147
Second degree heart block
Second degree heart block is identified by prolongation of the PR interval with occasional dropped
beats (it is of two kinds: Mobtiz type I and Mobtiz type II).
Mobtiz type I: site of block is AV nodal, PR interval gradually prolongs until a beat is dropped.
Mobtiz type II: site of block infranodal, PR intervals are of constant lengths before sudden
dropped beat.

Atrial fibrillation (AF)


EKG will reveal irregularly irregular, narrow QRS complexes with absent P waves.
AF is the most common chronic arrhythmia typically presents asymptomatically but may present
acutely with sudden onset of chest discomfort, palpitations, weakness and shortness of breath.
The causes of AF are numerous and include cardiac (HTN, acute ischemia, heart failure),
pulmonary (pneumonia, pulmonary emboli, hypoxia), metabolic (hyperthyroidism, catecholamine
surges), and drug-related etiologies (alcohol, cocaine, amphetamine, theophylline).
In unstable patients, immediate cardioversion should be performed.
In stable patient who has been in atrial fibrillation < 48 hours, cardioversion is appropriate.
In stable patient who has been in atrial fibrillation > 48 hours, 3 4 weeks of rate control (BCD)
and anticoagulation should precede cardioversion.

Chest pain
Chest pain in a young person with cardiovascular risk factors warrants a through cardiac work-up.
Aspirin should be the first drug administrated when suspicion of a coronary artery event is high due
to its ability to prevent platelet aggregation and coronary spasm.

Amiodarone
Amiodarone can cause pulmonary toxicity and should be avoided in patients with preexisting lung
disease.
Amiodarone lung toxicity is related to the cumulative total dose administered.

148
Dihydropyridine calcium-channel blocker
Amlodipine and Nifedipine are dihydropyridine calcium-channel blockers.
They can cause significant peripheral edema due to the property to dilate peripheral blood vessels.
It is NOT an allergic reaction. If the edema is significant, the drug should be discontinued.
They can cause peripheral vasodilatation and reflex bradycardia; as such they are contraindicated
in patients with acute coronary syndrome.

Infective endocarditis (IE)


Dukes criteria are used to make the diagnosis:
Major criteria:
Two or more blood cultures positive for a typical organism (S. viridans, S. aureus),
Endocardial involvement on ECHO, or
Presence of new murmur.

Minor criteria:
Presence of a predisposing condition (e.g. prosthetic valve, IV drug abuse),
Fever of 38.0C,
Suggestive vascular phenomena (e.g. septic emboli),
Immunologic phenomena (e.g. glomerulonephritis, Oslers nodes [painful fingertip nodules]) or
One positive blood culture

The presence of two major criteria, one major and three minor criteria or five minor criteria are
considered diagnosis of IE.

149
Leriche syndrome (Aortoiliac occlusion)
Leriche syndrome occurs as a result of atherosclerotic vascular disease and is characterized by triad
of bilateral hip, thigh and buttock claudication and pain, impotence and symmetric atrophy of the
bilateral lower extremities due to chronic ischemia.
Pain does not occur at rest or standing and occurs only after walking and gets relieved by rest.
Impotence is almost always present, in the absence of impotence think of another diagnosis.
Physical findings of patient with intermittent claudication may show absent or diminished pulses
below the level of stenosis, low temperature of the involved extremity, bruit, impaired wound
healing, increased venous filling time, shiny skin, hair loss, and atrophy of skin.

Chagas disease
A chronic protozoal disease caused by Trypanosoma cruzi (common throughout Latin America).
The two primary manifestations are megacolon/megaesophagus and cardiac disease.
Megacolon or megaesophagus (focal GI dilatation) occur secondary to destruction of the nerves
controlling the GI smooth muscle.
Pathophysiology of Chagas disease is not well understood, but probably represents a prolonged
myocarditis secondary to the protozoal infection.

Bradyarrhythmia vs. Tachyarrhythmia


A prolonged QRS interval or PR (i.e., typically bundle branch block) suggests a bradyarrhythmia.
Normal patients can maintain the cerebral blood supply with a heart rate from 30 to 180 bpm. When
the heart rate is lower than 30 bpm, cerebral perfusion is impaired, even if there is compensatory
increase in stroke volume.
A patient with a disease of the sinus node (sick sinus syndrome) or conduction system can have
intermittent bradycardia and typically manifests with recurrent syncopal episodes.

A prolonged QT interval suggests a tachyarrhythmia.


A prolonged QT interval is associated with polymorphic ventricular tachycardia called trosades
de pointes, in which the QRS complexes undulate over a point. The most important cause of
trosades de pointes is hypomagnesemia.

Postural hypotension is due to autonomic dysfunction.

150
Ventricular fibrillation
Ventricular fibrillation is a typical example of reentrant arrhythmia (Reentry).
Ventricular fibrillation is the most common cause of death in patients with acute MI. Decline in
mortality of patients hospitalized with acute coronary syndrome is largely attributable to the
effective detection and treatment of reentrant ventricular arrhythmias.

Amiodarone side effects


Pulmonary toxicity: the longer the patient uses amiodarone, the greater the chance of pulmonary
fibrosis developing.
Thyroid dysfunction (hypo and hyper): patients should have TSH levels checked before starting
treatment and also at 3-4 months interval.
Hepatotoxicity: up to 25% experience transient elevations of aminotransferases.
Corneal deposits: as amiodarone is secreted by the lacrimal gland.
Skin changes: blue-gray skin discoloration typically most prominent on the face.

Procainamide: nausea, drug-induced lupus, agranulocytosis and QT prolongation.

Lidocaine: CNS (drowsiness, confusion, agitation and seizures), and respiratory depression.

Quinidine: diarrhea, tinnitus, QT prolongation Quinidine syncope, thrombocytopenia.

Arteriovenous fistula
Symptomatic Arteriovenous fistula may be congenital, but usually it is after trauma.
Shunting of large amount of blood through the fistula from the arterial side to the venous side
increases cardiac preload and cardiac output.
Clinical signs include: widened pulse pressure, strong peripheral arterial pulsations (brisk carotid
upstroke), and tachycardia. The extremities are typically flushed.
There is left ventricular hypertrophy and the point of maximal impulse is displaced to the left.
Such patients are considered to have heart failure (high output cardiac failure) despite their high
cardiac output because the circulation is unable to meet the oxygen supply of peripheral tissues.
Other caused of high output cardiac failure include: thyrotoxicosis, Pagets disease, anemia, and
thiamine deficiency.

151
Post-MI hypertension
In post-MI hypertensive patient, beta-blockers and ACE inhibitors are preferred over diuretics and
calcium channel blockers.
Beta blockers decrease the myocardial oxygen demand by reducing heart rate and contractility.
ACE inhibitors are indicated when EF is decreased.

Lone atrial fibrillation


Patients with AF are at risk for development of mural thrombi and subsequent embolic strokes.
Lone atrial fibrillation is an atrial fibrillation in patient who is otherwise healthy (in the absence
of other risk factors of stroke such as; history of stroke or TIA, diabetes mellitus, hypertension, heart
failure, age over 65 years, valvular heart disease).
In patients with lone atrial fibrillation, anticoagulation with aspirin alone is sufficient.
In patients with atrial fibrillation and additional risk factors, anticoagulation with warfarin is
necessary with a goal of INR of 2-3.

When starting warfarin therapy, it takes at least 3-4 days to achieve the appropriate levels of
anticoagulation. In fact, warfarin therapy may actually causes hypercoagulability initially (this is
because the first vitamin K-dependant factors to be lost are proteins C and S, which are both
anticoagulants). In order to provide anticoagulation during this window of time, patients are often
bridged with a heparin product such as low-molecular weight heparin.

Thiazides ***Extremely high yield question for USMLE!!!

Decreased tolerance to glucose is a well-known side effect of thiazides leads to hyperglycemia.


Thiazides can also increase LDL cholesterol and plasma triglycerides.
Thiazides produce uric acid retention and gives risk for acute gouty arthritis.
Thiazides induce hyponatremia, hypokalemia, and hypercalcemia.
Despite these side effects, thiazides decrease cardiovascular mortality and morbidity at least equal
to that of the novel agent (ACE inhibitors, dihydropyridine calcium channel blockers).

Furosemide can cause ototoxicity, which presents as a hearing difficulty.


Aminoglycosides, vancomycin, quinine and chloroquine also cause ototoxicity.
Aspirin usually causes tinnitus.

152
Beta-blockers
Beta-blockers can worsen peripheral vascular disease by causing 2-mediated vasoconstriction of
the peripheral vessels.
Beta-blockers mask signs and symptoms of insulin-induced hypoglycemia.
Metoprolol, Atenolol, Carvidolol, and Esmolol (IV) are selective 1-adrenergic antagonists, when
administrated in high doses, they can block bronchodilatory 2-adrenergic receptors causing
bronchoconstriction in susceptible patients (e.g. asthma and COPD).

Hypovolemic or Hemorrhagic shock ***Extremely high yield question for USMLE!!!

Blood loss represents intravascular volume loss cardiac output, blood pressure, PCWP
stimulation of the sympathetic system generalized vasoconstriction ( SVR) and Heart rate.

Congestive heart failure (CHF)


CHF is a leading cause of peripheral and generalized edema.
In heart failure, there is a decrease in the effective circulatory volume, leading to renal
hypoperfusion, which leads to constriction of the kidneys efferent arterioles causing a resultant
increase in sodium retention within the nephron. These changes increase water retention and elevate
the total body volume, thus exacerbating CHF.
Additionally, renal hypoperfusion decreases glomerular filtration and therefore compromises the
bodys ability to get rid of excess fluid.

Amyloidosis
In amyloidosis, extracellular deposition of excess proteins results in organ dysfunction.
Deposition of amyloid fibrils in the kidney can result in proteinuria.
Binding of amyloid fibrils to the liver can inhibit the synthesis of coagulation factors, resulting in
increased bruisability.
Amyloid deposition in the heart leads to restrictive cardiomyopathy with symmetrical thickened
ventricular walls, and diastolic dysfunction. Systolic function is relatively preserved or slightly
reduced and ventricular dimensions remain unchanged.

153
Ventricular aneurysm
Ventricular aneurysm is a complication that may occur days to months after myocardial infarction.
Consequences of ventricular aneurysm include: symptoms of CHF, ventricular arrhythmia, mitral
regurgitation, and/or thrombus formation.
Persistent ST elevations are often seen on ECG in ventricular aneurysm.
Echocardiography is used to confirm diagnosis shows dyskinetic wall motion of a portion of the left
ventricle.

Vasovagal syncope
Excessive vagal tone is the mechanism.
Vasovagal syncope characterized by nausea, diaphoresis, tachycardia and pallor prior to syncope.
It frequently occurs in response to stress, pain, and certain actions (e.g. urination).

Boerhaave's syndrome
It is a spontaneous esophageal perforation and it produces pneumomediastinum.
The classic presentation is acute severe chest or epigastric pain after an episode of severe vomiting
or retching.
Fever and dyspnea are common, but hematemesis when present is not a prominent feature.
Physical examination often involves tachycardia, tachypnea, subcutaneous emphysema, and
unilaterally decreased breath sounds (usually left).
Palpable suprasternal crepitus may be found and it represents pneumomediastinum.
Chest x-ray reveals a unilateral pleural effusion (usually left), and widened mediastinum
(mediastinal emphysema).
Pleural fluid will have high amylase concentration, low pH, and may contain food particles.

Mallory Weiss tear is an incomplete esophageal tear, usually seen in patients with recurrent
vomiting or retching.

154
Hemochromatosis
Cardiac involvement may produce dilated cardiomyopathy leading to heart failure or various
conduction abnormalities.
Hemosiderin deposition in the pituitary gland can lead to hypogonadotropic hypogonadism
(testicular atrophy and decreased libido).
Hemosiderin deposition in the joints causes arthropathy (joint pain and swelling).
Hemosiderin deposition in the pancreas leads to diabetes.
Hemosiderin deposition in the skin leads to increased skin pigmentation (bronze diabetes).
Hemosiderin deposition in the liver leads to hepatomegally then cirrhosis.

RECOMMENDED DISCHARGE MEDICATIONS AFTER ACS


1) Aspirin (75-325 mg/day)
2) Beta-blocker
3) ACE-inhibitor
4) Lipid-lowering statin drug
5) Nitrates: all patients should prescribed short-acting nitrates (symptomatic benefits only).
6) Clopidogrel.
Clopidogrel should be prescribed to all patients with unstable angina/non-ST elevation MI (at
least for 12 months), as well as patients who are post PCI (at least for 30 days for bare metal
stents and 1 year for drug eluting stents as clopidogrel prevents subacute stent thrombosis).
Clopidogrel belongs to Thienopyridines, which also include ticlopidine.
Clopidogrel and Ticlopidine have anti-platelet effect by antagonizing ADP.
Clopidogrel is preferred over ticlopidine because it has less toxicity associated with it.
Clopidogrel is an alternative for aspirin for patients who can not tolerate it.

Premature ventricular contractions (PVCs)

155
PVCs identified by a wide QRS (> 120 msec [3 small squares), bizarre morphology, and
compensatory pause.
PVCs can be seen in normal individuals, but are more common in patients with cardiac pathology.
PVCs with increased frequency following myocardial infarction.
Though numerous PVCs may indicate worse prognosis, suppressing PVCs with antiarrhythmics has
actually been shown to worsen survival.
If the patient is asymptomatic, no treatment is indicated, just observation.
If the patient is symptomatic, beta-blockers are the first line of therapy.

Prinzmetal's angina or Variant angina


Prinzmetal's or Variant angina results from coronary vasospasm of a normal or minimally diseased
coronary artery, that causes transient transmural ischemia and ST segment elevation on EKG.
It typically occurs in young females and the greatest risk factor is smoking.
Classic picture of this disease, an episode of chest pain at night (from midnight to 8 am) and may
awake the patient up from sleep.
Transient ST elevation on the EKG, absence of Q waves and negative cardiac enzymes.
Calcium channel blockers and/or nitrates promote vasodilatation and prevent coronary vasospasm.
Non-selective -blockers and aspirin should be avoided as they can promote vasoconstriction.

Diastolic dysfunction
Diastolic dysfunction refers to impaired ventricular filling due to poor myocardial relaxation or
diminished ventricular compliance.
Systemic hypertension is the classic cause of diastolic dysfunction.
Contractility (i.e. ejection fraction) may remain normal but diastolic pressure is elevated,
consequently reducing cardiac output. This leads to typical findings of congestive heart failure.
Chronically elevated left ventricular diastolic pressure causes left atrial dilatation, which in turn can
lead to atrial fibrillation.
Treatment: diuretics and blood pressure control (Beta blockers are the initial drugs of choice).

156
Dressler's syndrome
Dressler's syndrome, an autoimmune pericarditis, is a late complication of acute MI that usually
develops between the second and tenth weeks post MI.
Classic EKG finding is diffuse ST elevation in all leads except aVR, where ST depression is seen.
Malaise and sometimes fever are also characteristic. ESR is typically elevated.
NSAIDs are the mainstay of therapy of Dressler's syndrome.
Anticoagulation should be avoided to prevent development of a hemorrhagic pericardial effusion.

Digitalis toxicity
Digitalis toxicity caused increased ectopy and increased vagal tone.
Atrial tachycardia with AV block is the most common arrhythmia specific for digitalis toxicity.

Aortic dissection
Aortic dissection presents as sudden, tearing chest pain that radiates to the back.
Hypertension is the most common predisposing factor.

Venous insufficiency
Venous insufficiency results from incompetence of the valves within the lower extremity veins.
Risk factors: old age, female gender, obesity, history of lower extremity surgery or DVT.
Chronic venous insufficiency is a leading cause of lower extremity edema.
Venous insufficiency manifestations ranging from mild varicosities, to leg pain, severe edema,
stasis dermatitis, skin fibrosis, ulceration, and immobility.
Therapies include leg elevation, compression stockings and wound care.

Aortic regurgitation (AR)


AR causes widening of the pulse pressure (systolic diastolic blood pressure).
Handgrip increases peripheral vascular resistance and afterload that causes louder murmur.
The primary medical therapy for AR is afterload reduction with vasodilators as nifedipine or ACE
inhibitors. If AR causes severe symptoms valve replacement is recommended.

157
Prinzmetal's angina or Variant angina
Prinzmetal's angina results from transient coronary vasospasm of a normal or minimally diseased
coronary artery, that causes transient transmural ischemia and ST segment elevation on EKG.
It typically occurs in young females and the greatest risk factor is smoking.
Classic picture of this disease, an episode of chest pain at night (from midnight to 8 am) and may
awake the patient up from sleep.
Prinzmetal's angina is associated with other vasospastic disorders, such as Raynauds phenomenon
and migraine headaches.
Transient ST elevation on the EKG, absence of Q waves and negative cardiac enzymes.
Calcium channel blockers and/or nitrates promote vasodilatation and prevent coronary vasospasm.
Non-selective -blockers and aspirin should be avoided as they can promote vasoconstriction.

Arterial occlusion of the lower extremity


Arterial occlusion of the lower extremity arises from: embolus, thrombosis or trauma.
All forms of arterial occlusion cause pain, diminished pulses, pallor, coolness to touch, neurologic
deficits (e.g. numbness), and muscle dysfunction in the affected extremity.

In embolic arterial occlusion, pain classically occurs suddenly and severe. diminished pulses or
absent pulses in the affected limb and normal in the unaffected limb. Majority of emboli arises from
ventricles following MI, and from the atria in atrial fibrillation.

Arterial thrombosis, results in slow, progressive narrowing of the vascular lumen in the affected
limb. Symptoms have an insidious onset. Pulses are diminished bilaterally.

SCREENING OF CARDIAC DISEASES


A detailed medical history and physical examination is the most effective way to screen a low risk
population for the presence of underlying cardiac diseases.
AHA recommends that all school and collegiate athletes be offered pre-participation screening by a
killed healthcare worker.

158
S4 in MI
A new fourth heart sound (S4) is one of the classic findings in MI.
Ischemic changes in the setting of MI may lead to diastolic dysfunction and stiffed left ventricle,
resulting in an atrial gallop (fourth heart sound).

Wolff-Parkinson-White syndrome (WPW)


In WPW, an accessory pathway conducts depolarization directly from the atria to the ventricles
without traversing the AV node.
In general, patients with atrial fibrillation and a rapid ventricular rate are treated with AV nodal
blockers like beta-blockers, calcium-channel blockers, digoxin and adenosine (IV). However, these
drugs in WPW serve to increase the accessory pathway, which can lead to ventricular fibrillation.
Thus, the best treatment for a patient with WPW who develops atrial fibrillation is conversion back
to sinus rhythm through electrical cardioversion or with antiarrhythmics like procainamide or
disopyramide (IV).

Stage I hypertension
BP ranging from 140-159/90-99 mmHg (stage I hypertension).
Life-style modification should be the first-line intervention for newly diagnosed stage I HTN.
Reduction of alcohol, sodium intake, weight loss (the most important intervention and the goal BMI
is 18.5-24.9 kg/m2) followed by aerobic exercise can all decrease blood pressure.
Quitting smoking will decrease a patients risk of cardiovascular events, but does not improve blood
pressure itself.

Massive pulmonary embolism (PE)


Massive PE is defined as a PE complicated by hypotension and/or acute right heart strain.
Syncope tends to occur in massive PE.
Jugular venous distension and new RBBB on ECG are indicators of right heart strain.
Treatment with fibrinolysis, with respiratory and hemodynamic support.

159
Statins
Elevated liver enzymes and myopathy (muscle pain) are well-known side effects of statins.
Statins are lipid lowering drugs by inhibiting HMG-CoA reductase, a rate-limiting enzyme in the
synthesis of cholesterol that converts HMG-CoA to mevalonate.
Mevalonate is used for the synthesis of cholesterol, dolichol, and CoQ10.
Reduced synthesis of CoQ10 has been implicated in the pathogenesis of statin-induced myopathy.
CPK levels should checked in any patient on statin who presents with muscle pain. If highly
elevated, the first step is to stop the statin.

B-type Natriuretic Peptide (BNP)


BNP is released from the cardiac ventricles in response to volume overload (elevated ventricular
filling pressure).
S3 is also associated with elevated ventricular filling pressure.

Measurement of plasma BNP is a useful laboratory test to distinguish between CHF and other
causes of dyspnea.
BNP level > 100 pg/mL diagnoses CHF with sensitivity, specificity and predictive accuracy of, 90,
76, and 83 respectively.

Cardiac tamponade
The classic physical findings of cardiac tamponade are pulsus paradoxus, and Becks triad:
hypotension, jugular venous distension (with a prominent 'y' descent),and muffled heart sounds.
Pathophysiology: when fluids accumulate in the pericardial sac it produces pressure above the
diastolic pressure of the ventricles, so the ventricles become less able to expand to accept venous
return to the heart. As a result of the decreased preload, stroke volume falls and thus cardiac output
is reduced. The decrease in cardiac output is further exacerbated during inspiration because the
lower intrathoracic pressure allows more allows more systemic venous blood to return to the right
ventricles, causing the intraventricular septum to bow into the left ventricle and further reduce the
left ventricular filling.
Electrical alternans is an important EKG finding for the diagnosis of pericardial tamponade. Other
findings include sinus tachycardia and low voltage QRS complexes.

160
Subacute bacterial endocarditis (SBE)
The most common causes of SBE are Viridans streptococci, enterococci, and coagulase-negative
Staph (such as S. epidermidis).
S. aureus is also common, but its presentation by acute, severe illness in a patient with normal heart
valves.
Cystoscopy can cause enetroccocal bacteremia in patients with chronic genitourinary infections.
Viridans streptococci (usually upper respiratory procedures).
Coagulase-negative Staph S. epidermidis (usually from the skin).

Dehydration
Elderly patients very poorly tolerate to fluid loss even if insignificant fluid loss (as an episode of
diarrhea, in case of decrease appetite or use of diuretics).
Mild hypovolemia may predispose an individual (especially elderly) to orthostatic syncope after
bed rest (like night-time sleeping), because it takes time from the body to adjust the sudden change
in the body position.
BUN/creatinine ratio is a sensitive indicator of the hydration status (but not very specific).
BUN/Cr > 20 indicates pre-renal azotemia.
Urine Na concentration decreased during dehydration (but taking salurtics makes it useless).

Heart failure
Hyponatremia is a bad prognostic factor in patients with heart failure. It indicates the presence of
severe heart failure and high level of neurohumoral activation.
Other important electrolyte abnormalities in patients with heart failure include hypo- and
hyperkalemia, that may reflect the activity of rennin-angiotensin-aldosterone system or may be due
to different drugs and drug combinations.
Decreasing the intake of water can help to control the electrolyte abnormalities.

161
Orthostatic hypotension
It is postural decrease in blood pressure by 20 mmHg systolic and 10 mmHg diastolic (sometimes
accompanied by an increase in the heart rate) that occurs on standing.
Decreasing baroreceptor sensitivity, arterial stiffness, decreased norepinephrine content in the
sympathetic nerve endings, and reduced sensitivity of the myocardium to sympathetic stimulation
(with aging) all contribute to increased incidence of orthostatic hypotension in elderly.

Subacute bacterial endocarditis (SBE)


SBE must be in the differential for all IV drug abusers presenting with fever an malaise.
IV drug abusers are at increased risk for SBE of both the right and left heart most commonly due to
coagulase-negative Staph S. epidermidis (usually from the skin).
If the tricuspid or Pulmonic valve is affected, SBE can present with embolic phenomena to the lung
(chest x-ray shows numerous round lesions in the lung periphery) and to the kidneys (proteinuria).
Tricuspid regurge is common in this setting (systolic murmur that increases with inspiration)

Malignant hypertension
Always suspect malignant hypertension in patients with very high blood pressure ( 200/140
mmHg).
Papilledema on ophthalmoscopy should be present in order to confirm the diagnosis.
Renal failure with elevated creatinine level and oliguria can develop rapidly in this case without
treatment, but its presence is not required to diagnose malignant hypertension.
The pathologic change responsible for end-organ damage in malignant hypertension is fibrinoid
necrosis of small arterioles.

Paroxysmal supraventricular tachycardia (PSVT)


PSVT is the most common paroxysmal tachycardia in people without structural heart disease.
The most common mechanism underlying PSVT is re-entry into the AV node.
Attacks begin suddenly and are characterized by heart rates between 160-220 bpm.
Mechanical and medical therapies for PSVT decrease AV node conductivity.

162
Vagal maneuvers, such as Valsalva, carotid sinus massage and immersion in cold water increases
vagal tone and decreases conduction through the AV node. This slows the heart rate and can often
break the rhythm.
Adenosine is a short-acting AV-nodal blocker is often used as well.

Loop diuretics (Furosemide)


Loop diuretics cause hypokalemia and hypomagnesmia. These electrolyte abnormalities can cause
ventricular tachycardia and also potentiate the side effects of digoxin.
When a patient experiences recurrent VT; the first thing to do after stabilizing the patient is to
search for the underlying cause.

Cocaine abuse
Cocaine inhibits catecholamines reuptake from neuronal synapses, and therefore sympathomimetic.
In some, cocaine-induced vasospasm can be so severe as to cause myocardial ischemia and /or
infarction.
Cocaine intoxication produces dilated pupils, hypertension (autonomic instability), tachycardia,
and may be myocardial ischemia and /or infarction.

Cocaine induced myocardial ischemia (non ST elevation and negative CK-MB & troponin) are
treated with benzodiazepines (e.g. diazepam), nitrates, and aspirin.
Elevated blood pressure can be treated with alpha-blockers.

Cocaine induced ST elevation myocardial infarction (STEMIs) are treated the same as classic
STEMIs, with PTCA or thrombolysis.

Beta blockers should be avoided as they may aggravate the cocaine mediated vasoconstriction.

Ventricular septal defect (VSD)


VSD is the most common congenital heart lesion.
VSD can cause failure to thrive, easy fatigability & heart failure.
VSD produces pansystolic murmur loudest at left sternal border plus diastolic flow murmur at the
apex (increased flow through the mitral valve).

163
Tetralogy of Fallot
Tetralogy of Fallot is the most common cyanotic congenital disease in children < 4 years of age.
Patients often present with cyanotic spells.
The classic tetrad includes (1) overriding aorta, (2) right ventricular hypertrophy, (3) subpulmonary
stenosis (single S2), and (4) ventricular septal defect (VSD) (pansystolic murmur).
Because a large unrestrictive VSD is always present, the right ventricular pressure is the same as the
left ventricular and aortic pressures.
Pulmonary artery pressure and flow are inversely proportional to the degree of subpulmonary
obstruction. The degree of cyanosis correlates precisely with the amount of obstruction.

Lidocaine
Lidocaine is a class IB anti-arrhythmic drug that is effective against a variety of ventricular
arrhythmias.
It has a wide therapeutic-to-toxic ratio.

Lidocaine is always given intravenously and can be administered via the endotracheal tube (when no
IV access exists).
Side effects include confusion, seizures and respiratory arrest.
It is widely used to control complex forms of ventricular arrhythmia (e.g., ventricular tachycardia)
in patients with acute coronary syndromes. At the same time, its use prophylactically to prevent the
development of ventricular fibrillation in patients with acute myocardial infarction is discouraged.
Clinical trials demonstrated that prophylactic lidocaine decreases the frequency of VPBs and
diminishes the risk of ventricular fibrillation (Choice A), but the overall prognosis is unaffected. The
problem is that lidocaine can increase the risk of asystole in these patients. Lidocaine has little
effect on myocardial contractility.
Lidocaine is not used prophylactically in patients with acute coronary syndromes. Although its
use decreases the risk of ventricular fibrillation, it may increase the risk of asystole.
Lidocaine has NO role in the treatment of atrial arrhythmias.

164
MAJOR FORMS OF SHOCK IN THORACIC TRAUMA
MAP HR PCWP PCWP
After fluid challenge

Hypovolemic/Hemorrhagic U or

Cardiogenic (LV dysfunction) V


Contusion, MI, traumatic valve
or septal rupture, arrhytmia

Cardiogenic (RV dysfunction) V , N U or


Contusion, MI

Extracardiac obstructive , N U or
Tension pneumothorax,
massive hemothorax

Extracardiac obstructive or
Pericardial tamponade

Neurogenic , V , N

Septic (hyperdynamic) , N

MAP = mean arterial pressure PCWP = pulmonary capillary wedge pressure


HR = heart rate V = variable
U = unchanged N = normal

Cardiogenic shock
Treatment is first with IV fluids to bring the PCWP to between 15 and 20 mmHg, followed by
positive inotropic agents e.g. dopamine or dobutamine to increase the cardiac output.
If the PCWP already high, give positive inotropic agents immediately.

Septic shock
The hyperdynamic type of septic shock is characterized by
1) Elevated cardiac output
2) Low systemic vascular resistance, right atrial pressure and pulmonary capillary wedge pressure
3) Frequently normal mixed venous oxygen concentration.

165
Indications of aortic valve replacement
1) All symptomatic patients with AS.
2) Patients with severe AS undergoing CABG or other valvular surgery.
3) Asymptomatic patient with severe AS with either; poor LV systolic function, LV hypertrophy >
15mm, valve area < 0.6 cm2 or abnormal response to exercise.

Selective alpha-blockers
Selective alpha-blockers are not recommended as first-line therapy anti-hypertensives.
Doxazosin, prazosin & terazosin are effective in patients with BPH by relaxing the sphincter
They are also have a favorable metabolic profile being beneficial in patients with glucose
intolerance and dyslipidemia.

Essential tremor
Essential tremor is a diagnosis of exclusion. It is a persistent, progressive condition that usually
begins in adulthood.
Essential tremor is characterized by a postural tremor (not resting tremor as in Parkinsons disease)
and usually disturbs the performance of fine motor tasks.
Most patients are only mildly affected and do not require treatment. Patients who require medication
are given either beta-blockers or primidone.
Beta-blockers are first-line treatment for essential tremors. The response in most patients is
excellent. Beta-blockers are not recommended if the patient is bradycardic or has severe COPD.
Primidone is an anticonvulsant agent which can be used to treat benign essential tremors. Its
administration can precipitate acute intermittent porphyria (colicky abdominal pain, confusion,
headaches, hallucinations and dizziness), which can be diagnosed by checking for urine
porphobilinogen.

ACE inhibitors
Ventricular remodeling in the weeks to months following myocardial infarction can lead to
dilatation of the ventricles. This process is lessened by ACE inhibitors.
ACE inhibitors should be initiated within 24 hours of myocardial infarction in all patients without
containdications.

166
ANEMIA
Anemia can be due to decreased RBC production, increased RBC destruction or frank blood loss.

Micocytic anemia
Iron studies are indicated in patients who present with microcytic/hypochromic anemia.
These are useful in confirming the diagnosis of iron-deficiency anemia, which is the usual cause of
microcytic/hypochromic anemia, and ruling out other causes.

Iron deficiency anemia


Iron deficiency is the most common cause of anemia in elderly patients.
Chronic gastrointestinal blood loss is the most common cause of iron-deficiency anemia in an adult
male or a post-menopausal woman.
Pica refers to an appetite to non-nutritive substances, such as ice, clay, dirt and paper products. Pica,
especially for ice, can be indicative of iron deficiency anemia.

Iron deficiency anemia: serum iron, serum ferritin and TIBC.


Anemia of chronic disease: serum iron, normal or serum ferritin and normal or TIBC.

Bone marrow stain is the most definite way to diagnose iron deficiency anemia.

Anemia of chronic disease (ACD) and Renal disease


Anemia of chronic disease (ACD) is seen in patients with chronic illnesses and is usually
normocytic. This is usually seen with infectious, inflammatory, or neoplastic diseases. Also, more
recently, this has been noted in patients with severe trauma, heart disease, and diabetes mellitus.
The pathophysiology involves defective utilization of the iron by the RBC precursors, secondary to
inflammatory mediators.
Anemia of chronic disease: serum iron, normal or serum ferritin and normal or TIBC.
Platelet and WBC counts are normal.

Treating the underlying cause of ACD will often improve the anemia.

Anemia of chronic kidney disease is due to erythropoietin deficiency.


One must be careful to ensure adequate iron stores prior to erythropoietin replacement because
erythropoietin-induced surge in RBC production can precipitate an iron-deficient state.

167
Lead poisoning
Basophilic stippling and microcytic hypochromic anemia in a child are important clues to the
diagnosis of lead poisoning.

Macrovascular traumatic hemolysis


In which intravascular fragmentation of red blood cells occur and leads to microcytic anemia
It is common in patients with artificial heart valves or severely calcified aortic valves.
Schistocytes (helmet cells) are the classic finding on peripheral blood smear.

Sideroblastic anemia
Sideroblastic anemia is seen in inherited or acquired defects affecting the biosynthesis of heme
within red cell precursors.
Hereditary form is due to defect in aminolevulinic acid synthase or abnormality in vitamin B6
(Pyridoxine) metabolism.

Sideroblastic anemia is characterized by serum iron levels and TIBC.


Usually two groups of RBC can be demonstrated on microscopy hypochromic and normochromic
(dimorphic RBC population).

It can progress to acute myelogenous leukemia (AML).


Prussian blue stain of RBCs in the marrow reveals ringed sideroblasts.
In patients with an identifiable cause of vitamin B6 (Pyridoxine) deficiency, (alcoholism,
chloramphenicol, isoniazid), the administration of pyridoxine can easily correct the problem.

168
THALASSEMIA
Hereditary underproduction of either alpha or beta globulin chains of the hemoglobin molecule
resulting in microcytic hypochromic anemia.

Alpha thalassemia
Alpha thalassemia is caused by abnormalities in the synthesis of alpha chains of hemoglobin.
It is usually a hypochromic, microcytic anemia, and a positive family history is usually present.
Peripheral blood smear shows hypochromic, microcytic anemia, and target cells.
If the patients anemia is not severe (as evidenced by the hemoglobin level, adulthood diagnosis, and
lack of severe symptoms), it is probably alpha-thalassemia minor. The treatment of choice is
reassurance and follow-up monitoring.

-thalassemia
-thalassemia minor occurs in people heterozygous for the -hemoglobin chain this results in
reduced hemoglobin synthesis and eventually hypochromic microcytic anemia due to poor
hemoglobinization of RBCs (hemoglobinopathy).
A patient with microcytic anemia non-responsive to iron supplementation is most likely to be -
thalassemia minor (particularly in someone of Mediterranean origin).
Peripheral blood smear shows hypochromic, microcytic anemia, and target cells.
If the patients anemia is not severe (as evidenced by the hemoglobin level, adulthood diagnosis, and
lack of severe symptoms), it is probably beta-thalassemia minor. The treatment of choice is
reassurance and follow-up monitoring.

Beta Thalassemia Major (Cooley Anemia)


-thalassemia major occurs when both -hemoglobin genes are defective, this results in severe
anemia and transfusion dependant at an early age (6-12 months of age).
Peripheral blood smear shows hypochromic, microcytic anemia, and target cells.
Growth failure, hepatosplenomegaly, jaundice, boney deformities, hemochromatosis, cirrhosis,
congestive heart failure and chronic anemia.
Treatment includes: blood transfusions once or twice a month, Deferoxamine, splenectomy, and
bone marrow transplantation.

169
Diamond-Blackfan Anemia Congenital Pure Red-Cell Anemia
The majority of cases are sporadic, although dominant and recessive inheritance is found in 15
percent of cases.
The primary pathology is an intrinsic defect of erythroid progenitor cells which results in increased
apoptosis (programmed cell death).
The condition often presents with pallor in the neonatal period.
Congenital anomalies (triphalangeal thumbs and craniofacial deformities) are present in over 50
percent of cases.
The macrocytic anemia of DBS is distinct from that of megaloblastic anemia because there is no
hypersegmentation of the nucleus in neutrophils and other blood cells in the former.
Very low reticulocyte count.
Increased RBC adenosine deaminase (ADA).
Electrophoresis reveals elevated fetal Hb levels.
Bone marrow biopsy shows significant decrease in RBCs precursors.
Therapy is mainly corticosteroids.
For unresponsive patients, transfusion and deferoxamine therapy is indicated.
Definitive treatment is stem cell transplant.

Fanconi Anemia
Fanconi's anemia is an autosomal recessive (chromosomal breaks) disorder marked by progressive
bone marrow failure characterized by progressive pancytopenia and macrocytosis ( MCV).
The average age at diagnosis is 8 years.
Associated deformities include cafe-au-lait spots (hypopigmentation of the skin), microcephaly,
microphthalmia, short stature, horseshoe kidneys, hypogonadism and absent thumbs.
Bone marrow hypoplasia.
Complications: increased risk of leukemia (AML) and other cancers.
Therapy is mainly corticosteroids and androgens.
Definitive treatment is bone marrow transplant.

170
Macrocytic anemia
Macrocytic anemia: MCV, MCH (90% of time) and normal MCHC.
The initial evaluation of patients with macrocytic or megaloblastic anemia includes measurement of
vitamin B12 and folic acid levels.
Folate and cobalamin deficiency both result in megaloblastic, macrocytic anemia with
hypersegmented neutrophils.
Folate and cobalamin deficiency both result in homocysteine levels and methionine levels
because both are involved in homocysteine metabolism to methionine.
Both folate and cobalamin are co-factors in the conversion of homocystine to methionine. Thus,
folate supplementation can improve the anemia of either disorder but will not improve the
neurologic changes of cobalamin deficiency.

Pernicious anemia Folic acid deficiency


Serum LDH Increased Normal
Achlorhydria Present Absent
Schilling test Positive Negative
Methyl malonyl CoA in urine Present Absent
Neurological signs Present Absent
Ass

Vitamin B12 (Cobalamin) deficiency


Vitamin B12 deficiency may occur due to: pernicious anemia (antibodies against intrinsic factor),
after total or partial gasterctomy, gastritis & elderly age (malabsorption), and strict vegetarian
after 3 to 4 years (low dietary intake).
Schilling test helps to differentiate between these causes; the patient is given a dose of oral radio-
labeled vitamin B12 and an intramuscular injection of unlabeled vitamin B12. Then, the urinary
excretion of radioactive vitamin B12 is measured. Normal urinary excretion of radioactive vitamin
B12 suggests normal absorption (B12 deficiency is most likely due to poor intake in diet).
Diminished urinary excretion of radioactive vitamin B12 suggests impaired intestinal absorption. To
differentiate between pernicious anemia and malabsorption, the next dose of radioactive vitamin
B12 is given with intrinsic factor; low excretion of radioactive vitamin B12 rules out pernicious
anemia and suggests a malabsorption syndrome e.g. pancreatic insufficiency, bacterial over growth...

171
Vitamin B12 is a necessary co-factor in the pathway leading to purine synthesis.
Vitamin B12 deficiency decreases DNA synthesis and erythropoiesis megaloblastic anemia.
Vitamin B12 deficiency presents with glossitis, neurologic changes & megaloblastic anemia.
Cobalamin (vitamin B12) deficiency can result in peripheral neuropathy or posterior column defects
due to defective myelin synthesis.

Pernicious anemia (PA) is the most common cause of megaloblastic anemia (common in elderly).
In PA, Vitamin B12 deficiency is due to reduced intrinsic factor (IF) secondary to gastric atrophy.
In the majority of cases, antibodies to parietal cells have been reported.
The diagnosis of PA is confirmed by achlorhydria, dcreased serum Vitamin B12, positive IF
antibodies, and extremely elevated LDH.
Pernicious anemia is associated with increase risk of gastric cancer.

Folic acid deficiency


The most common cause of folic acid deficiency is nutritional due to poor diet and/or alcoholism.
Alcohol abuse is the most common cause of folate deficiency in USA.
Some drugs impair the absorption of folic acid (e.g. phenytoin) and some drugs antagonize its
physiologic effects (e.g. methotrexate, trimethoprim).

172
HEMOLYTIC ANEMIA

Any hemolysis LDH, Bilirubin, Reticulocytes and Haptoglobin

Sickle cell disease


Sickle cell disease is an autosomal recessive disorder.
Sickle cell disease is characterized by chronic hemolysis of sickled cells, leading to a high RBC
turnover and anemia.
Hemolysis is mainly extravascular and leads to LDH, Bilirubin, Reticulocytes and
Haptoglobin
The hematocrit is generally 20-30 percent, owing to the decreased RBC volume.
The presence of hemoglobin F protects the infants from sickling during the first 4-6 months of life.

Painful crises are the most common manifestation of sickle cell anemia.
Dactylitis may be the initial presentation that warrants further work-up for sickle cell disease.
Sickle cell disease can cause childhood stroke.
Osteonecrosis is a common complication of sickle cell anemia due to vaso-occlusion of the bone. It
causes significant joint pain and functional limitation. The humerus and femur are the most
frequently affected bones.

Vasoocclusive crisis are one of the complications of sickle cell disease, which may manifest as
stroke, priapism, or intractable pain. Whenever such acute vasoocclusive crisis occurs, Exchange
transfusion is indicated.

Folic acid deficiency is much more common than vitamin B12 deficiency in sickle cell anemia.
Because of chronic hemolysis and compensatory reticylocytosis, increased demand for folic acid can
lead to folic acid deficiency. For this reason, patients with sickle cell anemia should be on folic acid
supplementation.
Folic acid supplementation is recommended in all patients with sickle cell anemia to prevent the
occurrence of aplastic crisis.

Patients at an early age become functionally asplenic; thus they are susceptible to infection with
encapsulated organisms such as S. pneumoniae, H. influenzae, and N. meningitidis.
Post-splenectomy patients or patients with auto-splenectomy are at increased risk for sepsis from
encapsulated organisms due to impaired antibody-mediated opsonization in phagocytosis.

173
Start penicillin prophylaxis at 2 months age.
Immunize the patient with regular vaccinations plus Pneumococcal vaccine, Polysaccharide
meningococcal vaccine H. influenzae type B vaccine, and hepatitis B virus vaccine.

Howell-Jolly bodies are nuclear remnants within RBCs typically removed by the spleen. They
appear in blood smear as single, round blue inclusions on Wright stain. Their presence stongly
suggests physical or functional hyposplenism.

Hyposthenuria is common in patients with sickle cell anemia and sickle cell trait. In which there is
an impairment of kidneys ability to concentrate urine leads to nocturia.

Autoimmune hemolytic anemia


Autoimmune hemolytic disease is extravascular hemolytic anemia; it is acquired.
A negative family history and positive Coombs' test, suggestive of autoimmune hemolytic anemia.
The peripheral blood smear may show spherocytes.
Autoimmune hemolytic: may be associated with various tumors.
Coombs' test or micro- Coombs' test is positive in such cases. Platelet and WBC counts are normal.

Hemolytic anemia in a patient with a malignant lymphoproliferative disorder is likely to be of the


warm autoimmune type, caused by anti-red blood cell IgG antibodies. If prednisone therapy is
ineffective, splenectomy is usually indicated.

Autoimmune hemolytic disease and hereditary spherocytosis are both extravascular hemolytic
anemias. However, Autoimmune hemolytic disease is acquired and hereditary spherocytosis has an
autosomal dominant transmission. A negative family history and positive Coombs' test are thus
suggestive of autoimmune hemolytic disease, whereas a positive family history is more suggestive
of hereditary spherocytosis. The peripheral blood smear in both conditions may show spherocytes.

174
Hereditary spherocytosis (HS)
HS is an autosomal dominant trait and is the most common hereditary hemolytic anemia in white
population.
There is congenital RBC membrane defect (spectrin) in HS leading to increased RBC membrane
osmotic fragility resulting in extravascular hemolysis occurring only in the presence of spleen.

It is characterized by positive family history, splenomegaly, anemia, spherocytosis, jaundice and


can cause cholecystitis due to pigmented gallstones.

Chronic leg ulcers may complicate the disease.


Patients are complicated by episodes of aplastic crisis (in which erythropoiesis is suppressed and
hemolytic process continues), usually because of Parovirus B19 infection
Severe anemia may also occur due to decreased intake of folate deficiency.

Peripheral blood smear demonstrate spherocytes with polychromatophilia.


Normal or MCV, normal MCH, but the MCHC is generally greater than 36%.
Increased reticulocyte count, increased bilirubin, negative Coombs test.
The osmotic fragility test is the diagnostic test for hereditary spherocytosis.

The treatment for most patients involves supportive care with oral folic acid for life and blood
transfusion during periods of extreme anemia.
Splenectomy is considered if the patients have moderate to sphere spherocytosis, or are refractory to
medical management.
Studies have shown that the risk of sepsis is present up to 30 years and probably longer after
splenectomy. Current recommendations state that patients should receive anti-pneumococcal,
Haemophilus, and meningococcal vaccines several weeks before the operation and daily oral
penicillin prophylaxis for three to five years following splenectomy.

175
Paroxysmal nocturnal hemoglobinuria (PNH)
PNH is an acquired disorder of hematopoietic cells.
It is characterized by an intravascular hemolytic anemia, a hypercoagulable state, and bone
marrow aplasia.

A red cell membrane defect (in PIG-A) causes increased binding of complement to the red cell
leading to increased intravascular hemolysis resulting in marked anemia.
These cells are more susceptible to lysis in an acidic environment. Due to relative hypoventilation at
night, there is mild acidosis. So the hemolysis is more at night and hemosiderinuria and hematuria
is common in the first morning urine.
Hemolysis is typically paroxysmal and complement mediated.

PNH may cause pancytopenia, and should be considered in all patients when accompanied by
reticulocytosis and hemolytic anemia.
Loss of iron in the urine may result in iron deficiency anemia.
The other characteristic feature of PNH is unusually high incidence of venous thrombosis in the
hepatic and portal veins (Budd-Chiari syndrome).

Lab tests show LDH, bilirubin, reticulocyte count and haptoglobin levels.
Low Leukocyte Alkaline Phosphatase is seen in paroxysmal nocturnal hemoglobinuria (PNH).
Test specific to PNH are sugar-water test and the acidified-hemolysis (Ham) test, which determines
the increased susceptibility of cells to lysis by complement.
Decay Activating Factor (DAF) is diminished in PNH.
Bone marrow examination shows hypocellular marrow.

Flow cytometry is screening and confirmatory test, it is simple and has high sensitivity and
specificity. The expression of the GPI-anchored proteins CD55 and CD59 can be analyzed using
monoclonal antibodies and flow cytometry.

Paroxysmal nocturnal hemoglobinuria (PNH) should be considered in the following situations:


1) Pancytopenia accompanied by hemolytic anemia (increased reticulocyte count and LDH and low
haptoglobin levels)
2) Recurrent thrombosis at unusual sites e.g., portal vein thrombosis or Budd Chiari syndrome.

Low Leukocyte Alkaline Phosphatase is seen in CML, hypophosphatemia and paroxysmal


nocturnal hemoglobinuria (PNH).

176
Glucose-6-phosphate dehydrogenase (G6PD) deficiency
It is an X-linked disorder, so there is typically a positive family history (common in African
Americans) and it is the most common enzymatic disorder of red blood cells in humans.

G6PD is an enzyme involved in creating NADPH, a cofactor required for creating glutathione and
prevent oxidation of hemoglobin.
Without G6PD, hemoglobin becomes oxidized and denatures into Heinz bodies.
Denatured hemoglobin disrupts red blood cell (RBC) membranes and causes hemolysis.
Episodes of hemolysis occur only due to oxidative stress from infection, fava beans or drugs
(primaquine, nitrofurantoin or sulfa drugs like sulphamethoxazole).

Variants of G6PD deficiency are G6PD A- (moderate enzyme deficiency) and G6PD Mediterranean
(severe enzyme deficiency), where hemolysis is precipitated by infection, drugs or fava beans.

The typical peripheral blood smear reveals bite cells and Heinz bodies.
Platelet and WBC counts are normal.
G6PD levels are often normal during the hemolytic episode.

Pyruvate kinase deficiency


Pyruvate kinase deficiency can also lead to a similar clinical picture of hemolytic anemia; however,
the hemolysis in such cases is not precipitated by sulfa drugs.
Furthermore, the typical peripheral smear does not include bite cells.

Aplastic anemia
Idiopathic aplastic anemia is an acquired disease that results in pancytopenia.
It may be due to chemicals (e.g., benzene, phenylbutazone), drugs (e.g., chloramphenicol,
sulfonamides), infectious agents (e.g., viral hepatitis) or ionizing radiation.
The peripheral blood smear does not show any abnormal morphology of cells.
Red blood cells (RBCs) are normocytic or macrocytic.
There is neither hemolysis of RBCs nor splenomegaly.
Reticulocyte count will be very low.
Aplastic anemia shows hypoplastic fat-filled marrow with no abnormal cells.
Bone marrow transplantation is the definitive treatment.

177
TTP-HUS
Unexplained hemolytic anemia, and thrombocytopenia in a patient with renal failure and
neurologic symptoms should raise strong suspicious for TTP-HUS.
Hemolytic uremic syndrome (HUS) and TTP come under a spectrum of diseases.
TTP-HUS is thought to be due to deficiency or autoantibody against a specific von Willbrand factor-
cleaving protease (ADAMTS-13) accumulation of large von Willbrand factor multimers and
platelet aggregation.
If the patient has more neurologic symptoms and less of renal failure, it is considered TTP.
If the patient has significant renal failure and less neurologic symptoms it is considered HUS.
Without prompt institution of appropriate therapy, TTP-HUS proves lethal in 80% of patients.
Both conditions are very serious and require emergent plasmapheresis (plasma exchange).
Rate of recovery usually defined as normalization of platelet count and LDH levels.
Renal function impairment and peripheral blood smear schistocytes may persist for several weeks
following clinical recovery.

Schistocytes (helmet cells) are fragmented erythrocytes. They are found in micoangipathic
hemolytic anemias (TTP, HUS & DIC) and due to RBC destruction by artificial heart valves or
severely calcified aortic valves (macrovascular traumatic hemolysis).

Thrombotic Thrombocytopenic Purpura (TTP)


TTP is a serious disorder, which presents with the following classical pentad:
1) Severe thrombocytopenia
2) Microangiopathic hemolytic anemia (RBC fragments)
3) Renal failure
4) Fluctuating neurological signs
5) Fever

HIV increases the risk for TTP.


The patients with TTP generally presents with fever, pallor, petechial, and confusion.
CBC: Anemia, thrombocytopenia.
Normal PT/PTT, bleeding time.
LDH, indirect bilirubin, Reticulocyte count (due to hemolysis), Coombs test \.
Blood smear shows schistocytes, helmet cells and RBC fragments.
Dipyridamole may help to treat TTP by preventing platelet aggregation.

178
Hemolytic uremic syndrome (HUS)
HUS is a serious disorder, which presents with the following classical triad:
1) Thrombocytopenia
2) Microangiopathic hemolytic anemia (RBC fragments)
3) Renal failure (BUN and creatinine levels are markedly elevated)

Hemolytic uremic syndrome (HUS) is typically a disease of young children.


HUS often follows mild viral illness (upper respiratory infection) or gastroenteritis (bloody
diarrhea) caused by E. coli O157:H7, Shigella, Salmonella, Yersinia or Campylobacter. In the past,
E. coli O157:H7 has been transmitted via the intake of undercooked hamburger meat from fast food
chains.
CBC: Anemia, thrombocytopenia.
Normal PT/PTT.
LDH, indirect bilirubin, Reticulocyte count (due to hemolysis), Coombs test \.
Blood smear shows schistocytes and giant cells.
Plasmapheresis is indicated in patients with hemolytic uremic syndrome.

ACUTE LEUKEMIA

Acute lymphoblastic leukemia (ALL)


Acute lymphoblastic leukemia (ALL) is the predominant type of leukemia in children from ages 2-
10 years. It is common in Down syndrome.
Approximately 30-50% of patients present with infections, and about half present with
lymphadenopathy and splenomegaly.
Lymphoblasts are typically seen on the peripheral smear.
Varying degrees of anemia, neutropenia, and thrombocytopenia have been noted.
The presence of more than 25% lymphoblasts in the bone marrow is diagnostic.
ALL is characterized by the presence of common ALL antigen (CALLA) and terminal
deoxynucleotidyltransferase (TdT).
Lymphoblasts lack peroxidase positive granules but often contain cytoplasmic aggregates of
periodic acid Schiff (PAS) positive material.

179
Acute myeloid leukemia (AML)
Acute myeloid leukemia (AML) is the predominant type of leukemia in adults.
In acute myelocytic leukemia (AML), the predominant cells are of myeloid origin.
AML is characterized by the presence of Auer rods (M3), myeloperoxidase & esterase.

Acute monocytic (FAB M5) leukemia


FAB M5 comprises less than 15% of all leukemias.
The onset is dramatic, with most chief complaints being headaches, fever, weight loss, and bleeding
from the gums or nose.
Signs include gingival hyperplasia and occasional skin lesions.
Peripheral smear reveals leukocytosis with a high proportion of blast forms (monoblasts,
promonocytes and Monocytes).
Monoblasts lack Auer rods, and are peroxidase negative and nonspecific esterase positive.
A positive alpha-naphthyl esterase test is characteristic.

Acute promyelocytic leukemia


APML is characterized by many hypergranular promyelocytes.
Each cell has many Auer rods.
There is a high incidence of DIC in these patients.

180
Chronic Myelogenous leukemia (CML)
CML is a disorder characterized by massive over production of myeloid cells.
CML is one of the myeloproliferative syndromes seen normally in adults (mostly age of 50).
It is characterized by leukocytosis, Basophilia, anemia and marked splenomegaly.

Night sweats, fatigue, and fever associated with increased metabolism due to granulocytic cell turn
over may occur.
Abdominal pain from massive enlargement of the spleen is common.
Lymphadenopathy is not a feature of CML.

Peripheral blood smear shows leukocytosis (predominantly of mature granulocytic forms such as
neutrophils and band forms with left sided myeloid series).
Presence of Philadelphia chromosome (9 and 22) and the Leukocyte Alkaline Phosphatase
makes the CML more likely than the leukemoid reaction.
Examination of a bone marrow biopsy specimen reveals hypercellularity with prominent
granulocytic hyperplasia.

Imatinib has changed the prognosis of patients with CML. It is tyrosine kinase inhibitor and works
by blocking signals within cancer cells and preventing a series of chemical reactions that cause the
cancer cells to grow and divide.

Elevated Leukocyte Alkaline Phosphatase is characteristic of Leukemoid Reaction.


Low Leukocyte Alkaline Phosphatase is seen in CML, hypophosphatemia and paroxysmal
nocturnal hemoglobinuria (PNH).

Leukemoid reaction
Leukemoid reaction is marked increase in leucocytes due to severe infection or inflammation.
Leukocyte Alkaline Phosphatase score typical for this condition.

181
Chronic lymphocytic leukemia (CLL)
The disease is seen mostly in older patients.
Patients are often asymptomatic, and are diagnosed when lymphocytosis is detected incidentally.
When patients are symptomatic, it is usually due to lymphadenopathy.
Patients may develop hypogammaglobulinemia and become more susceptible to infection.

The peripheral blood film typically reveals lymphocytosis with small, mature-appearing
lymphocytes.
"Smudge cells" are characteristic of chronic lymphocytic leukemia (CLL).
Other characteristic findings include splenomegaly, anemia and thrombocytopenia.
Lymph node biopsy confirms the diagnosis.

The presence of thrombocytopenia indicates a poor prognosis.


The staging system is directly related to the prognosis, as shown in the table below:

Stage Clinical Feature Prognosis


0 Lymphocytosis only Good
I Lymphocytosis + adenopathy Fair
II Splenomegaly Fair
III Anemia Intermediate
IV Thrombocytopenia Poor

Hairy cell leukemia


Hairy cell leukemia is a type of B-lymphocyte derived leukemia (leukemic reticuloendotheliosis).
Most patients present with pancytopenia and splenomegaly.
10-20% of patients can have leukocytosis.

Hairy cell leukemia is characterized by lymphocytes with fine, hair-like irregular projections (fine,
irregular cytoplasmic projections) and a tartrate resistant acid phosphates (TRAP) stain.
CD11c marker is relatively specific for hairy cell leukemia.
The bone marrow may become fibrotic; thus leading to dry taps.

Cladribine (purine analogue) is the drug of choice for patients with hairy cell leukemia.

182
Multiple myeloma
Multiple myeloma is a form of plasma cell leukemia, caused by the proliferation of a single
transformed plasma cell usually producing IgG.
MM presents in old age.

Classic tetrad of multiple myeloma CRAB: Calcium (hypercalcemia), Renal impairment (Urine
monoclonal proteins), Anemia (normocytic), and Bones (bone pain, bone lytic lesions, fractures).
Back pain is the most common manifestation.

Hypercalcemia is a common finding in a patient with multiple myeloma. Hypercalcemia may cause
severe constipation, anorexia, weakness, renal tubular dysfunction, and neurologic symptoms.
IgG antibodies or paraproteins, produced by the myeloma cells can collect in the glomeruli, causing
renal failure or myeloma kidney.
There is increased risk for infection due to total decrease in the functional antibodies and
leukopenia secondary to bone marrow crowiding with maligant plasma cells.

Normally, there is a 3-4 g/dL difference separating the serum total protein and albumin
concentration, the gap is greater in patients with multiple myeloma.
Laboratory findings include anemia, increased ESR (often more than 100), and Bence Jones
proteins in the urine (monoclonal proteins).
RBC morphology is significant for a rouleaux appearance.
X-rays reveal lytic lesions, especially in the axial skeleton, as well as osteoporosis and fractures.
Bone marrow biopsy shows 1020% plasma cells (normal is < 5%).

Serum immunoelectrophoresis demonstrates abnormal M-spike due to excess IgG production.

The complete work-up consists of CBC with differential and morphology, serum electrolytes,
kidney and liver screening profiles, skeletal survey, serum electrophoresis, and bone marrow biopsy.
Complications include renal failure, hypercalcemia, and hyperviscosity syndrome.

Bisphosphonates (e.g. Zoledronic acid) are the drugs of choice for mild to moderate hypercalcemia
due to malignancy.

183
Waldenstroms Macroglobulinemia (WM)
It is a rare chronic plasma cell neoplasm.
It is characterized by abnormal plasma cells which multiply out-of-control and invade the bone
marrow, lymph nodes and spleen.
Typically, there is production of excessive amounts of IgM antibodies in the blood that causes
hyperviscosity (thickening) of the blood.

Major Signs & Symptoms are:


1) Increased size of spleen, liver, lymph node
2) Tiredness, usually due to anemia(too few RBCs)
3) Tendency to bleed and bruise easily (little platelets)
4) Night sweats
5) Headache and dizziness
6) Visual problems (engorgement of the retinal veins caused by hyperviscosity of the blood)
7) Pain and numbness in extremities.

In multiple Myeloma, there is IgA/G not IgM. Also, No hyperviscosity like in WM.

Monoclenal gammopathy of undetermined significance (MGUS)


The characteristic laboratory findings of this condition include an M component (IgA, or IgG, or
IgM) < 3000 mg/dL, and fewer than 10 % plasma cells in the bone marrow.
Patients are initially asymptomatic, and may remain so for years.
In the early course of their disease, patients do not manifest with any lytic lesions, anemia,
hypercalcemia, or renal insufficiency (these are typically seen in patients with multiple myeloma).
The reason behind this is that the levels of M-proteins initially remain stable for years.
Nevertheless, patients with MGUS have a 25 % risk of development of a serious disease, which is
usually a late event. In addition, the evolution from MGUS to multiple myeloma, which occurs at a
rate of 1 %/yr, may be abrupt.
For these reasons, patients initially do not require any treatment, but proper education and
counseling are necessary.
Regular follow-up visits are recommended, and all patients are instructed to promptly obtain
medical evaluation if any clinical symptoms occur.

184
Hodgkin's disease
Neoplastic transformation of lymphocytes.
It present with enlarged, painless, rubbery lymph nodes (mostly cervical and supraclavicular).
Pruritus is common.
Patients with Hodgkin's lymphoma usually have normal blood smears (eosinophilia is common).
Reed Sternberg cells Owls eye cells are seen in the lymph node biopsy.

Hypercalcemia in cases of Hodgkin's disease is almost always produced by calcitriol.


Nephrotic syndrome is a well-known complication of Hodgkin's lymphoma, and is usually caused
by minimal change disease.

Non-Hodgkin lymphoma
Non-Hodgkin lymphoma is a complication of Sjogren's syndrome.
CHOP regimen is used for non-Hodgkin's lymphoma.
Anemia of lymphoproliferative disorders is due to bone marrow infiltration with cancerous cells
replacing RBC progenitor cells.

Burkitt lymphoma
Burkitt lymphoma is a neoplasm of mature B cells.
It is associated with the Epstein-Barr virus infection.
Most patients present with either a mass involving the mandible or abdominal viscera.
High mitotic index is typical.
Histological examination shows characteristic "starry sky appearance".
It is a very aggressive tumor but responds well to the high dose chemotherapy.

Tumors with high cell turnovers are frequently associated with tumor lysis syndrome.
Tumor lysis syndrome is characterized by hyperphosphatemia, hypocalcemia, hyperkalemia, and
hyperuricemia.
The tumors most often associated with this syndrome are the poorly differentiated lymphomas (e.g.,
Burkitt's lymphoma) and the leukemias (particularly ALL and less often AML). Although the
introduction of allopurinol has reduced the acute urate nephropathy to a large extent, the possibility
of tumor lysis syndrome remains.

185
Mucosa-associated lymphoid tissue (MALT) lymphoma
There is an important role for H. pylori infection in the pathogenesis of low-grade gastric MALT
lymphoma.
Such lymphomas regress after the eradication of H. pylori using antibiotic therapy.
Antibiotic therapy against H. pylori is the most accepted and recommended management of gastric
MALT lymphoma without any metastasis.
If antibiotic therapy fails to regress the lymphoma, chemotherapy is used (CHOP regimen or
CHOP+Bleomycin).

Idiopathic thrombocytopenia purpura (ITP)


ITP is a diagnosis of exclusion
It is characterized by thrombocytopenia and increased bleeding time.
All other cell lines, PT, and PTT will be normal.
Bone marrow biopsy shows megakaryocytes.

Autoimmune platelet destruction is a common cause of thrombocytopenia and should be


suspected in patients with ecchmoses, petechiae, and mucosal bleeding without signs or symptoms
of TTP/HUS, pancytopenia, bone marrow failure or splenomegaly.

ITP in children is typically acute and self-limited.


ITP in adults tend to be of insidious and chronic course, and treated with immunosuppression by
steroids.

Thrombocytopenia can be an initial manifestation of SLE, CMV, hepatitis or HIV.


Isolated thrombocytopenia may be a presenting feature of SLE, especially if the patient is a young
female, thus we need to use antinuclear antibody test to diagnose SLE.

Von Willebrand's disease


Von Willebrand's disease is the most frequently diagnosed inherited bleeding disorder in adults.
Labs would show an increase in bleeding time with an increase in PTT.
PT will be normal. Platelet count is normal.
vWF Level is decreased in Von Willebrand disease.
DDAVP increases the release of factor VIII:von Willebrand factor multimers from endothelial
storage sites.

186
Bernard-Soulier syndrome
Bernard-Soulier syndrome is an autosomal recessive rare bleeding disorder.
It is characterized by thrombocytopenia, giant platelets, and a bleeding tendency, which is typically
greater than expected bleeding for the degree of thrombocytopenia.
There is decrease or abnormality in the membrane glycoprotein (GP Ib) so the platelets can not
adhere to the endothelium.
Platelets from these patients do not aggregate in the presence of normal vWF and ristocetin because
of the decrease or abnormality in the membrane glycoprotein (GP Ib).

Hemophilia
Recurrent hemarthroses in patients with coagulopathies lead to a joint injury called 'hemophilic
arthropathy'.
Spontaneous hemarthrosis raises the suspicion for hemophilia, for which factor VIII assay is
diagnostic.
Prolonged PTT, normal prothrombin time, normal bleeding time, normal fibrinogen level and low
serum factor VIII activity are the typical lab findings.
The standard treatment for hemophilia is to replace the factor VIII.
However, mild hemophilia may be treated with desmopressin (DDAVP), which causes release of
factor VIII from the endothelial cells.

Vitamin K deficiency
The body gets vitamin K from two sources: exogenous from the food (absorbed in small intestine)
and endogenous from the bacterial production of vitamin K in the intestine (inhibited by antibiotics).
A 30-day store of vitamin K is stored in normal liver; however, an acutely sick person (e.g.
alcoholic, hepatocellular carcinoma, liver cirrhosis etc) will be vitamin K-deficient in 7-10 days.
Cystic fibrosis exocrine dysfunction of the pancreas malabsorption of fat soluble vitamins
deficiency of vitamin K deficiency of Factors II, VII, IX and X as well as protein C and S
prolonged PT.

Vitamin K deficiency leads to fall in all the prothrombin complex proteins (Factors II, VII, IX, X
and protein C and S) PT/INR levels, followed by PPT ( PT > PPT ).

187
Regardless of the cause, the first step in such setting (without an evidence of active bleeding) is
empiric vitamin K administration to correct any underlying coagulopathy.
Parenteral administration of vitamin K rapidly restores the stores in 8-12 hours.
Fresh frozen plasma (FFP) is indicated if the patient is actively bleeding.

Elevated PT/INR levels due to liver disease are not corrected by giving vitamin K; however, vitamin
K is still given first since such patients always have an underlying vitamin K deficiency due to
several co-morbidities.

Vitamin K may cause hyperbilirubinemia in premature infants.


Vitamin K reverses the action of warfarin, but takes 8-12 hours to be effective.
Fresh frozen plasma is indicated if the patient is actively bleeding, or if the patient needs immediate
surgery or an invasive procedure.

Liver disease
In chronic liver disease, factor VII is the first factor to be depleted.
Prolonged PT, and PTT.
Low platelets count.
Elevated PT/INR levels due to liver disease are not corrected by giving vitamin K; however, vitamin
K is still given first since such patients always have an underlying vitamin K deficiency due to
several co-morbidities.
Fresh frozen plasma is indicated if the patient is actively bleeding.

Chronic renal failure


Platelet dysfunction is the most common cause of abnormal hemostasis in patients with CRF.
PT, PTT, and platelet count are normal. BT is prolonged.
DDAVP is usually the treatment of choice, if needed. DDAVP increases the release of factor
VIII:von Willebrand factor multimers from endothelial storage sites.
Platelet transfusion is not indicated because the transfused platelets quickly become inactive.

188
Glanzmann's thrombasthenia
It is a rare bleeding disorder
It is an autosomal recessive disorder that results in deficient glycoproteins IIb-IIIa complex so
fibrinogen will not cross-connect.
The patient presents with increased bleeding episodes for some time.
Platelet counts may be normal but on the peripheral blood stream, platelets remain isolated and do
not exhibit clumping that is normally seen.
The bleeding time (BT) is a measure of the interaction of platelets with the blood vessel wall.
Bleeding time is markedly increased and clot retraction is decreased.
The diagnosis lies in qualitative platelet tests.
Epinephrine, collagen and thrombin fail to induce aggregation.
There is absence of primary wave of aggregation in response to ADP however platelet aggregation
studies with ristocetin are normal.
Von Willebrand factor is also normal.

Disseminated intravascular coagulation (DIC)


Causes: sepsis, transfusion reaction, neoplasia, trauma, and obstetric complications.
Physical examination: Bleeding: Venipuncture sites, epistaxis. Thrombosis: Digital gangrene,
hypotension.
CBC: Anemia, thrombocytopenia.
PT/PTT, bleeding time.
D-dimer, fibrin split products, fibrinogen.
Blood smear shows schistocytes.
Treat the underlying condition; transfuse with platelets and cryoprecipitate.

CHRONIC DIC
Migratory thrombophlebitis and atypical venous thromboses are suggestive of chronic DIC, which
is most likely due to cancer.
The typical laboratory findings of chronic DIC include mild prolongation of PT, low fibrinogen
levels and positive Fibrin split products (FSP).
The most common causes are malignancies of the lung, pancreas, stomach and prostate.

189
CT of the chest, abdomen and pelvis are thus indicated to identify and further evaluate the
underlying malignancy, along with age-appropriate cancer screening (e.g., digital rectal exam,
mammography, colonoscopy).

Deep vein thrombosis (DVT)


Deep vein thrombosis occurs when Virchows triad of stasis, endothelial injury, and
hypercoabulability are present.
Major surgery is a significant risk factor for DVTs.
Patients with DVT should be treated acutely with a heparin product and warfarin for several
months (with a goal INR of 2-3).
The goal of therapy is to prevent extension of the clot and development of future clots, not lysis of
the clot already present.
Whereas superficial thromboses do not need anticoagulation.

DVT in the setting of increased homocysteine levels


Homocysteine is a highly reactive amino acid. homocysteine levels predisposes to venous
thrombosis as well as atherosclerosis.
Homocysteine can be metabolized to cysteine or demethylated to form methionine. If either of these
pathways is disrupted by an enzyme or co-factor deficiency, elevation of homocysteine level occurs.
Homocysteine is catalyzed by cystathionine -synthase (using Pyridoxine as a cofactor) to
form cysteine.
Homocysteine is catalyzed by methylenetetrahydrofolate reductase and methionine synthase
(with folic acid and cobalamin as essential cofactors) to form methionine.
Independent of the underlying cause, homocysteine levels can be normalized by administration of
pyridoxine (B6) and folic acid.
Vitamin B12 should be added if a B12 deficiency is documented.
Thus, DVT treatment should include an attempt to correct the homocysteine level.

190
Antiphospholipid antibody syndrome
It is characterized by recurrent arterial or venous thrombosis, or recurrent fetal losses in the
presence of antiphospholipid antibodies.
There are three types of antiphospholipid antibodies. The first type is responsible for false-positive
syphilis serology. The second type is lupus anticoagulant, and it falsely elevates the APTT level.
The third type is anticardiolipin antibody.
Antiphospholipid antibody syndrome may occur as a primary condition, or it may be associated with
other autoimmune disorders such as SLE.

In patients with SLE and venous thromboembolic disease, the lupus anticoagulant, or anti-
phospholipid antibody syndrome, must be suspected.
Lupus anticoagulant is an IgM or IgG immunoglobulin prolongs PTT.

This syndrome places a pregnant patient at increased risk for a first or second trimester abortion,
and the use of heparin and aspirin reduces this risk.
Patients with arterial or venous thrombosis or fetal losses due to antiphospholipid antibody
syndrome is immediately treated with anticoagulant therapy.
Acute thrombosis is treated with heparin, and factor anti-X-a activity is measured, since APTT is
not reliable in such cases. Subsequently, such patient is anticoagulated with warfarin (unless the
patient is pregnant).

SLE
Pancytopenia (decreased RBCs, WBCs and Platelets) is common in patients with SLE. It occurs due
to formation of autoantibodies (warm IgG antibodies) against blood cells ( a form of type II
hypersensitivity reaction), leading to peripheral destruction of blood cells.

Warfarin
Warfarin is a vitamin K antagonist used for anticoagulation in numerous clinical settings.
Foods rich in vitamin K (e.g. dark green vegetables) will decrease its efficacy.
Vitamin supplements, alcohol, vitamin E, garlic, ginkgo biloba, ginseng, St. Johns wort, and several
types of antibiotics increase its activity.

191
Warfarin-induced skin necrosis
Warfarin is a vitamin K antagonist, inhibits the production of vitamin K-depebdant clotting factors
II, VII, IX, & X (half-lives around 60 hours). It also inhibits the blood natural anticoagulants,
Protein C and S (half-live is only 9 hours).
Therefore, warfarin in the first days of treatment can lead to hypercoagulable state (Protein C and S
deficiency) and placing the patient at risk of thrombus formation and skin necrosis.
Warfarin-induced skin necrosis presents with pain followed by bullae formation and skin necrosis.
The breasts, buttocks, thighs, and abdomen are commonly involved.
Vitamin K should be promptly administered in the early stages of the lesion, and warfarin is
discontinued if the lesion progresses.
Heparin should be used to maintain anticoagulation until the necrotic lesions heal.
Few patients require skin grafting.

Heparin
Elevated PTT is a therapeutic effect of heparin.

Thrombocytopenia (which manifests as prolonged bleeding from the venapuncture site), is


associated with paradoxical hypercoagulation (arterial/venous thrombosis), which may manifest as
an acute ischemic stroke, in patients receiving heparin therapy is highly suggestive of heparin-
induced thrombocytopenia (HIT) most likely caused by unfractioned heparin.

Patients on heparin should be regularly followed up with platelet counts.

There are two types of HIT:


HIT I occurs in the first two days of heparin therapy due to the direct effect of heparin on platelet
activation.
HIT II occurs within 4 to 10 days of heparin therapy and it is an autoimmune disorder
characterized by the formation of antibodies against heparin-platelet factor 4 complex and
resolution within 4 to 5 days of discontinuation of heparin.

The first step in management of patient with HIT is immediate cessation of all exposure to heparin,
including low-molecular weight heparin (LMWH).
Subsequently, an alternative means of anticoagulation is usually given; currently the two
recommended alternatives are danaparoid and direct thrombin inhibitor (e.g. lepirudin,
argatroban).

192
Blood transfusion
Patients who received the equivalent of more than one blood volume of blood transfusion or packed
RBCs over 24 hours may develop elevated plasma levels of citrate (a substance added to stored
blood). Citrates cheleates calcium and magnesium and may reduce their plasma levels, causing
paresthesias.

Calcium gluconate infusion is employed in rare cases of severe hypocalcaemia following massive
blood transfusion.
Warming the blood is recommended only during rapid massive transfusion to prevent hypothermia.

Individuals who received blood transfusions before 1992 should be screened for hepatitis C.
Those who received blood transfusions before 1986 should be screened for hepatitis B.

IgA deficiency significantly increases the risk of developing anaphylactic reaction to transfused
blood products. The risk of anaphylaxis may be reduced in susceptible patients by providing IgA-
deficient blood products or by performing additional washes on red blood cells or platelet products.
Anaphylaxis typically presents within minutes of starting transfusion and manifests as wheezing,
respiratory distress, tachycardia and hypotension. Similar mechanism seen in patients with
ahaptoglobinemia or anti-haptoglobin antibodies.

REACTION TO CYTOKINES
It is the most common transfusion reaction leads to febrile non hemolytic reactions (fever, chills,
malaise).
It is usually responds to NSAIDs and acetaminophen.
During blood storage, leukocytes release cytokines, which when transfused cause transient fever,
chills, and malaise.
Therefore, leukocyte depletion techniques (although not commonly employed due to high cost) can
reduce the probability of febrile transfusion reaction. These techniques are cell washing, use of
frozen deglycerolized red cells, use of leukocyte depletion RBC filters, etc.

193
ACUTE HEMOLYTIC TRANSFUSION REACTION
In which donor erythrocytes are rapidly destroyed by preformed recipient antibodies. This is
typically due to ABO mismatching.
It is classically presents as fever, chills, flank pain, and hemoglobinuria.
Diagnosis is made by positive direct antiglobulin test (Coombs), plasma free hemoglobin showing
pink plasma with hemoglobin concentration > 25 mg/dl, urine analysis will also show the presence
of hemoglobin, and repeated typing & cross matching revealing a mismatch.
Complications: DIC, ARF, cardiovascular collapse (shock) and even death.
Stop the transfusion, & supportive treatment (e.g. NSAIDs, acetaminophen, IV fluids ).

FRESH FROZEN PLASMA


FFP is the therapeutic agent of choice for coagulopathy in patients with liver failure.
Warfarin treated patients should be given fresh frozen plasma instead of vitamin K when emergency
surgical procedure is to be performed.
Patients who develop serious bleeding (e.g., intracerebral hemorrhage) due to excess anticoagulation
with warfarin should be given fresh frozen plasma (FFP) for the rapid reversal of anticoagulation.
Vitamin K reverses the action of warfarin, but takes 8-12 hours to be effective.
Patients with prosthetic valves usually tolerate cessation of oral anticoagulant therapy for about a
week without significant increase of the risk of thrombosis. So, reversing with FFP is a very good
option. It is difficult to anticoagulate after the operation if you give vitamin K in these people and it
takes several days to achieve therapeutic INR.

Carbon monoxide poisoning


Always consider carbon monoxide poisoning in patients with environmental risk factors (e.g.
enclosed space) who present with nausea, headache and dizziness as well as polycythemia.
CO binds hemoglobin decrease bloods oxygen-carrying capacity decrease oxygen delivery
body try to compensate by increasing red blood cell production polycythemia.

194
Obstructive sleep apnea (OSA)
Patients tend to be overweight or obese and have excessive daytime sleeping, snoring, morning
headaches, impotence and arterial hypertension.
Recurrent transient obstruction of the upper airway due to pharyngeal collapse during sleeping
short-term hypoxia stimulate the kidney to increase erythropoietin production creation of more
red blood cells polycythemia.
Treatment OSA causes the polycythemia to improve.

Polycythemia Vera (PV)


PV is a myeloproliferative Disease characterized by increased production of all three blood lines,
especially RBCs (increase in RBC mass and total blood volume RDW).

A typical patient is an old plethoric male who may complain of pruritus after hot bathing (due to
histamine release from an increased number of circulating basophils).

Up to 40% of patients with PCV suffer from gout.


Myeloproliferative Disease (MPD) catabolism and turnover of purines over-production of
uric acid serum uric acid precipitation of gouty arthritis.

Symptoms such as headache, dizziness, and paresthesias are frequently related to hyperviscosity.
They also have granulocytosis and thrombocytosis on peripheral blood smear, and splenomegaly.
Both thrombosis and bleeding can occur due to elevated platelet count and impaired platelet
function.
Reversible moderate hypertension frequently occurs as a result of expanded blood volume.

Bone marrow is virtually always hypercellular.


There is an elevated leukocyte alkaline phosphatase, normal oxygen saturation, and low
erythropoetin level.
In polycythemia ESR will be very low or close to '0'
Absence of measurable erythropoietin in Urine is an important diagnostic feature of PV.

195
Hereditary telangiectasia (Osler-Weber-Rendu syndrome)
It is an autosomal dominant disorder characterized by diffuse telangiectasias (ruby-colored papules
that blanch partially with pressure), recurrent epistaxis, and widespread AV malformations
(AVMs).
AVMs tend to occur in the mucous membranes, skin and gastrointestinal tract, but may be also
present in the liver, brain and lung.
AVMs in the lungs may present with hemoptysis and can shunt blood from the right to the left side
of the heart, causing chronic hypoxemia (digital clubbing) and reactive polycythemia.

Anabolic steroids
Anabolic steroids are commonly abused by athletes in an effort to improve performance.
Side effects include: suppressed endogenous testicular function (resulting in infertility),
gynecomastia, erythrocytosis ( Hematocrit), hepatotoxicity, cardiac disease, dyslipidemia
(lowered HDL and elevated LDL), increased coagulation, and premature epiphseal fusion (which
shunts growth).
Virilization is often seen in females who abuse steroids.

Infectious mononucleosis (IM)


IM is an acute, benign and self-limiting lymphoproliferative condition caused by Epstein-Bar Virus
(EBV). EBV is transmitted primarily by close contact with infectious oropharyngeal secretions.
Clinical manifestations include extreme fatigue, malaise, sore throat, fever, and a generalized
maculopapular rash.
Posterior cervical lymphadenopathy and palatal petechiae can be present.
Splenomegaly is also common.
Contact sports should be avoided to prevent the chances of splenic rupture. When rupture occurs,
the mortality is significant.
Hematological studies reveal leukocytosis with variant lymphocytes (atypical lymphocytes).
Heterophile antibodies are very sensitive and specific, but may be negative early in the illness.
Repeating the test may be helpful. For this reason, a negative antibody test does not exclude the
diagnosis of IM.

196
Senile purpura
Senile purpura is an ecchymotic lesion that occurs in areas susceptible to trauma in the elderly.
Senile purpura occurs because of perivascular connective tissue atrophy as people age.
The most commonly affected areas are dorsum of the hand and forearms.
The lesions develop rapidly and resolve over several days, typically leaving a brownish
discoloration from hemosiderin deposition.
Senile purpura is not a dangerous condition and requires no further investigation.

197
Ewing's sarcoma
It is another frequent tumor of adolescent population (usually <10yr age group).
The most common site is metaphysis and diaphysis of the femur.
The tumor is very aggressive and metastasize early to the lungs & lymph nodes.
Ewings sarcoma usually presents with local joint pain & swelling (usually around the knee).
Patients may initially be diagnosed with osteomyelitis, based on the misleading presentation of
intermittent fever, malaise, leucocytosis, anemia, and increased ESR.
X-ray shows osteolytic lesion with onion-skin appearance often followed by moth-eaten
appearance.

Osteosarcoma
It is the most common malignant bone tumor in youth.
It is more common in femur (distal end), humerus (proximal end) and shin (proximal end).
The most common symptom initially is pain.
X-ray shows Codmans triangle and sun-burst appearance.

Giant cell tumor or Osteoclastoma


It is a tumor of adults and it involves the epiphysis of the distal femur or proximal tibia.
It is benign but locally aggressive. It frequently recurs even after local curettage.
Radiology characteristically shows a soap-bubble appearance.
The best way to handle this tumor is to recognize it and leave it to experts (know how to recognize
bone tumors and refer them to appropriate experts).

198
LUNG CANCER
As the #1 killer in the USA, it is very important to know the different manifestations of lung
carcinoma. The first step is being able to recognize the acute presentation in the context of an at risk
patient and to associate it with malignancy as part of your differential.
Bone scan will be abnormal before the changes manifest in x-rays.

Until proven otherwise, suspect lung cancer in smokers who present with Horners syndrome (i.e.
partial ptosis, miosis & anhidrosis).
Simple chest x-ray should be the first test in patients with suspected lung cancer.
A hilar mass on the chest x-ray film of a smoker is most probably a lung cancer.
There is currently no data which proves that regular chest x-rays (CXRs) can prevent death due to
lung cancer.

Any chronic smoker with hypertrophic osteoarthropathy should have a chest x-ray to rule out
malignancy. Hypertrophic osteoarthropathy is associated with chronic proliferative periostitis of the
long bones, clubbing, and synovitis. It is usually associated with squamous cell carcinoma and
adenocarcinoma of the lungs.

Solitary pulmonary nodule


Solitary pulmonary nodule defined as a 3cm or less coin-shaped lesion in the middle to lateral one
third of the lung surrounded by normal parenchyma. Most of them are benign.
Calcification of the nodules favors a benign lesion.
Popcorn calcification is a hamartoma while "bulls eye" is a granuloma.
In low risk patients (i.e. < 40 years and non smokers) a solitary pulmonary nodule is not a sign of
immediate alarm.
The best approach is to ask for an old x-ray.
If there is no change in it for the last 12 months it is considered benign. It is followed by a CXR
every 3 months, for the next 12 months, and if there is no growth or no symptoms it is left as such.

High-risk patients i.e. smokers, require a full investigation work up. It starts with a CT scan chest
just to get better picture of the lesion and its extent, followed by fine needle aspiration. If the lesion
is missed, an open-lung biopsy is an option.

199
Adenocarcinoma of the lung
Adenocarcinoma is the most common lung cancer, even in smokers.
It has the least association with smoking history.
It is usually located peripherally and consists of columnar cells growing along the septa.
It is present as a solitary nodule and may be detected incidentally.
It metastasize early to adrenals, bone and CNS.
It is typically associated with hypertrophic pulmonary osteoarthropathy.

Squamous cell carcinoma of the lung


Suspect squamous cell carcinoma of the lungs in a patient with significant smoking history (i.e.
more than 45 years), hypercalcemia (remember sCa++mous), and hilar mass.
Hypercalcemia results from the effect of parathormone-related protein (PTHrP).

Small cell carcinoma of the lung


Squamous cell carcinoma of the lung usually produces ACTH and causing SIADH (Eaton-Lambert
syndrome) and cushings syndrome.
Squamous cell carcinoma of the lung is the most common cause of venocaval obstruction
syndrome.

Breast cancer
Advanced age is probably the single most important risk factor for breast cancer. Thats why women
of more than 40-50 years of age would need regular annual mammograms.
Only 10% of women diagnosed with breast cancer have positive family history.

Tamoxifen has a mixed agonist and antagonist activity on estrogen receptors.


Tamoxifen (anti-estrogen) reduces the risk of breast cancer in patients who have an increased risk
of developing breast cancer.
Tamoxifen (estrogenic effect) increases the risk of endometrial cancer and venous thrombosis.

Tumor burden is the single most important prognostic consideration in the treatment of patients
with breast cancer. It is based on TNM staging.

200
Local therapy (e.g. resection of metastases, local irradiation) is rarely curative in patients with
metastatic breast cancer, but it can be tried in patients with a resectable metastatic focus without
signs of systemic involvement.

Paget's disease
Paget's disease presents as a persistent dermatitis of the nipple. The lesion is typically red, oozing,
crusted, and often unresponsive to topical steroids and antibiotics.
Biopsy reveals the characteristic microscopic changes. Biopsy shows malignant cells that invade the
epidermis and have abundant pale-staining cytoplasm surrounding a hyperchromatic nucleus with
prominent nucleoli.
In almost every case of Paget's disease, there is an underlying breast carcinoma. This is most
commonly an infiltrating ductal carcinoma, but ductal carcinoma in situ (DCIS) may occasionally
be present.

Fibrocystic disease
It is benign condition. Very common in pre-menopausal females.
Patients present with bilateral painful, rubbery, firm, mobile masses; who experiences more
tenderness during her menses.
Fibrocystic disease is treated with aspiration of the cyst, which should yield clear fluid and result in
the disappearance of the mass.
Afterwards, patients are typically observed for 4 to 6 weeks.

OVARIAN cancer
Patients affected: Postmenopausal, > 50 years of age.
Risk factors: Nulliparity, breast cancer, family history.
OCP use is protective.
History: Often asymptomatic, but may present with abdominal girth from ascites, GI and GU
complaints, thrombophlebitis, and lower abdominal pain/pressure.
Physical examination: Early; Pelvic exam may be normal, or may present with a palpable adnexal
mass and pedal edema. Late: Palpable mass, ascites.
Screening tests: None (routine ultrasound and, CA-125 are not cost-efficient and has no role in
decreasing mortality from ovarian cancer).

201
Cancer colon
There are currently five recommended options for colorectal cancer screening:
1) FOBT (fecal occult blood test).
2) Sigmoidoscopy
3) FOBT + Sigmoidoscopy
4) Double contrast barium enema.

FOBT is the most commonly used screening test followed by sigmoidoscopy.


All patients older than 50 years of age should have an annual FOBT. Two samples from each of
three consecutive stools should be examined.
A positive FOBT warrants further evaluation using colonoscopy.

When a questionable ulcer is visiualized by flexible sigmoidoscopy, a biopsy is recommended for


two purposes:
1) To rule out possible cancerous lesions, and
2) To distinguish ulcerative colitis from Crohns disease (as treatment is different).

Patients with ulcerative colitis or pancolitis should begin surveillance after eight years from having
the disease.

Turcot's Syndrome
Turcot's syndrome refers to an association between brain tumors (primarily medulloblastomas and
gliomas) and FAP (Familial Adenomatous polyposis) or HNPCC (Hereditary nonpolyposis
colorectal cancer).
The majority of FAP-associated brain tumors are medulloblastomas, but gliomas have also been
described.
Patients with HNPCC are prone to high-grade gliomas.
It is autosomal recessive and mainly occurs in teens.

Gardner's syndrome
Gardner's syndrome is autosomal dominant where colonic polyps are seen with prominent
extraintestinal lesions. These include desmoid tumors, sebaceous or epidermoid cysts, lipomas,
osteomas (especially of the mandible), supernumerary teeth, gastric polyps, and juvenile
nasopharyngeal angiofibromas.

202
Peutz-Jeghers syndrome
Peutz-Jeghers syndrome is an autosomal dominant inherited disorder characterized by intestinal
hamartomatous polyps in association with mucocutaneous melanocytic macules.

Lynch syndrome
Lynch syndrome is also known as hereditary non-polyposis colorectal cancer (HNPCC).
To aid its diagnosis, criteria known as the Amsterdam Criteria I is used:
1) At least three relatives with colorectal cancer, one of whom must be a first degree relative of the
other two
2) Involvement of two or more generations
3) At least one case diagnosed before age 50
4) Familial adenomatous polyposis has been excluded

HNPCC is also classically divided into two subgroups on clinical grounds:


1) Hereditary site specific colon cancer (Lynch syndrome I)
2) Cancer family syndrome or (Lynch syndrome II: is distinctly associated with a high risk of
extracolonic tumors, the most common of which is endometrial carcinoma, which develops in up
to 43 % of females in affected families).

Familial colonic polyposis (FAP)


FAP is an autosomal dominant disease caused by mutations in the adenomatous polyposis coli
(APC) gene.
It accounts for less than 1 % of the total colon cancer risk in the United States.
A patient with FAP and hundreds of adenomas in the colon has a 100% risk of cancer, if he is not
treated appropriately with a procto-colectomy.

203
Cancer stomach
The only malignancy that have decreased universally over the last several decades is stomach
cancer. The reason for decrease in stomach cancer is not known and is not related to advances in
medicine.
Acanthosis nigricans might be a manifestation of occult gastrointestinal malignancy. Acanthosis
nigricans is dark, thick area of the skin, and the skin is velvety, most commonly in body folds.
Acanthosis nigricans most often associated with being overweight.

ESOPHAGEAL CANCER
The two major histological types are squamous cell carcinoma (SCC) and adenocarcinoma.
The risk factors for adenocarcinoma include Barrett's esophagus, chronic GERD, obesity, high
dietary calorie and fat intake, smoking, medications that promote GERD, etc. Patients with Barrett's
esophagus have a 1% per year risk of developing adenocarcinoma of the esophagus.
Adenocarcinoma generally occurs in patients who had GERD symptoms more than 20 years and
presents with weight loss, anorexia and asymmetric narrowing of the esophageal lumen.

The major risk factors for squamous cell cancer of the esophagus are smoking, alcohol, dietary
deficiency of beta-carotene, vitamin B-1, zinc, selenium, environmental viral infections, toxin
producing fungi, hot food and beverages, pickled vegetables and food rich in N-nitroso compounds,
etc.

Barium swallow followed by endoscopy is usually done when a patient is suspected of having an
esophageal cancer.
Esophageal cancer mimic achalasia. Features that favor esophageal cancer over achalasia are: short
history, rapid weight loss, asymmetric narrowing of the esophageal lumen, and inability of
endoscope to pass through the lower esophageal sphincter.

Squamous cell carcinoma of skin


Squamous cell carcinoma develops on sun-exposed skin surfaces in elderly patients.
The carcinogenic potential of tobacco is multiplicative.
Cyclosporine is a commonly used immunosuppressant associated with an increased risk of
squamous cell carcinoma of the skin and lymphoproliferative diseases.

204
Whenever an open wound fails to heal after a prolonged period, biopsies have to be obtained to
ensure that the ulcer has not degenerated into a squamous cell carcinoma. These ulcers are known as
Marjolin's ulcers.
Diagnosis is with a punch biopsy.
Treatment of such lesions by Surgical removal.

Basal cell carcinoma


Basal cell carcinoma is the most common form of skin cancer in the US.
It is the most common malignant tumor of the eyelid.
It usually occurs in fair-skinned individuals with a history of prolonged sun exposure.
The cancer usually remains local and almost never spreads to distant parts of the body, but it may
continue to grow and invade nearby tissues and structures, including the nerves, bones, and brain.
The five warning signs of basal cell carcinoma are:
1. Open sore that bleeds, oozes, or crusts and remains open for three or more weeks.
2. Reddish patch or irritated area
3. Shiny bump or nodule that is pearly or translucent and is often pink, red, or white.
4. Pink growth with a slightly elevated rolled border and a crusted indentation in the center.
5. Scar-like area which is white, yellow or waxy, and often has poorly defined borders.

Basal cell carcinoma has a shiny or "pearly" appearance and indurated.


The most common location is the lower eyelid margin, followed by the medial canthus, upper
eyelid, and lateral canthus.
Diagnosis is confirmed by shave or punch biopsy (full-thickness biopsy).
Basal cell carcinomas are histologically characterized by invasive clusters of spindle cells
surrounded by palisaded basal cells.

Treatment by Surgical removal.


Mohs surgery is a special type of surgery wherein microscopic shaving is done, such that 1-2 mm
of clear margins are excised. This technique currently has the highest cure rate for basal cell
cancer, but is indicated only in patients with high-risk features, as well as those with lesions in
functionally critical areas (e.g., perioral region, nose, lips, ears).
After treatment, new sites of basal cell cancer can occur; therefore, diligent monitoring of such
patients is necessary via regular follow-up examinations by the health care provider.

205
Malignant melanoma
Malignant melanoma is called "the fascinating disease." It can go to the most unimaginable places,
lie dormant for 15-25 years, then recur in surprising ways, even if the primary tumor was resected.
In many cases, malignant melanoma has spread to all sorts of places like the muscles of the heart
and the bile duct.
Think of malignant melanoma in a patient with a metastatic mass that is bleeding inside brain.
Malignant melanomas are notorious for causing bleeding inside the metastatic mass in the brain.

There are four tumors in the body that have not been reported to metastasize (almost never
metastasize) to the brain:
1) Non-melanomatous skin cancer
2) Oropharyngeal cancer
3) Esophageal carcinoma
4) Prostate cancer

Squamous cell carcinoma of the mucosa of the head and neck


Squamous cell carcinoma of the mucosa of the head and neck is common in patient with a
significant history of alcohol and tobacco use.
The first manifestation may be a hard (firm) unilateral solitary non-tender lymph node.
Such lymph nodes in the submandibular or cervical region in an older patient with a history of
smoking are highly concerning for head and neck cancer (mostly squamous cell carcinoma).

The initial test is lymph node biopsy followed by panendoscopy (triple endoscopy = esophagoscopy,
bronchoscopy, laryngoscopy) to detect the primary tumor.

Cryptorchidism (crypto = hidden, orchid = testicle)


Cryptorchidism, in which the testis fails to descend into its normal position in the scrotum, by age 12
months. If the baby was premature, you wait until they would have been 12 month old, e.g., if 3
month premature, wait until they are 15 month before labeling the testicle as really undescended.
Treatment is delayed until 12 month of age to give the testicle a chance to come down on its own.
The procedure is called orchiopexy, and frees up the bands of tissue that kept the testicle from
coming down. It is 99% effective and is done on an outpatient basis under general anesthesia.

206
Boys with cryptorchidism are at high risk for the development of testicular malignancy later in life.
(nearly four times higher than the risk in the general population).

Solid testicular mass


This is the tumor that we kill first and investigate later.
After the diagnosis of a solid testicular mass has been made, (a painless hard mass in testicle +
suggestive ultrasound), the initial management is removal of the testis and its associated cord,
orchiectomy.
The testis and abnormal tissue present is then examined under the microscope to determine the type
of cancer. Depending on the cell type of the cancer present other therapies, i.e. additional surgery,
radiation therapy, or possibly chemotherapy may be indicated. Using a combination of these
therapies, testicular cancer has one of the highest cure rates of all cancers.

TESTICULAR CANCERS
AFP, beta-hCG, and PLAP are tumor markers useful in diagnosis, staging, and monitoring patients
with testicular cancers.

Leydig cell tumors are the most common type of testicular sex cord stromal tumors principal
source of testosterone and increased aromatase expression increasing estrogen production
secondary inhibition of LH and FSH levels. The most common endocrine manifestation is
gynecomastia, however in prepubertal cases, precocious puberty is common.

Seminoma: PLAP

Embryonal cell carcinoma: AFP + beta-hCG (50%)

Choriocarcinoma: beta-HCG

207
Prostate cancer
The American Cancer Society urges physicians to offer yearly prostate specific antigen (PSA) blood
tests and digital rectal examinations (DRE) beginning at age:
1) 50 years - for men who have at least a 10-year life expectancy

2) 45 years - for men at high risk


African Americans
men with a first-degree relative diagnosed with prostate cancer at an early age (younger than
age 65)

3) 40 years - for men at even higher risk


men with several first-degree relatives who had prostate cancer at an early age. Depending on
the results of this initial test, further testing might not be needed until age 45.

Palliative radiation along with anti-androgen therapy, is the treatment of choice for metastatic
prostate cancer. Anti-androgen therapy consists of Leuprolide (LHRH analogue). Flutamide is
considered inferior to LHRH analogues.

Radiation therapy is the most appropriate for the management of progressive pain in patients with
prostate cancer and bony metastases after androgen ablation (orchidectomy). Focal external bean
therapy is an excellent choice as metastasis is localized to few sites.

Metastatic spinal cord compression


Metastasis of prostate cancer can lead to acute cord compression (i.e., absent rectal tone, urinary
incontinence, motor and sensory loss in the extremities).
Back pain is the 1st symptom of cord compression with tenderness at the site of metastasis.
Spinal cord compression or Cauda equina due to metastasis is a medical emergency and needs
prompt administration of intravenous steroids (dexamethasone) as the first step.
Next step would be an MRI of the spine. If MRI is not available or contraindicated (patients with
pacemakers), CT myelogram should be done.
Once the diagnosis of metastatic cord compression is confirmed radiotherapy should be considered.

208
Brain metastasis
Surgical resection followed by whole brain radiation is the standard of practice in the management
of solitary brain metastasis with stable extra-cranial cancer.
Multiple brain metastases are best treated with palliative whole brain radiation.

Thymoma
Thymoma is usually found in the anterior mediastinum.
The diagnosis is suspected when an anterior mediastinal mass presents in a young male or female.
About 20% of patients with myasthenia gravis have a thymoma.
Other anterior mediastinal masses include: retrosternal thyroid, teratoma, and lymphoma.
If the mass is large, patients may complain of chest heaviness or discomfort. Hoarseness, Horner's
syndrome, and facial and upper extremity edema may occur when the tumors invade locally.
Thymoma is associated with pure red cell aplasia. Everything except RBC, ESR, Hb, Hct, and
reticulocyte count are normal.
CT of the chest should be done to look for a thymoma in all newly-diagnosed myasthenia gravis
patients.

Thymoma is usually found in the anterior mediastinum.


Pericardial cysts are usually found in the middle mediastinum.
Nneurogenic tumors are located in the posterior mediastinum.

Multiple endocrine neoplasia (MEN)


MEN I: pituitary tumors, pancreatic tumors , and primary hyperparathyroidism.
Hypercalcemia and intractable ulceration can be due to MEN I.

MEN 2A MEN 2 B
Medullary thyroid cancer Medullary thyroid cancer
Pheochromocytoma Pheochromocytoma
Primary parathyroidism hyperplasia Mucosal neuromas (tongue, lips, eyelids, GIT)
Marfnoid habitus

Normal levels of calcium and PTH rules out MEN 2A.

209
MEN IIa is an autosomal disorder resulting from germline mutation in the RET proto-oncogene
localized on chromosome 10. In patients with suspected MEN IIa syndrome, DNA (genetic) testing
has replaced biochemical measurement of serum calcitonin as the recommended screening test. If
genetic analysis is positive for a RET proto-oncogene mutation, total thyroidectomy is indicated.

Pancreatic cancer
The known risk factors for the development of pancreatic cancer include: family history, Increasing
age (50 years), chronic pancreatitis, smoking, diabetes mellitus, obesity, and a diet high in fat.
Cigarette smoking is the most consistent reversible risk factor for pancreatic cancer.
Alcoholism, Gall stones, and Coffee intake are NOT a risk factor for pancreatic cancer.
Pancreatic carcinoma is the primary differential diagnosis in patients with chronic pancreatitis.
Pancreatic carcinoma most commonly presents with a dull upper abdominal pain that radiates to the
back, weight loss, or jaundice. Classic findings include a nontender but palpable gallbladder at the
right costal margin in a jaundiced patient (Courvoisier sign) or left supraclavicular adenopathy
(Virchow node) in a patient with metastatic disease.
Laboratory evaluation of those with pancreatic cancer typically demonstrates increased serum
bilirubin and alkaline phosphatase in conjunction with a mild anemia.
Abdominal ultrasound is usually the initial imaging performed on patients with jaundice. When the
ultrasound is nondiagnostic, the next step is to obtain an abdominal CT scan. Abdominal CT scan is
a very sensitive and specific tool used in the diagnosis of pancreatic carcinoma.
Endoscopic retrograde pancreatography (ERCP) is an excellent tool in the diagnosis of pancreatic
cancer, with a sensitivity and specificity of 90-95%. Because it is an invasive procedure, however, it
is most commonly reserved for those patients who have already undergone a nondiagnostic
ultrasound and a nondiagnostic CT scan.
The most commonly used tumor marker for pancreatic cancer is the cancer associated antigen (CA)
19-9, which has a sensitivity and specificity of 80-90%. CA 19-9 can be elevated in patients with
jaundice but no pancreatic cancer, however, which reduces its utility as a screening tool.
Postoperative monitoring of pancreatic cancer with CA 19-9 may be helpful in evaluating the
tumor response to chemotherapy.

210
VIPoma
VIPomas are cancerous pancreatic tumors that affect cells in the pancreas that produce vasoactive
intestinal peptide (VIP). The cause is not known. Remember the pancreatic cholera - VIPoma
VIPomas are diagnosed most commonly at age 50 or so. Women are more likely to be affected than
men.
VIPomas cause severe watery diarrhea, hypokalemia resulting in leg cramps, and a decrease in the
amount of acid in the stomach. Other symptoms include dehydration, abdominal pain and
cramping, weight loss, facial flushing and redness.
High level of VIP (vasoactive intestinal peptide) in the blood is diagnostic.
A CT scan or MRI is ordered to determine the location of the tumor.
The first goal of treatment is to correct dehydration. Intravenous (IV) fluids are often required to
replace fluids lost in diarrhea. The next goal is to slow the diarrhea. Some medications can help
control the diarrhea, such as octreotide. If the tumor has not metastasized, surgery can often cure it.

INSULINOMA is a rare beta-cell tumor that releases Insulin and causes hypoglycemia.
Elevated C-peptide levels and proinsulin levels are seen in patient with beta cell tumors.

Glucagonoma
Glucagonoma produces a characteristic skin rash (necrotizing dermatitis), weight loss, anemia, and
persistent hyperglycemia.
Glucagonoma is defined as a tumor of the islet cells of the pancreas, which secretes the hormone
glucagon.
The excess glucagon causes symptoms such as glucose intolerance and hyperglycemia.
It also causes a distinctive skin lesion called 'necrolytic migratory erythema.'
It is usually malignant, and metastasis to the liver may occur.
The diagnosis is confirmed by elevated levels of fasting glucose and serum glucagon, as well as a
pancreatic tumor revealed by CT scan.
The preferred treatment is surgical removal of the tumor (not respond to chemotherapy).

211
Juvenile angiofibroma (JNA)
Any adolescent who presents with a nasal obstruction, visible nasal mass, and frequent nosebleeds
(epistaxis) is considered to have a juvenile angiofibroma (JNA), unless proven otherwise.

This is typically found in the back of the nose or upper throat (nasopharynx) of adolescent boys.
It is a benign growth, but is capable of eroding and locally invading.
JNAs are deemed potentially very dangerous because these are composed of many blood vessels
which may bleed readily. In addition, its common areas of occurrence are difficult to access
surgically. For these reasons, such tumors should only be touched by a specialist.

In some cases, no treatment is necessary.


Treatment is required if the angiofibroma is enlarging, obstructing the airway, or causing chronic
nosebleeds. Surgical treatment includes removal of the tumor. Removal is often difficult because the
tumor is unencapsulated and may be deeply invasive. Recurrence of the tumor after surgical
resection is common.

TROUSSEAU'S SYNDROME MIGRATORY THROMBOPHLEBITIS


Patient often complaints of pain, itching and red streaks (thrombophlebitis) in some part of her
body with similar episode in another part weeks ago, and it went away on its own.

Patients with Trousseau's syndrome usually have an occult tumor, which may not always be
detectable at the time of presentation. The most common tumor is an adenocarcinoma.
Based on the reviews, 24% of patients have pancreatic carcinoma, 20% lung carcinoma, 13%
prostate cancer, 12% stomach cancer, 9% acute leukemia, and 5% colon cancer.
The thrombophlebitis of the atypical sites such as arm and chest is an important clue for the
underlying carcinoma.

Pancreatic cancer is the 4th leading cause of death from cancer in the U.S. The disease is slightly
more common in men than in women, and risk increases with age. The cause is unknown, but the
incidence is greater in smokers. Abdominal CT scan is the test of first choice when the suspicion of
Pancreatic cancer is so high.
Trousseau's syndrome that occurs in about 10% of patients with pancreatic adenocarcinoma is
attributed to the elaboration of platelet-aggregating factors and procoagulants from the tumor or
its necrotic products. Both venous and arterial thrombi can occur. So coagulation studies and
Doppler studies may be needed later but not at this point.

212
These venous thrombi can embolize to the lungs. A large "saddle" pulmonary embolus may result
in sudden severe dyspnea and demise of the patient. Smaller emboli, as in this case, may produce
pulmonary infarctions with some dyspneaand chest pain. In such a case a V/Q scan or spiral CT
chest will be needed.

CHRONIC DIC
Migratory thrombophlebitis and atypical venous thromboses are suggestive of chronic DIC, which
is most likely due to cancer.
The typical laboratory findings of chronic DIC include mild prolongation of PT, low fibrinogen
levels and positive Fibrin split products (FSP).
The most common causes are malignancies of the lung, pancreas, stomach and prostate.
CT of the chest, abdomen and pelvis are thus indicated to identify and further evaluate the
underlying malignancy, along with age-appropriate cancer screening (e.g., digital rectal exam,
mammography, colonoscopy).

Cervical lymph nodes


Small, rubbery cervical lymph nodes are a common observation in healthy children and young adults
and rarely pathologic. Nodes less than 1 cm in diameter are almost always due to benign process.
Patients with asymptomatic, soft lymph nodes can be observed for node growth or the development
of symptoms.

In contrast, nodes associated with cancer tend to be firm and immobile. Nodal diameter > 2 cm is
associated with a greater likelihood of malignancy or granulomatous disease.

Ondansetron (5-HT3 receptor antagonist) is the drug of choice for chemotherapy-induced emesis.
Megestrol acetate is the drug of choice for cancer-associated anorexia.

Biphosphanates (e.g.IV zoledronic acid, pamidronate) are the drug of choice for mild and moderate
hypercalcemia due to malignancy.
Vigorous hydration with IV saline and furosemide are used in emergency treatment of
hypercalcemic crises (vomiting, oliguria, anuria, somnolence and eventually coma).

213
Pain management
Give the appropriate pain medication to cancer patients.
Do not be afraid to use a narcotic drug if it is the most appropriate.
Although there are many versions, three key principles to the use of pain medications in patients
with terminal stages of cancer are as follows:
1) Try non-narcotic measures first, unless you are sure (using your clinical judgment) that the
patient is in severe pain.
2) Do not be afraid to give narcotic analgesics.
3) Prescribe adequate amounts of medication.

Tumor lysis syndrome


Tumors with high cell turnovers are frequently associated with tumor lysis syndrome.
The tumors most often associated with this syndrome are the poorly differentiated lymphomas (e.g.,
Burkitt's lymphoma) and the leukemias (particularly ALL and less often AML).

Although the introduction of allopurinol has reduced the acute urate nephropathy to a large extent,
the possibility of tumor lysis syndrome remains.
There are a number of metabolic abnormalities associated with tumor lysis syndrome. These include
hyperphosphatemia, hypocalcemia, hyperkalemia, and hyperuricemia.

Both potassium and phosphate are intracellular ions, and the breakdown of the cells release excess
amounts of these.
Released phosphate binds calcium and causes hypocalcemia. Hypocalcemia is also due to the release
of intracellular products by cell lysis.
Degradation of cellular proteins causes elevation of uric acid levels.

Prompt identification and immediate treatment of such metabolic abnormalities is necessary since
these may lead to fatal arrhythmias, acute renal failure, and even sudden death.

214
Febrile neutropenia
Febrile neutropenia is considered a medical emergency.
Fever in a neutropenic patient is defined as a single temperature reading of greater than
38.3C(100.9F) or a sustained T of greater than 38C(100.4F), over 1 hour.
Neutropenia is defined as an absolute neutrophil count < 500 cells/mm3.
Febrile neutropenia may be seen in patients who are undergoing chemotherapy. Chemotherapy
leads to impaired immunologic function and lead to disruption of the skin and mucosal barrier
(mucositis), which may result in seeding of bacteria into the blood stream (bacteremia). The most
common site of mucositis is the gastrointestinal tract, and the frequently identified organisms are
gram-negative organisms especially Pseudomonas aeruginosa.
Bacteria, fungi and viruses can all cause infection in neutropenic patients. Bacterial infections are
the most common and are frequently caused by endogenous skin or colon flora. Gram-negative
organisms especially Pseudomonas aeruginosa used to be the usual culprit, but now infections by
gram-positive organisms are increasing in number.
Empiric therapy for febrile neutropenia should be targeted against gram-negative bacilli especially
P. aeruginosa. Either monotherapy or combination therapy can be employed.
Monotherapy consists of ceftazidime, imipenem, cefepime or meropenem.
Combination therapy is equally effective and consists of an aminoglycoside plus an anti-
pseudomonal beta-lactam.
Fungal infections occur quite commonly in patients with prolonged neutropenia. When fever in
neutropenic persists despite empiric antibacterial therapy, antifungal agents like amphotericin B are
added to the empiric regimen.
Vancomycin is added to the empiric regimen when patient is hypotensive or he has severe
mucositis, evidence of skin or line infection, history of colonization with resistant strains of S.
aureus or pneumococcus or recent prophylaxis with fluoroquinolones.

215
Informed consent
A patients data may be used for research purposes only after obtaining informed consent (has to be
written; verbal consent is not recommended). This includes explaining the following to the patient:
1) The purpose of the search.
2) The foreseeable risks or discomforts that the patient may experience
3) That refusal or withdrawal will not affect current or future care that the patient is receiving
4) That the patient can withdraw from the study anytime he/she wants to

Breaking bad news ***High-yield for the USMLE


Breaking bad news is a very sensitive issue to both patients and physicians.
For this reason, many institutions have created guidelines to assure that such conversations are
carried out as smoothly as possible.
The following is an example of the recommended step-wise approach to breaking bad news to a
patient:
1) Make sure the patient is in a quiet, private and comfortable environment.
2) Ask the patient how much he knows, or what he thinks he might have. This will give you an idea
of what his expectations are. "What do you think of your symptoms?"
3) Ask the patient how much he wants to know. "How much would you like to know about your
condition?"
4) Give him a warning shot. "Unfortunately, the situation is more serious than what I earlier
thought."
5) Break the news if he wants you to. "The results show that you have advanced lung cancer."
6) Give his prognosis, but always keep him aware of all the options available to make his life as
comfortable as possible.
7) Try to explain everything as clearly and simple as possible.

216
Initial course of action
Chemicals in the eye deserve immediate attention; the best primary action is to flush the eye with
water best achieved under a faucet of running water for at least 15 minutes. Obtaining medical
care is appropriate once this treatment is initiated.
For foreign bodies or cuts/scratches of the eye, obtaining medical care is the first priortity.

Acetaminophen
In the United States, acetaminophen toxicity is the most common cause of acute hepatic failure, and
fulminant hepatitis responsible for as many as 40% of cases.
In patients who are within 4 hours of ingestion, administering activated charcoal in order to derease
absorption would be appropriate.

The Rumack-Matthew nomogram provides the likelihood of hepatotoxic effect from acetaminophen
overdose and the need of N-acetylcysteine. The first data point on the curve is at 4 hours.
The decision whether or not to administer the antidote can be made after 4 hours, based on the
acetaminophen level.
As long as N-acetylcysteine is administered within 8 hours of ingestion, the outcome will not be
adversely affected.

On the other hand, if the patient (adult) has ingested > 7.5gm of acetaminophen and levels will not
be available within 8hrs after ingestion, he/she should be given the antidote.
If the dose is below the amount, which is usually toxic to liver. We can wait for the acetaminophen
levels, and can decide once we interpret the levels on the nomogram.

Acute alcohol intake can reduce the risk of hepatic injury by the acetaminophen because it competes
with the CYP2E1 which results in decrease production of toxic metabolite.
Remember the chronic alcohol intake increases the risk of hepatic injury by stimulating P450 system
and decreasing the amount of glutathione (used for the metabolism of acetaminophen).

217
Alcohols
Alcohol intoxication causes poor coordination, unsteady gait, slurred speech, & nystagmus.

Ethylene glycol (anti-freeze) poisoning is associated with hypocalcemia and calcium oxalate
crystals (rectangular, envelope shaped crystals) deposition in the kidneys. This leads to flank pain,
hematuria, oliguria, acute renal failure and metabolic acidosis with elevated anion gap. Treatment
involves administration of fomepizole or ethanol to achieve ADH dehydrogenase inhibition, sodium
bicarbonate to alleviate acidosis and hemodialysis in the case of acidosis and/or end organ damage.

Methanol poisoning is associated with vision loss & coma. Methanol ingestion can lead to
headache, nausea, vomiting (hypokalemia) and epigastric pain and physical examination shows optic
disc hyperemia, and metabolic acidosis with elevated anion gap and very low bicarbonate.

ALCOHOL WITHDRAWAL
In any hospitalized patient with a suspected history of alcoholism, precautions should be taken to
prevent the symptoms of alcohol withdrawal.
Alcohol withdrawal can develop minor symptoms (eg, tremulousness, anxiety, sweating,
palpitations) within 6 hours of the last drink.
This stage is followed by alcoholic hallucinosis, which involves visual, auditory, or tactile
hallucinations (bugs crawling on the skin) independent of autonomic symptoms that typically
resolve within 24-48 hours (Alcoholic hallucinosis is the likely diagnosis in a known drinker who
develops altered behavior or mental status changes on the second or third day of hospitalization).
The last stage is that of delirium tremens, which is associated with hallucinations, disorientation,
tachycardia, hypertension, low-grade fever, sweating, and agitation.
All patients suspected of alcohol withdrawal should be placed in a protective environment, and
treated with Chlordiazepoxide [(Librium) is a long-acting benzodiazepine].

Carbon monoxide
Acute exposure presents with headache, nausea, vomiting, abdominal discomfort, confusion and
coma.
Patient often presents with pinkish-red skin hue.
Diagnosis is confirmed by carboxyhemoglobin levels.
Treatment consists of 100% oxygen or hyperbaric oxygen.

218
Caustics
The important clues to the diagnosis of caustic poisoning are:
1) Conscious patient (usually in severe pain)
2) White tongue
3) Heavy salivation
4) Furthermore, mouth burns, drooling of saliva and dysphagia are classic indicators of caustic acid
and alkali ingestion.

Damage caused by lie ingestion occurs nearly instantaneously and its effects are most pronounced in
the esophagus, where liquefactive necrosis occurs which can lead to perforation & mediastinitis or
peritonitis.
Patients often present with retrosternal/epigastric pain, hypersalivation and odynophagia/dysphagia.
Patients should be hospitalized, given intravenous hydration and receive serial abdominal and chest
x-rays.
Early upper gastrointestinal contrast studies and/or endoscopy are critical for evaluating the extent of
the damage and determining further treatment (if perforation is suspected, a Gastrografin study
should be performed).

Digoxin
Gastrointestinal symptoms are most common.
Paroxysmal atrial tachycardia is the most common arrhythmia.
Hyperkalemia occurs due to inhibition of Na+/K+ ATPase by digoxin.
Serum digoxin level should be ordered.

Opiates (heroin)
Opioid overdose is characterized by respiratory depression, hypotension, bradycardia, decreased
bowel sounds, pinpoint pupils (may be of normal size), hypothermia (as the result of environmental
exposure and impaired thermogenesis) and CNS depression (leading to coma).
Decreased respiratory rate is the best predictor of intoxication and is also a frequent cause of
mortality.
Naloxone is an opioid antagonist.

219
Opioid withdrawal
Opioid withdrawal causes central nervous system hyperactivity.
Symptoms opioid withdrawal include drug cravings, nausea or vomiting, abdominal cramps,
diarrhea, muscle spasms & aches, joint pains, restlessness, rhinorrhea, lacrimation, sweating, and
hot and cold flashes.
Signs of opioid withdrawal on exam include increased bowel sounds, pupillary dilatation, increased
respiratory rate accompanied by yawning, perspiration, and piloerection.
Autonomic instability (eg, hypertension) is common.
Methadone is proven effective for treatment of opioid withdrawal.

Cocaine
Cocaine is a CNS stimulant that causes hypertension, tachycardia, sweating, mydriasis, and
hypervigilance.
Cocaine and amphetamine intoxication present in a similar manner, but psychosis is more
commonly associated with amphetamine use.

Amphetamine intoxication has a presentation similar to that of cocaine intoxication, though it has
more prominent psychotic features.
Clinical manifestations include anxiety, aggression, agitation, and psychosis or delirium in the
setting of recent amphetamine use.
In addition, intoxicated patients develop two or more of the following signs: hypotension or
hypertension, bradycardia or tachycardia, cardiac arrhythmias, chest pain, respiratory depression,
sweating, pupillary dilatation, nausea or vomiting, insomnia, weight loss, psychomotor agitation or
retardation, muscular weakness, seizures, stroke, or coma.

Cannabis (marijuana)
Marijuana is the most commonly used illicit drug in USA.
Cannabis (marijuana) intoxication presents with behavioral changes (eg, social withdrawal,
euphoria, paranoia, anxiety) and two or more of the following: slowed reaction time, time distortion,
impaired short-term memory, confusion or increased appetite.
On physical exam, acute intoxication can cause conjunctival injection, tachycardia, dry mouth,
elevated blood pressure, and tachypnea.

220
Benzodiazepines
Benzodiazepines are used for treatment of anxiety, insomnia, seizures and alcohol withdrawal.
A patient with benzodiazepine overdose presents with slurred speech, unsteady gait and drowsiness
or sleepiness.
During benzodiazepine intoxication, pupil size tends to be normal, with mild respiratory depression.
Flumazenil is used for the treatment of benzodiazepine toxicity.

Barbiturates
Barbiturates are CNS depressants and would cause hypotension, respiratory depression, diminished
reflexes, ataxia, drowsiness, and slurred speech.

Hallucinogens
Phencyclidine (PCP or angel dust) is a commonly abused hallucinogenic drug, which
characteristically causes vertical nystagmus. It can also causes dissociative feelings, psychotic &
violent behavior, severe hypertension, hyperthermia and even seizures. Treatment includes urine
acidification, haloperidol in the case of psychotic behavior and close monitoring until the condition
resolves.

Iron
Iron poisoning disrupts basic cell processes by free radical production and lipid peroxidation leading
to systemic manifestations include nausea, vomiting, diarrhea, abdominal pain, gastrointestinal
bleeding, shock and metabolic acidosis.

Acute iron intoxication have five phases:


1) Gastrointestinal phase (I): occurs 30 minutes 6 hours after ingestion and it is the result of direct
mucosal damage. Patients experience nausea, vomiting, diarrhea, abdominal pain, and
gastrointestinal bleeding.
2) Latent phase (II): occurs 6 24 hours after ingestion, and is largely asymptomatic.
3) Phase III: occurs 6 72 hours after ingestion, manifested by shock and metabolic acidosis.
4) Phase IV: occurs 12 96 hours after ingestion, manifested by hepatotoxicity.
5) Phase V: occurs several weeks after ingestion, in which bowel obstruction occurs secondary to
mucosal scarring.

221
Iron poisoning commonly occurs in children of pregnant women taking pre-natal vitamins because
children often confuse brightly colored iron pills for candy.
Since iron is radio-opaque, the tablets can be seen in the stomach on abdominal x-ray.
The diagnosis of acute iron intoxication is made by measuring serum iron levels. Levels 350
mcg/dL are consistent with toxicity
Intravenous deferoxamine, an iron chelator binds to ferric iron, allowing its urinary excretion, is
used in moderate to severe cases of intoxication.

Antipsychotics
Fluphenazine is a long-acting injectable typical antipsychotic given every 2-3 weeks in
schizophrenic patients.
Fluphenazine is haloperidol and can cause hypothermia by inhibiting the bodys shivering
mechanism and/or inhibiting autonomic thermoregulation. For this reason, patients tacking
antipsychotic should be adviced to avoid prolonged exposure to cold temperatures.

Tricyclic antidepressants (TCAs)


TCA overdose causes CNS depression, hyperthermia, hypotension and seizures as well as
anticholinergic effects; dilated pupils, dry & flushed skin and decrease intestinal movements.
TCA overdose decreases myocardial conduction velocity by sodium channel blocker effect, leading
to QRS prolongation and risk of developing ventricular arrhythmia.
QRS complex duration is the best indicator of the extent of the overdose.
The major cause of mortality is believed to be secondary to TCA-induced hypotension.
The first priority in treatment TCA overdose is to secure the patients airway, breathing and
circulation.
Sodium bicarbonate (by alleviating the cardio-depressant action on sodium channels) improves
systolic blood pressure and shortens the QRS interval in TCA overdose, decreasing the likelihood of
arrhythmia.

222
Beta-blockers
Beta blocker overdose causes bradycardia, AV block, hypotension, wheezing (bronchospasm),
neurological effects (deleirium, seizures), hypoglycemia and potential cardiogenic shock (cold,
clammy extremities).
Atropine & IV fluids are the first line of therapy.
If atropine & IV fluids do not completely reverse the cardiac symptoms, glucagon should be
administered (glucagon leads to increase in cAMP and higher levels of intracellular calcium,
thereby augmenting cardiac contractility).

Organophosphate poisoning
In organophosphate poisoning, acetylcholinesterase is rendered non-functional, leading to symptoms
of cholinergic excess.
Common presentation includes, bradycardia, miosis, muscle fasciculations, salivation, vomiting,
lacrimation, diarrhea and urination.
To counteract the effects of organophosphate poisoning, atropine should be administered
immediately.
Of equal importance is the immediate removal of the patients clothing of to avoid continued
absorption of organophosphate through the skin.

Antihistamines
Diphenhydramine has both anticholinergic & antihistamine actions.
Diphenhydramine toxicity produces drowsiness, confusion or seizures as well as anti-cholinergic
effects (dry mouth, dilated pupils, blurred vision, dry flushed skin, reduce bowel sound, constipation
and urine retention).
It is one of the drugs for which urine screen is available.
Physostigmine, a cholinesterase inhibitor can counteract these anti-cholinergic effects.

Remember some antihistamines are over the counter drugs, which makes it important to know their
toxicity.
The other two new sedating drugs are terfenadine and astemizole and their overdose results in QT
prolongation and torsade de pointes.
Loratadine is one of the new non-sedating antihistamines.

223
Cyanide
Cyanide poisoning causes death within minutes, and is characterized by a burning sensation in the
mouth, bitter almond odor of the breath, initial hyperpnoea and headache followed by hypoxic
convulsions, respiratory arrest and death.

Neuroleptic Malignant Syndrome (NMS)


NMS is a drug induced idiosyncratic reaction.
The typical neuroleptic agents (such as haloperidol) are most frequently implicated in NMS but
other dopaminergic antagonists including the atypicals have also been implicated.
Symptoms of NMS begin within two weeks of initiation of the precipitating drug and include fever,
muscle rigidity lead pipe rigidity, autonomic instability and mental status change.
Hyperthermia and muscle rigidity of NMS may result in muscle necrosis, reflected by elevated
creatine kinase.
Leukocytosis and electrolyte abnormalities are also common.
Immediate cessation of the causative agent is recommended.
Dantrolene (muscle relaxant) is the most common drug used to reserve the condition followed by
bromocriptine (dopamine agonist) and amantadine (antiviral drug with dopamenergic properties).

Food allergy
Food, especially peanuts, allergies are the major cause of outpatient anaphylaxis in children.
In anaphylaxis, there is an IgE-mediated release of histamine and leukotrines from mast cells and
basophils, which cause relaxation of vascular smooth muscles and contraction of bronchial and
intestinal muscles.
Signs include bronchoconstriction, tachycardia, hypotension and urticaria.
Subcutaneous epinephrine (1 agonist causes vasoconstriction and raise blood pressure) is the first-
line of treatment for anaphylaxis in a patient with a patent airway.
In an anaphylactic patient with respiratory failure (cyanosis & obtundation), intubation should be the
first step.

Vancomycin side effects: nephrotoxicity, ototoxicity, and red man syndrome

224
Diabetic retinopathy
Diabetic retinopathy is the leading cause of acquired blindness among US adults.
It can be classified into non-proliferative and proliferative retinopathy.
Non-proliferative retinopathy is characterized by the dilation of veins, microaneurysms, retinal
hemorrhages, edema, and hard exudates.

Proliferative diabetic retinopathy is characterized by neovascularization. Most patients with


diabetic retinopathy are asymptomatic until very late in the disease, they may present with decreased
visual acuity, poor night vision, a curtain falling with a vitreous hemorrhage, or floaters during the
resolution of vitreous bleeds with macular edema. These changes may lead to retinal detachment.

Patients with hypertensive retinopathy usually do not show any symptoms associated with visual
impairment. There is initially focal spasm of arterioles, followed by progressive sclerosis and
narrowing. Fundoscopy findings may reveal AV nicking, copper wiring or silver wiring, exudates
and hemorrhages (specific finding depends on the grade of retinopathy).

Retinal detachment
Retinal detachment refers to the separation of the layers of the retina.
Retinal detachment usually occurs unilaterally and suddenly.
Common predisposing factors are myopia and surgical extraction of cataracts.
Patients describe it as "a curtain falling in front of the eye" or photopsia (flashes of light) and
floaters (obscuring spots in the visual field).
Ophthalmoscopic examination reveals an grey, elevated, detached retina.
Patients should lean their heads back.
Laser therapy and cryotherapy are done to create permanent adhesions between the neurosensory
retina, retinal pigment epithelium, and choroid.

225
Vitreous hemorrhage
Vitreous hemorrhage typically presents as a sudden loss of vision and onset of floaters (spots in the
visual field).
The most common cause is diabetic retinopathy.
An important diagnostic clue is that the fundus is hard to visualize, and even if it is visualized,
details may be obscured.
Immediate ophthalmoscopic consultation is required.
For patients with underlying medical conditions, conservative treatment (i.e., upright position during
sleep, which enhances settling of the hemorrhage) is recommended.

Age-related Macular degeneration (AMRD)


Macular degeneration affects central vision.
Peripheral fields and navigational vision are always maintained, but may become impaired by the
development of cataracts.
The condition results from degeneration and atrophy of the outer retina, retinal pigment epithelium,
Bruch's membrane and choriocapillaries.
The two forms are atrophic and exudative.
The atrophic type is characterized by drusen and patchy depigmentation in the macular region.
The exudative type is characterized by new blood vessels which can leak, bleed, and scar the retina.
Dry type ARMD usually presents as a slowly progressive, loss of vision in one or both eyes.
Wet type ARMD presents as acute or insidious painless, blurring of central vision, usually in one eye.
Testing reveals central scotoma.
Laser photocoagulation is the treatment of choice.

Central retinal artery occlusion (CRAO)


CRAO is also characterized by a sudden painless loss of vision in one eye.
Risk factors of CRAO: carotid artery embolic disease, temporal arteritis, cardiac thrombi,
Ophthalmoscopy of patients with CRAO reveals pale (whitening) retina, pallor of the optic disc,
cherry red spots at fovea, and boxcar segmentation of blood in the retinal veins.
Central retinal artery occlusion is emergently treated with an ocular massage and high-flow oxygen.
Other potential therapies are acetazolamide and thrombolytics.

226
Central retinal vein occlusion
Central retinal vein occlusion presents as a sudden painless loss of vision.
Ophthalmoscopy swollen disc, venous dilation, retinal hemorrhages, and cotton wool spots.

Amaurosis fugax
Amaurosis fugax is characterized by visual loss that is usually monocular, transient, and described
as 'like a curtain falling down'.
Ophthalmoscopy reveals zones of whitened, edematous retina following the distribution of the
retinal arterioles.
The condition is usually seen in patients with atherosclerosis, cardiovascular disease, or
hypertension.
It is caused by retinal emboli from the ipsilateral carotid artery. Rarely, cholesterol emboli or
plaques can be visualized.
Non-invasive evaluation of the carotids is useful in providing information regarding the degree of
stenosis.
Treatment of the atherosclerosis is important to reduce the risk of stroke.

Glaucoma
Glaucoma is the leading cause of blindness in North America.
This condition is characterized by an elevated intraocular pressure.
Drugs that dilate the pupil can precipitate glaucoma (e.g. anticholinergics, TCAs & ipratropium).
Increased pressure is transmitted through the aqueous humor, thereby damaging the optic nerve.

Open-angle glaucoma
Open angle glaucoma is usually asymptomatic in the earlier stages, has an insidious onset, with
gradual loss of peripheral vision over a period of years, and eventual tunnel vision.
It is more common in African Americans, and has an increased prevalence in those with a family
history of glaucoma and diabetes.
On examination, the intraocular pressure is high.
Ophthalmoscopic examination reveals cupping of the optic disc (> 0.3).
Annual examination in high-risk populations is of great benefit in prevention of the condition.

227
Beta-blockers e.g. timolol eye drops are effective in the initial management open angle glaucoma.
Laser trabeculoplasty is used as an adjunctive measure. If there is a continuous increase in
intraocular pressure, surgical trabeculectomy is done.

Closed-angle glaucoma
Acute glaucoma is a medical emergency.
Symptoms include a sudden onset of photophobia, eye pain, headache, and nausea.
On palpation, the affected eye may appear very hard and fleshy.
A non-reactive, mid-dilated pupil suggests acute glaucoma.
The best diagnostic test is tonometry.
Narcotics used to control pain
The increased intraocular pressur is reduced with Mannitol, acetazolamide, timolol or Pilocarpine.
Atropine should be avoided since can dilate the pupil and worsen the glaucoma.

Cataracts
Cataract is a vision-impairing disease characterized by progressive thickening of the lens.
Oxidative damage of the lens occurs with aging and leads to cataract formation.
Patients usually complain of blurred vision and glare (particularly when driving at night).
Definitive treatment is lens extraction. The three commonly used procedures are
phacoemulsification, extracapsular cataract extraction and intracapsular ataract extraction.

Congenital cataract
The most common cause of white reflex in the pediatric population is congenital cataract.

Even if congenital cataracts are more common, all children with white reflex should be referred to
an ophthalmologist immediately to rule out a retinoblastoma.

Consider galactosemia for a newborn or young infant with failure to thrive, bilateral cataracts,
jaundice and hypoglycemia. Early diagnosis and treatment by elimination of galactose from the diet
are mandatory.

228
Leukocoria
Every case of leukocoria is considered a retinoblastoma, until proven otherwise; therefore, such
cases should be promptly referred to an ophthalmologist.
Retinoblastoma is the most common intraocular tumor of childhood. The underlying pathology
involves inactivation of the Rb suppressor gene, which may be familial or sporadic.
Retinoblastoma is a highly malignant tumor, and failure to diagnose and treat the disease early may
lead to death from liver and brain metastases.
The other manifestations of the disease may include strabismus, decreased vision, ocular
inflammation, eye pain, glaucoma, and orbital cellulitis.
The diagnosis is highly suspected with US or CT scan findings of a mass with calcifications.

Allergic conjunctivitis
It is an acute hypersensitivity reaction that is caused by environmental exposure to allergens.
Intense itching, hyperemia, tearing, and conjunctival edema and eyelid edema are characteristic.
Some patients complain of photophobia and a burning sensation.
visual disturbances do not occur with allergic conjunctivitis.
There is usually a family or personal history of asthma, seasonal rhinitis, atopic dermatitis, food
allergies, and urticaria.
The condition usually subsides in 24 hours, even without treatment.
Reducing exposure to allergens can prevent conjunctivitis. Patients should be advised to never rub
their eyes and to use topical antihistamines, artificial tears, and cool compresses.
Topical steroids are useful in allergic conjunctivitis and sometimes inconjunctivitis caused by
Adenovirus.

Intraocular foreign body


Be overly suspicious for an intraocular foreign body in patients with high-velocity injuries (drilling,
grinding, hammering, etc.).
High-velocity injury has a greater probability of globe penetration and intraocular foreign body
formation.
If the initial pen light examination does not reveal any conjunctival and corneal abrasions or foreign
body, proceed with fluorescein examination.

229
Corneal ulceration
Corneal ulcer occurs due to infection of bacteria, viruses, or fungi.
Corneal ulceration presents as a foreign body sensation, blurred vision, photophobia, and pain.
A history of contact lens use, recent trauma, or ocular disease may be present.
A history of contact lens use, recent trauma, or ocular disease may be present.
The eye is erythematous, and ciliary injection is present.
Purulent exudates are seen in the conjunctival sac and on the ulcer surface.

Keratitis
HSV infection of the eye is the most common cause of corneal blindness in the United States.
Characteristic dendritic pattern over the cornea on fluorescein staining of the eye.
Oral acyclovir and topical trifluridin 1% solution are used in treatment.
Never use oral or topical steroids.

Pterygium
Pterygium is a triangular encroachment of the conjunctiva onto the nasal side of the cornea.
It is usually seen in patients over the age of 35, and is usually associated with constant exposure to
wind, sand, sun and dust.

Retinitis
Ophthalmologic problems occur in approximately half of patients with advanced HIV infection.
Retinitis can occur as a complication of opportunistic infections in AIDS patients.
Most cases of retinal disease are believed to be a reactivation of a previously acquired infection.

In HIV patients, both HSV and VZV can cause severe, acute retinal necrosis associated with pain,
keratitis, uveitis, and funduscopic findings of peripheral pale lesions and central retinal necrosis.

CMV retinitis is the most common serious ocular complication of HIV-positive patients; however,
it is typically painless, and funduscopy shows fluffy or granular retinal lesions located near the
retinal vessels and associated hemorrhages. It does not usually cause initial conjunctivitis or
keratitis.

230
Both varicella-zoster (VZV) and herpes simplex virus (HSV) can cause severe, devastating
intraocular inflammation.
In an immunocompromised individual, HSV retinitis may be characterized by rapidly progressing
bilateral necrotizing retinitis (referred to as the "acute retinal necrosis syndrome").
The initial symptoms are keratitis and conjunctivitis with eye pain, followed by rapidly progressive
visual loss.
Funduscopy reveals widespread, pale, peripheral lesions and central necrosis of the retina.

Dacryocystitis
It is an infection of the lacrimal sac.
It usually occurs in infants and adults over the age of 40.
Acute dacryocystitis is characterized by the sudden onset of pain and redness in the medial canthal
region. Sometimes, a purulent discharge is noted from the punctum.
A few patients present with fever, prostration, and an elevated leukocyte count.
Staphylococcus aureus and beta-hemolytic Streptococcus are the usual infecting organisms.
It usually responds to systemic antibiotic therapy.

Episcleritis
Episcleritis is an infection of the episcleral tissue between the conjunctiva and sclera.
Patients complain of an acute onset of mild to moderate discomfort, photophobia, and watery
discharge.
Examination reveals diffuse or localized bulbar conjunctival injection and episcleritic nodules may
be present.

Hordeolum
Hordeolum refers to an abscess located over the upper or lower eyelid.
It is usually caused by Staphylococcus aureus.
It appears as a localized red, tender swelling over the eyelid.
It is treated with warm compresses.
Incision and drainage is performed if resolution does not begin in the next 48 hours.

231
Chalazion
Chalazion presents as lid discomfort.
It is a chronic, granulomatous inflammation of the meibomian gland.
It appears as a hard, painless lid nodule.

Trachoma
Trachoma is caused by Chlamydia trachomatis serotype A-C.
It is a major cause of blindness worldwide.
The active phase of the disease is characterized by follicular conjunctivitis and pannus
(neovascularization) formation in the cornea.
Concurrent infection occurs in the nasopharynx, leading to nasal discharge.
The diagnosis can be made by Giemsa stain examination of conjunctival scrapings.
Oral tetracycline or erythromycin should be started immediately.
Repeated infections can lead to scarring of the cornea.

Orbital cellulitis
Orbital cellulitis is an infection within the orbit and posterior to the orbital septum.
Infection of the paranasal sinuses is the usual underlying cause.
It is unilateral and more common in children.
It is manifested by an abrupt onset of fever, proptosis, restriction of extraocular movements and
swollen, red eyelids.
Vision may be normal during the early stages, but spread to the cavernous sinus may lead to a
decrease in visual acuity.
The causative agents are Staphylococcus aureus, Streptococcus pneumoniae, and Haemophilus
influenza.
Gram stain and culture of the blood and soft tissue aspirates may help identify the pathogen.
Imaging with CT scan of the orbit helps delineate the extent of infection.
Immediate administration of intravenous antibiotics (oxacillin or nafcillin) is necessary.

232
Uveitis
Uveitis presents as unilateral painful red eye with blurred vision, conjunctival injection, and
constricted pupils as well as marked photophobia.
Pain occurs even when shining light in the unaffected eye.
It is associated with HLA B27-related conditions e.g. ankylosing spondylitis.
Keratic precipitates ("mutton fat") and iris nodules may be seen.
is seen in severe anterior uveitis.
Diagnostic finding, which distinguishes anterior uveitis from other causes of red eye, is presence of
pus in the anterior chamber (Hypopyon) on slit lamp eye examination. Another important slit lamp
finding is presence of haze in the anterior chamber.
Treatment with topical and sometimes systemic steroids.

Postoperative endophthalmitis
Postoperative endophthalmitis is the most common form of endophthalmitis.
It usually occurs within six weeks of surgery.
It is an infection within the eye, particularly the vitreous.
Patients usually present with pain and decreased visual acuity.
Examination reveals swollen eyelids and conjunctiva, hypopyon, corneal edema and infection.
The vitreous can be sent for Gram stain and culture.
Based on the severity, intravitreal antibiotic injection or vitrectomy is done.

Optic neuritis
Optic neuritis is frequently the early manifestation of multiple sclerosis.
Visual loss develops typically faster, and no exophthalmos is present.
Optic neuritis typically presents in people aged 20-45 years. Females are more commonly affected.
Patients experience rapid impairment of vision in one eye (or rarely both) and pain on eye
movement.
There are marked changes in color perception.
Afferent pupillary defect and field loss occur, usually with central scotoma.

233
Multiple Sclerosis (MS)
Suspect multiple sclerosis (MS) in a female with multiple neurologic presentations that occur at
non-predictable time intervals.
The age of onset is usually between the third and the fourth decade, with recurrent attacks of focal
neurologic dysfunction occurring at erratic and non-predictable time intervals.
Exacerbation of these neurological deficits by hot weather or exercise are a useful clue.
Optic neuritis is frequently the early manifestation of multiple sclerosis, results in blurring of vision,
and is often associated with retrobulbar pain.
Visual loss develops typically faster, and no exophthalmos is present.
Visual, auditory and somatosensory evoked response tests are of value in identifying silent lesions.
The pathology involves demyelination of focal regions in the white matter of the brain, with a
propensity to involve the periventricular and subpial white matter of the cerebrum, the optic nerves,
brain stem and spinal cord.
Multiple sclerosis is best diagnosed with MRI. The MRI shows white matter disease (demyelinating
lesions), which is very characteristic for multiple sclerosis.

Cavernous sinus thrombosis (CST)


The cavernous sinus is the most common dural sinus to be infected and thrombosed.
It occurs due to hematogenous spread from an infected, inflamed sinus.
Facial infections such as nasal furuncles and dental infections are the most common source of
primary infection.
It is characterized by periorbital edema, diplopia, exophthalmus, and chemosis.
Funduscopy reveals papilledema and dilated tortuous retinal veins.
It is often difficult to differentiate cavernous sinus thrombosis (CST) from orbital cellulitis;
however, the symptoms of CST are bilateral, and there is involvement of cranial nerves III (ptosis)
and V, as well as early visual loss.

234
Neurofibromatosis
Intensive axillary freckling and cafe-au-lait spots are suggestive of neurofibromatosis, type 1.
Optic glioma is a well-known complication of neurofibromatosis, type 1. It occurs in 15% of
patients with this disease. A history of slowly progressive unilateral visual loss and
dyschromatopsia are characteristic. Exophthalmos is usually present on physical examination. The
optic disk may be normal, swollen, or atrophic.

Amblyopia
The most common cause of amblyopia is strabismus (e.g. esodeviation i.e., medial deviation of eye)
Other causes include: (1) errors of refraction and (2) opacity of media along the visual axis.
The standard of treatment is occlusion of the normal eye.

Strabismus is the most common cause of amblyopia, which is a decrease in the visual acuity of one
or both eyes. The normal eye assumes the function of being the "preferred eye", leading to
amblyopia and eventual loss of vision in the deviated eye.
Amblyopia occurs only during the first decade of life, a period during which the visual cortex is still
maturing. During this stage, any anomaly (e.g., strabismus, abnormalities of refraction, or a media
opacity within the visual axis) compromising the formation of a normal image on the retina may
result in vision loss; therefore, the prompt diagnosis and management of such conditions are
essential.
The cover test is an important tool for the detection of strabismus. It is performed by asking the child
to fix his sight on a target as the examiner alternately covers one eye while observing the movement
of the other. A normal eye keeps the same position and does not move, whereas a misaligned eye
shifts to refixate the object when the normal eye is covered.
The initial treatment is occlusion therapy, which entails covering the normal eye to correct the
associated amblyopia. This procedure forces the affected eye to correct itself in order to be properly
functional, thus stimulating proper visual maturation. Thereafter, the misalignment can be addressed.

In case the amblyopia is secondary to abnormalities of refraction, appropriate lenses should be


prescribed so as to have a well-formed retinal image. The normal or less severely affected eye is
then covered for the same aforementioned reason.

If the amblyopia is caused by an opaque media, surgical removal of the media should be performed
before occlusion therapy.

235
Hyperosmolarity
Type 2 diabetics are prone to develop non-ketotic hyperosmolar syndrome (NKHS) in association
with physiologic stressors like infection.
Such stress causes elevations in the serum cortisol and catecholamine levels, substances that serve as
insulin counter-regulatory hormones.
Severe hyperglycemia (serum glucose > 600 mg/dL) in the absence of significant ketosis can result.
The resultant osmotic diuresis can cause dehydration and serum hyperosmolarity (often > 320
mOsm/L).
In NKHS, altered consciousness ranging from confusion to coma is the main symptom of the
metabolic disturbance.
Acute hyperglycemia can also causes blurred vision due to myopic increase in lens thickness and
intraocular hypotension secondary to hyperosmolarity.

Sympathetic ophthalmia
Sympathetic ophthalmia is also known as "spared eye injury". It is characterized by damage of one
eye (the sympathetic eye) after a penetrating injury to the other eye.
It presents by anterior uveitis, but panuveitis, papillary edema, and blindness may develop.
It is due to an immunologic mechanism involving the recognition of 'hidden' antigens. Some
antigens contained within the eye are protected from immunologic recognition by natural barriers.
Breaking these barriers results in the uncovering of 'hidden' antigens. An immune response against
these antigens can involve autoantibodies as well as a cell-mediated reaction.

236
INITIAL SURVEY
In management of trauma patient always secure airway, breathing and circulation first in this order before
doing any other intervention (ABC of resuscitation).

An airway is always patent (secure) in a patient who is conscious and able to speak.
An airway is needed in all unconscious patients.
In the field the best option would be a needle cricothyroidectomy.
In emergency room an orotracheal intubation with pulse oxymetry monitoring is the best option.

If the patient is conscious and able to speak, but he is tachypneic and has noisy respiration, therefore he
is in need for chin lift and oxygen by a face mask to clear his airway of any obstruction and to cut down
his respiratory rate.

In a totally apneic patient like this, an appropriate airway should be restored as soon as possible.
Orotracheal intubation and surgical cricothyroidectomy are preferred way to establish an airway in
apneic patient with head injury.
An orotracheal intubation needs hyperextension of neck and should be done only after a cervical spine
injury is ruled out or in case of apneic patient.
Though the apneic patient may have head injury and is also at risk of having a cervical injury the benefits
of orotracheal intubation outweighs the risk and so an orotracheal intubation should be done with care not
to move the head. Another option would be to do a surgical cricothyroidectomy.

Needle cricothyroidectomy is an excellent field procedure to establish an airway especially in children.

Intubation over a fiberoptic bronchoscope although very effective cannot be performed so rapidly to meet
the needs of an apneic patient.

Acute blood loss


Hemodynamically unstable trauma patients should be started on IV crystalloids immediately.

When hemorrhage occurs (acute blood loss), tachycardia and peripheral vascular constriction
(sympathetic response) are the first physiological changes. These responses act to maintain the blood
pressure within normal limits until severe blood loss has occurred.

In hemorrhagic shock, systolic blood pressure does not typically decrease until at least 30% of the
intravascular volume has been lost; level of consciousness is typically not altered until greater than
30% of the circulating volume has been lost.
In acute blood loss, if bleeding accounts for approximately 25-30% of the patient's blood volume (i.e.
approx. 1500mL in 70 kg man) transfusion is required.
Crystalloid resuscitation is adequate for blood loss that is less than 25% of the patient's blood volume.
Femur fracture can account for 1L of blood loss and more if associated with pelvic fracture.
An effective general guideline for treating a bleeding patient is to initially resuscitate with crystalloid. 2L
are administered very quickly, in less than 10min.

237
If the patient continues to show signs of hypovolemia after infusion of 2L of crystalloid, blood
transfusion should be initiated.
Hematocrit is not a good indicator in acute blood loss.
Unexplained acidosis, in anemic patients, may indicate blood transfusion.

Positive pressure mechanical ventilation


Positive pressure mechanical ventilation increases intra-thoracic pressure, which decreases venous return
to the heart and thereby decreases the ventricular preload and consequently decrease cardiac output.
In patients with hypovolemic shock, this effect may cause circulatory collapse if the patients
intravascular volume is not replaced before mechanical ventilation is attempted.

Cervical spine
It is important to rule out a fracture or dislocation of cervical spine as the first priority because of grave
consequences of missing a cervical spine injury.
At the site of accident, when cervical spine injury is suspected, the patient should be transported with the
neck stabilized with a collar.
Tenderness on palpation of cervical spine is indicative of a spine injury.
Approximately 5-10% of unconscious patients who present to the ED as the result of a motor vehicle
accident or fall, have a major injury to the cervical spine.
Patient with head trauma can have coexisting cervical spine injury, which should always be assessed
before mobilizing the patient.
Because of grave consequences of missing a cervical spine injury, it is important to rule out a fracture or
dislocation of cervical spine as the first priority. In the Emergency room this can be done best with a
lateral view x-ray of spine. It is 85-90% sensitive in diagnosing cervical spine injury.

Tetanus immunization
All the patients with traumatic wound should be assessed for need of Tetanus toxoid and/or tetanus
immunoglobulin.
The immunization of tetanus after an injury can be done with tetanus immunoglobulin (TIG), which
provides passive, temporary and immediate immunity or Tetanus toxoid (TT), which provides active,
prolonged but delayed immunity.
The current recommendations are as follows:
History of Tetanus immunization Clean minor wounds High-risk wounds
< 3 doses of tetanus toxoid TT: Yes TT: Yes
TIG: No TIG: Yes
3 doses of tetanus toxoid TT: Yes if last dose >10 years ago TT: Yes if last dose > 5 years ago
TIG: No TIG: No

238
In patients who have at least 3 doses of tetanus toxoid TT vaccination (a complete childhood series),
tetanus immunoglobulin TIG is not indicated following wounds, even if length of time since the
patients last vaccination is greater than 10 years.
TIG is only indicated if < 3 doses of tetanus toxoid and high risk wounds.

HEAD TRAUMA
Glasgow coma scale (GCS)
Patients with head injury or neurological symptoms can be triaged according to the GCS.
The GCS is as follows:
Eye opening Verbal response Motor response
Spontaneous 4 Oriented 5 Obeys 6
To verbal command 3 Confused 4 Localization 5
To pain (or shout) 2 Inappropriate 3 Flexion 4
None 1 Incomprehensible sounds 2 Abnormal flexion (Decorticate) 3
None 1 Extension (Decerebrate) 2
None 1

Head injuries can be classified according to the GCS score as minor (GCS score > 14), moderate (GCS
score < 13 and > 9), and severe (GCS score < 8).

Hyperventilation
Hyperventilation can be required to prevent or treat intracranial hypertension.
Hyperventilation causes cerebral vasoconstriction and thus decreasing cerebral blood flow.

Diffuse axonal injury


Diffuse axonal injury is the most significant cause of morbidity in patients with traumatic brain injuries.
It is frequently due to traumatic deceleration injury and results in vegetative state.
Sudden acceleration-deceleration impact produces rotational forces that affect the brain areas where the
density difference is the maximum, thus most of the diffuse axonal injury occur at gray white matter
junction.
Clinical features of patients with diffuse axonal injury are out of proportion with the CT scan findings.
Patient loses consciousness instantaneously and later develops persistent vegetative state.
CT scan characteristically shows numerous minute punctate hemorrhages with blurring of grey white
interface. However, MRI is more sensitive than CT scan for diagnosing diffuse axonal injury.

239
Epidural hematoma
Epidural hematoma will show as a biconvex hematoma on CT scan.
Epidural hematoma results from rupture of the middle meningeal artery due to head trauma. Because it is
under arterial pressure, it expands rapidly. Fluid resuscitation increases the rate at which the hematoma
expands, thereby precipitating neurological signs.
Focal neurological signs result from herniation of the parahippocampal uncus through the tentorial
incisure, which causes pressure on the ipsilateral oculomotor nerve and posterior cerebral artery as
well as compression of the ipsilateral cerebral peduncle against the edge of the tentorium.
Immediate surgical intervention for an epidural hematoma.

Transtentorial (uncal) herniation


Lesion Signs
Compression of the contralateral crus cerebri Ipsilateral hemiparesis
against the edge of the tentorium
Compression of the ipsilateral oculomotor Loss of parasympathetic innervation causes
nerve (CN III) by the herniated uncus mydriasis (occurs early)
Loss of motor innervation causes ptosis and the
eye looking down and out, due to unopposed
actions of the lateral rectus [trochlear IV] and
superior oblique muscles [abducent CN VI)].
(occurs later).
Compression of the ipsilateral posterior Contralateral homonymous hemianopia
cerebral artery by the herniated uncus
Compression of the reticular formation Altered level of consciousness, coma

Subdural hematoma will show as a lenticular hematoma on CT scan.

NECK TRAUMA
Neck blunt trauma
Angiograms are done following neck trauma to rule out carotid artery injury.
Angiograms are usually done for penetrating injury.
The angiograms will evaluate the aortic arch and its branches.
When there is an intimal flap of the carotid artery, surgery is recommended to repair the vessel.
Intimal flap injury can lead to vessel occlusion and symptoms and signs of ischemia. Despite the patient
being neurologically intact, exploration is mandatory, to prevent a stroke.
Today, with the availability of stenting, this may be an alternative option to surgery.

Cervical spine should be assessed in all the patients with severe blunt trauma to neck.

240
SPINAL CORD INJURIES
Patients treated with high-dose methylprednisolone within eight hours of spinal cord injury have
significant and sustained neurological improvement. Thus its use is warranted as the first priority
after stabilizing the patient.

Anterior cord syndrome


Anterior cord syndrome is commonly associated with burst fracture of the vertebra.
Selective damage of the corticospinal and spinothalamic tracts.
It is characterized by total loss of motor function below the level of lesion with loss of pain and
temperature on both sides below the lesion.
MRI is the best investigation to study the extent of neurological damage

Central cord syndrome


Central cord syndrome is characterized by burning pain and paralysis in upper extremities with
relative sparing of lower extremities.
It is commonly seen in elderly secondary to forced hyperextension type of injury to the neck
particularly in patients with degenerative disease (e.g. spondylosis).

Brown Squard syndrome


Brown Squard syndrome is acute hemisection of cord.
It is characterized by ipsilateral motor and proprioception loss and contra lateral pain loss below the
level of lesion.

Syringomyelia
Syringomyelia is a disease in which CSF drainage from the central canal is disrupted, leading to a fluid
filled cavity that compresses surrounding neural tissue, resulting in impaired strength and
pain/temperature sensation in the upper extremity.
The most common causes of syringomyelia are Arnold Chiari malformations and prior spinal cord injury.
3% to 4% of patients with spinal cord injuries (e.g. whiplash) will develop post-traumatic syringomyelia.
Symptoms develop months to years later.
MRI is used for definitive diagnosis.

241
CHEST TRAUMA
Tracheobronchial rupture
The right main bronchus is the most commonly injured in rapid deceleration chest trauma.
Persistent pneumothorax and significant air leak following chest tube placement in a patient who has
sustained blunt chest trauma suggests tracheobronchial rupture.
Other findings include pneumomediastinum & subcutaneous emphysema (palpable crepitus below skin).
The diagnosis is confirmed with high-resolution CT scanning, bronchoscopy, or surgical exploration.

Aortic rupture
The most common vascular injury following blunt thoracic trauma is aortic rupture.
The possibility of aortic disruption must be considered in every victim of severe deceleration injury.
The aortic rupture is usually partial and the most common site are the ascending aorta just above the
aortic valve and just distal to the proximal left subclavian artery.
Clinical signs can include hypertension in the upper extremities due to pseudocoartication, and normo-
or hypotension in the lower extremities and weak, hoarse voice due to pressure on the recurrent
laryngeal nerve by periaortic hematoma (unilateral vocal cord paralysis).
Chest x-ray, which may show mediastinal widening, blurring of the aortic knob, depressed left main
stem bronchus, left pleural effusion, deviation of nasogastric tube, and an apical cap.
Suspicion must be high for this injury and the next test it to obtain a spiral CT scan of the chest.
Another option in an unstable patient is to obtain an ECHO.
Once diagnosed, surgery is the treatment. Paraplegia is the most feared complication of surgery.

Traumatic rupture of the diaphragm


One diagnosis, which is frequently missed in the ER, is traumatic rupture of the diaphragm.
Patients usually have respiratory distress.
The rupture may be small or large and is usually on the left side. The liver protects the right side.
After blunt trauma to the chest, if an x-ray shows a deviated mediastinum with a mass in the left lower
chest, one should suspect a diaphragmatic perforation with herniation of abdominal viscera.
An elevated left diaphragm may be the only abnormal chest x-ray finding.
A barium swallow will be diagnostic.
All diaphragmatic ruptures require treatment. Surgery is best done via the abdomen in acute conditions
and via the chest in chronic cases.

Rib fractures
Rib fractures account for more than half of thoracic injuries from non-penetrating trauma.
Children have more elastic ribs and are less likely than elders to have a rib fracture.
Older patients are more prone to rib fractures and also to the complications like atelectasis, pneumonia
and respiratory arrest.

242
Rib fracture can be particularly deadly in elderly. This is due to severe pain, which causes
hypoventilation and atelectasis/pneumonia.
Thus pain management is the prime priority in managing rib fracture in elderly.
Oral agents, such as opiates and/or NSAIDs are most commonly utilized. Local nerve block can be used
if oral or systemic analgesics are not useful. Intercostal nerve blocks provide pain relief without affecting
respiratory function, although it caries some risk of pneumothorax.
First or second rib fractures are sometimes associated with vascular injury to the subclavian vessels.
Pulmonary contusion represents parenchymal bruising of the lung, which may or may not be associated
with rib fractures.
In hemothorax, lung is the usual bleeding source and it can occur secondary to rib fracture due to
fractured ribs causing lung trauma.
Rib fractures, periosteal spiral fractures (in children who cannot yet walk), and chip fractures are other
indicators that child abuse may be a concern.

Flail chest
In a patient with blunt thoracic trauma, suspect flail chest if the patient remains tachypneic and
hypotensive despite aggressive fluid resuscitation.
In flail chest, an isolated segment of the thoracic wall will exhibit paradoxical motion during inspiration.
Pain control and supplemental oxygen are the most important early steps in management.
Mechanical stabilization (post pressure mechanical ventilation) of chest wall is considered in case of flail
chest (to make the flail segment moves normally).

Pulmonary contusion
Pulmonary contusion is not uncommon after high-speed car accidents.
It represents parenchymal bruising of the lung, which may or may not be associated with rib fractures.
The clinical manifestations develop usually in the first 24 hours (often with in few minutes); chest pain,
tachypnea, tachycardia, and hypoxia are characteristic.
Physical examination typically reveals chest wall bruising and decreased breath sounds on the side of
pulmonary contusion.
Chest x-ray reveals patchy irregular alveolar infiltrate.
CT scan may be employed to make an early diagnosis.
ABG typically shows hypoxemia and, itself, is an indication to suspect pulmonary contusion in trauma
patients.
Management involves close monitoring, and intubation with mechanical ventilation in severe cases.
This is very important to differentiate pulmonary contusion from adult respiratory distress syndrome.
ARDS manifests 24-48 hours from the trauma; besides that, bilateral lung involvement is present.

243
ABDOMINAL TRAUMA
Blunt abdominal trauma
The most commonly injured viscera in blunt abdominal trauma is the spleen, followed by the liver and
kidney.
In hemodynamically unstable (BP 90/60 mm Hg) victim of blunt abdominal trauma, fluid
resuscitation should be initiated followed by Ultrasound examination (FAST).
If ultrasound is not definitive (as in obese patients, it is of low quality), then diagnostic peritoneal lavage
(DPL) is performed.
Those patients with confirmed peritoneal bleeding would then undergo laparotomy for surgical repair.
Physicians may skip straight to laparotomy in patients who can not be stabilized or have clear evidence
of pneumoperitoneum or diaphragmatic rupture.

In hemodynamically stable victim of blunt abdominal trauma, CT scan of the abdomen is the single best
study to evaluate solid organ damage.

In hemodynamically unstable victim of blunt abdominal trauma and pelvic fracture, both intraperitoneal
and retroperitoneal bleeding must be ruled out.
FAST and DPL are the appropriate tests to establish intraperitoneal bleeding.
If these test are negative, then the next step in management would be pelvic angiogram to search for
retroperitoneal bleeding.
Pelvic angiogram is not only useful in detecting the source of bleeding, but the bleeding can also be
resolved via embolization of the offending vessel.

An emergent exploratory laparotomy is required for all blunt abdominal trauma patients with
hemodynamic instability and/or peritoneal irritation (diffuse abdominal tenderness, guarding and
rigidity).

Laparotomy is the appropriate treatment for intraperitoneal bleeding.


Angiography is the appropriate diagnostic and therapeutic option for retroperitoneal bleeding.

Spleen
Spleen plays an important role in immune surveillance.
As blood flows through the spleen, antigens are sampled by dendritic cells and presented to helper T-
cells, which subsequently activate B-cells to differentiate into antibody-secreting plasma cells.
In the absence of specific antibodies, phagocytes are unable to recognize and engulf encapsulated
organisms, thereby allowing these organisms to multiply unchecked in the circulation.
All patients undergoing splenectomy should receive vaccinations against S. pneumoniae, N. meningitidis,
and H. influenzae either prior or immediately following surgery.
Pneumococcal vaccine boosters are required every 5 years.

244
Blunt splenic trauma
The immediate management of splenic trauma caused by blunt abdominal trauma depends on the
patients hemodynamic status and response to IV fluids.
There is poor correlation between the amount of free fluid in the abdomen and the necessity for surgical
intervention.
If the patient is hemodynamically unstable and unresponsive to fluid administration, then emergent
exploratory laparotomy is required.
If the patient responds to fluid administration and, is hemodynamically stable (SBP 100 mm Hg) and
does not need blood transfusion, performing an abdominal CT scan is the best next step.
In the case of CT scan-documented splenic injury, the decision for operative intervention is determined
by the grade of the injury.
If operative intervention is required, every effort is made to repair the spleen rather than remove it,
especially in children.
If removal is unavoidable, post-operative immunization against encapsulated bacteria is mandatory.

Pancreatic injury
Any form of blunt abdominal trauma can compress the neck and/or body of the pancreas against the
vertebral column leads to pancreatic contusion, crush injury, laceration or transaction of the pancreas.
Pancreatic injuries may be missed by CT scan during the first 6 hours following the trauma.
Serial CT scans are often required to detect the evolution of the injury.
Untreated pancreatic injury can later be complicated by a retroperitoneal abscess or pseudocyst.
Fever, chills and deep abdominal pain suggest a retroperitoneal abscess.

Duodenal hematoma
Isolated duodenal hematoma is treated conservatively with nasogastric tube and parenteral nutrition,
however, it is important to exclude other organ injury.
Surgery in needed only if there are other associated injuries or if the hematoma does not resolve within 2
3 weeks with nasogastric tube and parenteral nutrition.

Duodenal rupture
The 2nd part of the duodenum, being retroperitoneal and therefore the least mobile, is the commonly
injured part of the duodenum in blunt abdominal trauma.
Duodenal rupture us frequently diagnosed by noting retroperitoneal air on abdominal x-ray.
Duodenal injuries may be better characterized using CT scan of the abdomen with oral contrast.

245
Penetrating abdominal trauma
All hemodynamically unstable patients with penetrating abdominal trauma must undergo immediate
exploratory laparotomy to diagnose and treat the source of bleeding, as well as to diagnose and treat
perforation of any abdominal viscus in effort to prevent sepsis.

Diagnostic peritoneal lavage, CT scan, ultrasound or local wound exploration has no role in the
management of patients with a direct gunshot wound to the abdomen; laparotomy should be done.
Any gunshot wound of the abdomen requires exploratory laparotomy.
Any gunshot wound below the 4th intercostal space (level of nipple) is considered to involve the
abdomen.

EXTREMITIES
Femoral nerve
The femoral nerve innervates the muscles of the anterior compartment of the thigh (i.e., quadriceps
femoris, sartorius, pectineus), and is therefore responsible for knee extension and hip flexion.
It provides sensation to the anterior thigh and medial leg via the saphenous branch.

Tibial nerve
The tibial nerve supplies the muscles of the posterior compartment of the thigh, posterior
compartment of the leg, and plantar muscles of the foot.
These muscles control flexion of the knee and digits, and plantar flexion of the foot.
The tibial nerve provides sensation to the leg (except medial side) and plantar foot.

Obturator nerve
The obturator nerve innervates the medial compartment of the thigh (i.e., gracilis adductor longus,
adductor brevis, anterior portion of adductor magnus), and controls adduction of the thigh.
It provides sensation over the medial thigh.

Common peroneal nerve fibular nerve


The common peroneal nerve gives rise to the superficial and deep peroneal nerves. These two nerves
supply the muscles of the anterior and lateral leg.
These nerves provide sensation to the anterolateral leg and dorsum of the foot.

Peripheral artery aneurysm


Peripheral artery aneurysm manifests as a pulsatile mass that can compress adjacent structures
(nerves, veins), and can result in thrombosis and ischemia.
Popliteal and femoral artery aneurysm are the most common peripheral artery aneurysms. They are
frequently associated with abdominal aortic aneurysm.

246
Open fracture
In cases of open fractures, stabilization and delayed primary closure is the optimal therapy.
Open fractures should not be closed primarily because of the associated increased risk of infection and
subsequent osteomyelitis.
In managing such a patient with an open fracture always repair the bone injury first followed by the
vasculature and then by nerve repair.
It is very important to repair the bone first as it will stabilize the injured area.
This should be followed by repair of the vasculature as delay in repair can lead to distal ischemia and
gangrene.
Nerve injuries though important are not an emergency.

Fat embolism
Fat embolism occurs 12 72 hours after long bone fractures.
The clinical picture includes respiratory distress, tachypnea, tachycardia, hypotension, mental changes
(confusion, agitation and may progress to seizures or coma), thrombocytopenia, subconjunctival
hemorrhage and diffuse petechiae in upper extremities.
Diagnosis is confirmed by presence of fat droplets in urine or presence of intra-arterial fat globules
on fundoscopy.
Serial x-rays show increasing diffuse bilateral pulmonary infiltrates within 24 48 hours of onset of
symptoms.

BURNS
Escharotomy
When circumferential full-thickness burns involving the extremities or chest are present, escharotomy
may be necessary to prevent vascular compromise and respiratory difficulty, respectively.

Thermal inhalation injury


Burn victims are at high risk for respiratory compromise because the supraglottic airway is very
susceptible to direct thermal injury and acute obstruction by edema and blistering. In contrast, the
subglottic airway is protected from injury by reflex closure of the vocal cords upon exposure to
extremely hot air.
In burn victims, clinical indicators of thermal inhalation injury to the upper airway and/or smoke
inhalation injury to the lungs include: burns to the face, singing of the eyebrows, oropharyngeal
inflammation/blistering, oropharyngeal carbon deposits, carbonaceous sputum, stridor,
carboxyhemoglobin > 10%, or history of confinement in a burning building.
The presence of one or more of these indicators warrants early intubation to prevent upper airway
obstruction by edema.

247
Carbon monoxide poisoning
Early symptoms of carbon monoxide poisoning are typically neurologic and include agitation, confusion
and somnolence.
All patients with acute CO poisoning should be treated with 100% oxygen via non-rebreather facemask.

Burn sepsis
Patients with severe burns may develop hypovolemia from increased capillary leak and the major cause
of morbidity and mortality is hypovolemic shock.
In the setting of adequate initial fluid resuscitation, bacterial infection (usually bronchopneumonia or
burn wound infection) leading to sepsis and septic shock is the leading complication.
In the first week following a burn, S. aureus is the most common cause of wound infection.
After one week of a burn P aeruginosa is the most common cause of wound infection.

The diagnostic criteria for systemic inflammatory response syndrome (SIRS):


1) Fever or hypothermia (temperature < 36C)
2) Tachypnea
3) Tachycardia
4) Leukocytosis, leukopenia or bandemia (immature white blood cells).

Patients meeting at least 2 of the 4 criteria are considered to have SIRS.


SIRS due to infection is called sepsis.
Sepsis is considered severe when there is associated end-organ dysfunction such as oliguria, hypotension
(SBP < 90 mm Hg), thrombocytopenia (< 80,000 plt/microliter), metabolic acidosis or hypoxemia.
Septic shock is diagnosed in cases of infection-related SIRS where hypotension is unresponsive to fluid
resuscitation and vasopressors are required to maintain SBP > 90 mm Hg.

BITES AND STINGS


Bee sting can causes anaphylactic shock.
This is type 1 hypersensitivity reaction classically mediated by widespread IgE-dependant mast cells
and basophil degranulation following exposure to an antigen.
Patients experiencing anaphylaxis typically first experience pruritus, flushing and urticaria.
Respiratory symptoms such as chest tightness, stridor and a full or lumpy sensation in the throat
follow and may progress to respiratory distress and failure due to laryngeal edema.
Widespread histamine-mediated peripheral vasodilatation causes hypotension and circulatory collapse.
Patients should be promptly treated with subcutaneous epinephrine. Epinephrine is effective as it exerts
both alpha and beta adrenergic effects resulting in vasoconstriction, bronchial smooth muscle
relaxation and a decrease in vascular permeability thereby counteracting the primary pathophysiologic
disturbances in anaphylaxis.

248
ORTHOPEDICS
Joint fluid
Synovial fluid findings in different type of arthritis

Synovial fluid Normal Non-inflammatory Inflammatory Septic


Appearance Clear Clear, yellow Clear to opaque Opaque,
yellow yellow to green
WBC/mcL < 200 < 2000 > 2000 > 2000,
Often > 50,000
Neutrophil (%) < 25 < 25 50 75
Glucose (mg/dL) Serum level Serum level Between 25 and < 25
serum level
Gram stain/culture Negative Negative Negative Positive
Etiology Osteoarthritis, Rheumatic disease
No pathology Bacterial infection
trauma Crystal arthritis

In line with table above, the indications for surgical washout are: positive gram stain or culture, glucose <
25 mg/dL, > 75% neutrophils, or > 50,000 leucocytes per mcL.

Compartment syndrome
Compartment syndrome results from increased pressure within a limited anatomic space, acutely
compromising the circulation and ultimately threatening the function of the tissue within that space.
Compartment syndromes may be caused by direct trauma (hemorrhage), prolonged compression of an
extremity or may occur after revascularization of an acutely ischemic limb.
Diagnosis of compartment syndrome is made predominantly on clinical findings of severe pain, pallor,
pulselessness, paralysis and paresthesia.
The presence of pulses does not rule out compartment syndrome and suspicion should be high for this
diagnosis.
Pain on passive extension of fingers is the most sensitive physical sign of compartment syndrome.
All dressing around the incision should be checked to ensure that there is no bleeding from the suture
line, that there in no collection of blood (hematoma) and that the dressing in not to tight.
Fasciotomy is the treatment and must be done urgently, done in the operating room.

Volkmann's ischemic contracture


Volkmann's ischemic contracture is the final sequel of compartment syndrome in which the dead muscle
has been replaced with fibrous tissue.
Displaced anterior fat pad is a radiographic sign of supracondylar fracture of humerus, secondary to
fall on an outstretched hand, which may be complicated by Volkmann's ischemic contracture
Treatment consists of immediate fasciotomy.

249
Reflex sympathetic dystrophy
Reflex sympathetic dystrophy is a vague painful condition seen as a sequel of infection or trauma which
may be minor.
It is characterized by pain, hyperesthesia and tenderness, which are out of proportion to the physical
findings.
It does not have an acute onset and is not associated with compartment syndrome.

Femoral neck fracture


Femoral neck fractures are intra capsular fractures, commonly seen in elderly patients.
Diagnosis can be made with AP and lateral X-rays.
Garden classification for femoral neck fracture (based on radiographic findings) is as follows:
1) Type 1: Valgus impaction of femoral head commonly seen with stress fractures.
2) Type 2: Complete but non-displaced femoral neck fracture.
3) Type 3: Complete fracture with displacement < 50 %.
4) Type 4: Complete fracture with displacement > 50%.

Type 1 or stress fractures can be managed with toe touch weight-bearing until there is radiological
evidence of complete healing.
Garden type 2, 3 and 4 are called unstable fractures
Garden type 2 and 3 need open reduction and internal fixation as soon as the patient is stabilized.
Garden type 4 fractures and for Garden type 3 fractures, which cannot be satisfactorily reduced need
primary arthroplasty as soon as the patient is stabilized.
Elderly patients with displaced femoral neck fractures should be treated with primary arthroplasty.

Femoral shaft fracture


Femoral shaft fractures are generally seen after severe direct trauma.
Hypovolemic shock can be seen in such cases due to severe blood loss.
Most of the fractures of shaft of femur can be managed with closed intra medullary fixation of shaft.
This allows for early mobilization, improved knee and hip function during recovery and decreased
hospital cost. In this technique, closed reduction of fracture segment is followed by intramedullary nail
insertion through small skin insertion over the greater trochanter.
Closed intramedullary nailing is preferred over open nailing in closed femoral shaft fracture as it is
associated with less chances of infection, less soft tissue disruption and do not compromise periosteal
circulation.
External fixation is indicated in certain cases of open fracture but not in closed fracture.

250
Humeral shaft fracture
Radial nerve is commonly stretched in mid shaft humerus fracture leading to limited extension of wrist
(wrist drop). However, most of these are reversible upon closed reduction of humerus and do not need
exploration or nerve repair. Most of them resolve spontaneously after a few months.
Compression of the brachial artery could cause pulse asymmetry (feeble distal pulsation), not
necessarily indicating serious vascular injury.
Gentle traction to attempt alignment of the fragments of a fractured long bone is important to prevent
further vascular and neurological damage and it should be attempted immediately.
If successful it takes 30sec and will, to some extent, restore the normal anatomy of the limb.
It would decrease the patient's pain and discomfort. Then, the pulses must be checked again.
Failure to perform the alignment easily indicates that muscles, nerves, or vessels are entrapped in the
fracture site. In such a case, operation is required.

Gentle traction to attempt alignment of the fragments of a fractured long bone is important to
prevent further vascular and neurological damage and it should be attempted immediately.

Radial nerve injury


It is common after humeral shaft fractures.
Most of them resolve spontaneously after a few months.
It manifests by wrist drop and sensory loss on the posterior arm, forearm and lateral dorsal hand.

Ulnar nerve injury


It is common after medial epiconyle fractures.
It manifests by claw hand as a result of paralysis of most of the intrinsic muscles of the hand.

ROTATOR CUFF TEAR


It presents with severe pain and weakness of the shoulder abduction.
It results from impingement of the supraspinatous tendon.
A common cause of rotator cuff tear is fall on outstretched hands.
Rotator cuff is formed by the tendons of supraspinatous, infraspinatous, teres minor & subscapularis
muscle.
Movements of shoulder like positioning the arm above the shoulder aggravate pain.
Range of motion is limited only on active movement but is normal on passive flexion.
A positive drop arm sign, with inability to actively maintain 90 degree of passive abduction, may be
present in large tears.
Rotator cuff tendinitis can be distinguished from rotator cuff tear by injecting lidocaine that will result
in improvement in range of motion in cases of rotator cuff tendinitis but no effect in range of motion
in cases of rotator cuff tear.
MRI is used for definite diagnosis.

251
MRI is the investigation of choice for defining soft tissue injuries of the knee.

Medial meniscus tear


Medial and lateral meniscus is made up of fibro cartilage, which allows for a congruous fit between the
femoral condyles and tibia. Medial meniscus is 'C' shaped whereas lateral meniscus is 'O' shaped.
Meniscal injuries often result from twisting injuries with the foot fixed.
Medial meniscus is more commonly injured as compared to lateral meniscus. Bucket handle tear is the
most common type of medial meniscus tear.
Patients generally complain of pain and swelling of knee with popping sound at the time of injury.
Tenderness is generally felt along the medial side of the knee in case of medial meniscus injury.
Effusion with meniscus injury takes about 24 hours to form and is often not significantly bloody (unlike
ligamentous tears and osteochondral injuries, which may also associated with a popping sensation, but
cause rapid joint swelling due to hemiarthrosis).
Locking of the knee joint on extension is generally seen.
McMurray's sign is snapping, felt with tibial torsion with the knee flexed at 90 degrees.
Initial treatment in acute injury is conservative with immobilization and bracing.
Recurrent or persistent locking needs MRI or arthroscopic assessment and treatment.
Surgery (arthroscopic or open) is often necessary to correct the problem.

Anterior cruciate ligament tear


Patient gives a history of forceful hyperextension injury to knee.
Effusion is rapidly seen within hours of injury.
Lachman's test, anterior drawer test and pivot shift test are used for clinical diagnosis.
Lachman's test is the most sensitive physical test for diagnosis of anterior cruciate ligament injury.

Posterior cruciate ligament injury


Posterior cruciate ligament injury is seen with posterior directed forceful injury to proximal tibia like a
dashboard injury.
Posterior drawer test will help in clinical diagnosis.

Medial collateral ligament injury


Medial collateral ligament injury is associated with abduction injury to the knee.
It presents with pain along the medial joint line and is aggravated by walking.
It is caused by Valgus stress applied on the lateral aspect of the knee when it is partially flexed.
Medial knee pain is aggravated by, Valgus stress testing.
MRI is the investigation of choice for defining soft tissue injuries of the knee.
Surgery is rarely necessary for MCL tear; bracing and early ambulation is the preferred treatment.

252
Lateral collateral ligament injury
Lateral collateral ligament injury is very rare and would be seen in adduction injury to the knee.
Varus stress test will help in clinical diagnosis of these patients.

Developmental dysplasia of the hip (DDH)


Developmental dysplasia of hip is a congenital disorder and characterized by subluxatable and
dislocatable proximal femur and acetabulum.
Risk factors of DDH are: Caucasian race, female infant, first-born infant, breech position and family
history of DDH.
Screening for DDH should be done in all newborn infants.
On inspection uneven gluteal folds are seen.
Physical maneuver helpful for diagnosing DDH are: Barlow's maneuver: Used to determine if a hip is
dislocatable. Ortolani's maneuver: to see if the dislocated hip is reducible. Positive maneuver is
associated with a "click" as a result of stretching and snapping of joint capsule and tendons.
Positive Barlow and Ortolani maneuvers should prompt a radiologic diagnosis with ultrasound.
Ultrasound is the most sensitive tool to diagnose DDH for infants less than 4 months of age.
Treatment may involve use of hip (Pavlik) harness, spica cast or surgical reduction.

Legg-Calve-Perthes disease Idiopathic avascular necrosis of the femoral capital epiphysis


It occurs between the ages of 4-12 year old with the mean age of 7 years.
It presents with limp and the pain is absent or mild in a male child.
Antalgic gait is and range of motion of the hip is limited on internal rotation, flexion and abduction.
The patient may have abnormal limb length, and thigh muscle atrophy.
AP and lateral films of the pelvis show widening of the joint space and collapse of the femoral head.
Treatment is aimed at maintaining placement of the femoral head within the acetabulum.

Slipped capital femoral epiphysis


Slipped capital femoral epiphysis typically occurs in an obese adolescent (> 12 years).
It presents with a painful limp.
Loss of abduction and internal rotation are very characteristic and external rotation of thigh is seen
when hip is flexed.
Frog-leg lateral view X-ray of hip joint is the imaging technique of choice for diagnosis which shows
displaced femoral epiphysis.
Slipped femoral capitis is an emergency condition and should be promptly corrected with external
fixation of the hip joint with pins in order to prevent avascular necrosis of the femoral head.

253
Trendelenburg sign
Drooping of the contralateral hemipelvis below is normal horizontal level during monopedal stance
constitutes a positive Trendelenburg sign.
It is caused by weakness or paralysis of the gluteus medius and minimus muscles, which are innervated
by superior gluteal nerve.

Osgood schlatter disease Apophysitis of the tibial tubercle


It is osteonecrosis of the tibial tubercle resulting in tenderness over it.
The exact cause is unknown, although overuse and trauma play an important role.
It occurs between ages of 10-16yrs, specifically those whose growth centers in the bones are still active.
Patients present with activity-induced pain felt at the tibial tuberosity, which is the site of insertion of the
quadriceps tendon (pain worsened by contraction or stretching of the quadriceps).
Radiography shows irregularities of the tubercle contour.
Osgood-Schlatter disease is a self-limited condition.
Most cases respond to conservative therapy such as activity reduction, and only few severe cases
require cast immobilization.

Green stick fracture


It is incomplete fracture in which one cortex is broken, while the opposite cortex & periosteum is intact.
It is commonly seen in children due to plasticity of bones.
Unique fracture patterns like green stick fracture, torus fracture and plastic deformation are seen in
children because their bones are more porous and less brittle.

Nursemaid's elbow
Nursemaid's elbow, also called as subluxed radial head, is one of the most common elbow injuries in
children between the ages of 1-5 years.
Nursemaid's elbow occurs when infants or children are lifted or pulled by the hand or arm.
The mechanism of the injury involves radial head subluxation.
The child typically keeps the hand in a pronated position, and refuses (cries out in pain) attempted
forearm supination.
Diagnosis is made clinically as radiographs are often normal.
Treatment of this condition is closed reduction in the following steps:
First, extend and distract the elbow
Next, supinate the forearm.
Finally, hyperflex the elbow with your thumb over the radial head in order to feel reduction as it occurs

No post reduction films are needed.

254
Stress fracture
Stress fracture or March fracture or insufficiency fracture is commonly seen in young active adults
involved in vigorous and excessive exercises.
Bones most commonly involved in stress fractures are metatarsal, navicular, neck of femur and tibia.
Stress fractures classically occur in the anterior part of the middle third of the tibia in patients
involved in jumping sports while in the posteromedial part of the distal third of the tibia in runners.
This process starts with resorption of bone to make room for deposition of new stronger bone, but
continued loading can lead to gross failure of the bone weakened by resorption leading to stress fractures.
Patients generally complain of pain with activity or exercise and improves with rest.
Swelling and point tenderness are generally present.
X rays are normal in the initial stages, in which case CT and bone scan can be diagnostic.
Treatment includes restricting weightbearing along with short leg casts.
Healing normally occurs within 3-4 weeks.

Stress (hairline) fractures of the metatarsals are not uncommon in athletes and military recruits; the
second metatarsal is most commonly injured. Treatment is with rest, analgesia & a hard-soled shoe.

Fracture of metatarsal
History of a traumatic event and presence of crunching are compatible with fracture of metatarsal.

Anterior dislocation of shoulder joint


Anterior dislocation of shoulder is caused by forceful abduction and external rotation of the
glenohumeral (shoulder) joint.
Anterior dislocations are more common than posterior dislocation but they are more likely to be
associated with the axillary nerve compression.
Axillary nerve injury leads to paralysis of the deltoid and teres minor muscles as well as sensory loss
over the lateral upper arm deltoid (axillary nerve distribution).
Anterior dislocations allow the arm to be externally rotated and inability for internal rotation.
An AP and lateral X ray can diagnose anterior dislocation of the shoulder.

Posterior dislocation of shoulder joint


Tonic clonic seizures may result in posterior dislocation of the shoulder joint.
With posterior dislocation of the shoulder, patient holds the arm adducted and internally rotated,
inability for external rotation and intact sensations and reflexes.
Neurovascular impairment is unusual.
An AP and axillary view would diagnose posterior dislocation.

255
Scaphoid fracture
Scaphoid is the most commonly fractured bone among all the carpals.
Patient generally complains of pain at wrist joint.
Tenderness in anatomical snuffbox is a very sensitive marker of scaphoid fracture.
Initial X rays may be normal or may show fine radiolucent line in non-displaced scaphoid fractures.
Scaphoid views are necessary to avoid missing the scaphoid fractures.
If x-rays are negative but fracture is clinically suspected, a non-displaced scaphoid fractures (fractures
< 2 mm displacement and no angulation) should be presumed and the patient should be treated with a
thumb spica cast immobilization and repeat x-rays after 2-3 weeks.
Open reduction and internal fixation is required if initial X ray shows fracture displacement.

Trochanteric bursitis
In a middle-aged adult, superficial unilateral hip pain that is exacerbated by external pressure to the
upper lateral thigh (as when lying on the affected side in bed) suggests trochanteric bursitis.
On physical exam there is pain with external rotation or resisted abduction.
Trochanteric bursitis is inflammation of the bursa surrounding the insertion of the gluteus medius onto
the femurs greater trochanter.

Osteosarcoma Osteogenic sarcoma


It is the most common malignant bone tumor in youth.
It is more common in femur (distal end), tibia (proximal end) and humerus (proximal end).
The most common symptom initially is pain which is persistent and may worse at night.
Patients do not have systemic symptoms.
Serum alkaline phosphatase is elevated.
X-ray shows destructive (osteolytic) lesion, periosteal new bone formation with periostealinflammation
Codmans triangle and may also shows sun-burst appearance.

Ewing's sarcoma
It is another frequent (neuroectodermal) tumor of adolescent population (usually <10yr age group).
The most common site is metaphysis and diaphysis of the femur.
The tumor is very aggressive and metastasize early to the lungs & lymph nodes.
Ewings sarcoma usually presents with local joint pain & swelling (usually around the knee).
Patients may initially be diagnosed with osteomyelitis, based on the misleading presentation of
intermittent fever, malaise, leucocytosis, anemia, and increased ESR.
X-ray shows osteolytic lesion with onion-skin appearance often followed by moth-eaten
appearance.

256
Giant cell tumor or Osteoclastoma
It is a tumor of adults and it involves the epiphysis of the distal femur or proximal tibia.
It is benign but locally aggressive. It frequently recurs even after local curettage.
Radiology characteristically shows a soap-bubble appearance due to osteolytic quality of the tumor.
The best way to handle this tumor is to recognize it and leave it to experts (know how to recognize bone
tumors and refer them to appropriate experts).

Pre-renal failure
Oliguria, azotemia and an elevated BUN/Creatinine ratio > 20:1 in the post-operative state most likely
indicate pre-renal failure from hypovolemia.
The first step in management new-onset oliguria is to change the Foley catheter to ensure it isnt clogged.
Next, if pre-renal azotemia is suspected, a careful fluid challenge is indicated.

POSTOPERATIVE COMPLICATIONS
Postoperative fever
Common causes of postoperative fever are: atelectasis (1), pneumonia (2-3), urinary tract infection (3-5),
deep vein thrombosis (4-6) wound infection (5-7), and drug fever (>7).

Nosocomial infections
Nosocomial infections are defined as infections acquired as a result of hospitalization, and they manifest
at least 48 hours after hospitalization by fever and leucocytosis.
A urinary tract infection is the most common type of nosocomial infection; however, it is easy to treat
and has the best prognosis.
Commensal organisms, like E. coli and S. aureus are the most commonly found organisms.

Intravenous catheters are one of the most common causes of nosocomial infections.
Femoral central venous catheters carry a higher risk of causing bacteremia than subclavian catheters.
Intravenous catheter infections are most commonly caused by cutaneous organism such as Staphylococci,
but femoral catheters may also cause gram-negative bacteremia.
Classically, patients with intravenous catheter infection will have erythema or tenderness at the IV site.

Respiratory quotient (RQ)


RQ is the steady state ratio of CO2 produced to O2 consumed per unit time and may be used to make
assessments of the metabolism taking place in particular organs or in the body as a whole.
In steady resting state the RQ depends on the proportion of metabolic fuels being oxidized for ATP
production.
A steady state RQ close to 1.0 indicates predominant oxidation of carbohydrates & net lipogenesis.
The RQ for protein and lipid as sole sources of energy are 0.8 and 0.7, respectively.

257
Postoperative pulmonary complications
Risk factors: smoking, pre-existing pulmonary disease, age > 50, thoracic or abdominal surgery, surgery
lasting more than 3 hours and poor general condition.
Narcotic analgesics decreases respiratory drive, deep inspirations and coughing.
Increasing functional residual capacity (FRC) can help to prevent atelectasis & consequently pneumonia.
Encouraging lung expansion postoperatively decreases the risk of such complications and this is done by:
incentive spirometry (the most effective), deep breathing exercises, coughing, postural drainage frequent
repositioning/early ambulation (these methods can be used to increase the functional residual capacity).
Continuous positive airway pressure (CPAP) is more costly and has higher rate of complications.
Moving from the supine to sitting position can increase the FRC by 20% to 35%. In normal adults, this
can amount to several hundred cubic centimeters of lung volume.

Hypoventilation is a frequent occurrence after abdominal hernia repair.


Early physiotherapy and respiratory exercises (blowing against resistance) are mandatory to prevent
atelectasis, mucous plugging, and possible subsequent pneumonia development.

Delayed emergency from anesthesia is characterized by hypoventilation, which is evident by decrease in


respiratory rate, hypertension progressing to hypotension, tachycardia progressing to bradycardia,
restlessness and pallor/cyanosis.

Necrotized surgical infection


Diabetes is an important predisposing condition.
Signs and symptoms suggestive of necrotizing surgical infection include:
1) Intensive pain in the wound accompanied with fever and tachycardia
2) Decreased sensitivity at the edges of the wound
3) Cloudy-gray discharge.

The necrotizing surgical infection is usually caused by mixed gram-positive and gram-negative flora.
The presence of crepitus raises the suspicion that clostridial infection may be present, but some
streptococcal and other gas-forming organisms may also produce local crepitus.
The most important step in the management of this condition is early surgical exploration to assess the
extent of the process and debride the necrotized tissues.

Acute adrenal insufficiency


Acute adrenal insufficiency is a potentially lethal postoperative complication.
Preoperative steroid use is the main cause. Exogenous steroids depress pituitary-adrenal axis and
stressful situation can precipitate an acute adrenal insufficiency.
Acute onset of nausea, abdominal pain, vomiting, hypotension, and hypoglycemia after a stressful
event (e.g. surgical procedure) in a patient who is steroid-dependant is typical.

258
GASTROINTESTINAL SYSTEM
Boerhaave's syndrome
Spontaneous rupture of the esophagus (Boorhave syndrome) can occur during episodes of vomiting,
particularly when the patient is resisting the vomiting reflex.
Esophageal rupture in this setting occurs in the distal esophagus, typically few centimeters above the
gastroesophageal junction.
Patient presents with severe retrosternal and upper abdominal pain.
Exam reveals suprasternal crepitus represents pneumomediastinum that may be seen.
Contrast studies with gastrograffin are indicated in addition to upper gastrointestinal endoscopy when
a patient with acute alkali ingestion is suspected of having esophageal perforation.
Once an esophageal perforation has been diagnosed, primary repair of the esophagus should be done
immediately (via thoracotomy).

NOTE: Rupture of the esophagus may present with a left pleural effusion that has a high level of
amylase. Also seen on the x ray, will be subcutaneous emphysema.

Mallory Weiss tear


Mallory Weiss tear is an incomplete esophageal tear, usually seen in patients with recurrent vomiting.
Increased intragastric pressure during vomiting can cause tears in the mucosa of the cardia, and
sometimes of the distal esophagus.
These are called Mallory-Weiss tears, and account for 10% of upper GI bleeds.
Bleeding stops spontaneously in 90% of patients; however, vasopressin, endoscopic injection, or
electrocautery may be required in a few cases.

Ludwigs Angina
Ludwigs angina is a rapidly spreading bilateral cellulitis to the submaxillary and sublingual glands.
The infection classically arises from an infected 2nd or 3rd mandibular molar tooth.
Causative organisms are Streptococcus (S. mutans) and anaerobes.
The patient will present with fever, dysphagia, drooling, stridor and cyanosis.
Physical examination reveals firm induration in the submandibular space; the presence of anaerobes
may cause crepitus due to gas formation.
The most common cause of death is asphyxiation.
Hematogenous spread can cause endocarditis, osteomyelitis and septic joints.
Patients should be monitored for respiratory difficulty and intubated if necessary.
Treatment include removal of the infected tooth and intravenous antibiotics.

259
Parotid tumor
Parotid surgery to remove the deep lobe of the parotid carries a significant risk of facial nerve palsy.
Facial nerve (CN VII) carries motor innervation to the muscles of facial expression.
Destruction of facial nerve causes unilateral facial droop.

Parotitis
Parotitis can be a post-operative complication, & the most common infectious agent is S. aureus.
Acute parotitis presents with painful swelling of the parotid gland that is aggravated by chewing.
High fever, tender, swollen and erythematous parotid gland are common.
This post-operative complication can be prevented with adequate fluid hydration and oral hygiene pre-
and post-operatively.

Sialolithiasis
Sialolithiasis or calculus formation within the ductal system of salivary gland is most common in
submandibular gland followed by sublingual and parotid gland.
Stones are more common in the Wharton's duct.
Secretions rich in mucin and gravity dependent drainage of ductal system are the reasons for high
frequency of stones in submandibular gland.
Sialolithiasis presents as post-prandial pain and swelling.
Stones in Wharton's duct are radio opaque, hence x-ray of the involved salivary gland is diagnostic.
Sometimes a calculus close to the orifice of Wharton's duct can be palpated over the anterior aspect of the
floor of the mouth.
Treatment consists of dilation and incision of the involved duct to remove the calculus.

Sialadenitis
Sialadenitis typically presents with acute onset of swelling, pain, and tenderness of the involved salivary
gland.
The duct opening appears erythematous and pus can be extracted through it.

Biliary colic
Biliary colic occurs when the gall bladder distended as it contracts against an obstructed cystic duct.
The pain is typically related to meals (fatty) and resolves completely between episodes (intermittent).
There are no signs of acute cholecystitis (Murphy's sign, fever, elevated WBC and bilirubin).
Choledocholithiasis symptomatic of biliary colic and without any systemic toxicity is treated with
analgesic and spasmolytics and elective surgery is done at a later date.

260
Acute cholangitis
Cholangitis is an infection of the common bile duct, most commonly arises secondary to obstruction of
the common bile duct by stone or stricture
It is characterized by Charcot's triad of fever, severe jaundice, and right upper quadrant abdominal pain.
Confusion and hypotension may also be observed in those with suppurative cholangitis, producing
Reynold's pentad.
Laboratory findings include leukocytosis and liver function test abnormalities that are suggestive of
cholestasis.
Ultrasound is typically the first imaging study employed to confirm the diagnosis.
The treatment of acute cholangitis includes provision of supportive care and broad-spectrum antibiotics.
Patients who do not respond to this treatment regimen (persistent abdominal pain, hypotension despite
adequate fluids, high fever, or mental confusion) should undergo urgent biliary decompression,
preferably by endoscopic retrograde cholangiopancreatography (ERCP). ERCP is minimally invasive,
has a low rate of complications, and allows for diagnostic confirmation.

Acute cholecystitis
Acute cholecystitis is secondary to gallstone formation in over 90% of cases, most commonly arising
when a gallstone impacts in the cystic duct.
Sudden onset right upper quadrant abdominal pain, fever, vomiting, and leukocytosis are highly
suggestive of acute cholecystitis.
Treatment for acute cholecystitis includes supportive care (nothing by mouth, with intravenous
administration of antibiotics, alimentation, and analgesics) followed by laparoscopic cholecystectomy
within few days of hospitalization.
Cholecystectomy should be performed immediately in cases of perforation or gangrene.

Acalculous cholecystitis
Acute acalculous cholecystitis is an acute inflammation of the gallbladder in the absence of gallstones,
most commonly seen in hospitalized and severely ill patients.
It is most commonly seen in the following conditions:
1) Extensive burns
2) Severe trauma
3) Prolonged TPN
4) Prolonged fasting
5) Mechanical ventilation

It generally presents with right upper quadrant abdominal pain with fever, leucocytosis, and abnormal
liver function tests.
It can lead to complications like gangrene, perforation, and emphysematous cholecystitis.
The initial investigation of choice is an ultrasonogram, which shows signs of acute cholecystitis and no
gallstones; however, CT scan and HIDA scans are more sensitive and specific for the diagnosis.
The emergent treatment of choice is percutaneous cholecystostomy.

261
Gastric outlet obstruction
Major causes are complicated peptic ulcer disease (PUD), strictures secondary to ingestion of caustic
agent, pyloric stenosis and pancreatic cancer.
Characterized by early satiety, nausea, non-bilious vomiting, and weight loss.
Succession splash indicates the presence of hollow viscus filled with both fluid and gas.
Unceasing vomiting causes dehydration and hypochloremic metabolic alkalosis. This type of alkalosis
is frequently accompanied by hypokalemia. Potassium supplementation is usually indicated.

Dumping syndrome
Dumping syndrome is a common post-gastrectomy complication.
Pathophysiology involves rapid emptying of hypertonic gastric contents into the duodenum and small
intestine, which leads to fluid shift from the intravascular space to the small intestine, release of
vasoactive polypeptides, and stimulation of autonomic reflexes.
Patients complain of post-prandial abdominal cramps, weakness, light headedness & diaphoresis.
Symptoms usually diminish over time and dietary modification is helpful to control symptoms.
In resistant cases, octreotide should be tried.
Reconstructive surgery is reserved for intractable cases.

ACUTE PANCREATITIS
Acute pancreatitis is most commonly secondary to gallstones or alcohol abuse.
However, it may be also secondary to drug toxicity, idiopathic etiology, hypertriglyceridemia,
infection, severe trauma or burns, hypercalcemia, pancreatic abnormality, tumors, toxins or due to post
operative or post ERCP etiology.
Hypertriglyceridemia > 1000mg/dl can cause acute pancreatitis and during the attack the triglyceride
levels can be as high as 3000-5000 mg/dl. Acute pancreatitis secondary to hypertriglyceridemia often has
normal levels of amylase.
Drug-induced pancreatitis is mild and usually resolves with supportive care.
Remember the following scenarios for drug-induced pancreatitis:
o Patient on diuretics ?furosemide, thiazides.
o Patient with inflammatory bowel disease ?sulphasalazine, 5-ASA.
o Patient on immunosuppressive agents ?azathioprine, L-asparaginase.
o Patient with a history of seizures or bipolar disorder ?valproic acid.
o AIDS patient ?didanosine, pentamidine.
o Patient on antibiotics ?metronidazole, tetracycline
Acute onset of pain that radiates to the back and is accompanied with nausea & vomiting, and tenderness
is strongly suggestive of acute pancreatitis.
Complications: exudative left pleural effusion [when amylase concentration is high], abdominal
compartment syndrome, secondary paralytic ileus, intraabdominal hemorrhage, shock and pancreatic
pseudocyst.

262
Hypotension that can complicate acute pancreatitis is thought to arise from intravascular volume loss
secondary to local and systemic vascular endothelial injury (vascular damage) which increases vascular
permeability and transudation of plasma. Systemic vasodilatation may also contribute.
CT scan is diagnostic for pancreatitis.

A cystic lesion involving the pancreas on CT scan in the setting of acute pancreatitis with systemic
toxicity must be considered an abscess and should be treated with external drainage.
In the absence of systemic toxicity, a pseudocyst may be presumed and managed expectantly.

Pancreatic cancer
Complaints of daytime fatigue, anorexia, significant weight loss and visceral-type abdominal pain
interfering with sleep and abdominal examination reveals tender, full epigastrium. This combination
suggests a malignancy affecting the upper GI tract or associated solid organs.

Pancreatic cancer classically presents insidiously with a combination of constant visceral epigastric pain
radiating to the back, jaundice due to extrahepatic biliary obstruction, and anorexia with weight loss.
The known risk factors for the development of pancreatic cancer include: family history, chronic
pancreatitis, smoking, diabetes mellitus, obesity, and a diet high in fat. Alcoholism is not a risk factor for
pancreatic cancer
The most commonly used tumor marker for pancreatic cancer is the cancer associated antigen (CA) 19-9
[CA 19-9 can be elevated in patients with jaundice but no pancreatic cancer]. Postoperative monitoring of
pancreatic cancer with CA 19-9 may be helpful in evaluating the tumor response to chemotherapy.
Commonly, CT scan is effective in detecting bile and pancreatic duct dilation, mass lesions within the
pancreas, and indications of extrahepatic spread (eg, metastases or ascites).
Endoscopic retrograde pancreatography (ERCP) s most commonly reserved for
those patients who have already undergone a nondiagnostic CT scan.

Appendicitis
In acute appendicitis, the initial peri-umbilical pain is referred pain and visceral in nature; however,
pain shifts to the right lower quadrant with involvement of the parietal peritoneum and becomes
somatic in nature.

Acute appendicitis is a clinical diagnosis.


Patients with a classic presentation should be operated on immediately to remove the inflamed appendix
and prevent appendicular rupture.

Patients who present more than five days after the onset of symptoms, and have localized right lower
quadrant findings, should be treated with IV hydration, antibiotics and bowel rest. Non-operative
management is usually effective.
CT scan may reveal an abscess that can be drained percutaneously.

263
Antibiotics should cover enteric gram-negative organisms and anaerobes.
A second/third generation cephalosporin or a fluoroquinolone plus metronidazole are usually used.
Cefotetan has a good coverage of gram-negative organisms and anaerobes; therefore, this can be used as
monotherapy in this case.

In case of, gangrenous, ruptured, retrocecal appendix and the cecum adjacent to the appendix is
edematous and livid; experience has shown that right hemicolectomy with ileotransverse anastomosis
has best postoperative results, when resection of part of ascending colon is required.

Acute ruptured appendicitis may be complicated by pelvic abscess.


Pelvic abscess presents with abdominal pain, malaise, low-grade fever and rectal pain on defecation.
Finding of tender, boggy and fluctuant mass palpable only with the tip of the examining finger on rectal
examination indicates abscess in the rectovesical pouch.
Surgical drainage is indicated in the presence of a pelvic abscess.
Percutaneous drainage is the standard treatment approach for pelvic abscess.

Small bowel obstruction


Proximal SBO characterized by vomiting, abdominal discomfort, and abdominal contrast filling on X-ray
Mid or distal SBO typically presents as colicky abdominal pain, vomiting, abdominal distension,
constipation-obstipation, and dilated loops of bowel on abdominal x-ray.
Adhesions are the most common cause of SBO. They may be congenital in children (e.g. Ladds bands),
but typically results from abdominal operations or inflammatory processes.
The initial treatment for both complete and partial small bowel obstruction is IV fluids resuscitation,
electrolyte replacement, NPO and placement of nasogastric tube to decompress the stomach, prevent
aspiration and prevent further abdominal distension.
Colicky or paroxysmal abdominal pain with episodic hyperactive bowel sounds attributable to peristaltic
rushes, nausea, vomiting, abdominal distension, diffuse abdominal tenderness and obstipation are typical
signs of complete small bowel obstruction.
Surgery is typically required ultimately in patients with complete obstruction in order to rule out and
prevent strangulation (but patient should be prepared).
NOTE: Preparation for surgery with fluid and electrolyte replacement and gastric compression
will make the patient a better surgical candidate, UNLESS; strangulation or perforation of the
bowel is likely.

Upright abdominal x-ray is the first diagnostic test when intestinal perforation is suspected.

264
DIVERTICULITIS
The diagnosis of uncomplicated acute diverticulitis of the colon can usually be made clinically.
Diverticulitis is associated with a history of constipation (rather than diarrhea) and little fiber in their
diet, and usually produces left lower quadrant pain and fever.
It is usually seen in elderly patients.
Uncomplicated diverticulitis may be managed with bowel rest, IV antibiotics covering gram negative
and anaerobic organisms (e.g. ciprofloxacin & metronidazole) and observation till symptoms resolve.
When patients fail to respond to antibiotics, a complication must be suspected, such as formation of an
abscess, fistula, or frank perforation.
The best diagnostic test to evaluate such complications is via CT scan.
Any abscess in any location in the body require drainage.
CT guided percutaneous drainage is the standard treatment approach for pelvic abscess.
Sigmoidoscopy and colonoscopy can be disastrous in the presence of acutely inflamed bowel and are not
recommended during the acute episode because the risk of perforating the colon is very high.
CT scan is the best test for diagnosing and evaluating the abdomen of patients during an acute
episode of diverticulitis.

Psoas abscess
Psoas abscess usually resulting from hematogenous spread of bacteria.
S. aureus is the most common offending pathogen.
Patient presents with fever and right or left lower quadrant pain or back pain.
Exam does not reveal guarding, rigidity or rebound tenderness in the lower.
Deep abdominal palpation is required to elicit tenderness due to the deep location of the psoas on the
posterior abdominal wall. Psoas sign is positive.
CT scan is the investigation of choice to diagnosis intraabdominal abscess.
If clinical suspicion of psoas abscess is high and CT is negative, laparoscopy is indicated.
Any abscess in any location in the body require drainage.
Patients with psoas abscess are treated with drainage and systemic antibiotics.
CT guided percutaneous drainage is the standard treatment approach for pelvic abscess.

Meckel's diverticulum
Meckel's diverticulum, in which gastric mucosa is present in 50% of these diverticula. The acid
secretion may cause ulceration of the distal ileum with possible subsequent painless frank
hematochezia in children or in adults with intestinal obstruction.
Upper and lower endoscopies can not reveal lesions of the jejunum or ileum.
Technetium pertechnetate scintigraphy is the best diagnostic test for Meckel's diverticulum.

265
Hematochezia
Hematochezia is defined as bright red blood in the stool, it is typically represents for lower GI bleeding.
The three most common reasons for acute lower GI bleeding in patients over 50 years are diverticulosis,
angiodysplasia and neoplasms.
If direct tests such as colonoscopy fail to identify the site of the bleeding, then Technetium-99 labeled
erythrocyte scintigraphy should be used.
Labeled erythrocyte scintigraphy (tagged RBC scan), although not a very specific study, could be
helpful to define the site of bleeding, so that region can be further evaluated and treated by colonoscopy
or angiography.

Postoperative ileus
Postoperative ileus follows most surgeries where the peritoneal cavity is entered.
An ileus is a functional defect in bowel motility without an associated physical obstruction.
Signs and symptoms of postoperative ileus include, nausea, vomiting, abdominal pain, abdominal
distension, failure to pass flatus or stool, and hypoactive or absent bowel sounds (in contrast, mechanical
obstruction causes hyperactive tinkling bowel sounds).
Causes of defective bowel motility in the post-operative setting: splanchnic nerve sympathetic tone
following violation of the peritoneum, local release of inflammatory mediators and post-operative
morphine and other narcotics.

Bowel ischemia
Always consider bowel ischemia and infarction as an early complication of operation on the
abdominal aorta.
Infarction of the bowel results from inadequate colonic collateral arterial perfusion to the left and
sigmoid colon after the loss of inferior mesenteric artery during aortic graft placement.
Patients presents with abdominal pain and bloody diarrhea.
Fever and leucocytosis may also be present.
Colonoscopy shows discrete segment of cyanotic and ulcerated bowel.

Acute mesenteric ischemia


It classically presents with severe acute-onset of mid-abdominal pain out of proportion to physical
examination findings.
The most common cause is an embolus from the heart (e.g. in case of AF).
Progression to bowel infarction causes peritoneal signs and the passage of bloody stool.

266
Chronic mesenteric ischemia
Worsening postprandial pain that leads to avoidance of food is characteristic of chronic occlusion of
visceral arteries (abdominal angina).
The diagnosis of chronic mesenteric ischemia is suspected in patients with unexplained chronic
abdominal pain, weight loss, and food aversion.
Evidence of associated atherosclerotic disease is usually present.
Physical findings are usually nonspecific. Abdominal examination may reveal a bruit (50 % of patients).
Diagnosis requires angiography or a Doppler ultrasound.

Paralytic ileus
Absent bowel sounds with gaseous distension of both small and large bowels indicate a paralytic
(adynamic) ileus.
Paralytic ileus classically follows abdominal surgery, but it may occur in cases of retroperitoneal
hemorrhage associated with vertebral fracture.
Failure to pass stool or flatus, abdominal distension and absent bowel sounds.
Abdominal x-rays show air-fluid levels and distended gas-filled loops of both small and large
intestine.

267
BREAST
Evaluate a breast mass

Malignant lesions are generally hard, irregular and fixed.


Physical examination alone is insufficient to differentiate benign and malignant lesions.
A mammogram should be the first step in evaluation of all patients over age 35 with a palpable breast
lump. This is particularly important if there is a family history of breast cancer.
Ultrasound can distinguish a solid from cystic lesion.
FNA for cytological evaluation of palpable lesion can be done with or without preceding
ultrasonography.

Fibrocystic disease
It is benign condition. Very common in pre-menopausal females.
Patients present with bilateral painful, rubbery, firm, mobile masses; who experiences more tenderness
during her menses.
Fibrocystic disease is treated with aspiration of the cyst, which should yield clear fluid (serous
greenish and non-bloody) and result in the disappearance of the mass.
Afterwards, patients are typically observed for 4 to 6 weeks.

268
Fat necrosis of the breast
Fat necrosis of the breast is associated with breast trauma or surgery (e.g. reduction mammoplasty).
Fat necrosis can mimic breast cancer in its clinical and radiographic features as fixed mass, skin or
nipple retraction, evidence of calcification on mammography and appears solid (hypoechoic mass) on
ultrasonography.
Breast malignancy has micro-calcification, while fat necrosis has coarse calcification.
Excisional, core or fine needle biopsy is diagnostic and shows fat globules and foamy histocytes.
No treatment is indicated, as it is self-limited condition.
Standard follow up: monthly self- examination, annual clinical breast examination and routine screening
mammography starting at age of 40.

Inflammatory breast carcinoma


It is an uncommon form of breast cancer.
It presents as erythematous and edematous plaque with a peau dorange appearance overlying a
breast mass, commonly with axillary lymphadenopathy.
Spontaneous nipple discharge, it is another sign indicates the possibility of breast cancer
(nipple discharge in non-lactating woman should always raise suspicion for breast cancer).
25% of patients with this condition will have metastatic disease at the time of presentation.
Clinically inflammatory breast carcinoma can not be differentiated from infectious processes, therefore
biopsy for histology should be done first to exclude or confirm diagnosis.

Mastitis
Mastitis is not a contraindication to breastfeeding.
Some physicians actually encourage breastfeeding since this prevents engorgement, along with other
subsequent symptoms such as fissures and cracking of the nipple along with analgesics and antibiotics.

269
CARDIOTHORACIC SURGERY
Aortic dissection
Aortic dissection presents as sudden, tearing chest pain that radiates to the back.
Leg weakness can occur if the dissection progresses to involve the arteries feeding the spinal cord.
Check blood pressure in both arms (difference in BP greater than 30 mmHg between the arms is an
important clue)
Auscultate for aortic regurge (early decrescendo diastolic murmur at the left sternal border).
Mediastinal widening is often, though NOT always, seen on chest X-ray.
Normal EKG.
Transesophageal echocardiography is the preferred diagnostic tool or CT with contrast (invasive).
Before performing TEE, hypertension should be controlled by intravenous labetalol.
Type A dissections involve the ascending aorta and are treated with medical therapy and surgery.
Type B dissections involve the descending aorta and are treated with medical therapy only.
Intravenous labetalol (beta-blocker) is the most appropriate intervention for acute aortic dissection.

Abdominal aortic aneurysm (AAA)


AAA classically presents with a pulsatile abdominal mass.
Smoking cessation is the intervention with the greatest likelihood of slowing AAA progression.
Hypertension in patients with AAA should be treated with beta-blockers.
The study of choice for diagnosis and follow-up of abdominal aortic aneurysms is an abdominal
ultrasound.
Indications of surgery in patients with AAA include: diameter > 5 cm, presence of symptoms, or rapid
rate of growth.

A patient, who is hypotensive with abdominal pain and has a CT scan showing enlarged aortic silhouette,
has a diagnosis of ruptured abdominal aortic aneurysm. This patient needs immediate surgery.
CT scanning is only done in stable patients.
In unstable patients, the presence of aneurysm can be confirmed by bedside ultrasound.

After an AAA repair, diarrhea with blood in the stools should raise the suspicion of ischemic colitis. If
the CT scan is inconclusive a sigmoidoscopy/colonoscopy is recommended.

Spinal cord ischemia with lower spastic paraplegia is a rare complication of abdominal aortic
aneurism surgery.
The artery of Adamkiewicz arises from the aorta and supplies the anterior circulation of the middle and
lower spinal cord. Diminished flow through this artery may result from its thrombosis, ligation, or
systemic hypotension. Resultant neurologic dysfunction is due to ischemia of antero-lateral structures
of the spinal cord. It includes lower spastic paraplegia, pelvic organ dysfunction, and loss of
temperature and pain sensation over the lower extremities. Vibratory and proprioceptive sensation is
preserved because posterior circulation of the spinal cord is not affected.

270
Ruptured abdominal aortic aneurysm
When a patient represents with a pulsatile abdominal mass and hypotension, a presumptive diagnosis
of ruptured abdominal aortic aneurysm must be entertained and the patient should be taken
immediately to the operating room.

Central line placement


Complications from central lines may include pneumothorax, hemothorax, sepsis and tamponade.
Radiologic confirmation of the position of the catheter is a standard step in protocols after insertion of
central lines.
A cardinal rule for the prevention of myocardial perforation is radiologic confirmation that the catheter
tip is proximal to the cardiac silhouette. Ideally, the catheter tip should lie in the superior vena cava.
Prior to use of any newly placed central lines, the chest x-ray must be reviewed to ensure the absence
of pneumothorax, hemothorax and good positioning of the catheter tip.

Acute mediastinitis
Acute mediastinitis may occur following heart surgery.
Signs and symptoms include fever, chest pain, leucocytosis and mediastinal widening on chest x-ray.
Like an abscess, the treatment of mediastinitis requires drainage, debridment and antibiotic therapy.

Massive hemoptysis
A case of massive hemoptysis is a medical emergency.
Massive hemoptysis is defined as hemoptysis of greater than 100 to 600 ml of blood per 24 hrs.
Most of the times the volume is not measured; therefore any suspicion of massive hemoptysis based up
on history needs to be treated as an emergency.
Worldwide, pulmonary tuberculosis is one of the most common causes of massive hemoptysis.
RIGID BRONCHOSCOPY is the initial intervention of choice in massive hemoptysis because it allows
for rapid visualization of the bleeding site and it allows for control of bleeding through laser cautery or
other interventions.
Flexible bronchoscopy is used in the diagnostic aspects of hemoptysis and it is not a good choice for
emergency management of hemoptysis. Because of its small lumen it cannot provide much scope for
interventions.

N.B.: blood from the airway has an alkaline pH and is bright red and frothy in appearance.
N.B.: blood from GI origin is acidic, usually has a darker color and may contain food particles.

271
Solitary pulmonary nodule
Solitary pulmonary nodule defined as a 3cm or less coin-shaped lesion in the middle to lateral one third
of the lung surrounded by normal parenchyma.
Most of them are benign. Calcification of the nodules favors a benign lesion. "Popcorn" calcification is a
hamartoma while "bulls eye" is a granuloma.

In low risk patients (i.e. < 40 years and non smokers) a solitary pulmonary nodule is not a sign of
immediate alarm. The best approach is to ask for an old x-ray. If there is no change in it for the last 12
months it is considered benign. It is followed by a CXR every 3 months, for the next 12 months, and if
there is no growth or no symptoms it is left as such.

High-risk patients i.e. smokers, require a full investigation work up. It starts with a CT scan chest just
to get better picture of the lesion and its extent, followed by fine needle aspiration. If the lesion is
missed, an open-lung biopsy is an option.

Pilonidal disease
Acute pain and swelling of the midline sacrococcygeal skin and subcutaneous tissues are most
commonly due to pilonidal disease.
The acute presentation involves infection of a dermal sinus tract originating over the coccyx.
Treatment by drainage of abscesses and excision of sinus tract.

Clavicular fracture
Clavicular fracture is one of the most common injured bones in the body.
It commonly occurs as a result of fall on outstretched arm or direct blow to the shoulder.
The contralateral hand is classically used to support the weight of the affected arm.
Clavicular fracture presents with irregularity, crepitus and fullness over the fracture site and decreased
movements of the arm.
A careful neurovascular exam should accompany all fractures to the clavicle due to its proximity to the
subclavian artery and brachial plexus.

272
VASCULAR SURGERY
Occlusive arterial disease
The goal of therapy for occlusive arterial disease of the lower extremities is to relieve pain, prevent
limb loss, and maintain bipedal gait.
Most patients with intermittent claudication alone remain stable or even improve with appropriate
conservative treatment.
Intermittent claudication is best treated with aspirin and an exercise program.
Measurement of ankle brachial index may give some idea about the extent of disease severity.
An angiogram is usually not required for patients with intermittent claudication if the patient has no
evidence of ischemia, infection, gangrene, or loss of pulse. If surgery is not planned, a duplex arterial
study may suffice.
In patients with severe claudication and marked involvement of the tibial vessels, the disease can
progress to gangrene. If there are symptoms of nocturnal pain, ulceration of gangrene occurs,
angiogram and possible surgery may be required.

Deep vein thrombosis (DVT)


Preoperative DVT prophylaxis is dependent on the patient's individual risk factors and the type of
surgery being performed.
Surgical patients can be categorized according their risk of DVT:
1) Low risk: Minor surgery in a patient < 40 years old with no additional risk factors present.
2) Moderate risk: Patients > 40 years old, one or more additional risk factors, minor/non-major surgery.
3) High risk: Patients > 40 years old, additional risk factors, major operation (e.g., orthopedic procedures
of the lower extremity).
In low risk patients, prophylaxis other than early ambulation usually is not recommended.

For moderate risk patients, undergoing general abdominal, thoracic, or gynecologic surgery, either
LMWH or unfractionated heparin is recommended.
Patients in whom bleeding risks are unacceptable (e.g., intracranial/spinal cord surgery) should receive
intermittent pneumatic compression due to its efficacy and safety profile in this group of patients.

High-risk patients who are receiving general surgery can be given LMWH for prophylaxis.
Patients who are undergoing orthopedic procedures of the lower extremity (such as this patient) should
receive either LMWH or oral warfarin.

273
Venous ulcer
Ulcers on the medial aspect of the leg are generally from venous disease.
Edema, stasis dermatitis and venous ulceration result from lower extremities venous insufficiency.
The prolonged venous stasis results in pooling and extravasation of blood into the surrounding skin.
The venous hypertension prevents ulcer healing and has to be corrected before the ulcer can heal.
Venous hypertension may be due to vein varicosities and incompetent perforators.
Venous hypertension can be detected by observation of large venous varicosities and assessment of the
saphenous vein and its branches.

OTOLARYNGOLOGY
Nasopharyngeal carcinoma (NPC)
It is a tumor seen in the three-to-four decades of life and the incidence is higher in Asians.
Nasopharyngeal cancer usually presents initially as a painless neck mass.
Patients often present with recurrent otitis media, recurrent epistaxis and/or nasal obstruction.
Their etiology has been strongly linked to Ebstein-Barr virus infection.

Nasal septal perforation


The etiology of nasal septal perforations can be traumatic, iatrogenic (e.g. rhinoplasty), inflammatory
(e.g. sarcoidosis, Wegener granulomatosis), malignant, and cocaine-related.
It can cause a whistling sound with inspiration, crusting, bleeding, nasal discharge, parosmia, and
neuralgia.
If a patient develops a whistling noise during respiration following rhinoplasty, one should suspect nasal
septal perforation resulting from a septal hematoma.
Larger perforations can lead to atrophic rhinitis.

Nasal polyp
Nasal polyps are the most common nasal tumors.
It can be caused by an allergic or infectious process.
A history of allergic disorders (e.g., allergic rhinitis, asthma) is often present.
Sinonasal polyps may be associated with human papilloma virus (HPV) infection.
Septal papillomas are invariably a squamous papilloma with no malignant potential; however, most
lateral papillomas are inverted papillomas with malignant potential.
The typical symptoms are recurrent episodes of rhinitis, chronic nasal obstruction, altered taste
sensation, and diminished sense of smell. There is also a persistent postnasal drip.
Symptomatic treatment consists of steroids and decongestants.
Steroids are also indicated for recalcitrant nasal polyps.
If medical management fails, surgical removal of the polyp is indicated.

274
Aspirin is contraindicated in patients with asthma and nasal polyps because aspirin use may lead to
severe bronchospasm (aspirin triad or Samter's triad) in such patients.

Juvenile angiofibroma (JNA)


Any adolescent who presents with a nasal obstruction, visible nasal mass, and frequent nosebleeds
(epistaxis) is considered to have a juvenile angiofibroma (JNA), unless proven otherwise.

This is typically found in the back of the nose or upper throat (nasopharynx) of adolescent boys.
It is a benign growth, but is capable of eroding and locally invading.
JNAs are deemed potentially very dangerous because these are composed of many blood vessels which
may bleed readily. In addition, its common areas of occurrence are difficult to access surgically. For
these reasons, such tumors should only be touched by a specialist.

In some cases, no treatment is necessary.


Treatment is required if the angiofibroma is enlarging, obstructing the airway, or causing chronic
nosebleeds. Surgical treatment includes removal of the tumor. Removal is often difficult because the
tumor is unencapsulated and may be deeply invasive. Recurrence of the tumor after surgical resection is
common.

Torus palatinus
Torus palatinus is a benign bony mass (exostosis) located on the hard palate.
The cause of this mass is unknown and is unrelated to trauma.
It is basically an outgrowth from the hard palate and seen in a few adults.
The structure usually has been present for sometime and has a benign history.
The thin epithelium overlying the bony growth will tend to ulcerate and be slow to heal due to poor
vascularity.
No medical or surgical therapy is required unless the growth becomes symptomatic or interferes with
speech or eating.

275
UROLOGY
Urethral injury
Urethral injury can be of following types:
1) Anterior urethral injury due to injury to urethra anterior to the perineal membrane. Anterior urethral
injuries are most commonly due to blunt trauma to the perineum (straddle injuries), and many have
delayed manifestation.
2) Posterior urethra consists of the prostatic urethra and membranous urethra. Posterior urethral injuries
are most commonly associated with pelvic fracture.

Hallmark triad of urethral injury is:


1) Blood at urethral meatus
2) Inability to void
3) Distended bladder, (high riding prostate, and scrotal hematoma in posterior urethral injuries)

A retrograde urethrogram would be needed in this patient to assess the urethral injury but it will not
give any information about pelvic fracture.
Foley catheterization in the presence of urethral injury will predispose the patient to abscess formation
and worsening of the urethral damage.
Urethral injury is less common in females

Extra-peritoneal rupture of bladder


Hematuria, suprapubic tenderness, non-palpable bladder, and lower abdominal and perineal-edema in
trauma patient is highly suggestive of extra peritoneal rupture of bladder.
The most common site of extraperitoneal rupture is the bladder neck.
Retrograde cystogram with post void film is the investigation of choice for patients with suspected
bladder trauma.

Intraperitoneal rupture of the bladder


Intraperitoneal rupture of the bladder occurs in blunt trauma to a full distended bladder.
The dome is the most susceptible part of the bladder to injury by sudden increase in intravesical pressure.
Abdominal pain that refers to one or both shoulders suggests intraabdominal pathology that is causing
peritonitis and irritation of the diaphragm.
The dome of the bladder is the only region covered by peritoneum; thus the only injury that would permit
leakage of urine into the peritoneum causes chemical peritonitis.

276
Penile fracture
Fracture of penis is an uncommon condition seen during aggressive sex especially with female on top.
It is characterized by history of vigorous intercourse with bending of penis accompanied by cracking
sound and immediate pain, swelling and rapid loss of erection.
It generally involves the disruption of the tunica albuginea of one or both corpus cavernosa.
Penile fracture is a medical emergency and retrograde urethrogram followed by surgical exploration
of penis.

Kidney stones
Urinary calculi present as flank or abdominal pain radiating to the groin, along with nausea and vomiting.
The first-line modality for diagnosing a urinary stone is non-contrast spiral CT of the abdomen and
pelvis.
Ultrasonography can be used if the patient is pregnant (but can miss small stones).

Uric acid stones


Uric acid stones (needle-shaped crystals) occur when urine is saturated with uric acid in the presence of
an acidic urine and dehydration.
Ileus is possibly due to vagal reaction due to ureteral colic.
Uric acid stones which are radiolucent, have to be evaluated by either CT of the abdomen or
intravenous pyelography.
Ileus will be over when the ureterolithiasis is treated.
Stones less than 0.6 cm may pass spontaneously with hydration and analgesia.

Oxalate stones
Patients with Crohns disease, chronic pancreatitis or any other small intestinal disorder resulting in fat
malabsorption, are predisposed to hyperoxaluria (remember Ca++ role).
Increased absorption is the most common cause of hyperoxaluria and oxalate stone formation.

Hydrocele
A hydrocele is a fluid collection within the processus or tunica vaginalis.
Hydrocele can be differentiated from other testicular masses by transillumination.
Non-communicating hydrocele is referred to a fluid containing sac which is a remnant of processus
vaginalis. The upper limits of the mass is easily identified. Most cases will disappear spontaneously by
the age of 12 months. Hydroceles that do not resolve spontaneously should be removed surgically due to
the risk of inguinal hernia.
Communicating hydrocele, the upper limit cant be reached and it treated with surgery.

277
Varicocele
A varicocele is a tortuous dilatation of the pampiniform plexus of veins surrounding the spermatic cord
and testis in the scrotum, most common on the left side.
Patients classically presents with dull, aching pain in the testis.
On examination, the scrotum will feel similar to a bag of worms, valsalva maneuver will typically
cause the mass to enlarge and the mass reduces in size when the patient is recumbent.
Varicoceles that fail to empty when the patient is recumbent raises the suspicion for renal cell carcinoma,
which is most commonly due to obstruction of the gonadal vein where it enters the renal vein. CT scan of
the abdomen is most sensitive and specific for diagnosing the renal cell carcinoma.

Benign prostatic hyperplasia and hematuria


Hematuria in BPH is typically the result of rupture of dilated veins in the prostatic urethra.
In males over age 40, BPH is the most common cause of isolated microscopic hematuria (> 5 RBCs/hpf
without associated increased proteins, casts or leukocytes in the urine).
In males over age 60, BPH, nephrolithiasis, and urinary tract infection are the most common causes of
isolated microscopic hematuria (BPH is the most common cause).

278
HUMAN GENETICS
Chorionic Villus Sampling (CVS)
Transvaginal or transabdominal aspiration of chorionic villus tissue (theprecursor to the placenta).
Chorionic villus sampling is the best test for detection of fetal chromosomal abnormalities in the first
trimester of pregnancy.
Advantages: Has a diagnostic accuracy comparable to that of amniocentesis; availability at 1012 weeks
gestation.
CVS is indicated in women over 35 years following an abnormal ultrasound.
Disadvantages: A 1% risk of fetal loss and inability to diagnose neural tube defects.
Limb defects have been associated with chorionic villus sampling performed at 9 weeks (most
significant risk factor is the gestational age of the fetus, more risk if before 10 weeks).

Amniocentesis
Consists of transabdominal aspiration of amniotic fluid using an ultrasound-guided needle and evaluation
of fetal cells for chromosomal abnormalities.
There is ample fluid in weeks 1618 to perform the test.
Risks are fetal-maternal hemorrhage (12%) and fetal loss (0.5%).
Amniocentesis is indicated for the following:
In women who will be > 35 years of age at the time of delivery.
In conjunction with an abnormal triple screen (together they detect 65% of fetuses with trisomy 21).
In Rh-sensitized pregnancy to obtain fetal blood type or to detect fetal hemolysis.
To evaluate fetal lung maturity via a lecithin-sphingomyelin ratio 2.5 or to detect the presence of
phosphatidyl glycerol.

Percutaneous Umbilical Blood Sampling


Performed in the second and third trimesters, when umbilical cord vessels are large enough to puncture
safely.
Amniocentesis and chorionic villus sampling are now used preferentially because information can be
obtained at an earlier date and because the techniques have a lower rate of fetal loss.
Risks are fetal loss (1.8%), fetal-maternal hemorrhage, and infection.
Percutaneous umbilical blood sampling is used for the following:
Assessment and treatment of Rh isoimmunization/erythroblastosis fetalis.
Fetal karyotyping.
Fetal infection (e.g., CMV, toxoplasmosis, rubella).
Evaluation of genetic diseases (e.g., sickle cell disease, thalassemia).
Evaluation of fetal acid-base status.

279
TERATOGENIC DRUGS
The fetus is most susceptible during gestational weeks 38 (organogenesis).
TERATOGEN EFFECT
Alcohol Fetal alcohol syndrome (microcephaly, mental retardation, IUGR, cardiac defects,
and facial dysmorphology which involves midfacial hypoplasia, micrognathia,
flattened philtrum, microphtalmia, short palpebral fissure, and thin vermillion
border of the upper lip).
Cocaine Bowel atresias, IUGR, microcephaly.
Streptomycin CN VIII damage/ototoxicity.
Tetracycline Tooth discoloration, inhibition of bone growth, small limbs, syndactyly.
Sulfonamides Kernicterus.
Quinolones Cartilage damage.
Isotretinoin Heart and great vessel defects, craniofacial dysmorphism, deafness.
Iodide Congenital goiter, hypothyroidism, mental retardation.
Methotrexate CNS malformations, craniofacial dysmorphism, IUGR.
DES Clear cell adenocarcinoma of the vagina/cervix, genital tract abnormalities (cervical
hood, T-shaped uterus, hypoplastic uterus), cervical incompetence.
Thalidomide Limb reduction (phocomelia), ear and nasal anomalies, cardiac and lung defects,
pyloric or duodenal stenosis, GI atresia.
Coumadin Stippling of bone epiphyses, IUGR, nasal hypoplasia, mental retardation.
ACEIs Oligohydramnios, fetal renal damage.
Lithium Ebsteins anomaly, other cardiac diseases.
Carbamazepine Fingernail hypoplasia, IUGR, microcephaly, neural tube defects.
Phenytoin Neuroblastoma, nail hypoplasia, IUGR, craniofacial dysmorphism, microcephaly.
Valproic acid Neural tube defects, craniofacial and skeletal defects.

Lithium is associated with the congenital anomalies, classically Ebsteins anomaly. When a woman
with isotretinoin, she should receive strict contraception.

Fetal hydantoin syndrome presents with a small body size with microephaly, hypoplasia of the
distal phalanx of the fingers and toes, nail hypoplasia, low nasal bridge, hirsutism, cleft lip/palate and
rib anomalies (due to fetal exposure to hydantoin anticonvulsants, such as Dilantin and Phenytoin or
diphenylhydantoin, which is usually taken by epileptic mothers).
Neuroblastoma has been shown to be associated with exposure to phenytoin and other hydantoins in
utero.

Fetal alcohol syndrome presents with IUGR, microcephaly and facial dysmorphology which involves
midfacial hypoplasia, micrognathia, flattened philtrum, microphtalmia, short palpebral fissure, and thin
vermillion border of the upper lip.

280
Female offspring of women who ingested diethylstilbestrol during their pregnancy are at increased risk
of developing clear cell adenocarcinoma of the vagina and cervix, as well as cervical anomalies and
uterine malformations.

Oral hypoglycemic agents and ACE inhibitors are contraindicated in pregnancy.


Insulin is used for diabetes mellitus in pregnancy and hydralazine, methyldopa and labetalol are used for
hypertension in pregnancy.

Inhaled steroid are okay in pregnancy.

Radiation levels used for diagnostic exams are not associated with teratogenicity.

Physical exercise can be beneficial during pregnancy and is helpful in maintaining a feeling of well
being. It is usually recommended to keep it at the same level as before pregnancy.
All healthy pregnant women with uncomplicated pregnancies are encouraged to exercise for 30 minutes
daily at a moderate intensity that allows the mother to carry on conversation while exercising.

Low back pain is a very common complaint in the 3rd trimester of pregnancy. It is believed to be
caused by the increase in lumbar lordosis and the relaxation of the ligaments supporting sacroiliac
and other joints of the pelvic girdle, hormonal factors may contribute to this problem.

Physicians have to maintain their obligation to a patient's right to confidentiality, even in the event of a
pregnant minor wanting to withhold the diagnosis from her parents.

Labor should be allowed to proceed in patients where the fetus has been diagnosed with severe
congenital anomaly incompatible with life.

Physicians can refuse to perform elective abortions for personal or professional reasons. If you refuse to
perform any procedure then you should attempt to refer the patient to another physician who can & will.

Lupus anticoagulant, seen in SLE patients, leads to recurrent abortions and thromboembolic disease.
(antiphospholipid AB, such as lupus anticoagulant and anticardiolipin Ab, cause placental infarction
leading to fetal growth restriction or death.) These Ab are also associated with thromboembolic disease.

All patients with pseudocyesis need psychiatric evaluation (form of conversion disorder).

281
PHYSIOLOGY OF PREGNANCY
Human chorionic gonadotropin (HCG)
Human chorionic gonadotropin is secreted by the syncytiotrophoblast, production begins 8 days after
fertilization, and the levels double every 48 hours until they peak at 6 8 weeks gestation.
HCG is mainly responsible for maintenance of progesterone secretion by the preservation of the corpus
luteum in early pregnancy, until the placenta starts producing progesterone on its own.
Other biological functions include the promotion of male sexual differentiation and stimulation of the
maternal thyroid gland.
Increased HCG seen with hyperemesis gravidarum, H. mole, and choriocarcinoma, and can cause
hyperthyroidism during pregnancy too.

Cervix
Softening and cyanosis at approximately four weeks.
Thick mucus clot in the cervical os expelled at or near labor bloody show.
Mucus appears granular microscopically due to progesterone.

Uterus: Palpated above the pubic symphysis at 12 weeks.

Vagina: Thick acidic secretions; violet colorations (Chadwicks sign) from blood flow.

Cardiovascular
cardiac output (3050%), heart rate (by 1015 bpm), and stroke volume.
systemic vascular resistance (progesterone smooth muscle relaxation).
BP in the first trimester; reaches a nadir at 24 weeks and normalizes by 40 weeks.
Systolic murmur and audible S3 are normal; a new diastolic murmur is not.
The heart is displaced by the uterus upward and to the left cardiomegaly on CXR.

Endocrine
Thyroid hormone: estrogen levels thyroid-binding globulin Total and bound T3/T4,
normal free T4 and a normal TSH.
Human placental lactogen (HPL): Acts as an insulin antagonist to maintain fetal glucose level
postprandial hyperglycemia, fasting hyperinsulinemia/hypertriglyceridemia, and exaggerated
starvation ketosis response. Change can or worsen gestational diabetes.
Cortisol: total and free cortisol (produced by the fetal adrenal gland and the placenta).

GI
Nausea and vomiting (in up to 70% ); resolves by 1416 weeks when the hCG rise plateaus.
acid reflux from gastroesophageal junction sphincter tone.
Constipation from large bowel motility and water resorption.
biliary cholesterol saturation predisposes to gallstone formation.

282
Pregnancy has a protective effect against peptic ulcer disease and multiple sclerosis.

Hematologic
Physiologic anemia: Unequal in plasma volume (50%) and RBC mass (30%) hemoglobin and
hematocrit (dilutional effect). However, a hemoglobin < 11.0 mg/dL is never normal and is likely due to
iron deficiency.
WBC count: throughout pregnancy to a mean of 10.5 million/mL, but can during labor to > 20
million/mL.
Hypercoagulable state: DVT risk is highest in the puerperium. The leading non-obstetric cause of
postpartum death is thromboembolic disease (pulmonary embolism).

Musculoskeletal
motility of sacroiliac, sacrococcygeal, and pubic joints.
Low back pain is a very common complaint in the third trimester of pregnancy. It is believed to be
caused by the increase in lumbar lordosis and the relaxation of the ligaments supporting the joints of
the pelvic girdle.

Pulmonary
in tidal volume (VT) of 40%; in total lung capacity, residual volume, and expiratory reserve volume.
Respiratory rate (RR) is unchanged. minute ventilation (VT RR) alveolar and arterial PO2 and
alveolar and arterial PCO2.
Dyspnea of pregnancy is common and likely caused by VT and PCO2.

Renal
Kidneys dilate. Dilation of the collecting system may be mistaken for hydronephrosis.
GFR by 50%; renal plasma flow by 30%.
estrogen and progesterone stimulate the renin-angiotensin system. aldosterone contributes to water
retention and plasma volume.
BUN, serum creatinine, and hematocrit are often decreased in pregnant patients, and it is due to a
dilutional effect.

Skin
estrogen may changes that resemble cirrhotic disease, such as striae (on the abdomen, breast, and
thighs), spider angiomas, and palmar erythema.
Hyperpigmentation over the midline (linea nigra), face (chloasma), and perineum is due to -
melanocyte-stimulating hormone and steroids.
Diastasis recti: Rectus muscles may separate in the midline, leaving part of the anterior uterus covered
only by skin, fascia, and peritoneum.

283
Diseases improve in pregnancy
Graves disease (due to the relative immuno-tolerance of pregnancy)
Migraine (especially during 2nd trimester, avoid -blockers, it decreases placental blood flow and cause
growth retardation. In the 3rd trimester, imitriptyline or doxepin may be used in low dose)
Peptic ulcer (pregnancy associated with increased prostaglandins which protect the gastric mucosal
barrier.)
Multiple sclerosis

ANTENATAL CARE AND FETAL TESTING


The goal is to prevent, diagnose, and treat conditions that adverse out-comes in pregnancy.
In women with regular menstrual cycles, calculate the estimated date of delivery using Ngeles rule.
Ngeles ruledue date = last menstrual period (LMP) + nine months + seven days.

Gestational age (GA) can be determined by fundal height, quickening (at 1718 weeks), fetal heart tones
(at 10 weeks via Doppler), or ultrasound (fetal crownrump length at 512 weeks; biparietal diameter at
2030 weeks).

Patient's right to confidentiality


Physicians have to maintain their obligation to a patients right to confidentiality, even in the event of a
pregnant minor wanting to withhold the diagnosis from her parents. He should then encourage her to
confide in her mother as well.

IUGR
Smoking is the single most prevalent preventable cause of IUGR in USA.
IUGR may be symmetrical and asymmetrical.
In symmetrical growth restriction, the insult to the fetus begins before 28 weeks gestation and growth of
both the head and the body is deficient. It is usually caused by congenital infections, congenital
anomalies and vascular diseases of the mother.
Asymmetrical IUGR is a result of a late exposition to the insult past 28 weeks and is characterized by a
normal or almost normal head size and a reduced height, weight and abdominal circumference. It is a
result of late exposure to a maternal factors that does not allow optimal fetal growth e.g. hypertension,
preeclampsia, chronic renal disease, hypoxemia, smoking, vascular disease and toxic exposures. It has a
better prognosis than symmetrical IUGR.

IUGR can be diagnosed at an early stage by identifying risk factors, and through regular assessment of
fetal growth by means of fundal height measurement, starting from 22 weeks, and ultrasound.

Abdominal circumference is the single most effective parameter to estimate fetal weight, and this is
because it is reduced in both symmetrical and asymmetrical IUGR.

284
Once IUGR diagnosed, fetal well-being has to be closely monitored with NST and BPP twice weekly.
The mother can also contribute to this monitoring by assessment of a kick count.

NOTE: U/S performed between 16 and 20 weeks is the most accurate method for pregnancy dating.

In cases of IUGR, presence of oligohydramnios (amniotic fluid index < 7) is an indication for delivery.

STANDARD PRENATAL CARE


CATEGORY RECOMMENDATIONS
Weight gain Approximately 2535 lbs for an average woman (1525 lbs for obese women and more
for thin women).
An additional 300 kcal/day is needed during pregnancy and 500 kcal/day during
breast-feeding.
Nutrition Requirements for protein, iron, folate, calcium, and other vitamins/minerals in
pregnancy are as follows:
Folate: 1 mg daily; required to neural tube defects, especially three months prior
to conception.
Iron: Demand is by fetal needs and by expanding maternal blood supply.
Supplement with 3060 mg/day of elemental iron in the latter half of pregnancy.
Patients are advised to take prenatal vitamins, but most needs can be met through diet.
Prenatal labs
Initial visit Heme: CBC, blood type, Rh(D) antibody test, and antibody screen.
Infectious disease: UA and culture, rubella antibody titer, HBV surface antigen test,
syphilis screen (RPR/VDRL), cervical gonorrhea and chlamydia PCR or culture, PPD,
HIV (in high-risk groups).
Other: Pap smear, glucose testing, sickle prep.

1519 weeks Maternal serum -fetoprotein (MSAFP) or triple screen (MSAFP, estriol, -hCG).
Offer amniocentesis to patients > 35 years of age at the time of delivery.

1820 weeks Ultrasound to determine GA (if unknown or uncertain) and to survey fetal anatomy,
amniotic fluid volume, and placental location.

2628 weeks Glucose challenge test; repeat hematocrit, Rh(D) antibody test

28 weeks RhoGAM to Rh-antibody- women.

3236 weeks Cervical chlamydia and gonorrhea cultures in high-risk patients; repeat hematocrit.
Screen for group B streptococcus (GBS) with vaginal and rectal cultures. If , give
penicillin during labor to prevent transmission to the infant.

285
Screening for syphilis
It is recommended that all pregnant women be screened for syphilis regardless of risk factors.
Screening should be performed at the first prenatal visit and is typically via Rapid plasma regain test
(RPR) and venereal disease research laboratory test (VDRL).
When the screening test is positive, the diagnosis can be confirmed with fluorescent treponemal antibody
absorption (FTA-ABS) test.
Treatment with penicillin.

Rh(D) antibody test


At the first visit; blood typing and Rh antibody testing (in Rh-negative women) should be performed.
Unsensitized Rh-negative women potentially carrying Rh-positive fetus should have their Rh antibody
status retested between 24 and 28 weeks gestation.
Patients at risk of alloimmunization should be given anti-Rh immune globulin at 28 weeks gestation
and again at the time of delivery.

ABO antibodies: IgM, dont cross the placenta


Rh antibodies: Ig G, do cross the placenta.

Influenza vaccine
It is recommended that all pregnant women without contraindications be vaccinated against influenza.

Tuberculosis (TB) screening


High risk populations for tuberculosis are immigrants, women of low socioeconomic status, and HIV-
positive women. PPD is positive when it exceeds 10 mm in high-risk populations, and 5 mm in HIV
patients.
Positive PPD does not mean that the patient is infected.
However, all patients with positive PPD should undergo a chest x-ray in order to exclude active
tuberculosis before prescribing isoniazid prophylaxis.
Patients with a positive tuberculin skin test without any evidence of active tuberculosis are considered
to have a latent tubercular infection (LTBI). These patients should be treated with isoniazid for a
period of nine months.
When active tuberculosis arises in any individual (including a pregnant woman), it is cause for alarm and
warrants immediate treatment. Any concerns about possible drug toxicity are outweighed by the clear
benefits of curing an active tuberculosis infection. In pregnant women stricken with tuberculosis that is
not likely to be drug-resistant, the standard treatment regimen includes isoniazid (INH), rifampin, and
ethambutol for a period of nine months.
Streptomycin may be ototoxic for the fetus and should not be used in the pregnant woman.

-Fetoprotein (AFP)
Produced by the yolk sac and the fetal liver and found primarily in amniotic fluid. Small amounts cross
the placenta and enter the maternal circulation.

286
MSAFP (1520 weeks gestation): Results are reported as multiples of the median (MoMs) and depend
on accurate gestational dating.

Causes of elevated MSAFP (> 2.5 MoMs) include:


Incorrect gestational dating (most common cause)
Open neural tubedefects (anencephaly, spina bifida)
Abdominal wall defects (gastroschisis, omphalocele)
Multiple gestation
Fetal death
Placental abnormalities (e.g., placental abruption).

If MSAFP levels are found to be abnormal in a pregnant patient, the next step is ultrasonography.
Abnormally low MSAFP levels (< 0.5 MoM) warrant amniocentesis and karyotyping to rule out
chromosomal abnormalities.

Triple screen
The triple screen is more sensitive than MSAFP alone for detecting trisomies.
Sensitivity for detecting chromosomal abnormalities is by adding estriol and -hCG to MSAFP.
Trisomy 18 = AFP, estriol, -hCG.
Trisomy 21 = AFP, estriol, -hCG
A decrease in MSAFP and Estriol, and an increase in beta-hCG level is typical of Downs syndrome.

Maternal serum quadruple test


This test is done in the 2nd trimester in order to detect fetuses at increased risk of Down syndrome, neural
tube defects and Edwards syndrome.
Down syndrome (aneuploidy) = Estriol, MSAFP, -hCG, Inhibin A.

FETAL HEART RATE (FHR) MONITORING


Nonstress test (NST)
If the fetal movement decreases or becomes imperceptible by the mother, then a nonstress test should
be carried out to document fetal well-being
Performed with the mother resting in the left lateral supine position (to prevent supine hypotension).
Fetal heart rate is monitored externally by Doppler and correlated with spontaneous fetal movements as
reported by the mother.
Normal response: Two accelerations of 15 bpm above baseline for at least 15 seconds over a 20-
minute period.
Nonreactive: Perform further tests (e.g., a biophysical profile). Lack of fetal heart rate accelerations
may occur with any of the following: GA < 32 weeks, fetal sleeping, fetal CNS anomalies, maternal
sedative or narcotic administration, and fetal hypoxia (rare).

287
Contraction stress test (CST)
Used in high-risk pregnancies to assess uteroplacental dysfunction.
Fetal heart rate is monitored during spontaneous or induced (via nipple stimulation or oxytocin)
contractions.
Reactivity is determined as in the NST.
A positive CST is defined by late decelerations following 50% or more of contractions and raises
concerns about fetal jeopardy. Must have at least three contractions in 10 minutes.
A negative CST (no late decelerations) is highly predictive of fetal well-being in conjunction with a
normal NST.

Biophysical profile (BPP)


Real-time ultrasound is used to assign a score of 2 (normal) or 0 (abnormal) to five parameters: fetal
tone, breathing, movement, amniotic fluid volume, and NST.
A score of 8-10 is reassuring for fetal well-being, and should only be repeated once or twice weekly
until term for high risk pregnancies.
A score of 02 is extremely worrisome for fetal asphyxia, and strong consideration should be given to
immediate delivery if no nonhypoxic explanation is found.
When performing a BPP, remember toTest the Baby, MAN!
Fetal Tone
Fetal Breathing
Fetal Movement
Amniotic fluid volume
Non-stress test

PERINATAL INFECTIONS
Group B streptococcus
Screening cultures for group B streptococcus should be performed at 36-37 weeks gestation.
Positive cases should be treated with penicillin G during labor, even in the absence of frank
chorioamnionitis, thus reducing the risk of neonatal infections.

Toxoplasmosis in pregnancy
DOC in first trimester: spiramycin
DOC in 2nd -3rd trimester: pyrimethamine and sulfadiazine.
Elective termination of pregnancy is an option in 1st trimester.

Syphilis
Penicillin desensitization is considered to be the treatment of choice for the pregnant patients with
syphilis and having penicillin allergy.

288
HIV
The incidence of vertical transmission of HIV can be reduced administrating ZDV to pregnant women
and their offspring. ZDV is administered orally during pregnancy after the first trimester,
intravenously during labor and delivery, and orally to the neonate for the first 6 weeks of life.
Combining ZDV therapy and elective C. section reduces the risk of transmission more than either
intervention alone.
Forceps are to be avoided in this setting.

HBV
Vertical transmission of hepatitis B from pregnant females to the unborn child can occur with active
hepatitis B infection.
Newborns of mothers with active hepatitis B infection should be passively immunized at birth with
hepatitis B immune globulin (HBIG) followed by active immunization with recombinant HBV vaccine.

HCV
The overall incidence of vertical transmission of HCV is approximately 2-5%.
All patients, including pregnant patients, with chronic hepatitis C should receive vaccinations against
Hepatitis A and B if not already immune.

Polyhydramnios
An excess of amniotic fluid volume, defined by an AFI > 20 on ultrasound. May be present in normal
pregnancies, but fetal chromosomal or developmental abnormalities are common.
Etiologies include maternal DM, multiple gestation, isoimmunization, pulmonary abnormalities (e.g.,
cystic lung malformations), fetal anomalies (e.g., duodenal atresia, tracheoesophageal fistula,
anencephaly), and twin-twin transfusion syndrome.
Usually asymptomatic, or exam may reveal fundal height greater than expected.
Evaluation includes ultrasound for fetal anomalies, glucose testing for DM, and Rh screen.
Complications: Preterm labor, fetal malpresentation, cord prolapse.

Oligohydramnios
A deficiency of amniotic fluid volume defined as an amniotic fluid index (AFI) < 5 on ultrasound.
Without rupture of membranes (ROM), it is associated with a 40-fold in perinatal mortality.
Oligohydramnios almost always indicates the need to assess fetal well-being.
Etiologies include fetal urinary tract abnormalities (e.g., renal agenesis, polycystic kidney disease, GU
obstruction), chronic uteroplacental insufficiency (associated with a small-for-GA fetus), or ROM.
Usually asymptomatic, but IUGR or fetal distress may be present.
Rule out inaccurate gestational dates.
Treat the underlying cause if possible. Patients with ROM may benefit from amnio infusion.
Complications: Musculoskeletal abnormalities (e.g., clubfoot, facial distortion); pulmonary hypoplasia;
fetal hypoxia due to umbilical cord compression and IUGR.

289
Hyperemesis Gravidarum
Intractable nausea and vomiting that typically persist beyond 410 weeks gestation dehydration,
electrolyte abnormalities, and poor weight gain or weight loss.
Differentiate from morning sickness, acid reflux, and gastroenteritis.
More common in nulliparous and molar pregnancies (with -hCG).
Gestational trophoblastic disease should be ruled out in pregnant patients with severe vomiting.
Measurement of quantitative -hCG should be checked first. If the -hCG level is markedly elevated (i.e.
> 10,000) an ultrasonogram is then performed.
Evaluate for ketonemia, ketonuria, hyponatremia, and hypokalemic-hypochloremic metabolic
alkalosis.
Mild increase in ALT, AST, bilirubin, amylase and lipase are seen in 50% of hospitalized patients.
Elevated amylase and lipase are from salivary glands due to vomiting.
Treatment includes: frequent small meals; antiemetics and IV hydration.

Pregnant women with a current or previous diagnosis of anorexia nervosa are at risk for numerous
complications, including miscarriage, intrauterine growth retardation, hyperemesis gravidarum,
premature birth, cesarean delivery, and postpartum depression. Osteoporosis is also a common finding in
anorexic patients, whether pregnant or not.

LATE PREGNANCY BLEEDING


Any bleeding after 20 weeks gestation. Prior to 20 weeks, bleeding is referred to as threatened
abortion.
Complicates 35% of pregnancies.
The most common causes are placental abruption and placenta previa.
Other causes include bloody show, ruptured vasa previa, early labor, ruptured uterus, marginal placental
separation, genital tract lesions and trauma, and placenta accreta (placental adherence to myometrium).
Maintenance of airway, breathing and circulation is always the first priority. Hemodynamic resuscitation
has to be promptly initiated before starting any measure to diagnose the source of blood loss in patients
with antepartum hemorrhage who are unstable hemodynamically.

ABRUPTIO PLACENTA
Pathophysiology: Premature (before onset of labor) separation of normally implanted placenta.
Risk factors: Hypertension, abdominal/pelvic trauma, tobacco or cocaine use, previous abruption, rapid
decompression of over distended uterus.
Classic manifestations of acute abruptio placenta include: Painful third trimester dark vaginal
bleeding that does not spontaneously cease, abdominal pain, uterine contractions and uterine
tenderness. The absence of blood on pelvic examination does not rule out this condition.
Diagnosis: Primarily clinical. Transabdominal/transvaginal ultrasound sensitivity is only 50%; look for
retroplacental clot; most useful for ruling out previa (normal ultrasound is most likely).

290
Management of abruptio placenta depends on the severity of the bleeding and the age of the pregnancy.
Regardless of gestational age, patients with active, uncontrolled antepartum hemorrhage, who also
typically exhibit unstable vital signs and unreassuring fetal heart rates, should undergo emergent
caesarian section delivery.

Stabilize patient with mild abruption and premature fetus; manage expectantly (hospitalize, start IV and
fetal monitoring; type and cross blood, bed rest).
Moderate to severe abruption: Immediate delivery (vaginal delivery with amniotomy if mother and
fetus are stable; C-section is used only when there are obstetrical indications, or when there is maternal
or fetal distress [rapid deterioration of the state of either the mother or the fetus]).
Patients with severe placental abruption in labor have to be managed aggressively to insure a rapid
vaginal delivery.
After the events that are associated with excessive feto-maternal hemorrhage (e.g., placental abruption),
the failure to correct the dose of anti-D immune globulin may result in maternal alloimmunization.
Complications: Hemorrhagic shock. Coagulopathy: DIC in 10%. Recurrence risk is 516%; rises to 25%
after two previous abruptions. Fetal hypoxia.

PLACENTA PREVIA
Pathophysiology: Abnormal placental implantation:
Total: Placenta covers cervical os.
Marginal: Placenta extends to margin of os.
Low-lying: Placenta in close proximity to os.
Risk factors: Prior C-sections, grand multiparous, advanced maternal age, multiple gestation, prior
placenta previa.
Symptoms: Painless, third trimester, bright red bleeding that often ceases in 12 hours with or
without uterine contractions. First bleeding episode at 2930 weeks. Usually no fetal distress.
Diagnosis: Transabdominal/transvaginal ultrasound sensitivity is > 95%; look for abnormally
positioned placenta.
Management of placenta previa depends on the severity of bleeding and the age of pregnancy:
NO vaginal exam! [In the presence antepartum hemorrhage, pelvic examination must not be done
before ruling out placenta previa]
Stabilize patient with premature fetus; manage expectantly.
Give tocolytics (MgSO4).
Serial ultrasound to assess fetal growth; resolution of partial previa.
Assess fetal lung maturity with amniocentesis and augment with betamethasone.
Deliver by C-section. Indications for delivery include persistent labor, life-threatening bleeding,
fetal distress, documented fetal lung maturity, and 36 weeks GA.
Complete placenta previa requires delivery by C. section as the placenta completely covers the
cervical os and the fetus can not be delivered vaginaly.

291
Complications of placenta previa: risk of placenta accreta. Vasa previa (fetal vessels crossing the
internal os). Preterm delivery, premature rupture of membranes (PROM), IUGR, congenital anomalies.
Recurrence.

VASA PREVIA
Vasa previa is ruptured fetal umbilical vessels (abnormal umbilical vessels).
An antepartum hemorrhage with fetal heart changes, progressing from tachycardia, to bradycardia, to
a sinusoidal pattern occurring suddenly after rupture of membranes suggest the diagnosis of vasa previa.
The bleeding in this setting is fetal in origin, so maternal vital signs will remain stable while the fetus
exsanguinates.
Apt test which differentiates maternal from fetal blood can be performed to confirm the diagnosis.
Vasa previa or fetal vessel rupture necessitates immediate C. section crash C-section.

UTERINE RUPTURE
Risk factors include uterine scar (e.g. history of C. section) or abdominal trauma.
The hyperventilation, agitation, and tachycardia usually indicate an imminent rupture.
Uterine rupture presents with an sudden intense abdominal pain associated with vaginal bleeding can
range from spotting to massive hemorrhage, fetal heart rate abnormalities and loss of station of the
fetal head (e.g. from 0 to 2 .
Total abdominal hysterectomy is the treatment of choice for uterine rupture.
However, debridement and closure of the site of rupture can be considered in women with low parity
or who desire more children.

FIRST-TRIMESTER BLEEDING

SPONTANEOUS ABORTION (SAB)


Loss of the fetus prior to the 20th week of pregnancy.
Some 75% of cases occur before the 16th week, with 75% of these occurring before the eighth week.
Approximately 20% of clinically recognized pregnancies terminate in SAB.
risk is associated with a history of incompetent cervix, cervical conization or loop electrosurgical
excision procedure (LEEP), cervical injury, DES exposure, and anatomical abnormalities of the
cervix.
Additional risk factors are as follows:
First trimester: Fetal factors (chromosomal abnormalities).
Second trimester: Maternal factors (cervical incompetence, infection, hypercoagulable states); maternal
trauma, infection, dietary deficiency, DM, hypothyroidism, or anatomic malformation.

292
DIAGNOSIS
levels of hCG.
Ultrasound:
Can identify the gestational sac 56 weeks from the LMP, a fetal pole at six weeks, and fetal cardiac
activity at 67 weeks.
With accurate dating, a small, irregular intrauterine sac without a fetal pole on transvaginal ultrasound
is diagnostic of an abnormal pregnancy.
CBC is warranted in the presence of heavy bleeding.
Maternal Rh type should be determined and RhoGAM given if the type is Rh \.
All recovered tissue should be preserved and assessed by a pathologist and sent for chromosomal
analysis.

MISSED ABORTION
Description: Pregnancy has ceased to develop and dead fetus s still retained in the uterus. No POC is
expelled. No uterine bleeding; symptoms of pregnancy disappear. Brownish vaginal discharge. The
diagnosis is suspected when there is disappearance of the nausea and vomiting of early pregnancy, and
an arrest of uterine growth.
Diagnosis: Os is closed; ultrasound reveals no fetal cardiac activity. Fetal tissue is retained.
Treatment: Once the diagnosis of missed abortion is confirmed, surgical evacuation (dilation and
curettage) of the uterus has to be performed to avoid the patient serious complications, such as DIC and
sepsis and to minimize the extent of the hemorrhage. Prostaglandin (misoprostol) suppositories are an
alternative.
Complications: DIC is a serious but rare complication whose risk with GA.

THREATENED ABORTION
Description: No POC expelled; membranes remain intact and closed cervix. Uterine bleeding before the
20th weeks gestation is present; abdominal pain may be present. The fetus is still viable.
Diagnosis: Os is closed; ultrasound is normal.
Treatment: Reassurance and outpatient follow up. Avoid heavy activity. Pelvic rest for 2448 hours
with gradual resumption of activities, but abstinence from coitus and douching.

INEVITABLE ABORTION
Description: No POC expelled, but considered inevitable. Vaginal bleeding and lower abdominal
cramps that may radiate to the back and perineum, leading to cervix dilatation.
Diagnosis: Os is open +/ ROM.
Treatment: emergency Suction curettage (D&C) or expectant management. Surgical evacuation of
uterine contents. Prostaglandin suppositories are an alternative.

293
INCOMPLETE ABORTION
Description: Vaginal bleeding and lower abdominal cramps, leading to cervix dilatation with some,
but not all POC expelled. Visible tissue in the vagina or endocervical canal.
Diagnosis: Os is open; ultrasound reveals retained fetal tissue.
Treatment: emergency Suction curettage (D&C) remove remaining POC and to control bleeding.
Hemodynamic stabilization for heavy bleeding.

COMPLETED ABORTION
Description: Vaginal bleeding and lower abdominal cramps, leading to all products of conception
(POC) expelled; pain and uterine contractions cease, but spotting may persist.
Diagnosis: Os is closed; ultrasound shows empty uterus. POC is submitted to pathology to confirm fetal
tissue.
Treatment: conservative if an intrauterine pregnancy had been previously confirmed, otherwise serial -
hCG titers should be obtained weekly until negative to ensure an ectopic pregnancy has not been missed.

SEPTIC ABORTION
Description: Infection with abortion. Endometritis septicemia. Maternal mortality is 1015%.
Diagnosis: Hypotension, hypothermia, oliguria, respiratory distress if in shock; WBC.
Treatment: cervical and blood sampling, IV antibiotics and complete uterine evacuation with gentle
suction curettage.

RECURRENT ABORTION
Description: 2 consecutive SABs or a total of three SABs. If early, often due to chromosomal
abnormalities karyotyping of both parents. Incompetent cervix should be suspected with a history
of painless dilation of the cervix and delivery of a normal fetus between 18 and 32 weeks.
Diagnosis: Evaluate for uterine abnormalities. Cervical cultures for gonococcus, chlamydia, and group
B streptococcus should be obtained before the procedure.
Lupus anticoagulant, seen in SLE patients, leads to recurrent abortions and thromboembolic disease.
Treatment: Surgical cerclage procedures to suture the cervix closed until labor or ROM occurs with
subsequent removal prior to delivery. Restriction of activities.

FETAL DEMISE
Description: Absence of fetal cardiac activity.
Diagnosis: Uterus small for GA; no fetal heart tones or movement on ultrasound [The most appropriate
test to confirm the diagnosis of Intra Uterine Fetal Demise (IUFD) is real-time ultrasonography].
After diagnosis confirmed, a coagulation profile should be drawn to detect incipient DIC. A fibrinogen
level in the low-normal range can indicate developing DIC.

294
The best management of intrauterine fetal demise is delivery of the fetus to reduce risk of infection and
coagulopathy DIC.
Treatment options should be discussed with any patient prior to embarking on a plan so the patient can
make an informed decision about therapy with knowledge of the risks and benefits of all options.
Labor should be induced immediately in patients with intrauterine fetal demise who develop coagulations
abnormalities. A search should be undertaken to determine the cause after the first episode of
intrauterine fetal demise. Autopsy of the fetus and placenta should be performed in all cases of
stillbirth with the permission of the parents.

RHESUS ISOIMMUNIZATION
Rhesus (Rh) factor is an antigenic protein located on RBCs in Rh- individuals. Transmission is
autosomal dominant. When fetal RBCs leak into maternal circulation, maternal anti-Rh IgG antibodies
can form. These antibodies can cross the placenta hemolysis of fetal Rh- RBCs (erythroblastosis
fetalis). Hemolytic disease usually occurs during the second pregnancy owing to rapid production of
anti-Rh IgG antibodies by memory plasma cells.
Inquire about prior events that may have exposed the mother to Rh- blood, including ectopic
pregnancy, abortion, blood transfusions, prior delivery of an Rh- child, amniocentesis, or other
traumatic procedures during pregnancy.
Maternal: On initial visit, test for ABO and Rh blood groups and perform antibody screening (indirect
Coombs test). If \, repeat Coombs test at 2628 weeks. If , test serially for high titers of maternal
anti-Rh IgG (> 1:16).
Fetal: Assess during pregnancy using amniocentesis or ultrasound-guided umbilical blood sampling for
fetal blood type, Coombs titer, bilirubin levels, hematocrit, and reticulocytesor determine Rh status
and hematocrit postnatally using fetal cord blood.
Prevention:
If the mother is Rh \ at 28 weeks and (1) the father is Rh , (2) the fathers Rh status is unknown, or
(3) paternity is uncertain, give RhoGAM (Rh immune globulin).
If the baby is Rh , give RhoGAM postpartum as well.
Give RhoGAM to Rh-\ mothers who undergo abortion (therapeutic or spontaneous) or who have had
an ectopic pregnancy, amniocentesis, vaginal bleeding, or placenta previa/placental abruption.
RhoGAM is indicated in previously unsensitized Rh-negative women at 28 weeks gestation, and
within 72 hours after any procedure or incident (e.g. abortion, ectopic pregnancy), and delivery.

Sensitized Rh-\ mothers with titers > 1:16 should be closely monitored with serial ultrasound and
amniocentesis for evidence of fetal hemolysis.
In severe cases, initiate preterm delivery when fetal lungs are mature.
Prior to delivery, intrauterine blood transfusions may be given to correct a low fetal hematocrit.
RhoGAM (Rh immune globulin) destroys Rh-cells in maternal circulation and prevents Rh
sensitization.
Complications: Fetal hypoxia and acidosis, kernicterus, prematurity, death. Hydrops fetalis ( protein &
oncotic pressure, edema, jaundice, high-output cardiac failure) occurs when hemoglobin drops to < 7g/dL

295
Premature Rupture of Membranes (PROM)
Defined as spontaneous ROM before onset of labor.
May be precipitated by vaginal or cervical infections, abnormal membrane physiology, and cervical
incompetence.
Preterm PROM occurs at < 37 weeks gestation.
Prolonged ROM is defined as rupture > 18 hours prior to delivery.
Risk factors include low socioeconomic status (SES), young maternal age, smoking, and STDs.
Patients often report a gush of clear or blood-tinged amniotic fluid. Uterine contractions may be
present.
Sterile speculum exam reveals pooling of amniotic fluid in the vaginal vault, a Nitrazine paper test
(paper turns blue in alkaline amniotic fluid), and a fern test (a ferning pattern is seen under a
microscope after amniotic fluid dries on a glass slide).
Normal amniotic fluid index is > 5 and < 25.
Amniotic fluid sampling to measure fetal lung maturity is mandatory.
If Lecithin/Sphingomylein ratio < 2.0; prematurity is a major concern.
Ultrasound to assess amniotic fluid volume.
Cultures or smears to rule out infections.
Minimize infection risk; do not perform digital vaginal exams if gestational age is less than 34 weeks.
Check fetal heart tracing, maternal temperature, WBC count, and uterine tenderness for evidence of
chorioamnionitis.
Treatment depends on GA and fetal lung maturity.
In general, if there is no sign of infection, delivery is delayed with tocolytics in the form of -agonists
(e.g., ritodrine, terbutaline), magnesium, NSAIDs (e.g., indomethacin), or calcium channel blockers
(e.g., nifedipine).
Corticosteroids (e.g. dexamethasone) can be given to promote fetal lung maturity, when premature
rupture of membranes occur less than 32 weeks of gestation.
If a woman is admitted to the hospital after 32 weeks of gestation with PROM, delivery is
recommended.
Prophylactic antibiotics (Penicillin) are given to prevent infection in all patients with PROM whose
GBS status is unknown.
If signs of infection or fetal distress develop, give antibiotics (ampicillin and gentamicin) and induce
labor. If the patient is penicillin allergic, gentamicin and clindamycin are appropriate.
Complications: Preterm labor and delivery, chorioamnionitis, placental abruption, cord prolapse.

296
Chorioamnionitis
The differential diagnosis in patient with uterine tenderness, mild fever and leukocytosis is
intraamniotic infection (chorioamnionitis), abruptio placenta and a systemic viral infection.
Chorioamnionitis refers to infection of the amniotic fluid.
Symptoms of chorioamnionitis include maternal tachycardia (> 100 bpm), fetal tachycardia (> 160 bpm),
maternal leucocytosis with a left shift, uterine tenderness, and occasionally a foul discharge from the
uterine cavity.
Chorioamnionitis is typically polymicrobial infection.
Intraamniotic infection must be ruled out, amniotic fluid analysis (amniocentesis for gram stain, culture
and sensitivity and other studies).

PRETERM LABOR
Defined as onset of labor between 20 and 37 weeks gestation. Occurs in approximately 10% of all U.S.
pregnancies and is the 1 cause of neonatal morbidity and mortality.
Risk factors include multiple gestation, infection, PROM, uterine anomalies, previous preterm labor or
delivery, polyhydramnios, placental abruption, poor maternal nutrition, and low SES. Most patients have
no identifiable risk factors.
Patients may have menstrual-like cramps, onset of low back pain, pelvic pressure, and new vaginal
discharge or bleeding.
Preterm labor = regular uterine contractions + concurrent cervical change at < 37 weeks gestation

Requires regular uterine contractions ( 3 contractions of 30 seconds each over a 30-minute period) and
concurrent cervical change at < 37 weeks gestation.
Assess for contraindications to tocolysis (e.g., infection, non-reassuring fetal testing, placental
abruption).
Perform a sterile speculum exam to rule out PROM.
Obtain an ultrasound to rule out fetal or uterine anomalies, verify GA, and assess fetal presentation and
amniotic fluid volume.
Obtain cultures for chlamydia, gonorrhea, and group B strep. Obtain a UA and urine culture.
Treatment includes: Hydration and bed rest are the first step in stopping uterine contractions in early
stages of preterm labor and if these measures fail, tocolytics are indicated.
Unless contraindicated, begin tocolytic therapy and give steroids to accelerate fetal lung maturation.
Give penicillin or ampicillin for group B strep prophylaxis if preterm delivery is likely.
Advanced stage of premature labor should be managed more aggressively and tocolysis has to be
instituted at once. Magnesium sulfate is the drug of choice for tocolysis. (bed rest, cervical cultures,
antibiotics, steroid, Magnesium)
Complications: RDS, intraventricular hemorrhage, PDA, necrotizing enterocolitis, retinopathy of
prematurity, bronchopulmonary dysplasia, death.

297
POST-DATES PREGNANCY
In cases of post-term pregnancy (42-43 weeks), the non-stress test and biophysical profile should be
performed twice weekly and if there is oligohydramnios (amniotic fluid index 5) or if spontaneous
decelerations are noted, delivery has to be accomplished.

GESTATIONAL HYPERTENSION
The increased in BP that appears before 20 weeks gestation is either chronic HTN or hydatiform mole.
Gestational hypertension (formerly known as pregnancy-induced hypertension) is idiopathic
hypertension without significant proteinuria (< 300 mg/L) that begins in the second half of pregnancy,
during labor, or within 48 hours of delivery.
Some patients may go on to develop preeclampsia.
Retrospective diagnosis is made 112 weeks after delivery.

MILD PREECLAMPSIA
Preeclampsia is a hypertensive disorder that is defined by the association of hypertension, non-
dependant edema (in the hands and face), and proteinuria > 300mg/24hr and it occurs almost
exclusively in the young primigravida woman in her third trimester. Severe preeclampsia is defined by
BP greater than 160/110 and the presence of one or more of certain signs which include head-ache,
elevated serum creatinine and many others.
Beta-2 agonists may worsen the edema by decreased water clearance, tachycardia and increased
myocardial workload.
In cases of mild preeclampsia, if the pregnancy is remote from term and/or fetal lungs are not mature yet,
the patient is managed with bed rest, salt-reduced diet, and close observation.
Edema of the lower extremities in pregnancy is most commonly a benign problem. Also, leg cramps
are very common.
Pre-eclampsia should be suspected if the edema is associated with hypertension or proteinuria, or if it
is located on the hands and/or face. Know when to order Duplex.
The most effective treatment of preeclampsia and eclampsia is delivery and evacuation of the placenta.
In cases of mild preeclampsia, if the pregnancy is remote from term and/or fetal lungs are not mature
yet, the patient is managed with bed rest, salt-reduced diet, and close observation.
Methyldopa (centrally-acting alpha-adrenergic agonist) can be used to treat sustained blood pressure in
excess of 160/110 mm Hg.
Dexamethasone administration between 24 and 34 weeks gestation to accelerate lung maturity should
be considered.
Once the fetal lungs are deemed mature, delivery should be carried out.
Magnesium sulfate is used for the prevention of eclamptic seizures. In mild preeclampsia, it is
administrated during labor and within 24 hours of delivery.

298
SEVERE PREECLAMPSIA
Preeclampsia is defined as hypertension occurring in the second trimester of pregnancy and associated
with proteinuria. Risk factors include first conception, diabetes, renal failure, and extremes of age. The
hypertension regresses after delivery.
Severe preeclampsia is defined as a BP 160/110, proteinuria > 5 g/24 hours and/or the presence of
one or more of the following signs:
Oliguria (<500 mL/day)
Altered consciousness, headache, scotoma or blurred vision
Pulmonary edema or cyanosis
Epigastric or right upper quadrant pain
Microangiopathic hemolysis
Altered liver function tests
Elevated serum creatinine levels
IUGR, or oligohydramnios
Retinal hemorrhage is considered to be an extremely ominous sign of preeclampsia.
In severe preeclampsia, the patient has to be evaluated and stabilized before management decision is
taken. Bed rest and salt reduced diet are mandatory; patients with blood pressure greater than 160/110
mmHg necessitate antihypertensive therapy. If the response to treatment is prompt and the patient is
stabilized, the decision will then depend on the term: If the patient is at term or fetal lungs are mature,
delivery has to be done. In the opposite case, delivery can be delayed until 34 weeks gestation or until
fetal lungs become mature.

Magnesium sulfate
Magnesium sulfate is also used to prevent eclampsia, through IM injections.
The rate of administration is 1g/min until the seizures stop, without exceeding 4-6 g.
For prevention, it is used via IM injections.
Magnesium sulfate is given to women with eclampsia and severe preeclampsia in order to prevent the
further development of seizures while delivery of the fetus accomplished. Delivery is the most
important overall treatment of eclampsia.
In patients with mild disease, the prophylaxis is performed during labor, delivery, and within 24 hours
after delivery.
In patients with severe disease, the prophylaxis is started on admission and carried out throughout the
periods of evaluation and observation.
It is very important for patients on magnesium sulfate to be closely observed by regularly examining their
deep tendon reflexes.
The main inconvenience of magnesium sulfate is its toxicity; it manifests with hyporeflexia depressed
deep tendon reflexes (its first sign), respiratory depression, CNS depression, coma, and death by
cardiac arrest.
The treatment of MgSO4 toxicity is to immediately stop the infusion then to administer calcium
gluconate.

299
ECLAMPSIA
Eclampsia is diagnosed when unexplained convulsions (very short duration few seconds) occur in the
setting of preeclampsia.
Magnesium sulfate is given to women with eclampsia and severe preeclampsia in order to prevent the
further development of seizures while delivery of the fetus accomplished. Depressed deep tendon
reflexes is the earliest sign of magnesium sulfate toxicity which requires stopping of the magnesium
sulfate infusion and administration of calcium gluconate.
Delivery is the most important overall treatment of eclampsia.
The major cause of death in eclampsia is hemorrhagic stroke.

CHRONIC HYPERTENSION
hypertension that appears before 20-weeks gestation is either chronic hypertension or hydatiform mole.
Chronic hypertension is present before conception and at < 20 weeks gestation or may persist for >
12 weeks postpartum.
Treatment is as follows:
Monitor BP closely and treat with appropriate antihypertensives (e.g., methyldopa, -blockers,
hydralazine, calcium channel blockers).
Do not give ACEIs or diuretics, as ACEIs are known to uterine ischemia and diuretics can
aggravate low plasma volume to the point of uterine ischemia. Complications are similar to those of
preeclampsia [Labetalol is a perfect substitute of enalapril in pregnant women with diabetic
nephropathy].

If proteinuria or significant increase in BP is noted as the pregnancy progress, chronic hypertension


with superimposed preeclampsia is diagnosed.

Hypertension in a pregnant female in the setting of massive proteinuria, malar rash, and the positive
ANA titer is most likely due to systemic lupus erythematosus which frequently causes
glomerulonephritis.

Chronic hypertension is the most commonly identified risk factor for abruptio placenta.

Transient hypertension
Transient or late hypertension is hypertension that appears in the second half of pregnancy, during labor,
or delivery
It is not accompanied by proteinuria (<300mg/24hr).
Preeclampsia is diagnosed as soon as proteinuria exceeds 300mg/24hr.

300
HELLP SYNDROME
Combination of thrombocytopenia (low platelet count), microangiopathic hemolytic anemia (
indirect bilirubin & RBCs fragments on blood smear) and increased liver enzymes in a patient with
preeclampsia (mild hypertension & mild proteinuria) is defined as HELLP syndrome.
Right upper quadrant pain is typical due to distension of the hepatic Glissons capsule.
Delivery is the definitive treatment of HELLP syndrome in women beyond 34-weeks gestation.

Triad of renal failure, microangiopathic hemolytic anemia and thrombocytopenia occurs in hemolytic
uremic syndrome.

THYROID DISEASE IN PREGNANCY


Grave's disease is the most common cause of maternal hyperthyroidism. New onset, significant
arrhythmias (not premature beats) in a pregnant patient could be from hyperthyroidism. TSH should be
ordered as the next step.

In many patients with Grave's disease, the circulating levels of thyroid stimulating immunoglobulin
TSI (IgG autoantibodies) remain as high as 500 times the normal value for several months following
thyroidectomy.
Infants born to patients with Grave's disease treated with surgery are at risk for thyrotoxicosis because of
the passage of thyroid stimulating immunoglobulin (TSI) across the placenta.

In pregnancy, thyroid hormone requirements will be increased, and the patient should be monitored every
4-6 weeks for dose adjustments.
Estrogen replacement therapy increases the requirement for L-thyroxine in patients receiving estrogen
replacement therapy. The potential causes may include induction of liver enzymes, increased level of
TBG, and an increased volume of the distribution of thyroid hormones.
Grave's disease and migraine headaches improve in pregnancy.
Neonates of patients with Grave's disease treated with surgery are at risk for thyrotoxicosis because of the
passage of thyroid stimulating immunoglobulin across the placenta.

GLUCOSE INTOLERANCE IN PREGNANCY


Diabetes screening is performed between 24 and 28 weeks of gestation.
The screening test is the 1 hour 50 gram oral glucose tolerance test.
After 1 hour, if the blood glucose value is < 140 mg/dL, the gestational diabetes is ruled out.
If the blood glucose value is >140mg/dL, the 3 hour 100 g OGTT is used for confirmation.
Complications: the newborn of diabetic mother may complains of hypoglycemia, hypocalcemia,
polycythemia hyperviscosity (caused by elevated erythropotein from relative intrauterine hypoxia),
hyperbilirubinemia, respiratory distress syndrome and/or cardiomyopathy.

301
URINARY TRACT INFECTION IN PREGNANCY
The increased incidence of UTI seen in females is due to the shorter length of the urethra.
Other predisposing factors are the use of spermicidal contraceptives, a wet periurethral environment,
urethral termination beneath the labia and the close proximity of the urethra to the anus.
Voiding after intercourse has been shown to decrease the risk of UTI in sexually-active females.

Asymptomatic bacteruiria
Asymptomatic bacteruiria is a positive urine culture (clean-catch urine culture grows greater than
100,000 colonies/mL) in the absence of urinary tract symptoms.
In pregnant patients, asymptomatic bacteruiria increases the risk of developing cystitis and
pyelonephritis more than in the non-pregnant state.
All pregnant women with asymptomatic bacteruiria should be treated with antibiotics, especially women
at 12-16 weeks gestation, in order to decrease the risk of invasive tract infections (cystitis,
pyelonephritis), preterm deliveries, and low birth weight children.
Escherichia coli accounts for more than 70% of cases.
Recommended antibiotics in pregnancy are ampicillin, nitrofurantoin, or first generation oral
cephalosporins.

Acute fatty liver of pregnancy (AFLP)


AFLP is a rare complication of pregnancy that is characterized by the occurrence of an acute hepatic
failure in the third trimester or in early post partum.
The clinical picture is variable and may include nausea and vomiting, jaundice, abdominal pain,
hepatomegaly, and an altered mental status that progresses frequently to coma.
The work up usually reveals hyperammonemia, hyperbilirubinemia, hyperurisemia, hypoglycemia,
increased PT and PTT, and moderate elevation in transaminase levels.
Hypertension and proteinuria can be present in 50 % of cases, and DIC is often associated.
Histopathology shows microvesicular fatty infiltration of the hepatocytes, a histopathologic
presentation similar to that seen in Reye syndrome.
The treatment is supportive, and with early diagnosis and rapid delivery, the outcome is often
favorable.

Intrahepatic cholestasis of pregnancy (ICP)


ICP is a functional disorder of bile formation that develops in the second and third trimesters of
pregnancy.
The condition is characterized by intense pruritus and increased serum bile acid concentrations.
The etiology is unclear, but it is thought that genetic and hormonal factors (eg, higher levels of estrogen
or progesterone) influence the development of ICP.

302
Clinical manifestations of ICP include intense, often intolerable generalized pruritus that is especially
significant on the palms and soles and worsens at night.
Evidence of skin excorations may be present. Jaundice exists in 10-20% of patients.
On laboratory evaluation, serum total bile acids are typically increased and may be the only finding.
Liver function studies are sometimes suggestive of cholestasis, with alkaline phosphatase and the total
and direct bilirubin increased. GGTP is usually either normal or only mildly elevated. Serum
aminotransferases may be quite high (sometimes > 1000U/L), which requires the ruling out of viral
hepatitis.

The diagnosis of ICP is indeed one of exclusion.


It is founded on the presence of pruritus and elevated levels of serum bile acids and/or
aminotransferases in the absence of other diseases that could cause such findings.
Treatment is based on symptom relief and preventing complications in the mother and fetus.
Ursodeoxycholic acid is most promising, as it increases bile flow and can relieve pruritus.
The maternal prognosis of those with ICP is good, as the condition resolves shortly afterdelivery. There
are no hepatic sequelae. ICP may recur in subsequent pregnancies, and affected women are also at
increased risk of developing gallstones.

Intrahepatic cholestasis of pregnancy poses more danger to the fetus than to the mother, as fetal
prematurity, meconium-stained amniotic fluid, and intrauterine demise are all known complications.
Therefore, the preferred approach in managing the pregnancies of women with ICP includes an early
delivery once fetal lung maturity is established.

INTRAPARTUM EVENTS
Fetal Malpresentation
Defined as any presentation other than vertex (i.e., head closest to birth canal, chin to chest, occiput
anterior).
Risk factors include prematurity, prior breech delivery, uterine anomalies, poly- or oligohydramnios,
multiple gestations, preterm PROM, hydrocephalus, anencephaly, and placenta previa.
Breech presentations are the most common (3% of all deliveries) and involve presentation of the fetal
lower extremities or buttocks into the maternal pelvis. Subtypes include the following:
Frank breech (5075%): Thighs are flexed and knees are extended.
Footling breech (20%): One or both legs are extended below the buttocks.
Complete breech (510%): Thighs and knees are flexed.

DIAGNOSIS
Use Leopold maneuvers (a series of uterine palpations) to identify fetal lie (transverse or vertical) and
presentation (vertex or breech).

TREATMENT
Follow: Up to 75% spontaneously change to vertex by week 38.

303
External version: If the fetus has not reverted spontaneously, apply pressure to the maternal abdomen to
turn the infant to vertex. The success rate is roughly 75%. Risks are placental abruption and cord
compression, so be prepared for an emergency C-section if needed.
Trial of breech vaginal delivery: Attempt only if delivery is imminent; otherwise contraindicated.
Complications include cord prolapse and/or head entrapment.
Elective C-section: The standard of care in many hospitals, but it has not been shown to improve
outcome.
Before 37-weeks of gestation, fetuses in breech presentation need no intervention as they may convert to
vertex automatically.
Most of the breech presentations assume cephalic presentation by 34-weeks gestation.

Breech presentation
Breech presentation occurs when the buttocks or lower extremities of the fetus present first into the
maternal pelvis.
Approximately, 25% of fetuses less than 28 weeks gestation are in the breech presentation. Most of these
assume the cephalic presentation by 34 weeks gestation. That's why prematurity represents a major risk
factor. Breech presentation is also predisposed by any anomaly, fetal or maternal, that will prevent the
fetus from completing its rotation during this period.
For instance: congenital anomalies (hydrocephalus, anencephaly and spina bifida), placenta previa, pelvic
tumors, poly or oligohydramnios, multiple gestation, uterine anomaly, and contracted maternal pelvis.
There are three types of breech presentation:
1) Frank breech, in which the fetal legs are stretched in front of the fetal trunk;
2) Complete breech, in which both thighs and knees are flexed, and thus, the presenting parts are the feet
and the buttocks;
3) Footling or incomplete breech, in which one or both of the hips are not flexed in way that the foot or
knee lies below the buttocks.
The management of breech presentation depends on several maternal and fetal factors. And for vaginal
delivery to be decided, the mother as well as the fetus, should meet strict criteria, which are as follows:
Fetus in frank or complete breech.
Gestational age of at least 36 weeks.
Estimated fetal weight between 2500g and 3800g: if the fetal body is too thin or small, it can therefore
easily slip through the cervix before its full dilation, resulting in head entrapment.
Flexed fetal head.
Adequately large maternal pelvis, as assessed by pelvimetry or tested by a prior delivery.
No fetal or maternal indications of cesarean section.
Experienced obstetrician.
External cephalic version is used to convert a breech into a cephalic presentation. It is indicated
between 37 weeks gestation and the onset of labor.
Internal podalic version is indicated in twin delivery, to convert the second twin from transverse or
oblique presentation, to breech presentation.

304
STAGES OF LABOR
Labor is defined as the progressive cervical effacement, dilatation, or both resulting from uterine
contractions, which occur at least every 5 minutes and last 30 to 60 seconds.
Labor progresses through four stages:
Stage 1
The first stage extends from the onset of labor until full dilation of the cervix, and includes two phases:
Latent phase, during which dilation progresses at a slow rate until reaching 2-3cm. The length of the
latent phase is highly variable but is considered prolonged when it exceeds 20 hours in the primiparous
and 14 hours in the multiparous ;followed by
Active phase, during which the dilation is more rapid. The progression of the active phase is
evaluated by the rate of cervical dilation. At this phase, the cervix normally dilates at a rate of at least
1cm/hr in the primiparous and 1.2cm/hr in the multiparous.

Stage 2
The second stage of labor extends from complete dilation of the cervix to delivery of the baby.
It usually lasts 30 min to 3 hours in the primiparous, and 5 to 30 minutes in the multiparous.

Stage 3
The third stage of labor starts with the delivery of the baby, and ends with the delivery of the placenta.

Stage 4
The fourth stage extends from delivery of the placenta until 6 hours postpartum.
The mother should be closely observed during this stage because of the risk of postpartum hemorrhage.

305
CONDUCT OF NORMAL SPONTANEOUS LABOR
Prolonged latent phase
Prolonged latent phase can be caused by hypertonic uterine contractions, hypotonic contractions, or
premature or excessive use of anesthesia or sedation.
Hypertonic contractions, although intense, are ineffective. They are more painful and are associated
with increased uterine tone. Hypertonic activity of the uterus usually responds to therapeutic rest with
morphine sulphate or an equivalent drug.
Hypotonic contractions are less painful and are characterized by an easily indentable uterus during the
contraction. Sometimes, patients diagnosed withprolonged latent phase may actually be still in false
labor. Contractions of false labor are usually painless and sporadic, but can be rhythmic, occurring
every 10 to 20 minutes. Their main characteristic, however, is that they are not accompanied with
cervical changes. Patients with hypocontractile dysfunction are best treated with a diluted infusion of
oxytocin (Excessive use of oxytocin may cause water retention, hyponatremia and seizures water
intoxication).
Anesthesia may reduce uterine activity if administered in the latent phase. In the active phase, it has
either the opposite effect, or no significant effect. The only treatment when it is the cause of a prolonged
latent phase is to allow the responsible drug to be evacuated (therapeutic rest); the uterus usually
resumes its normal activity afterwards.

Prolonged or arrested active phase


Active phase arrest occurs when dilation fails to progress in the active phase of labor over a period of at
least two hours.
In the primigravida, cervical dilatation in the active phase progresses at a speed of 1.2 cm/hr.
So after 2 hours this patient must be at 7.5 cm.
Active phase arrest may be due to abnormal fetal lie, fetopelvic disproportion, inadequate uterine
contractions conduction anesthesia, excessive sedation,or cervical anomalies.
Midplevic contraction which is indicated by prominent ischial spines is an important cause of arrest
disorder of dilation (cervical dilation has been the same for over 2 hours, or the descent has not
progressed for more than 1 hour). Arrest disorder resulting from midpelvic contraction is treated with C.
section (low-transverese C. section)
Forceps can not be used unless the cervix is fully dilated.

ABNORMAL LABOR
False labor
In false labor, contractions are felt in the lower abdomen, irregular (but can be rhythmic, occurring
every 10 to 20 minutes), occur at an interval that does not shorten and do not increase in intensity.
In all cases of false labor contractions are not accompanied by progressive cervical changes and
discomfort is readily relieved by sedation.
All such patients need reassurance and discharge to the home.

306
Fetal Heart Decelerations
TYPE DESCRIPTION MOST COMMON CAUSE
Early Decelerations (15, 15) begin at Cephalic compression (no fetal distress).
approximately the same time as the
maternal contractions and end before or
with the contraction have ceased.
Variable Decelerations occur at any time during the Umbilical cord compression.
maternal contraction. Change mothers position (e.g., back to
side) and mask oxygen
Late Decelerations begin at the peak of the Uteroplacental insufficiency and fetal
contraction and persist until after the hypoxemia, possibly due to abruption or
contraction has finished. hypotension.
Further testing for reassurance is necessary.
If late decelerations are repetitive and
severe (Fetal distress), deliver the baby
ASAP (an indication for emergent
C.section).

Fetal sleep presents with decreased long-term variability.

Intrauterine infection presents with fetal tachycardia (HR > 160), which may be associated with other
signs of fetal distress.

In the presence of decreased fetal movements, fetal compromise should be suspected, and the best next
step in management in such case is the performance of a non-stress test (NST).

CESAREAN SECTION
Indications for Cesarean Section
MATERNAL FETAL AND MATERNAL FETAL
Prior classical C-section Cephalo-pelvic disproportion Fetal malposition (e.g.,
(vertical incision predisposes (most common cause of 1 C- posterior chin, transverse lie,
to uterine rupture with vaginal section) shoulder presentation)
delivery) Placenta previa/placental Fetal distress
Active genital HSV infection abruption Cord compression
(clear vesicles, shallow ulcers Failed operative vaginal Erythroblastosis fetalis (Rh
and inguinal adenopathy) delivery incompatibility)
Cervical carcinoma Post-term pregnancy (relative
Maternal trauma/demise indication)

307
OBSTETRIC ANESTHESIA
Anesthesia may reduce uterine activity if administered in the latent phase.

Epidural anesthesia
Hypotension is a common side effect of epidural anesthesia. The cause of hypotension is blood
redistribution to the lower extremities and venous pooling in the lower extremities.
Epidural anesthesia may cause overflow incontinence (caused by bladder denervation) as a transient side
effect. It is best treated with intermittent catheterization.

Amniotic fluid embolism


Amniotic fluid embolism may occur after amniocentesis or during labor.
Abrupt onset of hypoxia with respiratory failure, cardiogenic shock and seizures, in a patient who had
undergone amniocentesis or delivered, is most likely due to amniotic fluid embolism.
Disseminated intravascular coagulation (DIC) is the most feared complication in patients with
amniotic fluid embolism.
Respiratory support is always the first step of management.
If facemask ventilation does not correct the hypoxia, intubation should be the next step.

Postpartum discharge
A significant granulocytic leukocytosis may be seen in immediate postpartum period.
Normal Lochia rubra (sweetish odor, and bloody discharge along with some small blood clots on the
introitus and vaginal walls), is characteristic of the first few days postpartum.
After 3 to 4 days, the color becomes paler and the discharge is then named lochia serosa. It turns
afterwards white or yellow and becomes lochia alba.

POSTPARTUM HEMORRHAGE
Defined as a loss of > 500 mL of blood for vaginal delivery or > 1000 mL for C-section occurring
before, during, or after delivery of the placenta.
Complications include acute blood loss (potentially fatal), anemia due to chronic blood loss (predisposes
to puerperal infection), and Sheehans syndrome.

UTERINE ATONY
Risk factors: Uterine overdistention (multiple gestation, macrosomia, polyhydramnios). Exhausted
myometrium (rapid or prolonged labor, oxytocin stimulation). Uterine infection. Conditions interfering
with contractions (anesthesia, myomas, MgSO4).
Diagnosis: Palpation of soft, enlarged, boggy uterus. Most common cause of postpartum
hemorrhage (90%).

308
Treatment: Bimanual uterine massage (usually successful). Oxytocin infusion. Methergine
(methylergonovine) if not hypertensive. Prostin (PGF2) if not asthmatic.

GENITAL TRACT TRAUMA


Risk factors: Precipitous labor. Operative vaginal delivery (forceps, vacuum extraction). Large infant.
inadequate episiotomy repair.
Diagnosis: Manual and visual inspection of lower genital tract for any laceration > 2 cm long.
Treatment: Surgical repair of the physical defect.

RETAINED PLACENTAL TISSUE


Risk factors: Placenta accreta/increta/percreta. Placenta previa. Uterine leiomyomas. Preterm delivery.
Previous C-section/curettage.
Diagnosis: Manual and visual inspection of placenta and uterine cavity for missing cotyledons.
Ultrasound may also be used to inspect uterus.
Treatment: Manual removal of remaining placental tissue. Curettage with suctioning (take care not to
perforate the uterine fundus).

SHEEHANS SYNDROME (POSTPARTUM PITUITARY NECROSIS)


Defined as pituitary ischemia and necrosis anterior pituitary insufficiency 2 to massive obstetric
hemorrhage and shock decrease in prolactin, TSH and FSH.
The 1 cause of anterior pituitary insufficiency in adult females. The most common presenting syndrome
is failure to lactate (due to prolactin levels).
Other symptoms include weakness, lethargy, cold insensitivity, genital atrophy, and menstrual disorders.

POSTPARTUM FEVER
In the immediate postpartum period, a low-grade fever, leukocytosis and vaginal discharge are normal
findings. The vaginal discharge (lochia) is initially bloody, then serious and finally white to yellow in
color days following delivery.
Puerperal fever is defined as an increase in temperature ?38C for more than 2 consecutive days in the first
10 days of postpartum.
The 7 Ws of post-partum fever:
Wind (atelectasis 0, pneumonia 2-3)
Water (UTI 1-2)
Womb (endomyometritis 2-3)
Wound infection (incision, episiotomy 4-5)
Walk (DVT, pulmonary embolism 5-6)
Weaning (breast engorgement, abscess, mastitis 7-21)
Wonder drugs (drug fever)

309
Postpartum Infections
Genital tract infection with a temperature 38C for at least two of the first ten postpartum days (not
including the first 24 hours).
Endometrial infection is most common. Incidence is after C-section.
Risk factors include emergent C-section, PROM, prolonged labor, multiple intrapartum vaginal exams,
and intrauterine manipulations.
Etiologies are outlined in the mnemonic the 7 Ws. Perform a pelvic exam to rule out hematoma or
lochial block.
Consider UA and culture (for UTI) and blood cultures (for sepsis).
Consider other diagnoses (e.g., pelvic abscess, septic pelvic thrombophlebitis) if the three-drug regimen
is not effective after 48 hours.
Hospitalize and give broad-spectrum IV antibiotics (e.g., clindamycin and gentamicin) until patients
have been afebrile for 48 hours. Add ampicillin for complicated cases.

Endometritis
Endometritis is the most common cause of puerperal fever, occurs on the 2nd and 3rd day postpartum.
Predisposing risk factors:
Prolonged labor
Prolonged and premature rupture of membranes
Manual removal of the placenta
Repeated pelvic examinations
More frequent after C section or operative vaginal delivery (episiotomy)
Clinically it presents with fever, enlarged & tender uterus and foul smelling lochia.
Postpartum endometritis is most commonly polymicrobial infection; composed of gram positive, gram
negative, aerobic and anaerobic organisms. It is due in 70% of cases to anaerobic organisms such as
Peptostreptococcus, Peptococcus and streptococcus, often mixed with Bacteroides fragilis. Of aerobic
pathogens, E. coli is the most common, followed by enterococci.
When endometritis is diagnosed, cultures for aerobic and anaerobic organisms should be obtained.
Antibiotic therapy should be instituted as early as possible and should provide coverage of both
aerobic and anaerobic organisms; the most appropriate antibiotic combination is clindamycin (effective
against anaerobes) + gentamicin (excellent gram-negative coverage) or clindamycin + ampicillin.

Thrombophlebitis
Pelvic thrombophlebitis is a diagnosis of exclusion.
It is a condition predisposed by the pelvic venous stasis usually present after delivery and occurs when
there is a large inoculum of anaerobic pathogen on that level.
Clinically, it is suspected in the setting of a persistent spiking fever for 7 to 10 days postpartum, which
fails to respond to antibiotic therapy.

310
When suspected, heparin therapy should be added promptly to antibiotics and maintained for 2 to 3
weeks. When anticoagulant therapy is started, a rapid response must be noted, or else, pelvic abscess
should be suspected.

Superficial femoral vein is a deep vein of the thigh and a thrombus of this vein requires anticoagulation.

Lactation and Breast-Feeding


Physiology: During pregnancy, estrogen and progesterone breast hypertrophy and inhibition of
prolactin release. After delivery of the placenta, hormone levels markedly and prolactin is released,
stimulating milk production. Periodic infant suckling further release of prolactin and oxytocin, which
stimulate myoepithelial cell contraction and milk ejection (let-down reflex).
Breast-feeding: Colostrum (early breast milk) contains protein, fat, secretory IgA, and minerals. Within
one week postpartum, mature milk with protein, fat, lactose, and water is produced.
High IgA levels in colostrum provide passive immunity for the infant and protect against enteric bacteria.
Other benefits include incidence of allergies, facilitation of mother-child bonding, and maternal weight
loss.
Some contraindications to breastfeeding include certain active maternal infections (i.e., HIV,
tuberculosis, herpes simplex lesions on or near the nipple, malaria, sepsis, typhoid fever), eclampsia,
nephritis, breast cancer, substance abuse, and use of certain medications (e.g., tetracycline,
chloramphenicol, warfarin).
Transmission of HIV by breastfeeding is well documented; therefore, the presence of maternal HIV
infection is an absolute contraindication to breastfeeding.
Breast milk jaundice is not a contraindication to breastfeeding.
Hemolytic disease of the newborn (erythroblastosis fetalis) is not a contraindication to breastfeeding
because antibodies in the mother's milk are inactivated in the intestinal tract and do not contribute to
further hemolysis of the infant's RBCs.

There is no medications in the suppression of breast milk production (No bromocriptine for lactation
suppression anymore).
Patients are advised to wear a tight-fitting bra, avoid nipple manipulation and use ice packs and
analgesics to relieve associated pain.

311
GYNECOLOGIC NEOPLASIA
Endometrial cancer-HTN and DM
Breast cancer- late childbirth and pauciparity
Cervical cancer-smoking, HPV
OCPs reduce the risk of ovarian and endometrial cancers, as well as benign breast disease

ENDOMETRIAL NEOPLASIA
When to perform endometrial biopsy to rule out endometrial carcinoma in patients with DUB? If the
patient is older than 35 years, obese, diabetic or with chronic hypertension.
Premenopausal women with simple or complex hyperplasia without atypia respond to therapy with
cyclic progestins.
All patients should undergo repeat biopsy after 3-6 months of treatment.
The risk of progression to endometrial cancer in patients with complex hyperplasia with atypia is low and
therefore even if the patient does not want more children, hysterectomy is not warranted.

CERVICAL INTRAEPITHELIAL NEOPLASIA (CIN)


An abnormal Pap smear in an asymptomatic woman with no grossly visible cervical changes.
No specific symptoms or signs; the cervix appears grossly normal.
Pap smear: Reveals dysplasia or carcinoma in situ.
Pap smear should be performed annually in all women three years after become sexually active but no
later than 21 years of age. After 3 normal results 1 year apart, perform the screening every 2 to 3 years.
Women who (1) are immunocompromised, (2) had in utero exposure to DES, or (3) have a history of
CIN II/III or cancer should have annual Pap smears, even if their prior Pap smears were negative.

Pap smear findings

312
Colposcopy is indicated if Pap smear is abnormal.
Cervical dysplasia in a high risk patient should be investigated with colposcopy.
If Pap smear reveals a dysplasia, perform colposcopy.
If Pap smear reveals an inflammatory atypia (ASCUS according to Bethesda classification), repeat after
4 to 6 months. If a repeat Pap smear after 12 months is also abnormal then perform Colposcopy.

Colposcopy: Reveals an atypical transformation zone with white patches and vascular atypia that
indicates areas of greatest cellular activity.
Colposcopically directed punch biopsy and endocervical curettage.

During pregnancy, the primary goal of colposcopy is the exclusion of invasive cancer. Any woman with
cytologic specimen suggesting HSIL should undergo colposcopy and direct biopsy. If the biopsy is
negative, a second biopsy is recommended 6-8 weeks after delivery.

TREATMENT

Low grade squamous intraepithelial lesion (LSIL) (Mild dysplasia or CIN I)


Adolescents Repeat Pap in 12 months
Premenopausal woman Colposcopy
Postmenopausal
Colposcopy
Reflex HPV testing, if positive Colposcopy

Atypical squamous cells of undetermined significance (ASC-US)


Adolescents Repeat Pap in 12 months
Non-adolescent woman Reflex HPV testing; if positive Colposcopy.

CIN I: Pap smear or colposcopy every 34 months for one year. With resolution, patients can return
to an annual Pap smear.
CIN I

CIN I preceded by low grade abnormalities CIN I preceded by high grade abnormalities
(ASC-US; ASC-H, LSIL) (HLSIL; AGC-NOS)

Repeat cytology after 24 months Immediate diagnostic excisional procedure
(post child bearing period)
Still abnormal after 24 months (Or)
Close clinical follow-up
Diagnostic excisional procedure (Women planning future pregnancy)

313
CIN II and CIN III:
Exocervix: Laser or cryotherapy.
Endocervix: Conization with the loop electrosurgical excision procedure (LEEP) or cold knife biopsy.
Excisions in pregnant women should be considered only if a lesion suggestive of invasive cancer is
detected at colposcopy.

CERVICAL CANCER
Patients affected: Premenopausal, > 40 years of age.
Risk factors for cervical cancer include the following:
Multiparity, smoking, early initiation of intercourse, number of sexual partners, HIV infection,
venereal warts, family history.
HPV: Types 6 and 11 mild dysplasia; types 16, 18, and 31 higher-grade cellular changes.

HISTORY/PE
Often asymptomatic. May present with postcoital bleeding, menorrhagia, pelvic pain, or vaginal
discharge. Look for cervical discharge/ulceration or pelvic mass.

DIAGNOSIS
Screening tests: Annual Pap smear for sexually active women [Pap smear should be performed annually
in all women once they become sexually active. After 3 normal results 1 year apart, perform the
screening every 2 to 5 years].
Precursor lesion: Cervical intraepithelial neoplasia (CIN) is a common diagnosis; most often seen in
women 2540 years of age. Associated with HPV.
Pap smear. However, treatment is never justified until a definitive diagnosis has been established
through biopsy [For gross lesions, biopsy is needed and Pap smear is no longer appropriate. Know and
understand the utility and indications of each procedure aiming at screening or diagnosing cervical pre
malignancies].
Cervical biopsy and endocervical curettage or conization to determine the extent and depth of invasion of
the cancer.
Clinical staging (under anesthesia) to estimate gross spread.
The depth of penetration of malignant cells beyond the basement membrane serves as a reliable clinical
guide to the extent of cancer within the cervix as well as a predictor of the likelihood of metastases.
Imaging: Abdominal and pelvic CT or MRI.

TREATMENT
Carcinoma in situ (stage 0):
Women who have completed childbearing: TAH.
Women who wish to retain the uterus: Cervical conization or ablation of the lesion with cryotherapy or
laser.

314
Invasive carcinoma:
Microinvasive carcinoma (stage IA): Simple extrafascial hysterectomy.
Stage I (limited to cervix): Hysterectomy.
Radiation therapy:
Stage II: Upper two-thirds of the vagina or parametria.
Stage III: Lower third of the vagina or pelvic side wall; all cases ofhydronephrosis.
Stage IV: Bladder or rectal involvement or distant metastases.
Radical surgery fewer long-term complication than irradiation and may allow preservation of ovarian
function; it is therefore the preferred mode of therapy in younger women.
Close follow-up with Pap smears every three months for one year and every six months for another year
is necessary after cryotherapy.

Summary of Treatment
Early: Radiotherapy, radical hysterectomy and lymphadenectomy.
Advanced: Irradiation/chemotherapy (surgery may harm the bladder and rectum without being effective)

COMPLICATIONS
Uremia is the most common cause of death in end-stage cervical cancer.
Metastases to regional lymph nodes occur with frequency from stage I to IV.
GU complications include the following:
Ureters are often obstructed lateral to the cervix hydroureter, hydronephrosis, and renal insufficiency.
Almost two-thirds of untreated patients die of uremia when ureteral obstruction is bilateral.

UTERINE NEOPLASIA
Fibroid uterus
Submucosal fibroids often interfere with implantation of the embryo resulting in infertility.
Fibroids are the most common benign uterine tumors in women.
Fibroids are the most common indication for hysterectomy.
They are estrogen-dependent tumors; therefore, they increase in size with oral contraceptive pills
(OCPs) or pregnancy, and often regress after menopause.

Leiomyomata
Leiomyomata or leiomyomatous uterus can be very difficult to distinguish from Adenomyosis since both
can present with dysmenorrhea, menorrhagia and large sized uterus.
Nevertheless, the consistency of the uterus is softer in Adenomyosis.
For women above 35, it is mandatory to perform an endometrial curettage, or even a hysterectomy, to
rule out endometrial carcinoma.

****Presence of dysmenorrhea, menorrhagia, and enlarge uterus is almost diagnostic of either


adenomyosis or fibroid uterus.

315
Adenomyosis
Adenomyosis is defined as the presence of endometrial glands in the uterine muscle. This invasion
can extend to the full thickness of the myometrium, or in some instances, to the serosa of the uterus.
Adenomyosis occurs most frequently in women above 40 and typically presents with severe
dysmenorrhea and menorrhagia.
The physical exam reveals an enlarged and generally symmetrical uterus.
The consistency of the uterus is softer in Adenomyosis.
Presence of dysmenorrheal, heavy menses, and enlarged uterus is almost diagnostic of either
adenomyosis or fibroid uterus.
For women above 35, it is mandatory to perform an endometrial curettage, or even a hysterectomy, to
rule out endometrial carcinoma.

OVARIAN NEOPLASIA

Meigs syndrome: ovarian fibroma + ascites + right hydrothorax.


Krukenbergs tumor: metastasis of stomach cancer to both ovaries.

Granulosa cell tumors produce excessive amounts of estrogen, and can present with precocious puberty
in younger children and postmenopausal bleeding in elderly patients.
This has to be differentiated from heterosexual precocious puberty or virilizing symptoms which are
usually produced by excessive androgens.

316
Solid ovarian tumors (viewed by ultrasonography) are almost always malignant and demand immediate
and aggressive evaluation and treatment in all age groups, except in pregnancy.
Bilateral solid ovarian masses discovered incidentally in a multiparius African-American pregnant
woman most likely signify a pregnancy luteoma. This is a benign self-limited condition and requires
no treatment.

Sudden onset of hirsutism or virlization during pregnancy

No ovarian mass Bilateral cystic Bilateral solid Unilateral solid


Abdominal CT scan to Theca Lutein cysts Mostly pregnancy Laparotomy or
rule out adrenal mass (Rule out high -HCG luteoma laparoscopic biopsy to
states) rule out malignancy

VULVAR NEOPLASIA
Hypertrophic dystrophy of vulva is most commonly seen in postmenopausal women and is treated with
local application of 1% corticosteroid ointment three times a day for 6 weeks. Hyperplastic dystrophy
cannot be distinguished clinically from cancer of the vulva and biopsy is therefore required every time
the diagnosis is suspected.
In a postmenopausal female who has vulvar itch and dryness, lichen sclerosis must be suspected.
However, vulvar carcinoma in situ must also be in the differential and a biopsy obtained if suspicion is
high.
Lichen sclerosis is usually seen in postmenopausal women but may develop at any age. It present with
pruritus, burning and dyspareunia and is treated with 'superpotent topical corticosteroids' such as
clobetasol or halobetasol.
*Itchy spot in a postmenopausal women needs biopsy.

317
GESTATIONAL TROPHOBLASTIC NEOPLASIA (GTN)
A range of proliferative trophoblastic abnormalities that can be benign (e.g., hydatidiform mole) or
malignant (e.g., choriocarcinoma).
Risk factors for GTD include extremes of age (< 20 or > 40), a diet deficient in folate or -carotene,
and blood group (e.g., a type A woman impregnated by a type O man).
Molar pregnancy may progress to malignant GTD, including invasive moles (1015%) and
choriocarcinoma (25%).
Hydatidiform mole (or molar pregnancy) accounts for approximately 80% of cases of GTD.
Preeclampsia in the first trimester is pathognomonic for hydatidiform mole.
Subtypes are as follows:
Complete moles: Usually result from sperm fertilization of an empty ovum; have a chromosomal
pattern of 46,XX and are paternally derived.
Incomplete (partial) moles: Result when a normal ovum is fertilized by two sperm (or a haploid sperm
that duplicates its chromosomes). Usually have a chromosomal pattern of 69,XXY and contain fetal
tissue.

Presents with first-trimester uterine bleeding (most common), hyperemesis gravidarum,


preeclampsia/eclampsia at < 24 weeks, uterine size greater than dates, and hyperthyroidism
(hypertension, tachycardia, tachypnea). No fetal heartbeat is detected.
Pelvic exam may reveal enlarged ovaries with bilateral theca-lutein cysts that will resolve after
treatment. It may also reveal possible expulsion of grapelike molar clusters into the vagina or blood in
the cervical os.
Look for markedly serum -hCG (usually > 100,000 mIU/mL) and a snowstorm appearance on
pelvic ultrasound with no gestational sac or fetus present.
CXR may show lung metastases.
D&C reveals cluster-of-grapes tissue.
Type and screen is critical; follow -hCG closely and prevent pregnancy for one year to ensure
accurate monitoring.
Treat malignant disease with chemotherapy (methotrexate or dactinomycin) and residual uterine
disease with hysterectomy.
Chemotherapy and irradiation are highly successful for metastases (90%).
Complications: Malignant GTD, pulmonary or CNS metastases, trophoblastic pulmonary emboli, acute
respiratory insufficiency.

318
URINARY INCONTINENCE
Defined as the involuntary loss of urine due to either bladder or sphincteric dysfunction.

UA and urine culture to exclude UTI.


Serum creatinine to exclude renal dysfunction.

Even though it is 'gold standard', routine urodynamic testing is not recommended in incontinent
patients because it is invasive, expensive, and requires special training.
Oxybutynin has antocholinergic properties and inhibits smooth muscle contraction. This drug is used for
urge incontinence.
Urethropexy is indicated in stress incontinence.

Exclude fistula in cases of total incontinence.


Look for neurologic abnormalities in cases of urge incontinence (spasticity, flaccidity, rectal sphincter
tone) or distended bladder in overflow incontinence.

319
Cystogram to demonstrate fistula sites and descensus of the bladder neck.
Treatment for overflow incontinence include cholinergic agents and intermittent self-catheterization.

Pelvic muscle exercises (Kegel exercises) and urethropexy are the recommended treatment options for
stress incontinence. Exercises should be tried first because the surgery is invasive and can be associated
with significant morbidity.

Interstitial cystitis result in urge incontinence.

ENDOMETRIOSIS
Patients with endometriosis present with dysmenorrhea, dyspareunia and dyschezia and the physical
exam reveals tender adnexal mass and firm nodularity located in the broad ligaments, the uterosacral
ligament or in the cul-de-sac.
Oral contraceptive pills are first line agents in the treatment of endometriosis in young women desiring
future fertility.
Laparoscopy is the gold standard for the diagnosis of endometriosis.

ECTOPIC PREGNANCY
In all secondary amenorrheas, pregnancy should be the first diagnosis to suspect.
In the presence of vaginal hemorrhage, abdominal pain and/or amenorrhea in a woman of childbearing
age, suspect ectopic pregnancy.
Positive pregnancy test with no evidence of an intrauterine or extrauterine pregnancy; differential
diagnosis includes early viable uterine pregnancy, ectopic pregnancy, or nonviable intrauterine
pregnancy (completed abortion).
Serial -hCG levels are required when the results of the initial U/S findings are indeterminate.
-hCG levels should double every 48 hours in viable pregnancies, but increases in a slower rate in
ectopic pregnancy and nonviable intrauterine pregnancy.
Intrauterine pregnancy should be seen with transvaginal ultrasonogram at -hCG levels from 1,500 to
2,000 mIU/mL.
If -hCG level < 1,000 mIU/mL, both -hCG and transvaginal ultrasonogram should be repeated in 2 3
days.

320
SEXUALLY TRANSMITTED DISEASES (STDS)
Lymphogranuloma venereum
Lymphogranuloma venereum is a sexually transmitted disease caused by Chlamydia trachomatis
serotypes L1, L2 and L3 and manifests with generalized malaise, headaches and fever followed by a
papule, which subsequently transforms into a painless ulcer and reactive inguinal adenitis. Classic
'groove' sign is usually seen in men and in the second stage of disease.

Granuloma inguinale (GI)


It is a STD caused by the bacterium Donovania granulomatis.
It is characterized by an initial papule, which rapidly evolves into a painless ulcer with irregular borders
and a beefy-red granular base.
Microscopic exam reveals Donavan bodies: Giemsa or Wright stain of tissue smears reveals reddish
encapsulated bipolar staining organisms that are found within large mononuclear cells momocytes.
Patients are managed with doxycycline, erythromycin, tetracycline, or TMP/SMX.

Behcets disease is a rare multisystem disorder with an autoimmune etiology and manifests with
recurrent ulceration in the mouth and genital area associated with uveitis.

Condylomata acuminata Genital warts


Condylomata acuminata are skin colored or pink, verrucous and papilliform skin lesions present around
the anus or vulva.
Vulvar papillomatosis, or condylomata acuminata, are genital lesions caused by human papilloma virus
(HPV) serotypes 6 and 11.
Condylomas present as exophytic lesions with a raised papillomatous or spiked surface and may grow
into large and cauliflower-like formations.
There are three treatment options for condyloma acuminata, which are chemical or physical agents,
immune therapy and surgery.
The choice of treatment depends upon the number and extent of lesions.
Chemical agents are trichloroacetic acid, 5-florouracil epinephrine gel and podophyllin.
Immune modulators are imiquimod and interferon alfa.
Surgical options are cryosurgery, excisional procedures and laser treatment.
Podophyllin is one of the available treatment options. It is an antimitotic agent and leads to cell death.
It is teratogenic and is contraindicated in pregnancy.

321
Syphilis
The chancre characterizes the primary stage of syphilis: it is a painless, indurated ulceration with a
punched-out base and rolled edges. Serologic testing is not reliable at this stage and includes a high rate
of false-negatives, so diagnosis in the first stage is made via spirochete identification on dark field
microscopy.
In primary stage of syphilis, serologic testing is not reliable and includes a high rate of false-negatives, so
diagnosis is made via spirochete identification on dark field microscopy.

PELVIC INFLAMMATORY DISEASE (PID)


Cervicitis
The most common cause of mucopurulent cervicitis is Chlamydia trachomatis.
The second most common communicable disease in USA is Gonorrhea.
Cervical ectopy created by OCPs predisposes to colonization with C. trachomatis.
The nucleic acid amplification test for Chlamydia is an effective screening method.
A single dose of azithromycin (1 gram) is treatment of choice for genital chlamydia infections.
7 days of doxycycline can also be used for chlamydia but because of the compliance issues single dose
azithromycin is preferred.
Given the high-rate of co-infection with Gonorrhea and Chlamydia and the poor sensitivity of gram stain
for Chlamydia; when gram stain confirms cervical gonorrhea infection (gram-negative diplococci),
antibiotic management must be aimed at eradication of both organism; Ceftriaxone plus azithromycin.
In chlamydial infection, empiric treatment of gonorrhea is not recommended.
But in Gonococcal infections empiric treatment of Chlamydia is recommended.
Untreated patients are at risk of developing complications such as pelvic inflammatory disease and
infertility, they are also more likely to spread disease for others.

Tubo-ovarian abbesses
Tubo-ovarian abbesses are usually managed with triple antibiotic therapy.
Drainage is indicated if there is no response to antibiotic therapy after 24 to 48-hours.

Chorioamnionitis
Patients with chorioamnionitis exhibit fever greater than 38C(98.7F), uterine tenderness and irritability,
elevated WBC count and fetal tachycardia. It is frequently associated with preterm or prolonged rupture
of membranes.

322
Endometritis
Endometritis is the most common cause of puerperal fever on the 2nd and 3rd day postpartum.
Predisposing risk factors include prolonged labor, prolonged and premature rupture of membranes,
manual removal of the placenta and repeated pelvic examinations. It is also more frequent after cesarean
delivery or operative vaginal delivery. Clinically it presents with fever, uterine tenderness and foul
smelling lochia. Endometritis is a polymicrobial infection; it is due in 70% of cases to anaerobic
organisms such as Peptostreptococcus, Peptococcus and streptococcus, often mixed with Bacteroides
fragilis. Of aerobic pathogens, E. coli is the most common, followed by enterococci. When endometritis
is diagnosed, cultures for aerobic and anaerobic organisms should be obtained. Antibiotic therapy should
be instituted as early as possible and should provide coverage of both aerobic and anaerobic organisms;
the most appropriate antibiotic combination is clindamycin with either aminoglycoside or ampicillin.

VAGINAL DISCHARGE
Copious vaginal discharge that is white or yellow, nonmalodorous, and occurs in the absence of other
symptoms or findings on vaginal exam is referred as physiologic leucorrhea.
Symptoms of pathologic vaginal discharge include a history of pruritus, burning and malodorous vaginal
discharge and erythema, edema and friability of vaginal mucosa; tenderness of the cervix and green or
curd-like vaginal discharge on vaginal exam.

Vaginosis
Bacterial Vaginosis: Tx of choice is metronidazole cream or clindamycin cream.
An STD caused by the bacteria Gardnerella vaginalis coccobacilli.
Presents with a profuse ivory to gray malodorous discharge.
Vaginal pH of 5 -6.5.
When add KOHfishy odor.
Identifying clue cells (vaginal epithelial cells with adherent bacteria) on a wet mount preparation of
the discharge.
Itching and burning are not usual.
The treatment of choice for Bacterial Vaginosis in a pregnant lady is clindamycin cream (not orally) or
metronidazole cream.

Candida vaginitis
C. albicans is a frequent cause of ectocervicitis that is typically associated with vulvovaginitis.
Candida vaginitis is not considered a sexually transmitted disease and occurs in presence of risk factors
such as diabetes mellitus, oral contraceptive pills, pregnancy and immunosuppressive therapy.
Pseudohyphae are characteristically seen on wet mount preparations of vaginal discharge from patients
with candida vulvovaginitis.
Vaginal ph 4.0 4.5.
Symptomatic patients are treated with an azole antifungal such as fluconazole. Clotrimazole cream is
an effective treatment for Candidal vaginitis. Sexual partner do not require treatment.

323
Trichomonas vaginitis
T. vaginalis is a frequent cause of ectocervicitis that is typically associated with vulvovaginitis.
It usually causes gray-green, thin, frothy vaginal discharge, pruritus, dysuria, and dyspareunia,
though it can be asymptomatic.
Pear-shaped motile organisms with 3 to 5 flagella on wet mount clinches the diagnosis.
Vaginal ph 5.0 6.0.
Metronidazole is the treatment of choice and should be prescribed to both the patient and the partner.
If alcohol is taken during metronidazole therapy, a disulfiram-like reaction may result in which
acetaldehyde accumulates in the blood stream. This causes flushing, nausea, vomiting and hypotension.

Interstitial cystitis (IC) Painful bladder syndrome


Interstitial cystitis is a chronic condition of uncertain etiology and pathophysiology.
It is characterized by urinary urgency, urinary frequency and chronic pelvic pain in the absence of
another disease that could cause symptoms.
Pain is exacerbated by sexual intercourse, filling of the bladder, exercise, spicy foods and beverages.
Pain is relieved by voiding.
Interstitial cystitis results in urge incontinence.
Cystoscopy demonstrates submucosal petechiae and ulcerations.

OCPs
Oral contraceptive pills are first line agents in the treatment of endometriosis in young women desiring
future fertility.
Serious side effects of OCPs OCPs are protective against
Venous thromoboembolism Ovarian cysts and cancer
Cardiovascular events/myocardial infarction Endometrial cancer
Cerebrovascular events/stroke Benign breast disease
Elevation of triglyceride levels Dysmenorrhea (anemia)
Cholestasis or cholecystitis
Benign hepatic tumors
Diabetes mellitus
Hypertension

Even though OCPs may worsen diabetes they do not precipitate diabetes in a non-diabetic patient.
OCPs are steroid compounds and therefore cause sodium and water retention; they will worsen this
patient's hypertension.
As for breast cancer, the risk does not seem to change with their use.

Weight gain has not been associated with the use of combination OCPs in controlled studies.
Weigh gain has been reported in association with medroxyprogestrone use.

324
Post-coital or emergency contraception has become necessary because it is estimated that around 60% of
pregnancies in the US are unplanned.
Levonorgestrol (progestin-only method Plan B) is the recommended method of emergency
contraception and is given up to 120 hours after unprotected intercourse. Patients should be warned that
pregnancy may still occur.

Atrophic vaginitis
Atrophic vaginitis is treated with estrogen; this latter should be balanced with medroxyprogesterone
(provera) if the uterus is still present.
If the patient is not willing to use oral hormones, Premarin (estrogen) cream twice daily may be used.

MENSTRUAL PHYSIOLOGY
Abdominal pain in a young female in the middle of her cycle with a benign history and clinical
examination is most likely Mittelschmerz.

Early follicular phase immediately follows menstruation. Cervical mucus is thick, scant and acidic
(does not allow penetration by spermatozoa).

In the ovulatory phase, cervical mucus is profuse, clear and thin. It will stretch to approximately 6
cm, its pH is 6.5 (more basic than other phases and allow penetration by spermatozoa) and exhibit
ferning on microscopic slide smear preparation.

The mucus of the post and pre-ovulatory phases, which is scant, opaque and thick.
Evaluation of the cervical mucus is part of infertility workup.
Normally, cervical mucus extends to at least 6 cm when lifted vertically, and its pH is 6.5 or more.

Initial menstrual cycles in pubertal females (first 1 2 years) are anovulatory; and typically irregular
and may be complicated by menorrhagia; this is due to immaturity of the developing hypothalamic-
pituitary-gonadal axis, that does not produce adequate quantities and proportions of the hormones (i.e.
FSH and LH) required to induce ovulation.

Abdominal pain in a young female with regular menstrual periods (not taking OCPs = ovulating), in the
middle of her cycle with a benign history and clinical examination is most likely mittelschmerz
(midcycle pain).

Primary dysmenorrhea usually appears 6 to 12-months after menarche. It is caused by increased levels of
prostaglandins and presents with lower abdominal pain with menstruation. NSAIDs are highly effective
for treatment; oral contraceptive pills inhibit ovulation and are also effective.

325
Systemic and topical corticosteroids can induce an acneiform eruption characterized by monomorphous,
erythematous follicular papules distributed on the face, trunk and extremities. Comedones are absent.
Adolescent acne is characterized by open and closed comedones and inflammatory nodules.

PRECOCIOUS PUBERTY
Etiologies of precocious puberty in females can be subdivided into
(1) Central True isosexual precocious puberty, which is secondary to activation of the hypothalamic-
pituitary-ovarian axis, and
(2) Peripheral Pseudoisosexual precocious puberty, in which the increase in female steroid hormones
and the resultant apparition of secondary sexual characteristics occurs without activation of the
hypothalamic-pituitary axis, mostly from ovarian tumors, but also adrenal tumors, exogenous
estrogen exposure, advanced hypothyroidism and McCune-Albright syndrome.

Central precocious puberty presents with increased FSH and LH levels.


All patients with central precocious puberty should receive brain imaging with CT or MRI.
Treatment with GnRH analogue therapy.

Peripheral precocious puberty presents with decreased FSH and LH levels

In patients who present with precocious pubarche, it is very important to differentiate between precocious
puberty that is caused by premature activation of the hypothalamus-pituitary-gonad (HPG) axis and
precocious pseudo-puberty that is caused by a gonadotropin-independent process, typically an excess
of sex steroids.
GnRH stimulation test serves to differentiate between true isosexual and pseudoisosexual precocious
puberty. In GnRH stimulation test, 100 micrograms are administered as an I.V. bolus: A significant
release of LH indicates an activation of the pituitary gland; therefore, true isosexual precocious puberty.

Idiopathic precocious puberty represents 40% of the cases of precocious puberty in females and results
from the premature activation of the hypothalamic-pituitary-ovarian axis.
Patients with idiopathic precocious puberty should be treated with GnRH agonist therapy in order to
inhibit secretion of estrogens, preventing, thus, premature epiphyseal plate fusion.

Precocious pubarche with signs of severe androgen excess (i.e. severe cystic acne, significant growth
acceleration) is suggestive of precocious pseudo-puberty that is caused by a gonadotropin-independent
process (typically an excess of sex steroids e.g. 21-hydroxylase deficiency). It can be caused by late-
onset congenital adrenal hyperplasia.

Premature adrenarche is characterized by the isolated appearance of axillary hair before the age of six
years. This change results from premature androgen secretion of the adrenal glands. The condition is
generally benign and has no clinical significance. On the other hand, premature pubarche (pubic hair
growth before the age of 8) is more alarming, as it is associated in 50% of cases with a CNS disorder.

326
McCune-Albright syndrome
McCune-Albright syndrome is a rare condition characterized by precocious puberty, pigmentation (cafe
au lait spots) and polyostotic fibrous dysplasia (multiple bone defects).
It may be associated with other endocrine disorders, such as hyperthyroidism, prolactin- or GH-
secreting pituitary adenomas, and adrenal hypercortisolism.
McCune-Albright syndrome is sporadic and has been recently attributed to a defect in the G-protein
cAMP-kinase function in the affected tissue, thereby resulting in autonomous activity of that tissue.

HCG may be produced by hepatomas and result in precocious puberty; however, it occurs almost
exclusively in boys.
Testosterone level increased in female: androgen comes from ovary
DHEAs level increased in female: androgen comes from adrenal.

Dysfunctional uterine bleeding (DUB)


DUB refers to heavy vaginal bleeding that occurs in the absence of structural or organic disease.
After menarche and before menopause it is considered physiologic.
DUB is the most common cause of abnormal uterine bleeding.
Due to its benign nature, it is a diagnosis of exclusion.
The most common cause of dysfunctional uterine bleeding (DUB) in adolescent women is anovulation.
In which patients with DUB do you perform endometrial biopsy to rule out carcinoma? When the
patient is older than 35 years, obese, diabetic or has chronic HTN*****.
DUB is treated with cyclic progestin therapy from day 14 25 of each cycle or by daily combination
OCPs.
Cases not controlled by hormonal therapy may undergo endometrial ablation or hysterectomy.
In cases of uncontrolled bleeding, IV estrogen is the drug of choice for, to suppress the bleeding, and to
ensure cardiovascular stability. Once cardiovascular stability is achieved, D&C should be performed.
Once the bleeding is stopped, advise the patient to take the following: 25 mg of oral conjugated estrogen
daily for 25 days; add 10 mg of medroxyprogesterone for the last 10 to 15 days of treatment; then allow
5-7 days for withdrawal bleeding. This sequence is to mimic the menstrual cycle.

327
PRIMARY AMENORRHEA
First know two important points
1) Absence of breasts indicates lack of estrogen.
2) Presence of uterus indicates normal mullerian development and absence of Y-chromosome.

Examine Breast and Uterus first


1) Both breast and uterus are present
Think of two possibilities:
Outflow tract problem e.g. imperforate hymen, vaginal septum,
Anovulation e.g. pregnancy before the first menses, anorexia nervosa, excessive excrecise

2) No Breast and Uterus present


Always next step is to measure FSH
FSH (primary gonadal problems)
Gonadal dysgenesis/agenesis do Karyotyping

FSH (central or hypothalamic problem)


GnRH deficiency
CNS tumor/infection
Stress
GnRH stimulation test is used to differentiate hypothalamic from pituitary cause.

3) Breast present (BP) and Uterus absent


Presence of Breast indicates normal estrogen
Absence of Uterus indicates either presence of Y-chromosome or no mullerian development
Always next step is to measure serum testosterone and do karyotyping
Mullerian agenesis (Mayer-Rokitansky-Kuster-Hauser syndrome)
46 XX (no uterus)
Normal female phenotype
Normal testosterone

Complete androgen insensitivity


46 XY but Y does not work because of insensitivity of androgen receptors (so, Y is useless).
Normal female phenotype
But Y produces testosterone, so raised testosterone is hallmark.

328
Mayer-Rokitansky-Kuster-Hauser syndrome (MRKH)
Genetically normal female (46, XX)
MRKH is the result of a idiopathic absence of Mullerian duct derivatives: fallopian tubes, uterus,
cervix, and upper vagina; lower vagina originates from urogenital sinus.
Patients have normal secondary sexual characteristics (ovarian function is intact), normal pubic &
axillary hair, amenorrhea, and absent uterus.
Serum testosterone levels are in range typical for females.
These patients should be treated with surgical elongation of the vagina.

Aromatase deficiency
Genetic disorder, in which there is total absence or poor functioning of the enzyme that converts
androgens into estrogens.
In utero, the placenta will not be able to make estrogens, leading to masculinization of the mother that
resolves after delivery.
High levels of gestational androgens result in a virilized XX child with normal internal female
genitalia but ambiguous external genitalia.
Clitoromegaly are often seen due to excessive androgens in utero.
Later in life patients will have delayed puberty, osteoporosis, undetectable circulating estrogens, high
concentration of gonadotropins (i.e. FSH & LH) and polycystic ovaries.

329
Androgen insensitivity syndrome
The combination of primary amenorrhea, bilateral inguinal masses, and normal breast development
(why? Because testes secret some estrogen, unopposed to testosterone) without pubic or axillary hair is
strongly suggestive of androgen insensitivity syndrome.
The internal productive organs do not develop (no uterus) because the testes are still present and secrete
mullerian inhibiting factor.
Serum testosterone levels are in range typical for males.
A blind vaginal pouch and karyotype of 46 XY are other clues.
Lower part of vagina exists, sine it is from urogenital sinus, not from Mullerian tube: give rise to uterus,
tubes,upper part of vagina. Presence of MIF is the cause.
A gonadectomy should be performed to avoid the risk of testicular carcinoma.
Estrogen therapy, can be used at the time of expected puberty if a gonadectomy is performed prepubertal.

Hypogonadotropic hypogonadism
Hypogonadotropic hypogonadism is a decrease in circulating sex hormones due to decreased
concentrations of LH and FSH.
This condition can be caused by strenuous exercise, anorexia nervosa, marijuana use, starvation, stress,
depression, and chronic illness.
Patients presents with amenorrhea and may suffer from infertility.
This condition increases a patients risk of osteoporosis (decreases bone mineral density) and decreased
muscle bulk.

Kallmann's syndrome
Kallmann's syndrome consists of a congenital absence of GnRH secretion (i.e. low FSH and LH)
associated with anosmia and a normal karyotype (46 XX).
Therefore, patients have hypogonadotropic hypogonadism, and present with amenorrhea, anosmia
and absent secondary sexual characteristics, although, internal reproductive organs are present.
Abnormal development of the olfactory bulbs and tracts result in the hyposmia or anosmia (decreased
sense of smell).
Phenotypic appearance includes eunuchoid stature and incomplete sexual development.

Turner syndrome (45, XO)


Turner's syndrome is inherited in a sporadic fashion, and not as an autosomal dominant trait.
Turner syndrome is characterized by primary ovarian failure, amenorrhea, multiple congenital
anomalies, short stature webbing of the neck (due to lymphatic channel abnormalities) and bilateral
streaked ovaries.
Turner syndrome may be associated with renal malformation, pigmented nevi, tendency to keloid
formation, hypoplastic nails, hearing loss of perceptive nature, glucose intolerance and hypertension.

330
Patients with Turner syndrome have ovarian dysgenesis, which results in low estrogen levels and
inability to menstruate.
Inhibin is a marker of ovarian function, and therefore will be low in Turner syndrome.
Testosterone levels are near normal, as testosterone is produced by ovarian stromal cells.
FSH elevation, greater than LH elevation, is classic for primary ovarian failure in Turner syndrome.
Aortic coarctation (elevated blood pressure in the upper extremities and low in the lower extremities) is
a common finding in Turner syndrome.
Patients with Turner's syndrome should be treated with growth hormones, not steroids, when their
height falls below the 5th percentile on the normal growth curve. Therapy is continued until the bone age
is greater than 15 years, and the growth rate falls to less than 2 cm.

Turner's syndrome with 46 XY karyotype is associated with a higher incidence of gonadoblastoma;


hence, prophylactic bilateral gonadectomy is indicated in the management of such patients.

In all patients of childbearing age presenting with amenorrhea, it is of utmost importance to first rule out
pregnancy, by measuring serum beta-hCG levels, before any other step, and this regardless of the
presence or not of any symptoms or signs. If beta-hCG result is negative, the problem is most likely due
to oral contraceptives and can be treated accordingly.

In a patient with primary amenorrhoea and no breasts, estrogen is low and FSH measurement should be
ordered: if it is decreased, GnRH stimulation test is the next step; if it is increased, karyotype is the next
step.
All patients with primary amenorrhea and high FSH levels need to have a karyotype determination.

331
Amenorrhea in lactating mothers
High levels of circulating prolactin inhibitory effect on the production of the hypothalamic hormone
GnRH production and release of LH and FSH by the anterior pituitary suppression of ovulation.
Because lactation suppress ovulation, it is used as a contraceptive method (but not a reliable method).

Premenstrual syndrome (PMS)


Physical symptoms include bloating, fatigue, headache and breast tenderness.
Psychological symptoms may include anxiety, mood swings, difficulty concentrating, decreased libido
and irritability.
Symptoms usually begin one to two weeks prior to menses and disappear near menses and the patient is
symptom free until her next ovulation.
A menstrual diary for at least 3 cycles is a useful aid for confirming the diagnosis in suspected cases;
PMS is confirmed when one or several particular symptoms occur repeatedly at the same time of the
several cycles.
Once the diagnosis is confirmed, treatment depends on the patient's complaints. There is no universally
accepted treatment. Reduction of caffeine intake may reduce breast symptoms. An exercise program
may be effective in improving the general well being of the patient. Recently, selective serotonin
reuptake inhibitors have been shown to be beneficial in double blind controlled trials and is now the
drug of choice for mood decline.

PMS has to be differentiated primarily from major depression because there is an association between
puerperal depression, menopausal depression and PMS.

HORMONAL CONDITIONS
Galactorrhea
Galactorrhea presents as guaiac negative bilateral nipple discharge.
Prolactinoma, hypothyroidism, overstimulation of the nipple, OCPs, and medications which lower
dopamine levels are common causes of galactorrhea.
Prolactin production is inhibited by dopamine and stimulated by serotonin and TRH.
Workup includes; ruling out pregnancy, measuring serum prolactin and TSH levels, and possible
MRI of the brain to rule out prolactinoma.

332
HIRSUTISM

The risk factors for osteoporoses include: Thin body habitus, smoking, alcohol intake, steroid use,
menopause, malnutrition, family history of osteoporosis, and Asian or Caucasian race.

The major source of estrogen in menopausal women is from the peripheral conversion of adrenal
androgens by fat tissue.

Raloxifene is a mixed agonist/antagonist of estrogen receptors. In breast tissue and vaginal tissue, it is an
antagonist, whereas in bone tissue, it is an agonist and may be used to treat osteoporosis. It increases the
risk of thrombo-embolism.

POLYCYSTIC OVARIAN SYNDROME (PCOS)


It is characterized by anovulation, signs of androgen excess and ovarian cysts.
History is positive for amenorrhea or irregular bleeding and infertility.
Exam reveals hirsutism, obesity, male pattern baldness and increased acne.
Pelvic exam reveals bilateral enlarged, smooth, mobile ovaries.
Pathophysiology: abnormal GnRH secretion stimulates the pituitary secrete excessive LH and
insufficient FSH.
Excess LH stimulate ovarian theca cells excess androgen production hirsutism, acne, and
androgenic alopecia.
Anovulation can be associated with amenorrhea or irregular menses occasionally complicated by
menometrrohagia.
Patients with PCOS are at high risk to develop endometrial hyperplasia and endometrial carcinoma
due to unbalanced estrogen secretion (peripheral conversion of androgens to estrogen, while in
progesterone).

Metformin is indicated in polycystic ovarian syndrome patients with impaired glucose tolerance. It helps
in preventing type 2 diabetes mellitus and correcting obesity, hirsutism, menstrual irregularity, and
infertility.

333
POLYCYSTIC OVARIAN DISEASE
Patients with polycystic ovarian disease (PCOD) usually have elevated DHEA levels.
Although ACTH levels are normal in these women, ACTH stimulation test produces an exaggerated
response of DHEA because of increased sensitivity of the adrenal gland to ACTH.
PCOD is characterized by an unbalanced estrogen secretion that may result in endometrial hyperplasia.
Patients are treated with combined oral contraceptive pills or cyclic progestins.
Patients with polycystic ovarian disease are at risk of developing type II diabetes.
The clinical triad of polycystic ovarian disease (PCOD) is amenorrhea, hirsutism and obesity.

INFERTILITY
Semen analysis is a simple test that helps to identify male factor as the cause of infertility. Semen
analysis should be performed early in the evaluation of the infertile couple, usually as the initial
screening test.

The most common cause of cause for decreased fertility in women in their fourth decade who are still
experiencing menstrual cycles is age-related decreased ovarian reserve (Oocyte aging).

The first step in a patient with secondary amenorrhea is to rule out common situations; that is, pregnancy,
then hypothyroidism (TRH increasesprolactin increasesGnRH inhibited) and hyperprolactinemia.
All patients with secondary amenorrhea should receive pregnancy test, prolactin level, and FSH level.
The subsequent step should be the determination of the patients estrogen status with progestin
challenge test.

334
Infertility in women with irregular menstrual cycles is often due to anovulation. Ovulation can be
tested by measuring a midluteal phase progesterone level.

Primary Premature ovarian failure


Primary ovarian failure is defined as primary hypogonadism in a woman under age 40.
Causes: idiopathic, chemotherapy, radiation, autoimmune disorders (e.g. Hashimotos thyroiditis,
Addison disease, type I diabetes), Turners syndrome, and fragile X syndrome.
Symptoms: amenorrhea, hot flushes, vaginal and breast atrophy and anxiety, depression or irritability.
Pathophysiology: impaired follicular development decrease estrogen levels loss of feedback
inhibition of estrogen on FSH and LH elevation of FSH and LH levels.
Primary ovarian failure results in decreased estrogen and increased FSH and LH as a result of loss of
feedback inhibition of estrogen and inhibin on these hormones. FSH elevation, more than LH
elevation, is diagnostic of primary ovarian failure.
FSH elevation in the setting of 3 months of amenorrhea in a women under age 40 confirms the
diagnosis of premature ovarian failure.
Patients with premature ovarian failure lack viable oocytes, so the only option to treat infertility in
premature ovarian failure is egg donation in-vitro fertilization.

The initial tests to assess the ovulatory function are BBT and midluteal progesterone level.

The peritoneal factor is the most common cause of female infertility.

Luteal phase defect


Luteal phase defect is confirmed by endometrial biopsy.
Luteal phase defect is first attempted with progesterone supplements.
Clomiphene citrate or hMG can be tried if progesterone gives no results.
Clomiphene citrate (estrogen analogue) acts by binding to hypothalamic estrogen receptors and
suppressing the inhibitory effect estrogen has on GnRH production.

Major complication of ovulation induction are multiple gestation and OHSS.


OHSS (ovarian hyperstimulation syndrome) is a complication that occurs in 1-3% of patients under
ovulation induction. OHSS may be complicated with ovarian torsion, ovarian rupture, thrombophlebitis
and renal insufficiency.

335
Anabolic steroids
Anabolic steroids (testosterone analogs) are commonly abused by athletes in an effort to improve
performance.
Side effects include: suppressed endogenous testicular function ( suppress the production of GnRH,
LH and FSH, resulting in infertility), azoospermia, decreased testicular size, gynecomastia,
erythrocytosis ( Hematocrit), hepatotoxicity, cardiac disease, dyslipidemia (lowered HDL and
elevated LDL), increased coagulation, acne and premature epiphseal fusion (which shunts growth).
Virilization is often seen in females who abuse steroids.
Behavioral effects include aggressiveness and psychotic symptoms.
Exogenous testosterone analogs feedback inhibition of the hypothalamus GnRH production
LH and FSH production; LH and FSH are trophic on the testes.

MENOPAUSE
Symptoms of menopause include irregular or absent menses, heat intolerance, flushing, insomnia, and
night sweats.
In menopause, the circulating estrogen decreases, resulting in decrease in the feedback inhibition on the
hypothalamic-pituitary axis, resulting in elevation of serum FSH and LH levels.
The major source of estrogen in menopausal women is from the peripheral conversion of adrenal
androgens by the aromatase enzyme present in adipose tissue. This process is increase in obese women
who may have increased menopausal symptoms.
Hyperthyroidism and menopause can have similar presentations, and serum TSH and FSH levels should
be checked in patients with these symptoms.

Menopause results in increased LDL, decreased HDL, osteoporosis, and high risk of heart disease due
to lack of effect of estrogen on cholesterol balance, and also due to altered vascular endothelium
reactivity due to decreased estrogen.
Hypertrophic dystrophy of vulva is most commonly seen in postmenopausal women and is treated
with local application of 1% corticosteroid ointment three times a day for 6 weeks. Hyperplastic
dystrophy cannot be distinguished clinically from cancer of the vulva and biopsy is therefore required
every time the diagnosis is suspected.
Estrogen is used for atrophic vaginitis.

Think of trichotillomania in patients with uncontrollable urges to pull out their hair, resulting in alopecic
patches.

336
Lichen sclerosis
Lichen sclerosis is usually seen in postmenopausal women but may develop at any age.
It present with pruritus, burning and dyspareunia.
Lesions are characterized by a decrease in subepithelial fat with atrophic changes of the vulva and
perineum such as thinning of the labia majora and loss or fusing of labia minora.
In a postmenopausal female who has vulvar itch and dryness, lichen sclerosis must be suspected.
However, vulvar carcinoma in situ must also be in the differential and a biopsy obtained if suspicion is
high.
It is treated with 'superpotent topical corticosteroids' such as clobetasol or halobetasol.
***Itchy spot in a postmenopausal women needs biopsy.

Hormone replacement therapy (HRT)


Hormone replacement therapy (HRT) is now only recommended for the short-term use of controlling
menopausal symptoms.
Large studies have shown that long-term use of combined HRT can slightly increase the risk of coronary
heart disease and stroke.
HRT protects against osteoporosis and to a minor degree against colon cancer.
HRT increase incidence of venous thromboembolism and endometrial hyperplasia/cancer (only with
unopposed estrogen).
Estrogen replacement therapy increases the metabolism of thyroid hormones (induction of liver enzymes)
increases the level of thyroid-binding globulin (TBG), and increased volume of the distribution of thyroid
hormones (salt & water retention); resulting in increases the requirements for L-thyroxine in patients
receiving estrogen replacement therapy.

Osteoporosis
The risk factors for osteoporoses include: thin body habitus, smoking, alcohol intake, steroid use,
menopause, malnutrition, family history of osteoporosis, and Asian or Caucasian race.
Bone demineralization and low bone mass is typical for osteoporosis.
A compression fracture of the vertebrae is a common complication of advanced osteoporosis. It usually
manifests as acute back pain without an obvious preceding trauma in a predisposed patient. Neurologic
examination will be normal.
Normal serum calcium, normal serum phosphate and normal serum parathyroid hormone levels are seen
in patients with osteoporosis as well as normal serum alkaline phosphate level.
Dual energy x-ray absorptiometry (DEXA) is currently the gold standard and most widely used
investigation for detecting osteoporosis and osteopenia.
Biophosphates (e.g. alendronate, reisedronate) inhibit osteoclastic activity.
Raloxifene is a selective estrogen receptor modulator (SERM), has a mixed agonist/antagonist of
estrogen receptors. In breast and vaginal tissue, it is an antagonist, whereas in bone tissue, it is
antagonist, that increases bone mineral density. It is a first-line agent for the prevention of osteoprosis,
and it decreases breast cancer risk, but it also increases thromboembolism risk.

337
BREAST DISEASES
Work-up for a breast mass

The differential diagnosis of a breast mass includes fibrocystic disease, fibroadenoma,


mastitis/abscess, fat necrosis, and breast cancer.

A young woman who presents with a breast lump can be asked to return after her menstrual period for
reexamination (which may reveal regression of the mass) if no obvious signs of malignancy are present.

Fibrocystic Breast Disease


Exaggerated stromal tissue response to hormones and growth factors.
Microscopic findings include cysts (gross and microscopic), papillomatosis, adenosis, fibrosis, and ductal
epithelial hyperplasia.
Primarily affects women 3050 years of age; rarely found in postmenopausal woman.
Associated with trauma and caffeine use.

HISTORY/PE
Presents with painful, often multiple masses in both breasts.
Nipple discharge may also be present.
Rapid fluctuation in the size of the masses is common.
During the premenstrual phase, pain and mass size .

338
DIAGNOSIS
Mammography is of limited use.
Ultrasound can help differentiate a cystic from a solid mass.
Aspiration of a discrete mass that is suggestive of a cyst is indicated to alleviate pain as well as to
confirm the cystic nature of the mass.
Fine-needle aspiration (FNA) cytology or needle biopsy of suspicious lesions may be used.
Perform an excisional biopsy if no fluid is obtained or if the fluid is bloody on aspiration.
There is an risk of breast cancer if ductal epithelial hyperplasia or cellular atypia is present.

TREATMENT
Danazol may be given for severe pain but is rarely used owing to its side effects (acne, hirsutism,
edema).
Simple mastectomy or extensive removal of breast tissue is rarely indicated.

FIBROADENOMA
A benign, slow-growing breast tumor with epithelial and stromal components.
The most common breast lesion in women < 30 years of age.
Cystosarcoma phyllodes is a large fibroadenoma.

HISTORY/PE
Presents as a round or ovoid, rubbery, discrete, relatively mobile, non-tender mass 15 cm in diameter.
Does not occur after menopause unless the patient is on HRT.

DIAGNOSIS
Breast ultrasound can differentiate cystic from solid masses.
Needle biopsy or FNA.
Excision with pathologic exam if the diagnosis remains uncertain.

TREATMENT
Excision is curative, but recurrence is common.

MASTITIS
Cellulitis of the periglandular tissue caused by nipple trauma from breast-feeding coupled with
introduction of S. aureus from the infants nostrils into the nipple ducts.
Symptoms often begin 24 weeks postpartum; are usually unilateral; and include focal breast tenderness,
erythema, edema, warmth, and possible purulent nipple drainage.
Differentiate from simple breast engorgement. Infection is suggested by focal symptoms, a breast milk
culture, WBC count, and fever.
Continued breast-feeding and PO antibiotics (e.g., penicillin, dicloxacillin, erythromycin).
Incision and drainage of breast abscess if present.

339
Breast Cancer
The most common cancer (affects one in nine women) and the second most common cause of cancer
death in women (after lung cancer).
The distribution by quadrant is as follows: 45% occur in the upper outer quadrant, 15% in the upper
inner quadrant, 5% in the lower inner quadrant, 10% in the lower outer quadrant, and 25% in the
subareolar area.
Risk factors include the following:
Female gender, older age.
Breast cancer in a first-degree relative.
BRCA1 and BRCA2 mutations (associated with early onset).
A personal history of breast cancer.
A high-fat and low-fiber diet.
A history of fibrocystic change with cellular atypia.
exposure to estrogen (nulliparity, early menarche, late menopause).
First full-term pregnancy after age 35.
Late menarche is associated with risk.

HISTORY/PE
Early findings: May present as a single, nontender, firm-to-hard mass with ill-defined margins or as
mammographic abnormalities with no palpable mass.
Later findings: Skin or nipple retraction, axillary lymphadenopathy, breast enlargement, redness, edema,
pain, fixation of mass to the skin or chest wall.
Late findings:
Ulceration; supraclavicular lymphadenopathy; edema of the arm; metastases to the bone, lung, and
liver.
Prolonged unilateral scaling erosion of the nipple with or without discharge (may be Pagets disease of
the nipple).

Metastatic disease:
Back or bone pain, jaundice, weight loss.
A firm or hard axillary node > 1 cm.
Axillary nodes that are matted or fixed to the skin (stage III); ipsilateral supraclavicular or
infraclavicular nodes (stage IV).

Breast cancer stages:


Stage I: Tumor size < 2 cm
Stage II: Tumor size 25 cm
Stage III: Axillary node involvement
Stage IV: Distant metastasis

DIAGNOSIS
Diagnostic measures are as follows:

340
Mammography: Look for density with microcalcifications and irregular borders.
Ultrasound: To distinguish a solid mass from a benign cyst.
Tumor markers for recurrent breast cancer: CEA and CA 15-3 or CA 27-29.
Receptor status of tumor: Determine estrogen receptor (ER), progesterone receptor (PR), and
HER2/neu status.
Metastatic disease:
Labs: ESR, alkaline phosphatase (liver and bone metastases), calcium.
CXR: To evaluate for pulmonary metastases.
CT of the chest, abdomen, pelvis, and brain: To look for metastases.
Bone scan: uptake indicates metastases to bone.

TREATMENT
Pharmacologic:
All hormone receptor patients should receive tamoxifen [Tamoxifen decreases the overall
mortality in patients with breast cancer by preventing recurrences, and cancer development in the
opposite breast].
ER-\ patients should receive chemotherapy.
Trastuzumab, a monoclonal antibody that binds to HER2/neu receptors on the cancer cell, is highly
effective in HER2/neu-expressive cancers.

Surgical options include the following:


Partial mastectomy plus axillary dissection followed by radiation therapy.
Modified radical mastectomy (total mastectomy plus axillary dissection).
Contraindications to breast-conserving therapy include large tumor size, subareolar location, multifocal
tumors, fixation to the chest wall, or involvement of the nipple or overlying skin.
Invasive cancer requires axillary dissection.

Stage IV: Treat with radiotherapy and hormonal therapy; mastectomy may be required for local
symptom control.

TNM staging (IIV) is the most reliable indicator of prognosis.


ER- and PR- status is associated with a favorable course.
Cancer localized to the breast has a 7590% cure rate.
With spread to the axilla, the five-year survival is 4050%.
Aneuploidy is associated with a poor prognosis.

COMPLICATIONS
Pleural effusion occurs in 50% of patients with metastatic breast cancer;
Edema of the arm is common.

Local heat, bed rest and NSAIDs are the mainstay of treatment of superficial thrombophlebitis.
Anticoagulants are indicated only when clot extends into the deep vein system.

341
Streptococcal pharyngitis
Clinical features of abrupt onset of high fever, inflamed tonsils and soft palate, purulent exudate, tender
adenopathy) are suggestive of streptococcal pharyngitis.
In such a setting, a positive rapid test for streptococcal antigen is sufficient to warrant treatment with a
single dose of IM Benzathine penicillin G.
When the rapid test for streptococcal antigen is positive, there is no further need to obtain a throat.
Although 10 day course of oral penicillin V is also acceptable for streptococcal pharyngitis therapy, it is
fess preferred because it needs to be given for much longer periods, and is thus more likely to be
associated with medication nonadherence, especially once the patient's symptoms abate.
Erythromycin can be used as second line in penicillin-allergic patients.

Retropharyngeal abscess
Retropharyngeal abscess though more common in children can also occur in adults.
Onset is generally slow following contiguous spread of upper respiratory, oral infections or local
penetrating trauma.
It is caused by mixed flora, aerobes and anaerobes.
Patients often complain of a sore throat, fever and difficulty swallowing (dysphagia), pain with
swallowing (odynophagia), and pain with neck movements, particularly with neck extension.
The common signs are posterior pharyngeal edema, nuchal rigidity, cervical adenopathy, and fever.
Trismus is generally not present.
Upper airway patency should be monitored in patients with retropharyngeal abscess.
CT of the scan and/or lateral radiographs of the neck to fully evaluate the extent of the infection.
Intravenous broad-spectrum antibiotics are indicated in the treatment of this condition, to prevent the
spread of infection into the mediastinum.
Transoral drainage can also be effective.
The most feared complication of retropharyngeal abscess is spread of infection into the mediastinum,
which can lead to acute necrotizing mediastinitis. Mediastinal extension is characterized by its
widening on chest x-ray, chest pain dyspnea, and fever.

Peritonsillar abscess
The space between the capsule of the palatine tonsil and the pharyngeal muscles comprise, loose
connective, tissue, which is vulnerable to abscess formation called peritonsillar abscess.
Peritonsillar abscess or "quinsy" is a suppurative complication of acute tonsillitis.
It is characterized by marked extension into the peritonsillar space.
Streptococcus pyogenes is the most commonly associated organism with peritonsillar abscess.
The typical scenario involves an ill-appearing patient with fever, unilateral sore throat, neck pain,
referred earache, dysphagia, and muffled voice or hot potato voice.
A muffled voice should make one consider a diagnosis other than uncomplicated pharyngitis or tonsillitis
There is excessive salivation due to inability to swallow leading to drooling.

342
The patient can also have trisrnus, inability to open mouth, hot potato speech, and jugulo digastric
lymphadenopathy.
The tonsils are swollen, which pushes the uvula to the contralateral side.
Unilateral prominent cervical lymphadenitis and deviation of the uvula (can be helpful in
distinguishing a peritonsillar abscess from epiglottitis).
Management includes needle drainage (patient should be in the Trendelenburg position), close
monitoring, and intravenous antibiotics.
Failure to obtain pus is an indication for surgical incision and a more formal exploration.
Any delay in treatment increases the risk of spontaneous rupture.

Epiglottitis
Epiglottitis is inflammation and cellulitis of the soft tissue above the vocal cords glottis, which may cause
life' threatening airway obstruction.
Epiglottitis presents with rapid onset and progression of symptoms as in this patient.
It presents with sore throat, dysphagia, drooling, muffled voice, and cough.
The patient assumes a tripod position, sitting up on hands with the tongue protruding out and head
leaning forward.
Examination reveals cervical lymphadenopathy.
Stridor (harsh shrill noise with respiration.), laryngeal tenderness, and respiratory distress are the
commonly associated signs and symptoms.
Some common causes of epiglottitis are as follows:
H. Influenza type B is the most common cause in children and adults.
Streptococci group A is the second most common cause in adults.
Streptococcus pneumoniae, H. Parainfluenza, K. Pneumoniae, Candida albicans, Staphylococcus
aureus, N. Meningitidis and Varicella zoster.
Incidence is particularly higher in countries where they do not vaccinate against H. Influenza.
Indirect laryngoscopy is considered to be the best for diagnosis.
Immediate intubation maybe required in patients with severe airway obstruction.
Antibiotics, antipyretics, racemic epinephrine, and steroids are the drugs of choice.
Regardless oft cause, all incidences of epiglottitis are considered serious medical emergencies.

343
Leukoplakia
Leukoplakia is a whitish patch or plaque that cannot be clinically or pathologically characterized as any
other disease, and is not associated with any physical or chemical causative agent, except the use of
tobacco.
These are localized, plaque-like lesions that have a granular appearance and are hard to remove.
Leukoplakia is usually caused by chronic irritation to the oral mucosa due to smoking, alcohol, or ill-
fitting dentures.
Vitamin A and B deficiencies and syphilis are also known to cause leukoplakia.
These lesions have a risk of transformation into squamous cell carcinoma.
Hence, an incisional biopsy or exfoliative cytologic examination should always be done.

Oral candidiasis appears as curd like patches over erythematous base. These lesions can be easily
removed by rubbing off. This condition is common in denture wearers, diabetics, and immunosupressed
individuals.

Hairy leukoplakia is a white, painless lesion that appears "hairy" and is often found in AIDS patient o
the lateral aspect of tongue. It is caused by Epstein Barr Virus (EBV).

Nasal polyps
Nasal polyps are the most common nasal tumors. It can be caused by an allergic or infectious process.
A history of allergic disorders (e.g., allergic rhinitis, asthma), or aspirin- or NSAID-induced
bronchospasm in a condition known as aspirin-exacerbated respiratory disease is often present.
Sinonasal polyps may be associated with hum papilloma virus (HPV) infection.
Septal papillomas are invariably a squamous papilloma with no malignant potential; however, most
lateral papillomas are inverted papillomas with malignant potential.
The typical symptoms are recurrent episodes of rhinitis, chronic nasal obstruction, altered taste
sensation, and diminished sense of smell.
There is also a persistent postnasal drip.
Symptomatic treatment consists of steroids and decongestants.
Steroids are also indicated for recalcitrant nasal polyps.
If medical management fails, surgical removal of the polyp is indicated.
Aspirin is contraindicated in patients with asthma and nasal polyps because aspirin use may lead to
severe bronchospasm (aspirin triad or Samter's triad) in such patients.

Deviated nasal septum also presents with symptoms of nasal obstruction, rhinitis, or epistaxis;
however, close inspection and systematic facial analysis will reveal a crooked, twisted nose.

344
Rhinitis
Rhinitis is defined as nasal hyperfunction and tissue inflammation that leads to nasal congestion,
rhinorrhea, pruritus, nasal obstruction, and sneezing.
It is divided into allergic and non-allergic types.

Allergic rhinitis
Allergic rhinitis is an IgE-mediated reaction, and usually involves a variety of allergens.
The typical allergic triggers are: inhalants, foods, and chemicals; these can cause sneezing, nasal
congestion, rhinorrhea, cough, itching of the nose and eyes, headache, and epistaxis.
Patients usually have dark, puffy eyelids, which are called "allergic shiners."
Patients may also have a red crease over the nose; this results due to constant rubbing of the nose,
which is called "the allergic salute."
The nasal discharge is clear, and the nasal mucosa is pale or bluish.
Management includes avoidance of allergens and use of medications such as decongestants and
histamine blockers.
Poorly controlled symptoms can lead to a surprising amount of disability, significant loss of productivity,
and decreases quality of life.

Viral rhinitis
Viral rhinitis presents with rhinorrhea, sneezing, and nasal congestion.
These patients also complain of general malaise.
Nasal examination shows an erythernatous mucosa, in contrast to the pale mucosa in allergic rhinitis.

Otitis externa
Otitis externa is usually an acute bacterial infection of the ear canal, although it can also be caused by
fungal infections.
It affects all age groups. Its risk factors include swimming, any source of water trapped in the ear can;
trauma to the ear canal, and a hot humid environment.
Pain on traction of the pinna is the hallmark of otitis externa.
Otitis externa is always painful and tender. When a patient has pain without tenderness, this is
suggestive of a middle rather than an external ear infection.
Examination reveals edema, erythema, and purulent discharge.
The tympanic membrane appears normal, although it may be hard to visualize due to edema
The treatment includes analgesics and antibiotic eardrops with or without steroids.

345
Malignant otitis externa
Malignant otitis is a serious infection of the external ear, which extends into the deeper tissues adjacent to
the external auditory canal.
It is usually caused by Gram-negative rods (Pseudomonas aeruginosa).
It is usually seen in elderly patients with poorly-controlled diabetes, malnutrition, leukemia,
immunocompromised patients
It presents with foul smelling ear discharge (greenish), deep otalgia, granulations in the ear canal.
Progression of the infection can lead to osteomyelitis of the skull base and destruction of the facial
nerve and involvement of cranial nerves VII, IX-XII.
CT scan or MRI should be done in all case of suspected malignant otitis externa.
Topical neomycin and polymixin +/ hydrocortisone are sufficient for uncomplicated otitis externa.
Treatment includes in-hospital administration of parenteral anti-pseudomonal antibiotics (IV
ciprofloxacin is the most effective treatment).
Possible surgical debridement of devitalized tissue.
Serial measurements of the ESR can be used for the follow-up of this condition.
Persistent infection may cause osteomyelitis of the skull base.

Acute otitis media (AOM)


AOM is an inflammation of the middle ear with signs and symptoms of acute illness, such as fever.
It is more common in children than adults.
AOM also occurs in patients with AIDS due to immunosuppression; however, it usually presents as
earache, fever, and decreased hearing.
Some of its common causes are as follows:
Bacteria:
S. Pneumoniae: 30-50% of cases.
H. Influenza: 20-30% of cases, common in older children, adolescent, and adults.
M. Catarrhalis: 2-15% of cases.
Group B Streptococci: 20% in neonates and young infants.
Staphylococcus aureus
Viruses: Rhinovirus, Influenza virus, Adenovirus, Entero virus, Parainfluenza virus.
AOM usually follows an upper respiratory tract infection, which is evidenced by a history of nasal
discharge, boggy nasal mucosa, and postnasal drip.
Patients usually present with ear discharge, and decreased hearing.
Earache is also common, but my be absent in young patients.
Non-specific symptoms including fever, irritability and diarrhea can sometimes be present as well.
Infants are usually asymptomatic, aside from extreme irritability.
Inflammation of the tympanic membrane and diminished movement of the membrane with air
insufflation are hallmark signs.
Antibiotics sensitive to S. pneumoniae, H. influenza, and Moraxella sp. should be administered to
children (e.g. Amoxicillin). Antipyretics and analgesics should also be prescribed.

346
Serous otitis media
Serous otitis media is the most common middle ear pathology in patients with acquired
immunodeficiency syndrome.
It is due to the auditory tube dysfunction arising from HIV lymphadenopathy or obstructing lymphoma.
It is characterized by a hearing difficulty (conductive hearing loss), dull tympanic membrane which is
hypomotile on pneumatic otoscopy, and air fluid levels in the middle ear (non-infectious effusion).
Ventilating tubes are seldom helpful.

Chronic otitis media


Chronic otitis media usually presents as a purulent aural discharge. The tympanic membrane appe<
thickened with calcific patches and perforation.

Mastoiditis
Mastoiditis is the most common complication of otitis media; however, effective use of antibiotics has
substantially decreased the incidence of this condition.
Mastoiditis has been classified into 2 types, classic and latent. Classic mastoiditis refers to an acute
disease following AOM. Latent or masked mastoiditis refers to a more chronic disease, which can be
subclinical, often secondary to partial treatment of AOM with antibiotics.
The risk factors of mastoiditis are recurrent AOM, cholesteatoma, and immunocompromised host.
The common causative organisms are Staphylococcus sp., Streptococcus pneumoniae, H. influenza,
and Moraxella sp.
Patients present with otalgia, pain behind the ear, and fever.
Examination reveals erythema, edema, tenderness over the mastoid area and the auricle is protruded.
CT imaging may be used to confirm the clinical diagnosis; it demonstrates fluid-filled middle ear and
demineralization of the mastoid trabeculae.
Intravenous antibiotics are the immediate treatment of choice.

Cholesteatoma
In spite of having the suffix "-oma", a cholesteatoma is not a tumor.
It is an epithelial cyst that contains desquamated keratin, and is generally located medial to the
normal position of the tympanic membrane.
It can be congenital or more likely acquired, secondary to otitis media or Eustachian tube
dysfunction.
Infection of keratin debris is usually due to P. aeruginosa.
Patients generally present with repeated infections or progressive conductive hearing loss.
The most common signs are drainage and granulation tissue that are unresponsive to antibiotic
treatment.

347
Cholesteatomas is characterized by intact or marginal tympanic membrane perforation, but it may
sometimes present as an ear canal filled with mucous, pus and granulation tissue.
Cholesteatomas destroy bones; therefore, any bony structure in or around the middle ear and mastoid
cavity can be eroded and subsequently infected. There can even be erosion into the inner ear, as well as
intracranial spread. CT scan can detect bony defects.
Therapy consists of surgical removal.

Temporomandibular joint (TMJ) dysfunction


Always think of referred otalgia when a patient presents with ear pain, but has no significant or prior
history of ear problems.
TMJ dysfunction is a common cause of referred otalgia, and pain aggravated by chewing, along with
psychogenic grinding (bruxism), further supports the diagnosis.
Suspect TMJ syndrome in a patient with unilateral and chronic pain of the muscles of mastication.

Ramsay Hunt syndrome is a facial nerve palsy that is caused by herpes zoster. Patients present with
lesions of the external ear with or without tympanic membrane involvement, vertigo, tinnitus, and
deafness due to geniculate ganglion involvement.

Glossopharyngeal neuralgia is characterized by a sharp, lancinating pain associated with swallowing,


chewing, talking and yawning. This condition has also been associated with multiple sclerosis.

Otosclerosis
Otosclerosis is an autosomal dominant condition characterized by an osseous dyscrasia limited to the
temporal bone leads to hypertrophy and fixation of stapes footplate.
It presents as conductive hearing loss by the third decade of life.
Hearing loss and tinnitus are the usual presenting symptoms.
Tuning fork examination reveals bone conduction greater than air conduction (conductive hearing loss).
Audiometry, also shows conductive hearing loss and loss the stapedial reflex.
Treatment includes oral sodium fluoride, which has a variable response, and is used sporadically for
labyrinthine otosclerosis. It has also been used for postoperative medical management of obliterative
otosclerosis. Hearing loss can be managed using amplification.
Surgical treatment is usually required.

348
Presbycusis
Presbycusis is a high-frequency sensorineural hearing loss occurring in older individuals.
It is a significant geriatric problem.
There are no associated symptoms such as tinnitus.
Patients may give a history of excessive noise exposure.
This can be explained by the underlying pathology, which is a high-frequency hearing loss in both ears
that results in difficult in speech discrimination.
Patients often complain of difficulty of hearing in crowded or noisy environments and trouble hearing
high-pitched voice or noises.
Examination of the ear reveals no abnormalities.
Amplification devices are used to be with the patient's hearing, and some patients may benefit from
cochlear implants.
Presbycusis is a diagnosis of exclusion; therefore, all other possible etiologies of hearing loss in elderly
individuals should be considered before giving this diagnosis.

Dizziness
Dizziness can be classified as vertigo, presyncope or disequilibium.
Severe spinning sensation accompanied by nausea is most characteristic of vertigo.
Next step is to classify vertigo as either central or peripheral.
Peripheral vertigo tends to last for shorter intervals than central vertigo.
Symptoms of ear-fullness suggests a peripheral vertigo.
Meniere's disease is the likely cause of vertigo if the patient also has a sensation of ear fullness.

Meniere's disease
Meniere's disease is a disorder of unclear etiology resulting from distension of the endolyphatmic
compartment of the inner ear. It may be unilateral or bilateral.
Meniere's presents with recurrent episodes of rotational vertigo (last for 20 minutes 24 hours), low-
frequency, sensorineural hearing loss, tinnitus, and aural fullness.
It may be accompanied by vomiting and postural instability.
Nystagmus may be seen during an acute attack.
It is more common in females and occurs in patients with syphilis, or following head trauma.
The diagnosis is made clinically.
It may resolve spontaneously and treatment today is empirical.
First line therapy consists of environmental and dietary modifications, including maintenance of a low-
salt diet (2 3 grams of sodium/day).
Triggers that increase the endolymphatic retention should be avoided, including alcohol, caffeine,
nicotine, and foods high in salt.
Symptoms other than hearing loss improve in the majority of patients without treatment.

349
Benign positional vertigo may be hard to distinguish from Meniere's disease. Every effort must be
made to reproduce vertigo with positional changes to exclude Menieres disease. BPV is not usually
associated with hearing loss or tinnitus.

Acoustic neuroma may produce vertigo and tinnitus. However, the symptoms will be progressive an
unremitting. In addition, there may be other neurologic side effects and a MRI scan may be required to
detect an acoustic neuroma.

Eustachian tube dysfunction


Eustachian tube dysfunction is a common cause of conductive hearing loss in children.
Patients generally present with aural fullness, an audible pop while swallowing or yawning,
intermittent ear pain, and hearing loss.
Such symptoms usually follow an upper respiratory tract infection or episode of allergic rhinitis.
Otoscopy reveals retraction and decreased mobility of the tympanic membrane.
The hallmark of eustachian tube dysfunction is a middle ear effusion.
Upper respiratory tract infection caused by rhinovirus, respiratory syncytial virus, influenza virus, and
adenovirus has been associated with auditory tube dysfunction.
It can be treated with auto-insufflation and oral & nasal decongestants.

Medication-induced ototoxicity
Furosemide
Furosemide is a loop diuretic that can cause ototoxicity (hearing loss and/or tinnitus).
The occurrence of loop diuretic ototoxicity depends on several factors, including dose, infusion rate,
history of renal failure, and coadministration of ototoxic agents such as aminoglycosides (gentamicin).
Patients usually complain of a hearing difficulty soon after taking the drug.
Few patients may present with tinnitus and dysequilibrium.
Among loop diuretics, ototoxicity caused by ethacrynic acid seems to develop more gradual and takes
longer to resolve than that caused by furosemide or burnetanide:
Discontinuation of the drug reverses the hearing impairment, although few patients may require hearing
aids to facilitate hearing.
Overall, ototoxicity is usually self-limited and reversible in adult patients, but irreversible hearing loss
has been reported in neonates.
Prevention of ototoxicity caused by loop diuretics consists of using the lowest doses possible to achieve
desirable effects and avoiding rapid infusion rates.
In addition, the risk factors associated with administration of these drugs must be diligently assessed,
including coadministration of other ototoxic medications and history of renal failure.

Aspirin usually causes tinnitus; but in very higher doses it can cause (6 to 8 grams/day) hearing loss

350
Tinnitus
Tinnitus or ringing of the ears can occur in various situations.
It can sometimes occur in patients taking aspirin and quinine.
For decades, quinine has been used for treating cramps in dialysis patients. Interestingly, only one
double-blind placebo controlled trial of nine hemodialysis patients has shown its beneficial affect.
Tinnitus can also occur in patients who are depressed.
It may be a feature of Meniere's disease and acoustic neuroma.
It can cause disruption of sleep, concentration and depression.

351
Gonococcal conjunctivitis
Gonococcal conjunctivitis is acquired through contact with infected genital secretions.
It occurs 2-5 days after birth.
It presents as copious purulent eye discharge with swollen eyelids and chemosis.
Gram stain of the conjunctival exudates should be done to look for intracellular gram-negative
diplococci and diagnosis can be confirmed by culture of the discharge on Thayer-Martin media.
Immediate treatment is required to prevent corneal perforation.
A single intramuscular dose of ceftriaxone is sufficient for an uncomplicated infection.

Neonatal chlamydial conjunctivitis


Chlamydial trachomatis is transmitted from the genital tract of infected mothers.
It occurs later than gonococcal conjunctivitis, about 5-15 days afterbirth.
Neonatal chlamydial conjunctivitis presents with mild hyperemia and scant mucoid eye discharge that
is not as purulent as gonococcal conjunctivitis.

Chemical conjunctivitis
Chemical conjunctivitis is the most common cause of conjunctivitis.
It occurs within the first 24 hours after birth.
A history of silver nitrate eye drops instillation is usually present.

Cavernous sinus thrombosis


Cavernous sinus thrombosis is the most common, late complication of infection of: the central face
(around the eyes and nose) or paranasal sinuses.
Headache, malaise, and fever precede the development of ocular symptoms such as swelling and pain.
Both eyes are involved, and the characteristic chronology of symptoms (i.e., with one eye developing
symptoms before the other) is pathognomonic.
In addition, there may be features suggestive of dysfunction of cranial nerve (CN) III, IV, V, and VI,
since these pass through the cavernous sinus.
Ptosis, mydriasis (CN III dysfunction)
Absent corneal reflex (CN V dysfunction).
Headache (neuropathic pain as a result of irritation of V1 and V2 branches of Trigeminal nerve)
Lateral gaze palsy and double vision (Abducens nerve CN VI dysfunction)
High resolution CT is the definitive diagnostic procedure.
The mainstay of treatment is early and aggressive antibiotic administration.
Anticoagulants and corticosteroids may be used as adjunctive therapy.

352
Orbital cellulitis
Orbital cellulitis is an infection within the orbit, located posterior to the orbital septum.
Infection of the paranasal sinuses is the usual underlying cause.
Causative agents are Staphylococcus aureus, Streptococcus pneumoniae, and Haemophilius influenza.
It is unilateral and more common in children.
The typical presentation includes the abrupt onset of fever, painful eye, proptosis, restriction of
extraocular movements and swollen, red eyelids.
There is no cranial nerve dysfunction or visual disturbances in the early stages, unless the infection
spreads to involve the cavernous sinus.
Vision may be normal during the early stages, but spread to the cavernous sinus may lead to a decrease in
visual acuity.
Gram stain and culture of the blood and soft tissue aspirates may help identify the pathogen.
CT scan of the orbit helps delineate the extent of infection and to identify a potential abscess.
Immediate administration of intravenous broad-spectrum antibiotics is necessary.

It is important to differentiate orbital cellulitis (infection involving the fat and muscles within the orbit)
from preseptal cellulitis (infection involving the superficial tissues).

Hypertensive retinopathy
Systemic hypertension affects the retinal circulation as well as the choroidal circulation.
The sequence of events begins with focal spasm of arterioles, followed by progressive sclerosis and
narrowing.
If hypertension is left uncontrolled, this eventually leads to flame-shaped hemorrhages from ruptured
vessels, formed exudates and papilledema.
Keith-Wagener classification:
Grade I Slight AV nicking
Grade II Copper wiring, AV depression with humping ends
Grade III Silver wiring, flame shaped hemorrhages, exudates
Grade IV Flame shaped hemorrhages, exudates.and papilledema

Diabetic retinopathy
Diabetic retinopathy is the leading cause of blindness in the USA.
It occurs in both insulin dependent and non-insulin dependent diabetes mellitus.
There are 3 main categories:
1) Background or simple retinopathy consists of microaneurysms, hemorrhages, exudates, and retinal
edema
2) Pre-proliferative retinopathy with cotton wool spots
3) proliferative or malignant retinopathy consists of newly formed vessels.
Patients are usually asymptomatic at first, despite early signs of retinopathy (e.g., microaneurysms).
Visual impairment occurs with the development of macular edema.
Argon laser photocoagulation is the suggested treatment for the prevention of complications.

353
Central retinal vein occlusion (CRVO)
CRVO presents as a sudden, painless, unilateral loss of vision.
It occurs in patient with a history of hypertension, clotting disorders, diabetes mellitus, glaucoma,
cardiovascular diseases, head trauma, or uveitis.
Pupils, cornea, and conjunctivae are normal in the earlier stages.
Ophthalmoscopy reveals disc swelling, venous dilation and tortuosity, retinal hemorrhages and cotton
wool spots.
CRVO requires emergent ophthalmologist referral.

Central retinal artery occlusion is also characterized by sudden painless loss of vision in one eye;
however, funduscopic examination reveals pallor of the optic disc, cherry red fovea and boxcar
segmentation of blood in the retinal veins.

Presbyopia is a common age-related disorder. It is due to decrease in lens elasticity leads to difficulty
in near vision. Patients usually note their inability to focus objects at a normal reading distance, and
complain of having to hold books at arms length to read. It is corrected with convex lenses.
Amblyopia refers to reduced vision that develops in early childhood. Strabismus is the most common
cause of amblyopia, and results in loss of vision if not corrected.
Hyperopia is also known as farsightedness, which occurs when the light rays fall behind the retina. It
usually seen in children with a family history. Objects that are closer to the eye are not seen clearly,
unless accommodation is used. Other common complaints are eye fatigue, headache, and squinting while
reading. In children and adolescents, cycloplegic drops should be administered before eye examination to
eliminate accommodation and to correctly determine the refraction error.

354
USMLE STEP 2 CK HIGH YIELD NOTES (EPIDEMIOLOGY)

Incidence
new cases in population over a given time period
Incidence = /total population at risk during that time
Incidence is new incidents.
When calculating incidence, dont forget that people previously positive for a disease are no longer
considered at risk.

Prevalence
total cases in population at a given time
Prevalence = /total population at risk
Prevalence incidence disease duration.
Prevalence > incidence for chronic diseases (e.g., diabetes).
Prevalence = incidence for acute disease (e.g., common cold).
The classic question about incidence and prevalence: when a disease can be treated and people can be
kept alive longer but the disease cannot he cured, what happens to the incidence and prevalence?
Answer: nothing happens to incidence, hut prevalence will increase as people live longer.
An epidemic occurs when the observed incidence greatly exceeds the expected incidence.

Disease
\
a b
Test

\ c d

Sensitivity SNOUT = SeNsitivity rules OUT


Ability to detect disease. Percent of people with disease who test positive.
Tests with high sensitivity are used for screening.
High value is desirable for ruling out disease. They may have false positives but do not miss many
people with the disease (low false-negative rate).
Mathematically, sensitivity is calculated by dividing the number of true positives by the number of
people with the disease [a / (a + c) = 1 percent false-negatives].

355
USMLE STEP 2 CK HIGH YIELD NOTES (EPIDEMIOLOGY)

Specificity SPIN = SPecificity rules IN


Ability to detect health (or non-disease). Percent of people without disease who test negative.
Mathematically, specificity is calculated by dividing the number of true negatives by the number of
people without the disease [d / ( d + b ) = 1 percent false-positives].
Tests with high specificity arc used for disease confirmation.
High value is desirable for ruling in disease. They may have false negatives but do not call anyone sick
who is actually healthy (low false-positive rate).

Sensitivity and specificity of a test do not depend on the prevalence of the disease in the population.

Generally, the more sensitive the test is, the less specific and vice versa.
A perfectly valid diagnostic test should have sensitivity and specificity equal to 1, but this is seldom
possible.
The ideal confirmatory test must have high sensitivity and high specificity; otherwise, people with the
disease may he called healthy.

The trade-off between sensitivity and specificity is a classic statistics question.


Understand how the cut-off glucose value in screening for diabetes (or changing the value of any of
several screening test) will change the number of true and false negatives and true and false positives.
If the cut-off value is raised, fewer people will be called diabetic (more false negatives, fewer false
positives).
If the cut-off value is lowered, more people will be called diabetic (fewer false negatives, more false
positives)

An arbitrary cutpoint is chosen to differentiate normal from abnormal results.


Choosing the appropriate cutpoint value is important; however, overlapping normal and abnormal results
make this choice difficult.

356
USMLE STEP 2 CK HIGH YIELD NOTES (EPIDEMIOLOGY)

Any cutpoint demonstrates a trade-off between sensitivity (the proportion of true positive results) and 1
specificity (the proportion of false positive results).
The curve presented is called the receiver-operating characteristic (ROC) curve.
Increasing the cutpoint marker value increases the specificity of the test (lower false-positives) and
ruling in possibility P1 on the curve, but decreases the sensitivity of the test.
On the other hand, decreasing the cutpoint marker value increases the sensitivity of the test and ruling
out possibility, but decrease the specificity P3 on the curve.
P3 is a point of high sensitivity, but relatively low specificity; increasing sensitivity increases negative
predictive value, or 'ruling out' possibility.
Therefore, it is reasonable to assume that P3 cutpoint corresponds a lower serum marker value than P1
cutpoint.

We use the "CAGE" questionnaire in clinical practice to screen patients for alcoholism. Usually if we get
2 out of 4 positive responses to the "CAGE" questions, the chance of a patient being alcoholic is 100%. If
we change this criteria so that 3 out of 4 positive responses to the "CAGE" questions label the patient as
alcoholic, what is the affect on the sensitivity and specificity of this test?
As the number of questions is increased from 2 to 3 for a patient to be labeled as an alcoholic, we will
increase specificity and decrease the sensitivity because there will be fewer people who will answer "yes"
(decrease false positives and true positives). Those answering "yes" will now have a greater chance of
having alcoholism (increase false negatives and true negatives). So, when these values are put in the
formula for calculating sensitivity and specificity we will find that sensitivity is decreased and specificity
is increased.

Positive predictive value (PPV)


a
PPV =
Percent of positive test results that are true-positive. (a + b)
When a test comes back positive for disease, PPV measures how likely it is that the patient has the
disease (probability of having a condition, given a positive test).
PPV is calculated by dividing the number of true positives by the number of people with a positive test.
PPV depends on the prevalence of a disease (the higher the prevalence, the greater the PPV), it makes
intuitive sense, because the more common the disease is in the population, the more likely patient with a
positive test is actually diseased (that is, the test is true-positive).
PPV depends on sensitivity/specificity of the test (e.g. an overly sensitive test that gives more false
positives has a lower PPV).

357
USMLE STEP 2 CK HIGH YIELD NOTES (EPIDEMIOLOGY)

Negative predictive value (NPV)


d
NPV =
Percent of negative test results that are true-negative. (c + d)
When a test comes back negative for disease, NPV measures how likely it is that the patient is healthy
and does not have the disease (probability of not having a condition, given a negative test).
NPV is the probability of being free of a disease if the test returns negative.
NPV depends on the pretest probability of the disease.
NPV is high if the pre-test probability of the disease is low.
NPV is low if the pre-test probability of the disease is high.
NPV is calculated by dividing the number of true negatives by the number of people with a negative test.
NPV depends on the prevalence of a disease (the higher the prevalence, the lower the NPV) because
the probability of true negative result for a patient who tested positive is high in a population with low
prevalence of the disease.
NPV depends on sensitivity/specificity of the test (e.g. an overly sensitive test with lots of false
positives will make the NPV higher).

As the cut-off point moves from X to A, the TP will increase, and the FN will decrease at the same time.
However, the denominator will be unchanged because as TP is increased, FN will be decreased the
same time. So the result will be an increase in sensitivity, as sensitivity = TP/TP+FN. So, the end result
will be decrease in PPV (as PPV = TP/TP+FP)

Moving the cut-off point from X to B how that effects sensitivity. The result will be an increase in TN
and decrease in FP. However, the denominator will be unchanged because so the result will be an
increase in specificity (sensitivity = TN/TN+FP).

358
USMLE STEP 2 CK HIGH YIELD NOTES (EPIDEMIOLOGY)

Probability
Pre-test probability of the disease should be estimated according to available clinical information.
If the patient belongs to the low-risk groups, that means that the pre-test probability of a disease is low.
Post-test probability can be calculated using the information on sensitivity and specificity of the test and
the pre-test probability of the disease.
If a test returns negative, the probability of the disease is 1 - negative predictive value.
Positive predictive value shows the probability of the disease if the test is positive.
1 - specificity shows false positive ratio and 1 - sensitivity shows false negative ratio.

Odds ratio (OR) a/b ad


Odds Ratio = =
Used only for: c/d bc

Retrospective studies (e.g. case control), OR


Compares disease in exposed and non-disease in unexposed populations with disease in unexposed and
non-disease in exposed populations to determine whether there is a difference between the two. Of
course, there should be more disease in exposed than unexposed populations and more non-disease in
unexposed than exposed population, OR
It is a less than perfect way to estimate relative risk (RR), if prevalence of disease is not too high.

Odds ratio is considered clinically significant if it is greater than or less than 1.


OR > 1 means that the factor under study is a risk factor for the outcomes.
OR < 1 means that the factor under study is a protective factor in respect to the outcome.
OR = 1 means there is no significant difference in outcomes in either the exposed or the unexposed.
For example, OR of 2.4 for colon cancer in patients who eat high fat diet means that the high fat diet
increases 2.4 times the likelihood of having colon cancer, and thus is a risk factor for colon. An OR of
0.05 for colon cancer in patients who eat green leafy vegetables means that patients eating green leaf
vegetables have 0.05 times chance of having colon cancer as compared to those who do not eat green
leafy vegetables. In other words, eating green leafy vegetables decreases the risk of colon cancer by 95
%. Thus, eating green leafy vegetable is a protective factor for colon cancer.
If the outcome of a case-control study is not common in the population, the odds ratio is close to the
relative risk.

359
USMLE STEP 2 CK HIGH YIELD NOTES (EPIDEMIOLOGY)

Relative risk (RR) RR =


a/(a+b)
c/(c+d)
Prospective or experimental studies (e.g. cohort).
Relative risk(RR) is the ratio of the risk in an exposed group to the unexposed group.
Calculated as percent with disease in exposed group divided by percent with disease in unexposed group
RR = 1 means that there is no exposure-outcome association.
RR values between 0 and 1 means a negative association (exposure has a protective effect against the
outcome).
RR greater than 1 is clinically significant.
RR > 1 means that there is positive association between the risk factor and outcomes.
Farther the value of RR from 1, stronger is the association (e.g. 2.0 is stronger than 1.5)
RR < 1 means that there is negative association between the risk factor and outcome.

Relative risk is a point estimate of association, but it does not account for random error.
You can realize intuitive that there is a certain probability that any value of relative risk (even if it is
extreme) can arise by chance.
P value helps us understand what the probability is of obtaining the result by chance alone.

a c
Attributable risk (AR) AR = -
(a+b) (c+d)
Number of cases attributable to one risk factor; in other words, the amount by which you can expect
the incidence to decrease if a risk factor is removed.
ARP represents the excess risk in exposed population that is explained by the risk factor.

ARP = (risk in exposed - risk in unexposed)/risk in exposed.


The easier way to obtain ARP is to derive it from relative risk (RR): ARP = (RR-1)/RR.

For example, if the incidence rate of lung cancer in the general population is 1/100 and in smokers it is
10/100, the attribute risk of smoking in causing lung cancer is 9/100 (assuming a properly matched
control).

Retrospective studies (e.g. case control), OR


Compares disease in exposed and non-disease in unexposed populations with disease in unexposed and
non-disease in exposed populations to determine whether there is a difference between the two. Of
course, there should be more disease in exposed than unexposed populations and more non-disease in
unexposed than exposed population, OR
It is a less than perfect way to estimate relative risk (RR), if prevalence of disease is not too high.

360
USMLE STEP 2 CK HIGH YIELD NOTES (EPIDEMIOLOGY)

Risk
Risk is a measure of incidence of a disease that shows the probability of getting the disease over a certain
period of time
To calculate the risk in a group, we divide the number of diseased subjects by the total number of
people in the group.

Precision
Reliability, or precision, is the reproducibility of results.
Precision is the measure of random error in the study.
Inter-rater reliability measures the similarity of results when tests are interpreted by different people.
Random errorreduced precision in a test.
A reliable test gives similar or very close results on repeat measurements. Reliability is maximal when
the random error is minimal.
The tighter the confidence interval, the more precise is the result.
Increasing the sample size, increases the precision.

Accuracy
Accuracy is the trueness of test measurements (validity).
Accuracy and validity represent the measure of systematic error (bias).
Validity, or accuracy, measures how well a test measures what it intends to measure.
In order to determine the validity of the test, we have to compare the results given by this test to the
truth or so-called gold standard, which is considered the most valid and reliable test available
(although typically not the most convenient or safe).
Accuracy is reduced if the result does not reflect the true value of the parameter measured.
Test-retest probability (intra-observer variability) assesses the similarity of results when a single person
interprets a test repeatedly.

361
USMLE STEP 2 CK HIGH YIELD NOTES (EPIDEMIOLOGY)

Generalizability
Generalizability, or external validity, of a study means the applicability of its results beyond the cohort
where in it was studied.
External validity answers the question, how generalizable are the results of a study to other
populations?
Do not expect that the results of a study conducted in middle-aged women maybe applicable to elderly
men.

Number needed to treat (NNT)


The number of people in the general population who must be treated to prevent disease in one patient.
Inverse of the absolute risk reduction associated with an intervention.
NNT = 1 (rate in untreated group rate in treated group).

Median
It is important to know the difference between the measures of the center of a dataset.
The median of a dataset the number that divides the right 50% of data from the left 50% (it is in the
middle of the dataset).
If the number of observations is even, finding the median becomes tricky, you should find the middle two
values, add them together, and divide by two.

Mean
To find the mean of a dataset, you should add all the observations and divide that sum by the number of
observations.

Mode
Another measure of the center of a dataset is the mode. Finding the mode is the easiest.
The mode is the most frequent value of a dataset.
In the scenario described, the mode is 22 mmHg (Choice D).

Statistical distribution
Normal Gaussian bell-shaped (mean = median = mode).
Bimodal is simply 2 humps.
Positive skew is asymmetry with tail on the right (mean > median > mode).
Negative skew has tail on the left (mean < median < mode).

362
USMLE STEP 2 CK HIGH YIELD NOTES (EPIDEMIOLOGY)

Outlier
An extreme and unusual value observed in the dataset is called an outlier.
The outlier may be the result of a recording error, a measurement error, or a natural phenomenon.
You should understand how an outlier can affect the measures of center and the measures of dispersion
of the dataset.
Mean is extremely sensitive to outliers and easily shifts toward them.
Median is much more resistant to outliers, because it is located in the middle of the dataset where the
observations usually do not differ much from each other
Mode is not affected by outliers, because they do not change the most frequent value observed.
Standard deviation is sensitive to outliers, because it is the measure of dispersion within the dataset and
outliers significantly increase the dispersion.

Standard deviation SD
Normal distribution is a nice symmetrical bell-shaped distribution.
What makes this distribution so nice is the possibility to predict what proportion of observations lie
within particular limits from the mean.
The degree of dispersion from the mean is determined by standard deviation.
With a normal or bell-shaped distribution:
1 SD from the mean holds 68% of values
2 SD from the mean hold 95% of values
3 SD from the mean hold 99.7% of values

Correlation coefficient (r)


The correlation coefficient assesses a linear relationship between two variables.
Correlation coefficient shows the strength and the direction (positive, negative) of linear association
between two variables. It does not necessarily imply causality.
r is always between 1 and +1.
The null value for correlation coefficient is 0 (no association).
The sign of the correlation coefficient indicates positive or negative association; the closer the value is to
its margins (-1 or 1) the stronger association.
It is extremely imports to know that the correlation coefficient shows the strength of association, but does
NOT necessarily imply causality.
For example, if r is negative, so as the level of X increases, the Y decreases.
Coefficient of determination = r2.

363
USMLE STEP 2 CK HIGH YIELD NOTES (EPIDEMIOLOGY)

Confidence interval (CI)


Range of values in which a specified probability of the means of repeated samples would be expected to
fall.
CI = confidence interval.
CI = range from [mean Z (SEM)] to [mean + Z (SEM)] = mean Z (SEM)
Z = standard score.
SEM = standard error of mean
The 95% CI (corresponding to p = .05) is often used. For the 95% CI, Z = 1.96.
Larger confidence interval means that there is a wider range of possible effects.
Smaller confidence interval means that there is a narrower range of possible effects.
If the confidence intervals of different groups overlap then the groups are considered to have no
statistically significant difference.

The null value for relative risk(RR) is 1.0.


The RR of 1.0 means, that there is no association between the exposure and the disease.
The RR of more than 1.0 means that there is an association between the exposure and the disease, and the
exposure is detrimental.
But reporting only RR value gives us no clue whether such a finding can be explained by chance alone.
Confidence interval gives us such a clue
What is very important in this case, the confidence interval doe not contain the null value (RR=1.0),
therefore these findings are what we call 'statistically significant'.
Statistically significant results have p value < 0.05, which means there is less then 5% chance that the
results as extreme as that can be observed by chance alone.

364
USMLE STEP 2 CK HIGH YIELD NOTES (EPIDEMIOLOGY)

Scatter plots

Scatter plots are useful for crude analysis of data.


These can demonstrate the type of association (linear, nonlinear), if any is present.
If a linear association is present, the correlation coefficient can be calculated to provide numerical
description of the linear association.
The scatter plot shows an almost perfect linear association between the blood glucose level at admission
and the number of episodes of significant ventricular arrhythmias.
The scatter plot above demonstrates a positive association (i.e., an increase in the blood glucose level is
associated with an increase in the number of episodes of significant ventricular arrhythmias); therefore,
the correlation coefficient is positive.
The correlation coefficient of an almost perfect linear association is close to 1.
Crude analysis of the association using scatter plots does not account for possible confounders (e.g.,
severity of the disease, degree of sympathetic activation, etc.), and does not necessarily imply causal
relationships between variables.

Different types of studies


There are two main objectives of epidemiological studies.
One is descriptive epidemiology, which deals with rates, ratios and distribution and explains the
determinants of disease inform of time, place and person.
Second is the analytical epidemiology, which tests the hypothesis, created by descriptive epidemiology.
Analytical epidemiology consists of observational studies and experimental studies.
Observational studies include case control, cohort and cross sectional studies.

365
USMLE STEP 2 CK HIGH YIELD NOTES (EPIDEMIOLOGY)

Experimental
Compares therapeutic benefits of two or more treatments, or treatment and placebo.
Highest-quality study when randomized and double-blinded.

Cohort
An observational study in which a cohort of exposed and non-exposed individuals are followed to
determine if disease develops.
Smokers had a higher risk of developing COPD than did nonsmokers.
Usually prospective, but can also be retrospective.
Prospective or longitudinal cohort study divides the study group-into exposed and not exposed. Then, it
follows each subject prospectively till the onset of disease. It is a stronger study than case control study
and cross sectional study.
Retrospective cohort study starts at some point between the exposure and outcome, the researcher goes
back past and classifies subjects as exposed and not exposed .and then follows them till the outcome.
Thus in cohort study, the study subjects are free of the outcome at the time study begins.
Advantages:
Data are collected in real time.
recall bias.
Allows the effects of rare exposures and multiple outcomes of an expo-
sure to be examined.
Relative risk (RR), incidence, and OR can be determined.
Limitations:
Studies are time consuming, require many subjects, and are very expensive.
Selection bias and confounding variables may complicate result interpretation (exposure is not
randomly distributed).
Rare diseases cannot be studied.
Losses occur to follow-up.

Case-control/Retrospective
The case-control study method is used to compare exposure experience of the people with disease
(cases) and exposure experience of the people without disease (controls).
The main measure of association in case-control studies is the exposure odds ratio (OR).

366
USMLE STEP 2 CK HIGH YIELD NOTES (EPIDEMIOLOGY)

Case-control studies are very popular in exploring an exposure-disease association, because they are
relatively cheap and less time-consuming than cohort studies.
It starts with the outcome and identifies the prior exposure the risk factors in study population, thus it
moves from effect to cause.
Case control study starts by classifying the subjects as cases and controls according to their outcome
status.
The cases and controls are assessed retrospectively to look for the presence of risk factors.
Exposure status is determined by personal interviews, health records, laboratory reports etc. in the
individual.
Information about past exposure is used to calculate an odds ratio (OR).
Patients with COPD had higher odds of a history of smoking than those without COPD.
Matching is a study design tool, used in case-control studies, that makes cases and controls have similar
distribution of some important confounding variables. It is an efficient tool to control confounding.
One of the major drawbacks of case-control studies is the fact that the risk cannot be derived directly
from their results.
It compares the odds of exposure in cases to the odds of exposure in controls.
It is NOT the same as relative risk. Relative risk can be calculated in follow-up studies by comparing the
risk in exposed individuals to the risk in unexposed individuals.
Calculation of relative risk directly, is not possible in case-control studies, because the study design does
not include following people overtime.
But, sometimes, relative risk can be approximately equal to the odds ratio. If the prevalence of the
disease is low exposure, the odds ratio approximates the relative risk. This, statement is. called the rare
disease assumption and represents one of the fundamental epidemiologic concepts.
If the outcome of a case-control study is not common in the population, the odds ratio is close to the
relative risk.
Advantages:
Small study groups.
Inexpensive.
Useful for rare diseases or outcomes.
Limitations:
Can be used to examine multiple potential etiologic factors.
Data may be inaccurate owing to recall bias and survivorship bias (those with more aggressive disease
may already have died).
Prevalence, incidence, and relative risk (RR) cannot be calculated.

367
USMLE STEP 2 CK HIGH YIELD NOTES (EPIDEMIOLOGY)

Cross-sectional survey/Prevalence survey observational


A survey of the population at a single point in time [Collect data from a group of people to assess
frequency of disease (and related risk factors) at a particular point in time].
In cross-sectional study, exposure and outcome are measured simultaneously at a particular point of time
(you can remember it as a 'snapshot' study). In other study designs, a certain time period separates
exposure and outcome.
Since both exposure and outcome are present for some time before the study, it is not possible to
determine the temporal association between the exposure and outcome from a cross sectional study.
Prevalence odds ratio is calculated in cross-sectional studies to-compare prevalence of a disease
between different populations.
Advantages: Can be used to estimate disease prevalence and to form hypotheses.
Limitations: Because risk factors and presence of disease are determined simultaneously, causal
relationships cannot be established [Can show risk factor association with disease, but not causality] (vs.
case-control and cohort studies).

Randomized Controlled Clinical Trial (RCCT)


Randomized control trial is a type of experimental study, and is considered gold standard to study the
efficacy of the treatment or procedure.
An experimental, prospective study in which subjects are assigned to a treatment or control group
(Subjects are randomly assigned to experimental or control group).
Randomization bias and confounding.
May be blinded (the patient does not know to which group he/she is assigned) or double blinded (neither
the patient nor the researcher knows the group assignment).
This type of study and helps to show strong causal relationship.
Advantages:
Highest-quality study.
Minimizes the effect of bias and confounding (has the least bias).
Can potentially demonstrate a causal relationship.
Limitations:
Very costly and time intensive.
May be difficult to blind some interventions (e.g., education, exercise, surgery).
For ethical reasons, one cannot compare a new treatment to a placebo if there is a standard of care.

368
USMLE STEP 2 CK HIGH YIELD NOTES (EPIDEMIOLOGY)

Meta-analysis
A statistical combination of data from several studies (often via a literature search).
Advantages:
Can the statistical power of a study to allow for the evaluation of small differences.
May resolve conflicting studies in the literature.
Limitations:
Cannot overcome the limitations of individual studies.
One must make sure that the pooled data evaluate similar populations and interventions.
Analyses are complicated because errors are introduced when means and variances from different
studies are combined.

Chi-squared test (2)


It is used to compare 2 or more percentages (%) or proportions of categorical outcomes (nonnumeric
data, also called nominal data).
It can be done using a 2 x 2 table by comparing the observed values to the values expected under the
hypothesis of no association.
If the difference between observed and expected values is large, an association between the exposure and
the outcome is assumed to be present.

t-test
Checks difference between the means of 2 groups (Mr. T is mean).
Several statistical approaches can be used, but the basic information needed to perform two-sample t test
is the two mean values, the sample variances, and the sample size.
After the calculations, t statistic is obtained that is used to calculate p value.
If p value is less then 0.05, null hypothesis (no difference between two groups) is rejected and the two
means are assumed to be statistically different.
If p value is large, null hypothesis is retained.

Analysis of variance (ANOVA)


checks difference between the means of 3 or more groups (ANOVA = ANalysis Of VAriance of 3 or
more variables.).

369
USMLE STEP 2 CK HIGH YIELD NOTES (EPIDEMIOLOGY)

Type I error ()
Stating that there is an effect or difference when none exists (to mistakenly accept the
experimenthypothesis and reject the null hypothesis).
= you saw a difference that did not existfor example, convicting an innocent man.
p = probability of making a type I error.
p is judged against , a preset level of significance (usually < .05).
If p < .05, then there is less than a 5% chance that the data will show something that is not really there.

Type II error ()
Stating that there is not an effect or difference when one exists (to fail to reject the null hypothesis when
in fact H0 is false).
is the probability of making a type II error.
= you did not see a difference that does existfor example, setting a guilty man free.

Power (1 )
Power of a study is the ability to detect the difference between two groups (treated versus non-treated,
exposed versus non-exposed).
Increasing the sample size increases the power. As a result, the confidence interval of the point estimate
(e.g., relative risk) is getting tighter.
It depends on:
1) Total number of end points experienced by population
2) Difference in compliance between treatment groups (differences in the mean values between groups)
3) Size of expected effect

P value
P value shows the probability of obtaining the result of a study by chance alone.
Relative risk is a point estimate of association, but it does not account for random error.
You can realize intuitive that there is a certain probability that any value of relative risk (even if it is
extreme) can arise by chance.
P value deals with random variability.
When the p value is less than 0.05, this is usually considered statistically significant.

370
USMLE STEP 2 CK HIGH YIELD NOTES (EPIDEMIOLOGY)

If someone tells you that P < 0.05 for a given set of data, there is less than a 5% chance (because 0.05 =
5%) that these data were obtained by random error or chance.
If P value < 0.05, the 95% confidence interval does not contain 1.0 (a null value for relative risk).
If P < 0.01, the chance that the data were obtained by random error or chance is less than 1%.
For example, if I tell you that the blood pressure in my controls is 180/100 mmHg but decreases to
120/70 mmHg after administration of drug X and that p < 0.10, there is less than a 10% chance that the
difference in blood pressure was due to random error or chance. However, there is up to a 9.99999%
chance that the result is due to random error or chance.
For this reason, p < 0.05 is commonly used as the cutoff for statistical significance.
Three points to remember:
1) The study may still have serious flaws
2) A low p-value does not imply causation,
3) A study that has statistical significance does not necessarily have clinical significance. For example, if
I tell you that drug X can lower the blood pressure from 130/80 to 128/80, p <
0.000000000000000001, you still would not use drug X.

Null hypothesis (H0)


Hypothesis of no difference (e.g., there is no association between the disease and the risk factor in the
population).
The null hypothesis is always the statement of no relationship between the exposure and the outcome.
To state the null hypothesis correctly, you should think about the design of the study.
In cross-section, you cannot decide whether the exposure preceded the outcome or the outcome precede
the exposure and, of course, you cannot directly compute the risk.
In cohort study, the outcome is the same for subjects with or without the risk factor NO relationship!

Alternative hypothesis (H1)


Hypothesis that there is some difference (e.g., there is some association between the disease and the risk
factor in the population).
Alternative hypothesis states that there is a relationship between the exposure and outcome


371
USMLE STEP 2 CK HIGH YIELD NOTES (EPIDEMIOLOGY)

Bias
Defined as any error in the design, implementation, or analysis of a study conclusions differing from
the truth.
If the sample is not representative of the population that is being evaluated, it is considered a biased
sample. Bias can seriously threaten the validity of the study

Selection bias
Produces a sample that is not representative of the study population of interest; overestimation or
underestimation of the association between an exposure and outcome.
Selection bias results from the manner in which the subjects are selected for the study, or from the
selective losses from the follow-up.
Loss to follow-up in prospective studies creates a potential for selection bias.
Subtypes are as follows:
Self-selection bias: Patients who choose or do not choose to participate (non-respondent bias) may
yield results that are not representative of the population. As an example, patients with refractory
disease may be more likely than others to enroll in an experimental study. Selection bias can be
controlled by choosing a representative sample of the population for the study and achieving high rates
of follow-up.
Enrollment bias: Subjects are assigned to a study group in a nonrandom fashion. An example would be
assignment of healthier patients to the intervention group.

Information bias
Yields misclassification of subjects on the basis of exposure and/or outcome; differing quality of data
between study groups.
Information bias occurs due to imperfect assessment of association between the-exposure and outcome as
a result of errors in measurement of exposure and outcome status.
It can be minimized by using standardized techniques for surveillance and measurement of outcomes,
and use of trained observers to measure exposure outcome.
Subtypes are as follows:
Lead Time bias refers to the chronology between the diagnosis and treatment of different cases. Such
difference in chronology between treatment and diagnosis could affect the outcomes the study. Lead-
time bias should always be considered while evaluating any screening test. It is apparent prolongation
of survival in patients to whom this test was applied (diagnosed earlier), without actually prolonging

372
USMLE STEP 2 CK HIGH YIELD NOTES (EPIDEMIOLOGY)

true survival (the natural course of the disease is not altered no change in prognosis). Patients
screened appear to live, longer because the disease was detected earlier then it would have been if
diagnosed from clinical symptoms, but the overall prognosis stays the same.
Measurement bias: refer to misclassification of outcome and/or exposure (e.g., labeling diseased as
non-diseased and vice versa) and are related to the design of the study. Information gathering distorts
data; distorted results and conclusions. An example would be an assay that inaccurately estimates a
biological parameter.
Responder bias: is present when the outcome is obtained by the patient's response, and not by
objective diagnostic methods (e.g., migraine headache). Participants responses to subjective questions
are affected by their awareness of their study arm.
Observer bias: occurs when an observers decision is affected by the knowledge of the exposure
status, that may be affected by awareness of the hypothesis. Eliminated with double-blind study design.
Observer's bias is controlled by blinding technique.
Recall bias: Errors of memory that occur in retrospective cohort or case-control studies (the difference
in the ability of certain persons to recall facts regarding risk factor exposure). People who develop a
disease or who have a negative outcome may be more likely to remember risk factors or to exaggerate
their history of exposures.
Late-look biasinformation gathered at an inappropriate time

Ascertainment bias can be avoided by selecting a strict protocol of case ascertainment.

Unacceptability bias refers to participants' response with desirable answers which leads to
underestimation of the risk factors. E.g. medical students know the risk of smoking and may not care to
reveal their smoking status, especially to the Public Health Department. Therefore a lower number of
cigarettes smoked may be reported, than actually smoked. This will affect the results of the study.

Admission rate bias refers to distortion in risk ratio due to different-hospitals admission of cases. For
instance, patients with cardiac diseases may prefer to be admitted to a certain hospital which may
interfere with the outcome of interest.

Sample distortion bias is seen when estimate of exposure and outcome association is biased because
the study sample is not representative of target population with respect to the joint distribution of
exposure and outcome.

373
USMLE STEP 2 CK HIGH YIELD NOTES (EPIDEMIOLOGY)

Confounding bias
Confounding is referred to the bias that can result when the exposure-disease relationship is mixed up
with the effect of extraneous factors called confounders.
Confounders influence both the exposure and the outcome overestimation or underestimation of the
relationship between exposure and outcome.
In order to be confounder, an extraneous factor should have some properties linking it with the outcome
of interest and the exposure. It should be related to the outcome of interest, and it should be related to the
exposure. If these properties are present, we could observe at least part of the effect of smoking on cancer
of the oral cavity by examining the association between alcohol consumption and cancer of the oral
cavity.
If a factor is a confounder, it should be controlled for in the analysis.
Example: If vitamin consumption is associated with a healthy diet, it may appear that individuals who
take vitamins are protected from a particular disease when it is actually diet that is the protective factor.
For example, cigarette smoking can be a confounding factor in studying the association between the
maternal alcohol drinking and low birth weight baby, as cigarette smoking is independently associated
with both alcohol consumption and low birth weight baby.
Reduced by study design (randomization or matching for case control) or by statistical adjustment
(e.g., multivariate analysis).
The methods used to control confounding in the analysis stage of a study are stratification and modeling.
Matching is a study design tool that makes cases and controls have similar distribution of some
important confounding variables. Matching is an efficient method to control confounding.
Randomization is a method to control confounders in the design stage of a study. Its purpose is to
balance various factors that can influence the estimate of association between treatment and placebo
groups so that the un-confounded effect of the exposure of interest can be isolated. A very important
advantage of randomization, when compared to other methods, is the possibility to control the known
risk factors (e.g., age, severity of heart failure) as well as unknown and difficult-to-measure
confounders (e.g., level of stress, socioeconomic status)

Ways to reduce bias:


1) Blind studies (double blind is better)
2) Placebo responses
3) Crossover studies (each subject acts as own control)
4) Randomization: the similarity of baseline characteristics of the patients in the treatment and placebo
groups in clinical trials indicates that the randomization was successful.

374
USMLE STEP 2 CK HIGH YIELD NOTES (EPIDEMIOLOGY)

Effect modification
Effect modification is a phenomenon when the effect of the main exposure on the outcome, is modified
by the level of another variable.
Effect modification is NOT a bias! and it is not due to flaws in design or analysis phases of the study
which can result in, for example, selection bias.
For example, the effect of oral contraceptives on the incidence of breast cancer is modified by family
history for the women with family history the risk is increased, for the women without family history the
risk is not increased.
There are other well-known examples of interaction you probably are familiar with, the effect of
estrogens on the risk of venous thrombosis is modified by smoking; the risk of lung cancer in people
exposed to asbestos greatly depends on smoking status, etc.

Hawthorne effect
Hawthorne effect is the tendency of the study population to affect the outcome, due to the fact that they
are being studied.
The study population changes their behavior because they are being studied and this can seriously affect
the validity of the study.
It is commonly seen in study that concerns with behavioral outcomes or outcomes that can be influenced
by behavioral changes.
Randomized control trials have a sense of uncertainty and risk due to randomization, which maybe more
potent behavior modifiers than mere observation.
In order to minimize the potential of Hawthorne effect, studied subjects can be kept unaware that they
are being studied, however it may pose ethical problems.

Latent period
The concept of latent period is an important issue in chronic disease epidemiology.
Exposure must be continuously present for a certain period of time (called latent period) to influence the
outcome.
Latent period from getting exposed to developing outcome is relativity short in infectious diseases.
Unlike that, chronic diseases, e.g., cancer or coronary artery disease may have a very long latent period.
Moreover, extended period of continuous exposure may be necessary to affect the outcome.
Latent period is a natural phenomenon, not bias!

375
USMLE STEP 2 CK HIGH YIELD NOTES (EPIDEMIOLOGY)

376
USMLE STEP 2 CK HIGH YIELD NOTES (EPIDEMIOLOGY)
Epidemic: occurrence of disease in excess of the expected rate.
Pandemic: a worldwide epidemic.
Endemic: the usual, expected rate of disease over time.

Per-year rates commonly used to compare groups:


1) Birth rate: live births/1000 population.
2) Fertility rate: live births/1000 population of females age15-45 yr
3) Death rate: deaths/1000 population
4) Neonatal mortality rate: neonatal deaths (in the first 28 days)/l000 live births
5) Perinatal mortality rate: neonatal deaths + stillbirths per 1000 total births
The major cause is prematurity
The neonatal mortality rate is roughly 6/1000 (higher in blacks)
The fetal mortality rate is roughly 9/1000 (higher in nonwhites)
The perinatal mortality rate is roughly 15/1000
A stillbirth (fetal death) is defined as a prenatal or natal death after 20 weeks' gestation.
6) Infant mortality rate: deaths (from 0-1 year old)/1000 live births (the top three causes, in descending
order, are congenital abnormalities, low birth weight, and sudden infant death syndrome)
7) Maternal mortality rate: maternal pregnancy-related deaths (deaths during pregnancy or in the first 42
days after delivery)/100,000 live births
The top three causes are pulmonary embolism, pregnancy-induced hypertension, and hemorrhage.
The rate increases with age and is higher in blacks.

Leading causes of death in the United States by age


Infants: Congenital anomalies, short gestation/low birth weight, sudden infant death syndrome, maternal
complications of pregnancy, respiratory distress syndrome.
Age 114: Injuries, cancer, congenital anomalies, homicide, heart disease.
Age 1524: Injuries, homicide, suicide, cancer, heart disease.
Age 2564: Cancer, heart disease, injuries, suicide, stroke.
Age 65+: Heart disease, cancer, stroke, COPD, pneumonia, influenza.

Disease prevention (PDR)


1 Prevent disease occurrence (e.g., vaccination).
2 early Detection of disease (e.g., Pap smear).
3 Reduce disability from disease (e.g., exogenous insulin for diabetes).

377
USMLE STEP 2 CK HIGH YIELD NOTES (EPIDEMIOLOGY)
Important prevention measures
Risk factor Services
Diabetes Eye, foot exams; urine tests
Drug use... Hepatitis immunizations; HIV, TB tests
Alcoholism... influenza, pneumococcal immunizations; TB test
Overweight... Blood sugar tests for diabetes
Homeless, recent immigrant, inmate... TB test
High-risk sexual behavior HIV, hepatitis B, syphilis, gonorrhea, chlamydia tests
123

Reportable diseases: B. A. SSSMMART Chicken or youre Gone


Hep B, Hep A
Salmonella, Shigella, Syphilis, Measles, Mumps, AIDS, Rubella, Tuberculosis
Chickenpox, Gonorrhea

Medicare and Medicaid


MedicarE is for Elderly & MedicaiD is for Destitute.
Medicare and Medicaid are federal programs that originated from amendments to the Social Security
Act.
Medicare is health insurance for people who are eligible for Social Security (primarily people > 65 years
old as well as the permanently and totally disabled and patients with end-stage renal disease). Nursing
home care is paid by Medicare only in the short term after a hospital admission; then it is paid by the
patient (if the patient has no money, the state usually pays). Medicare Part A = hospital; Part B = doctor
bills.
Medicaid covers the indigent and poor who are deemed eligible by the individual states.

Coronary Heart Disease is the leading cause of death in both men and women of all races and ethnicities
in United States.
USPSTF strongly recommends routine screening for lipid disorders in men aged 35 years and older, and
women aged 45 years and older.

USPSTF recommends screening for men ages 20 to 35 years and for women ages 20 to 45 years in the
presence of any of the following:
1) Diabetes
2) Family history suggestive of familial hyperlipidemia
3) Presence of multiple risk factors for coronary artery disease
4) Family history of cardiovascular disease before the age of 50 years in a male relative or before the age
of 60 years in a female relative.

378
USMLE STEP 2 CK HIGH YIELD NOTES (EPIDEMIOLOGY)
USPSTF recommends the use of total cholesterol and HDL cholesterol for the purposes of screening.
If abnormal values are found, full fasting lipid panel should be ordered.

However, for treatment purposes, the recommendations made by ATP 3 are based on total risk
assessment and LDL cholesterol levels.

Total to HDL cholesterol level is a better predictor of risk of CHD than LDL to total cholesterol;
however, it is not used to make decisions regarding the treatment.

An optimal level of LDL cholesterol is < 100 mg/dL.


An optimal level of total cholesterol is <200 mg/dL.

The ATP 3 recommendations for the treatment are:


LDL goal LDL Levels at which LDL level at which
lifestyle modification drug therapy starts
CHD or CHD < 100 mg/dL 100 mg/dL 130 mg/dL
equivalent
Two or more risk < 130 mg/dL 130 mg/dL 160 mg/dL
Zero or 1 risk factor < 160 mg/dL 160 mg/dL 190 mg/dL

ATP 3 recommendations for management of low HDL are as follows:


1) Lowering LDL cholesterol is the primary target of therapy and should be reached before treating low
HDL
2) When the low HDL is associated with high triglycerides (>200 mg/dL), first, therapy should be
directed at achieving non-HDL goals.
3) Life-style modifications to treat low HDL include weight management, increase physical activity
and smoking cessation.
4) Medical treatment of low HDL includes the use of either fibrates or nicotinic acid. Fibrates are
considered be the drug of choice, as nicotinic acid has many side affects and is poorly tolerated.
5) If possible, medication that lowers HDL cholesterol, such as beta-blockers, benzodiazepines,
androgens, etc., should be discontinued.

The main effect of fibrates (e.g. Gemfibrozil) is to lower plasma triglycerides and elevate HDL
levels; however, severe risk of muscle toxicity increases if given with statins (pravastatin has the least
risk of this side effect).

Nicotinic acid lowers LDL, VLDL and elevate HDL levels: However, it has poor tolerability:

Statins are the drug of choice to lower LDL cholesterol.

379
USMLE STEP 2 CK HIGH YIELD NOTES (EPIDEMIOLOGY)
Type A personality
Type A personality is characterized primarily by time pressure (i.e. feeling rushed most of the time) and
competitiveness.
Human studies have demonstrated a significant-association between type A personality and exaggerated
cardiovascular response.

Pneumococcal vaccine
It is recommended; single dose for:
1) For adults over the age of 65 years.
2) All individuals with chronic debilitating illnesses like chronic cardiovascular, pulmonary, liver, and
renal disease.
3) Individuals with immunosuppression and diabetes mellitus.
4) Chronic alcoholics, and individuals in chronic-care facilities, should also receive this vaccination.
A one-time booster is given 5 years after the primary vaccination in elderly patients because of the
weaning effect of vaccination.
Pneumococcal vaccine contains capsular polysaccharides of the 23 most prevalent types and it produces
T cell independent B cell response.
Polysaccharides can't be presented to T cell because it's a polysaccharide. Only peptides can be presented
by macrophages B Cell to T cell in association with MHC II.

Influenza vaccine
Influenza vaccine immunization is recommended on annual basis for:
1) All individuals aged 55 years and older
2) Individuals of any age with chronic debilitating illnesses like: cardiovascular, renal or pulmonary
disorders, diabetes mellitus.
3) All immunocompromised adults.
4) Nursing home residents.
5) Pregnant women in the second or third trimester during influenza season.
6) Other high-risk individuals such as physicians, nurses, employees of nursing.homes, and family
members patients infected with influenza.
7) Adults in close contact with children aged 0 59 months.

Routine influenza vaccine is recommended in all patients with cystic fibrosis; but pneumococcal
vaccine is not routinely indicated because patients with CF only rarely become infected with
Streptococcus pneumoniae. However, since it is readily available and safe, many physicians consider
giving this vaccine to this patient population.

380
USMLE STEP 2 CK HIGH YIELD NOTES (EPIDEMIOLOGY)
A number of vaccines are made from embryo or has egg products in its formulation, and the patients
with an egg allergy are at risk of allergic reaction/anaphylaxis when given such vaccines. Influenza
vaccine and yellow fever vaccine can cause severe allergic reactions in patients with severe egg allergy.
A test dose skin testing is recommended before administrating this vaccine in patients with egg allergy.
An egg allergy is also a relative contraindication to MMR vaccine. However, recent studies have shown
that allergic reaction against MMR are more due to gelatin content of the vaccine rather than the egg, and
thus, many physicians recommend MMR vaccine even in patients with an egg allergy.

Two doses of hepatitis A vaccine are indicated only in adults who are at increased risk of the disease.

HAV vaccine, HBV vaccine S. pneumoniae and influenza vaccine are recommended for HIV-infected
individuals.

HIV-infected individuals are at an increased risk for developing measles. Although MMR is a live
vaccine, MMR is highly recommended for all HIV-infected patients who are asymptomatic and not
severely compromised (CD4 counts > 200/micro-L).

USPHS guidelines, HIV-infected patients should receive tetanus and diphtheria immunizations according
to the normal dosing schedules. Td vaccine, which is given as a booster every 10 years.

HIV-infected patients should not receive live vaccines (e.g. BCG, varicella chickenpox, varicella zoster
shingles, anthrax, oral typhoid, transnasal influenza, oral polio and yellow fever).

The vaccination schedule for preterm infants should be conformed to the childs chronological age, not
the gestational age.
The exception is that children should be 2 Kg prior to receiving the 1st hepatitis B vaccine.

USPSTF strongly recommends routine screening for chlamydial infection in all sexually active women
aged 24 years and younger, and in other asymptomatic women at increase risk for this infection.

USPSTF recommends screening for colorectal cancer in men and women after 50 years of age. If there
is a family history of colon cancer in first degree relative, the screening should be started from 40. Fecal
occult blood test, periodic colonoscopy, or a combination of both may be used for screening.

USPSTF recommends routine annual mammography screening, with or without clinical breast
examination (CBE), for women aged 50 and older. In high risk patients baseline mammogram at 35 can
be done.

All adults should be immunized against diphtheria and tetanus every 10 years.
A three-dose schedule of diphtheria and tetanus is given to those who have not received an initial
immunization series in childhood.

381
USMLE STEP 2 CK HIGH YIELD NOTES (EPIDEMIOLOGY)
Female patients over the age of 50 are required to undergo annual mammograms until the age of 75. In
general screening studies are not routinely recommended for people older than 80.

Screening for bladder cancer is not recommended, even in patients who are at high risk of developing the
disease.

USPSTF recommends screening for strabismus, amblyopia, and refractive errors in children aged 0 5
years.

USPSTF recommends that providers discuss the benefits and limitations of PSA testing with their
patients.

USPSTF recommends screening male active or former smoker aged 65 75 years with a one-time
abdominal ultrasound to evaluate for an abdominal aortic aneurysm.

No serologic or radiographic test has proven to be effective for screening for pancreatic cancer in
asymptomatic adults.

Patients with chronic liver disease should receive a number of vaccinations. In addition to being up-to-
date with childhood immunizations, they should be vaccinated against hepatitis A, hepatitis B,
influenza, and invasive pneumococcal disease (PCV). Like all adults, they should get a Td booster at
least every 10 years.

A patient with diabetes mellitus should be up-to-date on Td and receive invasive pneumococcal disease
vaccine (PCV) and inactivated influenza vaccine.

Dual energy x-ray absorptiometry (DEXA)


DEXA is currently the gold standard and most widely used investigation for detecting osteoporosis and
osteopenia.
DEXA focuses mainly on 2 areas i.e. spine and pelvis.
DEXA is the test of choice in most settings.
USPSTF recommends a one-time screening of all women who are 65 years or older with a DEXA scan.
Patients with a T-score of less than 1.5 plus risk factors for osteoporosis or a T-score of less than 2.0
should receive preventive medications. The preferred agents are oral bisphosphonates or raloxifene.

Quantitative CT of the spine is the most sensitive diagnostic test for osteopenia but it is not a gold
standard because of its poor reproducibility.

382
USMLE STEP 2 CK HIGH YIELD NOTES (EPIDEMIOLOGY)
If someone is exposed to HBV and has a documented response to HBV vaccination, he/she does not need
anything else but reassurance.
For those who are vaccinated, but there is no documentation of the response to vaccine, a second course
of vaccination is needed unless they shows positive anti-HBsAg at the time of exposure.
If someone is a non-responder to previous vaccination, they should be given HBIG on exposure to HBV.
When someone is exposed to HBV and is not vaccinated previously, he/she should be given HBIG within
24 hours and a vaccination against HBV should be started.

Pap smear
Cervical cancer screening with a Pap smear should beginning at age 21 or three years after the initiation
of sexual intercourse, whichever comes first.
Screening is usually started at 18 and stopped at 60-75 years.
If 3 consecutive Pap smears are normal, screening may be performed less frequently (every 2 3 years)
in a low-risk woman i.e. patients who have single sexual partners, no history STDs etc.
However, even in low-risk women, a Pap smear needs to be repeated within one year if the patient has a
new sexual partner.
Women who have undergone subtotal hysterectomy and still have a cervix should be screened according
to the above guidelines.
If a woman has undergone a hysterectomy for cancer or cervical dysplasia she needs yearly Pap smear
screening of the vaginal epithelium and inspection of the vulvar and perianal epithelium. But if a women
who have had hysterectomies for benign indications, there is no consensus regarding cytologic screening.

Cancer cervix
Women who should have annual Pap smears, even if their prior Pap smears were negative;
1) Immunocompromised
2) Had in utero exposure-to-DES
3) Have a history of CIN cancer.
Carefully planned treatment and follow-up can prevent up to 95% of cervical cancers in patients with
CIN.
Patients who underwent treatment for biopsy-confirmed CIN II/III (moderate to severe dysplasia) should
be screened using cytological test or a cytological test plus colposcopy every 4 to 6 months.
After three consecutive negative results of screening, they should be screened annually using a
cytological test.
It is not necessary to do a biopsy if the screening tests do not reveal any abnormalities.

383
USMLE STEP 2 CK HIGH YIELD NOTES (EPIDEMIOLOGY)
Cancer colon
The guidelines as follows:
Patients at average risk of developing colon cancer should begin screening at age 50 with fecal occult
blood testing (FOBT), flexible sigmoidoscopy, a combination of the two, colonoscopy, or double-
contrast barium enema, once a year, until the age of 80.
Patients with an affected first-degree relative should begin screening 10 years before the age the relative
was diagnosed.
Colonoscopy for people > 50 years old. Once in every 10 years.

Ovarian cancer
The screening with CA 125 combined with vaginal ultrasound seems reasonable in patients who are at
higher than average risk of ovarian cancer (who have family history ovarian cancer, but not family cancer
syndrome).
Positive family history of early ovarian cancer (before age 40) or combined ovarian and breast cancer
may indicate the presence of a family cancer syndrome and are considered high-risk group.
Prophylactic oophorectomy is considered in patients with some family cancer syndromes (e.g., women
with BRC or BRCA2 mutations) along with an intensive screening program.

Bupropion has been approved by FDA for smoking cessation program.

TORCH infections (toxoplasmosis, rubella, CMV, HSV and syphilis) cause a syndrome characterized by
microcephaly, hepatosplenomegaly, deafness, chorioretinitis and thrombocytopenia.

Dysthymia
Dysthymia = depressed mood for most days for at least two years..
Additional features is two or more of: decreased or increased appetite, poor energy, insomnia or
hypersomnia, impaired concentration, low self-esteem, and feelings of worthlessness.

384
Herpes simplex encephalitis
Herpes mainly affects the temporal lobe of the brain, and presents with an acute onset (<1 week
duration) of focal neurological findings such as altered mentation, focal cranial nerve deficits,
hemiparesis, dysphasia, aphasia, ataxia, or focal seizures.
Fever is present in approximately 90% of patients.
CSF examination characteristically reveals lymphocytic pleocytosis, increased number of erythrocytes
(hemorrhagic destruction of temporal lobes), and elevated protein levels; low CSF glucose levels.
HSV polymerase chain reaction analysis is the gold standard.
Acyclovir can cause crystalluria with renal tubular obstruction during high-dose parenteral therapy,
especially in inadequately hydrated patients.

Bacterial meningitis
The characteristic CSF findings in bacterial meningitis are markedly increased PMN leukocytes and
decreased glucose levels.
Gram stain may show organisms.

Waterhouse-Friderichsen syndrome
The Waterhouse-Friderichsen syndrome is acute adrenal gland insufficiency caused by profound,
usually, bilateral adrenal glands hemorrhage because of disseminated intravascular coagulation.
It is a very serious complication of meningococcal meningitis caused by Neisseria meningitidis.
Patients usually experience circulatory collapse and septic shock.
The typical signs are petechial rash or purpura, cyanotic pallor and cutaneous bleeding with subsequent
necrosis.

Cryptococcal meningitis
It is often seen in immunocompromised patients (e.g., HIV patients with a history of IV drug abuse).
CSF examination illustrates chronic lymphocytic meningitis, with an increase in opening pressure,
elevated proteins, pleocytosis (usually lymphocytes), and low glucose levels.
Organisms can be detected by India ink preparation.

385
Progressive multifocal leukoencephalopathy (PML)
PML is an opportunistic infection seen in immunocompromised patients. It is caused by the JC virus, a
human polyomavirus.
PML predominantly involves the cortical white matter.
The most common presenting symptoms are hemiparesis and disturbances in speech, vision and gait.
An immunocompromised patient with focal neurological deficits should raise the suspicion for PML.
Progressive multifocal leukoencephalopathy lesions are non-enhancing, and do not produce mass effects.
Classic MRI findings in PML consist of multiple demyelinating, non-enhancing lesions with no mass
effects.

Suspect progressive multifocal leukoencephalopathy in an HIV-infected patient with focal neurological


signs and multiple non-enhancing lesions with no mass effect on the CT scan.

NOTE: A positive Toxoplasma serology is quite common in normal subjects in the United States

Primary CNS lymphoma is the second most common cause of mass lesions in HIV-infected
patients. Patients presents with an altered mental status. It presents as a solitary weakly ring-
enhancing periventricular lesion on MRI, but is usually solitary, weakly enhancing and periventricular.
The presence of EBV DNA in CSF is quite specific for the diagnosis.

Toxoplasmosis is a good differential diagnosis; however, MRI reveals multiple, ring-enhancing,


spherical lesions in the basal ganglia.

An isolated, round, smooth-bordered, ring-enhancing intracranial lesion on contrast CT scan of the


brain in an immunocompetent patient with a known extracranial bacterial infection is most likely a
brain abscess. Aerobic & anaerobic streptococci and Bacteroides species (anaerobes) are the most
common causative organisms.

386
Cranial nerves
Olfactory (CN I): Smell
Optic (CN II): Sight
Oculomotor (CN III): Eye movement, pupil constriction, accommodation, eyelid opening.
Trochlear (CN IV): Eye movement
Trigeminal (CN V): Mastication, facial sensation (ophthalmic branch V1 controls corneal sensation,
when it is damaged corneal anesthesia, patients can suffer from corneal injury without awareness).
Abducens (CN VI): Eye movement
Facial (CN VII): Facial movement, taste from anterior 2/3 of tongue, lacrimation, salivation
(submaxillary and sublingual glands), eyelid closing
Vestibulocochlear (CN VIII): Hearing, balance
Glossopharyngeal (CN IX): Taste from posterior 1/3 of tongue, swallowing, salivation (parotid gland),
monitoring carotid body and sinus chemo- and baroreceptors
Vagus (CN X): Taste from epiglottic region, swallowing, palate elevation, talking, thoracoabdominal
viscera, monitoring aortic arch chemo- and baroreceptors
Accessory (CN XI): Head turning, shoulder shrugging
Hypoglossal (CN XII): Tongue movement.

Supratentorial lesion: partial or complete hemiparesis


Posterior column lesion: ataxia
Cerebellar lesion: posterior fossa symptoms e.g. ataxia, nausea, vomiting
Upper thoracic spinal lesion: paraplegia, bladder & fecal incontinence, absent of sensation from the
nipple downwards.
Lower thoracic spinal lesion: absent of sensation from the umbilicus downwards

387
Spinal cord compression
Spinal cord compression is characterized by signs and symptoms of upper motor neuron dysfunction
distal to the site of compression
These include bilateral decreased sensation, weakness, hyperreflexia, and extensor plantar response
(positive Babinski sign: when stroking the sole of the foot elicits extension of the great toe).
Cord compression is a medical emergency requiring prompt diagnosis by a spinal MRI.

Cauda equine syndrome


Cauda equine syndrome is characterized by low back-pain, paraplegia, variable sensory loss, saddle
anesthesia, urinary and fecal incontinence and it would not occur secondary to compression/fracture
vertebra.
The cauda equina consists of spinal nerves before they exist the spinal cord.
This is a surgical emergency. High-dose of dexamethasone should be started immediately once the
diagnosis is suspected.
Cord compression is a medical emergency requiring prompt diagnosis via spinal MRI to identify the site
of compression/fracture, followed by surgery.

BACK PAIN
May arise from paraspinous muscles, ligaments, facet joints, disks, or nerve roots; typically resolves
within four weeks.
Prolonged bed rest is contraindicated.
Risk factors for malignancy include age > 50, a previous history of cancer, pain not relieved by lying
down, symptoms > 1 month, pain that worsens at night, and constitutional symptoms.
It is important to be familiar with the basic strategy of the management of patients with acute back pain.

388
Disk herniation
Causes include degenerative changes, trauma, or neck/back strain or sprain.
Most common in the lumbar region, especially at L4L5 and L5S1.
Common among middle-aged and older men.
Disc herniation presents as acute onset of electricity-like low back pain radiating down the buttock and
below the knee usually preceded by several months of aching, discogenic pain. The pain is caused due
to impinged nerve.
Herniated disk manifests with sciatica-type pain that radiates along the thigh and typically below the
knee. Exacerbated by straining (e.g., coughing).
Associated with sciatica, paresthesias, muscle weakness, atrophy, contractions, or spasms.
Positive straight leg suggests nerve root irritation due to impingement.
Crossed straight leg test is very specific for disc herniation.
At the same time, no neurologic deficit is present and the perianal area is intact (it is always important to
rule out cauda equina syndrome).

Obtain a plain radiograph if other causes of back pain are suspected (e.g., infection, fracture).
In patients with acute 'mechanical' back pain without significant neurologic deficit, conservative
approach is preferred for a period of 4-6 weeks. This includes early mobilization, muscle relaxants, and
NSAIDs. Bed rest and physical therapy has not been shown to be helpful.
If the pain persists after 4-6 weeks of conservative treatment or progressive neurologic deficit evolves,
high-resolution diagnostic modalities are usually employed: MRI or CT with or without contrast
myelography.

Emergency surgical decompression is indicated in case of significant or rapidly progressive neurologic


deficit (foot drop, weakness of the legs) or cauda equina syndrome.
Severe or rapidly evolving neurologic deficits and are indications for discectomy.

Lumbar Spinal Stenosis


A narrowing of the lumbar spinal canal that can compression of the nerve roots.
Narrowing of the spinal canal results from encroaching osteophytes at the facet joints, hypertrophy of the
ligamentum flavum and protrusion of intervertebral disks.
Lumbar spinal stenosis is associated with aging, especially after 60 years age.
Back pain associated with calf pain.
Back pain that radiates to the buttocks and legs; leg numbness and weakness.
Pain becomes better with sitting or leaning forward due to flexion of the lumbar spine or hips and gets
worsened by extension of the lumbar spine.
Leg cramping is worse at rest, with standing, and with walking (pseudo-or neurogenic claudication).
Symptoms improve with flexion at the hips.
In some patients gait disturbance is so prominent that they complain of having "spaghetti legs" or
walking "like a drunken sailor".
The preservation of pedal pulses helps distinguish from vascular claudication.

389
MRI is the investigation of choice for suspected lumbar spinal stenosis.
Mild to moderate: NSAIDs and abdominal muscle strengthening.
Advanced: Epidural steroid injections can provide relief.
Refractory: Surgical laminectomy may achieve significant short-term success, but many patients will
have a recurrence of symptoms.

Metastasis from carcinoma


It is the most common malignant tumor of the skeletal system.
In patients with history of malignancy, back pain raises suspicion for bone
metastasis.
Progressive low back pain not relieved by rest and worse at night suggests
vertebral body metastasis.
Secondaries to the bone mainly arise from primaries of the prostate, breast,
lung, multiple myeloma, and lymphoma.
Technetium 99 scinti scanning is the most effective method to assess bone
metastasis.

Lumbar strain
Usually occurs following twisting of the back while lifting heavy weights.
There is usually no point tenderness. Para spinal muscles are involved.

Lumbago
Usually related to physical strain and has less dramatic onset.
Usually paravertebral muscle tenderness rather than spinal tenderness is the feature.

Compression fracture of the vertebrae


It is a common complication of advanced osteoporosis.
It usually manifests as acute back pain without any history of trauma in a predisposed patient to
osteoporosis e.g. old age, postmenopausal women and glucocorticoid therapy.
Neurologic examination will be normal. Local tenderness usually present.
Malignancy presents with similar picture. X-ray spine and bone scan is indicated to differentiate these
two.

Note: Absent ankle reflex can be seen in elderly patients as part of the normal aging process.
Absent Babinski sign suggests no upper motor neuron-type lesion.
Lower extremity weakness or numbness suggests associated nerve injury.

390
SPINAL CORD INJURIES
Patients treated with high-dose methylprednisolone within eight hours of spinal cord injury have
significant and sustained neurological improvement. Thus its use is warranted as the first priority
after stabilizing the patient.

Syringomyelia
Syringomyelia is a disease in which CSF drainage from the central canal is disrupted, leading to a
fluid filled cavity that compresses surrounding neural tissue.
Presence of cord cavity is the most characteristic feature.
The most common causes of syringomyelia are Arnold Chiari malformations, prior spinal cord
injury and idiopathic.
3% to 4% of patients with spinal cord injuries (e.g. whiplash) will develop post-traumatic
syringomyelia. Symptoms develop months to years later.
It results in impaired strength (areflexic weakness) and pain/temperature sensation in the upper
extremity in a cape distribution.
MRI is used for definitive diagnosis.

Subacute combined degeneration


Occurs with vitamin B12 deficiency. Low serum vitamin B12.
Patients complains of distal paresthesia and weakness of extremities followed by spastic paresis and
ataxia.
Treatment with vitamin B12.

Anterior cord syndrome


Anterior cord syndrome is commonly associated with burst fracture of the vertebra.
Selective damage of the corticospinal and spinothalamic tracts.
It is characterized by total loss of motor function below the level of lesion with loss of pain and
temperature on both sides below the lesion.
MRI is the best investigation to study the extent of neurological damage
Patients treated with high-dose methylprednisolone within eight hours of spinal cord injury have
significant and sustained neurological improvement, thus its use is warranted as the first priority after
stabilizing the patient.

Central cord syndrome


Central cord syndrome is characterized by burning pain and paralysis in upper extremities with
relative sparing of lower extremities.
It is commonly seen in elderly secondary to forced hyperextension type of injury to the neck
particularly in patients with degenerative disease (e.g. spondylosis).

391
Brown Squard syndrome
Brown Squard syndrome is acute hemisection of cord.
It is characterized by ipsilateral motor and proprioception loss and contralateral pain loss below the
level of lesion [TWO LEVELS BELOW THE LESION].

Ongoing ischemia
Paresthesias, change in temperature, and non-detectable radial arterial pulsations.
Immediate anticoagulation with heparin and surgical intervention (i.e., embolectomy) are crucial,
because this condition can lead to tissue death and amputation if not effectively treated within hours.

Transient ischemic attack (TIA)


It typically presents as acute, focal neurological deficits (e.g., right-sided weakness, expressive
dysphasia) that completely resolve within 24 hours.
There are essentially 3 underlying pathophysiological mechanisms of TIA:
1. Blood vessel abnormality (e.g., atherosclerosis, inflammation, arterial dissection, developmental
malformation, venous thrombosis)
2. Embolic source (e.g., heart, extracranial artery)
3. Inadequate cerebral blood flow (due to decreased perfusion pressure or increased blood viscosity)

Normal CT scan is also consistent with the diagnosis, since ischemic strokes are not always visible on the
CT scan within the first 24 hours of presentation.
In elderly patients, the most common causes of TIAs are atherosclerosis and emboli.
In younger patients, atherosclerosis is less likely, and one should consider other causes such as emboli,
vasculitis, dissection, malformations and hypercoagulable states. To identify the underlying cause,
transthoracic echocardiogram can be performed. A normal echocardiogram warrants further work-up
to check for possible hypercoagulable conditions (e.g., Factor V Leiden mutation, prothrombin gene
mutation, hyperhomocysteinemia, lupus anticoagulant, etc.).

There are three subtypes of TIA:


1. Embolic (artery-artery or cardioaortic)
2. Large vessel atherothrombotic (large artery low flow TIA)
3. Small vessel atherothrombotic (lacunar low flow TIA)

Normal MRI or CT angiography makes the diagnosis of large vessel low flow TIA unlikely.
All patients with atherothrombotic TIA should receive an antiplatelet agent if there is no contraindication
to its use.
Aspirin is the initial agent of choice for this purpose, Clopidogrel (2nd line) and Ticlopidine (3rd line).
Aspirin is used in combination with dipyridamole when the patient still develops a TIA despite being on
aspirin.
Anticoagulation is considered when the TIA was caused by emboli from the heart.

392
Note: Hemorrhage is an important cause of strokes. Symptoms almost never resolve within 24 hours, and
CT scan usually shows a hyperdense lesion. Be able to differentiate TIA from hemorrhagic strokes.

Cerebrovascular accident
Cerebrovascular disease is responsible for 200,000 deaths each year in the United States, and occurs most
commonly in the middle and late years of life. The etiopathology can either be ischemic (85%) or
hemorrhagic (15%).

Atherothrombotic strokes occur at rest and have a gradual onset. Often, the patient experiences
successive strokes with increasing frequency. Atherosclerotic risk factors (age, obesity, long history of
hypertension, and hypercholesterolemia) are frequently present in such patients. The neurologic exam
reveals hemiplegia of the pyramidal type (brisk deep tendon reflexes and Babinski sign), which is
suggestive of an upper motor neuron lesion due to obstruction of a major cerebral artery. The most
common site of obstruction is the lateral striate arteries (arteries of stroke), which are the penetrating
branches of the middle cerebral artery. These supply the internal capsule, the caudate nucleus, putamen
and globus pallidus. The asymmetrical pupils are due to the increased intracranial pressure, which
resulted from ischemia and subsequent cerebral edema.

Cardioembolic strokes have a more sudden onset and often involve a large cerebral artery. The most
common site of obstruction is the lateral striate arteries (arteries of stroke), which are the penetrating
branches of the middle cerebral artery. These supply the internal capsule, the caudate nucleus, putamen
and globus pallidus.

A hemorrhagic stroke usually occurs while the person is awake or under stress. It is less common
than ischemic cerebrovascular accidents, and is most commonly caused by a hypertensive crisis. Other
causes include transformation of a cardioembolic stroke, arteriovenous malformation, amyloid
angiopathy and coagulation disorders.The most commonly affected structures are the basal ganglia. The
associated neurologic symptoms result from the mass effect of the extravasated blood.

Cerebral emboli are one of the most dreaded consequences of endocarditis. They occur when pieces of
infected valvular vegetations break off and enter the CNS circulation. Treatment with sntibiotics.

The blood supply of the brain can be explained as follows:


1. Anterior vasculature - comprised of the internal carotid artery and its branches, especially the
paired anterior and middle cerebral arteries.
2. Posterior circulation - comprised of the paired vertebral arteries, which unite to form the basilar
artery, which further divides into the paired posterior cerebral arteries.

The most common site of atherosclerosis is the origin of the internal carotid artery.

393
Internal carotid artery occlusion is most commonly manifested as ocular disturbances (transient mono-
ocular blindness or amaurosis fugax) and ischemia in the middle cerebral artery territory.
It is important to stress that atherosclerosis is a common form of carotid artery disease. The risk of TIA
and stroke is related to the degree of carotid artery stenosis. The most common site of atherosclerosis is
the origin of the internal carotid artery (most severe in the first 2 cm).
The internal carotid artery (ICA) supplies blood to the ipsilateral brain via the anterior and middle
cerebral arteries. It also perfuses the retina and optic nerve via the ophthalmic artery. The sudden and
transient, painless, monocular loss of vision is called amaurosis fugax, and is due to transient embolic
occlusion of the retinal artery.
A combination of amaurosis fugax and ischemia in the left middle cerebral artery territory is therefore
highly suggestive of left internal carotid artery occlusion. Many times, however, it might not be possible
clinically to distinguish between MCA and ICA occlusion.

Amaurosis fugax is characterized by visual loss that is usually monocular, transient, and described as
'like a curtain falling down.' Ophthalmoscopy reveals zones of whitened, edematous retina following
the distribution of the retinal arterioles. Non-invasive evaluation of the carotids is useful in providing
information regarding the degree of carotid artery stenosis.

Middle cerebral artery (MCA): stroke is characterized by contralateral motor and/or sensory
deficits (more pronounced in the upper limb than lower limb) and homonymous hemianopia. If the
dominant lobe (left) is involved, the patient may have aphasia; whereas, if the non-dominant lobe (right)
is involved, the patient may have neglect and/or anosognosia.

Anterior cerebral artery (ACA): contralateral weakness and sensory loss in the leg more than
upper extremities. Other features that may be seen include urinary incontinence, primitive reflexes (e.g.,
grasp and sucking), personality changes, confusion, & foot drop.

Posterior cerebral artery (PCA): Homonymous hemianopsia, visual hallucinations, memory deficits.

Occlusion of the penetrating branches of PCA CN III palsy with contralateral hemiplegia (Weber's
syndrome) or CN III palsy with contralateral dyslexia/alexia (Benedikt's syndrome).

Basilar artery: Coma, locked-in syndrome, cranial nerve palsies, apnea, visual symptoms, drop
attacks, dysphagia.
Locked-in syndrome: occlusion of paramedian branches of basilar artery presenting as quadriparesis
with intact vertical eye movements.

394
The following is a summary of the abovementioned. (Memorize this!)
Lesion Manifestation
Middle cerebral artery Contralateral hemiplegia, conjugate eye deviation toward side of infarct,
occlusion hemianesthesia, homonymous hemianopia, *aphasia (dominant hemisphere),
*hemineglect (non-dominant hemisphere)
Anterior cerebral artery Contralateral weakness that predominantly affects the lower extremity,
occlusion abulia, akinetic mutism, emotional disturbances, deviation of head and eyes
toward the lesion, sphincter incontinence.
Vertebrobasilar system 'Alternate' syndromes, with contralateral hemiplegia and ipsilateral cranial
lesion(*supplying the nerve involvement
brain stem)
Posterior limb of internal Motor impairment without any higher cortical dysfunction and visual field
capsule (lacunar infarct) abnormalities

Lacunar stroke: Pure motor or sensory stroke, dysarthriaclumsy hand syndrome, ataxic hemiparesis.
Always suspect lacunar stroke if a patient presents with a limited neurologic deficit.
The lacunar stroke scenarios are pure motor stroke, pure sensory stroke (thalamus), ataxic-
hemiparesis, and dysarthria-clumsy hand syndrome.
Lacunar strokes are due to microatheroma and lipohyalinosis in small penetrating arteries of the brain.
The principal cause of lacunar stroke is hypertension then diabetes.

Memorize this table with the 4 common lacunar syndromes:


Syndrome Pathology Presentation
Pure motor Lacunar infarction in the Unilateral motor deficit (face, arm, lesser
hemiparesis posterior limb of the internal extent, leg); mild dysarthria, sensory, visual
capsule
Pure sensory Stroke in the Unilateral numbness, paresthesia,
stroke ventroposterolateral nucleus of hemisensory deficit involving the face,
the thalamus trunk, and leg.
Ataxic-hemiparesis Lacunar infarction in the Weakness that is more prominent in lower
posterior limb of the internal extremity, along with ipsilateral
capsule and leg incoordination
Dysarthria-clumsy Lacunar stroke at the basis Hand weakness, mild motor aphasia, sensory
hand syndrome pontis abnormalities

Thalamic stroke (Dejerine-Roussy syndrome): caused by a stroke involving ventral postero-lateral


(VPL) nucleus of the thalamus, which transmits sensory information from the contralateral side of the
body. Hemi-sensory loss with severe dysesthesia (numbness and tingling burning feeling) of the
affected area is typical for a thalamic stroke.

395
CT scan without contrast should be performed in all patients who present with signs and symptoms of
stroke, to differentiate ischemic from hemorrhagic stroke.
If there is evidence of ischemic stroke, carotid Doppler and TEE are performed to evaluate the possible
source of embolism.
MRI: to identify early ischemic changes (e.g., diffusion-weighted MRI is specific for acute stroke).
ECG and an echocardiogram if embolic stroke is suspected.
Vascular studies: For extracranial disease (carotid ultrasound, MRA, or traditional angiography) and for
intracranial disease (transcranial Doppler or MRA).
Screen for hypercoagulable states with a history of bleeding, first stroke, or patients < 50 years of age.

If a patient presents within 3 hours after the onset of an ischemic stroke,


thrombolytic therapy with tPA (after CT scan) should be started (Patients
must first be screened for contraindications).
The use of streptokinase in stroke patients has not shown any benefits. The
drug actually increases the risk of bleeding. The FDA currently recommends
only tPA in the management of stroke patients.
Aggressive blood pressure control is not advised as this may impair the
cerebral autoregulation and worsen the neurological outcome. Treatment is
indicated when the systolic blood pressure is greater than 220mm Hg or
diastolic blood pressure is greater than 130 mm Hg. HTN should be reduced
over a period of several hours. The preferred drugs are the calcium channel
blockers (e.g., nicardipine) and ACE inhibitors.
Maintain systolic BP at 20 mmHg above the patients normal BP.
Aspirin: Associated with morbidity and mortality in acute ischemicstroke
presenting 48 hours from onset.
Prevention and long-term treatment:
o Aspirin, clopidogrel: If stroke is 2 to small vessel disease or thrombosis or
if anticoagulation is contraindicated.
o Carotid endarterectomy: If stenosis is > 70% in symptomatic patients or > 60% in asymptomatic
patients (contraindicated in 100% occlusion).
Anticoagulation: In cases of emboli, new AF, or hypercoagulable states (target INR = 23)

Fibrinolytic therapy improves neurologic outcomes in patients with ischemic strokes when given
within 3 hours of symptom onset. Before administrating tPA, a non-contrast head CT scan should be
performed to rule out hemorrhagic stroke and the patient should be screened for other contraindications
for therapy.

Complete occlusion (100 % stenosis) of the carotid artery is a contraindication to surgery.


Surgery (carotid endarterectomy) is the best treatment for an irregular atherosclerotic lesion in the
carotids with blockage of 60% or greater and is of great benefit in preventing a stroke.
When the lesion is 40%, the disease can be followed every 6-12 months with duplex ultrasound.

396
Brainstem Infarction
Brainstem lesions typically involve the cranial nerves and sensory loss of one half of the face and the
contralateral half of the body.
1. Medial medullary syndrome: results from Vertebral artery occlusion. As a result, the patient will
demonstrate contralateral paralysis of the arm and leg, contralateral loss of tactile, vibratory, and
position sense, and tongue deviation to the injured side.
2. Lateral midpontine syndrome: results from lesion of the lateral pons can cause the, which results in
impaired sensory and motor function of CN V (the trigeminal nerve) with accompanying limb ataxia.
3. Medial midpontine syndrome: an ischemic lesion of the medial pons. Common characteristics
include ipsilateral limb ataxia and contralateral eye deviation and paralysis of the face, arm, and leg.
Impairment of touch and position sense is variable.
4. Wallenberg syndrome: occlusion of posterior inferior cerebellar artery lesion of the lateral
medulla, presenting as an ipsilateral Horner syndrome; loss of pain and temperature sensation of the
face; weakness of the palate, pharynx, and vocal cords; and cerebellar ataxia. Loss of pain and
temperature sensation on the contralateral side of the body.

Hypertensive CVA
Hypertension is the most important risk factor for an intraparenchymal brain hemorrhage.
In elderly patients with multiple medications, it is very important to regularly assess their general well-
being and current medication list, as well as to promote patient adherence to the appropriate regimen.
Hemorrhages are seen as hyperdense areas on CT scan, while infarcts characteristically have hypodense
parenchymal areas on CT scan.

Always suspect parenchymal brain hemorrhage whenever a patient presents with a history of
uncontrolled systemic hypertension and focal neurological signs.

Putamen hemorrhage
The most common site of hypertensive hemorrhage is the putamen (35%).
The internal capsule lies adjacent to the putamen and is almost always involved, thereby leading to
hemiparesis.
Other neurological signs include hemisensory loss, homonymous hemianopsia, stupor and coma.
The eyes are deviated away from the paralytic side.

Cerebellar hemorrhage
Cerebellar hemorrhage accounts for 16% of cases of hypertensive intraparenchymal hemorrhages.
The typical presentation is patient having ataxia, vomiting, occipital headaches, gaze palsy, and facial
weakness.
There is no hemiparesis.

397
Pontine hemorrhage
Pontine hemorrhage accounts for 5-12% of cases of hypertensive intraparenchymal hemorrhages.
Patients present with a deep coma and paraplegia that developed within a few minutes.
The pupils are pinpoint and reactive to light. There are no horizontal eye movements.
There is decerebrate rigidity.

Subarachnoid hemorrhage
AVM is the most common of the subarachnoid hemorrhage in children. The history of seizures and
migraine-like headaches is characteristic.
Patients with subarachnoid hemorrhage have a sudden, dramatic onset of severe headache.
There are usually no focal neurological signs.
The most common causes are saccular aneurysm and vascular malformations.
Vasospasm (with symptomatic ischemia and infarction) is the major cause of morbidity and mortality in
patients with subarachnoid hemorrhage (SAH). In affected patients, the signs of ischemia usually appear
in about 7 days after the SAH. Calcium channel blockers (e.g., nimodipine) are used to prevent
vasospasm in patients with SAH.

Subarachnoid hemorrhage (SAH) can be caused by an intraventricular


hemorrhage, which is common in premature infants. Accumulation of the
blood in the subarachnoid space may lead to destruction of the arachnoid villi
and cisterns, thereby blocking the flow or decreasing the absorption of CSF,
and leading to communicating hydrocephalus.
SAH is the most common cause of communicating hydrocephalus.

The most common cause of a subarachnoid hemorrhage in children is arteriovenous malformation


(AVM) with rupture into the subarachnoid space. AVM typically produces seizures and migraine-like
headaches. Therefore, the history of seizures helps in making the correct diagnosis.

Hyponatremia is one of the important complications of subarachnoid hemorrhage "Cerebral salt-wasting


syndrome".
SIADH is also commonly seen in patients with intracranial hemorrhage.

Vasospasm following subarachnoid hemorrhage (SAH), occur in approximately 30% of patients, and
is the major cause of morbidity and mortality in such patients. In affected patients, the signs of ischemia
usually appear in about 7 days after the SAH. Calcium channel blockers (e.g., nimodipine) are used to
prevent vasospasm in patients with SAH.

398
Intraventricular hemorrhage (IVH)
Results from bleeding in the germinal matrix, and occurs predominantly in premature and low-birth-
weight (LBW) infants.
Patients may present with pallor, cyanosis, hypotension, seizures, focal neurologic signs, bulging or
tense fontanel, apnea and bradycardia; however, many cases remain asymptomatic, thus mandating
transfontanel ultrasound for all newborns with predisposing risk factors.

Cerebral hemorrhage due to excess anticoagulation


The risk of bleeding increases with an increase in the INR.
Correction of excess anticoagulation is dependent upon the INR value and the presence of clinically
significant bleeding.

INR <5, no significant bleeding omit next warfarin dose


INR 5-9, no significant bleeding stop warfarin temporarily
INR > 9 stop warfarin, give oral vitamin K

Patients with serious intracranial bleeding should have rapid correction of excess anticoagulation, with a
target INR of less than 1.5. Fresh-frozen plasma (FFP) reverses the action of warfarin, works
immediately, and lasts for a few hours; therefore, FFP should be administered emergently. In addition,
emergency neurosurgical evaluation should be done for decompression of the increased intracranial
pressure.
Vitamin K reverses the action of warfarin, but takes 8-12 hours to be effective. In clinical practice,
patients with significant bleeding secondary to anticoagulation are given both fresh frozen plasma and
intravenous vitamin K.
For the USMLE, however, since fresh frozen plasma works immediately, its administration is the most
appropriate next step in the management of such patients.

Subdural hematoma
Subdural hematoma will show as a lenticular hematoma on CT scan.
Bleeding is due to tearing of bridging veins between cortex and venous sinuses.
Treatment of acute subdural hematoma is essentially conservative if no midline shift is present on CT
scan. Treatment is centered on prevention of intracranial hypertension by head elevation,
hyperventilation and if needed, use of acetazolamide or mannitol.
Craniotomy is indicated in acute subdural hematoma with midline shift of structures on CT scan but is
associated with grave prognosis.

399
Epidural hematoma
Epidural hematoma will show as a biconvex hematoma on CT scan.
Bleeding is due to middle meningeal artery injury due to head trauma. Because it is under arterial
pressure, it expands rapidly. Fluid resuscitation increases the rate at which the hematoma expands,
thereby precipitating neurological signs.
Focal neurological signs result from herniation of the parahippocampal uncus through the tentorial
incisure, which causes pressure on the ipsilateral oculomotor nerve and posterior cerebral artery as
well as compression of the ipsilateral cerebral peduncle against the edge of the tentorium.
Immediate surgical intervention for an epidural hematoma.

Transtentorial (uncal) herniation


Lesion Signs
Compression of the contralateral crus cerebri Ipsilateral hemiparesis
against the edge of the tentorium
Compression of the ipsilateral oculomotor Loss of parasympathetic innervation causes
nerve (CN III) by the herniated uncus mydriasis (occurs early)
Loss of motor innervation causes ptosis and the
eye looking down and out, due to unopposed
actions of the lateral rectus [trochlear IV] and
superior oblique muscles [abducent CN VI)].
(occurs later).
Compression of the ipsilateral posterior Contralateral homonymous hemianopia
cerebral artery by the herniated uncus
Compression of the reticular formation Altered level of consciousness, coma

400
Cavernous sinus thrombosis
The cavernous sinus is the most common dural sinus to be infected and thrombosed.
Facial infections such as nasal furuncles and dental infections are the most common source of primary
infection by hematogenous spread.
Presents with severe headache, followed by fever and periorbital edema, diplopia, exophthalmus,
chemosis & visual loss.
Funduscopy reveals papilledema and dilated tortuous retinal veins.
Nasal discharge and blood should be cultured.
CT scans of the cavernous and air sinuses, orbit, and brain should be performed.
Treatment with high-dose IV antibiotics, nafcillin or cefuroxime should be started, pending culture
results.
Surgical drainage of the infected air sinus may be indicated, especially if there is no response to the
antibiotics in 24 h.
The prognosis is grave; the mortality rate remains about 30%, despite antibiotic therapy.

It is often difficult to differentiate cavernous sinus thrombosis (CST) from orbital cellulitis; however, the
symptoms of CST are bilateral, and there is involvement of cranial nerves III (ptosis) and V, as well as
early visual loss.

Bells palsy
The absence of forehead furrows indicates Bell's palsy (peripheral seventh nerve palsy), and rules out
the diagnosis of a central facial paresis.
Patients with central lesions still have forehead furrows because the contralateral motor innervation of the
forehead remains intact.

Dominant hemisphere is responsible for speech, language and calculation.


Posterior part of dominant frontal lobe contains motor speech area of Broca and damage to this area
results in expressive aphasia in which patient can't articulate speech or write normally but can
comprehend written and spoken language.
Frontal lobe also contains motor cortex and premotor cortex and damage to these areas result in
contralateral paresis.

Sensory cortex is present in the parietal lobe and damage to sensory cortex results in contralateral
hemianesthesia.

Unilateral lesion of occipital lobe causes contralateral hemianopia.

401
Brocas aphasia
BROcas is BROken speech.
Motor (nonfluent/expressive) aphasia with good comprehension
Posterior part of dominant frontal lobe contains motor speech area of Broca and damage to this area
results in expressive aphasia in which patient can't articulate speech or write normally but can
comprehend written and spoken language.

Wernickes aphasia
Wernickes is Wordy but makes no sense.
Sensory (fluent/receptive) aphasia with poor comprehension

Hemi-neglect syndrome is characterized by patients ignore the left side of a space and respond only to
the stimuli coming only from the right side. It is caused by lesion of the right (non-dominant) parietal
lobe, which is responsible for spatial organization.

Medial medullary syndrome: is caused by occlusion of the vertebral artery or one of its branches
and is typically associated with contralateral spastic hemiplegia, contralateral vibratory and
proprioception loss, and tongue deviation to the injured side.

Lateral medullary syndrome may result in well-known abnormalities such as Wallenberg syndrome.
This syndrome typically involves an ipsilateral Horner syndrome; loss of pain and temperature
sensation of the face; weakness of the palate, pharynx, and vocal cords; and cerebellar ataxia. There is
also loss of pain and temperature sensation on the contralateral side of the body.

402
Seizures and epilepsy

Complex partial *Remember the automatisms*


Complex partial seizures typically involve the temporal lobe.
A complex partial seizure is characterized by brief (i.e., lasting for a few minutes) episodes of impaired
consciousness, failure to respond to various stimuli during the episode, staring spells, automatisms (e.g.,
lip smacking, swallowing, picking movements of the hand), and post-ictal confusion.
The EEG pattern is usually normal or may show brief discharges.
Hyperventilation during the EEG can not simulate complex partial seizures.
Rule out a mass by MRI or CT without contrast.
CT scan of the head without contrast is the initial diagnostic test of choice when a patient presents with
impaired consciousness or seizures or focal neurologic signs.
Phenytoin or Carbamazepine are considered first-line therapy.
In children, phenobarbital is the first-line anticonvulsant.

Absence (petit mal) seizures


Petit mal seizures are characterized by a sudden cessation of mental activity.
An episode is very short, but may occur repeatedly throughout the day.
Suspect absence seizures in a 4 8 year old child with frequent daydreaming episodes, brief staring
spells, decline in school performance, without alteration of consciousness and no post-ictal phase.
There are no associated complex automatisms or tonic-clonic activity.
The diagnosis is best confirmed by EEG studies.
Hyperventilation during the EEG reveals a generalized 3Hz spike-and-wave pattern on a normal
background.
Rule out a mass by MRI or CT with contrast.
The preferred treatment is ethosuximide or valporic acid.

Atypical absence seizure lasts longer. The characteristic EEG pattern is slow spike-and-wave activity
with a frequency less than 2.5Hz.

NOTE: Todd's palsy is characterized by transient hemiplegia that occurs after a seizure.

403
Generalized (grand mal) seizures
Primarily idiopathic. Partial seizures can evolve into secondarily generalized tonic-clonic seizures.
Sudden onset of tonic contractions of muscles throughout the body [extension of the back and
extremities, continuing with 12 minutes of repetitive, symmetric clonic movements] followed by
intermittent relaxation of various muscle groups.
Marked by incontinence and tongue biting.
Patients may appear cyanotic during the ictal period.
Consciousness is slowly regained in the postictal period; muscle aches and headaches may be present.
EEG typically shows 10-Hz activity during the tonic phase and slow waves during the clonic phase.
1 generalized tonic-clonic seizures: valporic acid is the first-line therapy, Lamotrigine is the 2nd-line
therapy.
Secondarily generalized tonic-clonic seizures: Same as for partial seizures.
Valporic acid works by increasing the availability of GABA. Side effects include ataxia, tremor,
hepatotoxicity, thrombocytopenia, GI irritation and hyponatremia.
Lamotrigine works by decreasing glutamate release. Side effects include diplopia, ataxia, rash and
Stevens-Johnson syndrome.

Infantile Spasms (West Syndrome)


Affects males more often than females; the initial event occurs between 3 and 12 months of age.
Associated with a family history.
Tonic, bilateral, symmetric head jerks that tend to occur in clusters of 510; arrest of psychomotor
development at the age of seizure onset. The majority of patients have mental retardation.
Abnormal interictal EEG (very high amplitude slow waves).
Hormonal therapy with ACTH, prednisone, and clonazepam or valproic acid.

Status Epilepticus
A medical emergency consisting of prolonged (> 30-minute) or repetitive seizures without a return to
baseline consciousness.
Common causes include anticonvulsant withdrawal/noncompliance, anoxic brain injury, EtOH/sedative
withdrawal or other drug intoxication, metabolic disturbances (e.g., hyponatremia), trauma, and infection.
Determine the underlying cause with pulse oximetry, CBC, electrolytes, calcium, glucose, ABGs, LFTs,
BUN/creatinine, ESR, and toxicology screen.
Defer EEG and brain imaging until the patient is stabilized.
Perform LP in the setting of fever or meningeal signs (only after having done a CT scan to ensure the
safety of the LP).
Maintain ABCs; consider rapid intubation for airway protection.
Administer IV benzodiazepine (lorazepam or diazepam) plus a loading dose of phenytoin.
If seizures continue, intubate and load with an IV sedative (midazolam or pentobarbital) and initiate
continuous EEG. monitoring.

404
Glucose, thiamine, and naloxone may also be given to presumptively treat other potential etiologies.

Vertigo
Vertigo is a sensation of excessive motion compared to physical reality.
It is most commonly due to dysfunction with the vestibular labyrinth system (inner ear).

Dizziness
Dizziness can be classified as vertigo, presyncope or disequilibium.
Severe spinning sensation accompanied by nausea is most characteristic of vertigo.
Next step is to classify vertigo as either central or peripheral.
Peripheral vertigo tends to last for shorter intervals than central vertigo.
Symptoms of ear-fullness suggests a peripheral vertigo.
Meniere's disease is the likely cause of vertigo if the patient also has a sensation of ear fullness.

Tinnitus
Tinnitus or ringing of the ears can occur in various situations.
It can sometimes occur in patients taking aspirin and quinine.
For decades, quinine has been used for treating cramps in dialysis patients. Interestingly, only one
double-blind placebo controlled trial of nine hemodialysis patients has shown its beneficial affect.
Tinnitus can also occur in patients who are depressed.
It may be a feature of Meniere's disease and acoustic neuroma.
It can cause disruption of sleep, concentration and depression.

Aminoglycosides can cause nephrotoxicity and ototoxicity.


Gentamicin sometimes causes severe vestibulotoxicity, resulting in potentially permanent vertigo and
ataxia (gait imbalance).
Aspirin usually causes tinnitus; but in very higher doses it can cause (6 to 8 grams/day) hearing loss
Furosemide is a loop diuretic that can cause ototoxicity (hearing loss and/or tinnitus).

405
Menieres disease
Meniere's disease is a disorder of unclear etiology resulting from distension of the endolyphatmic
compartment of the inner ear. It may be unilateral or bilateral.
Meniere's presents with recurrent episodes of rotational vertigo (last for 20 minutes 24 hours), low-
frequency, sensorineural hearing loss, tinnitus, and aural fullness.
It may be accompanied by vomiting and postural instability.
Nystagmus may be seen during an acute attack.
It is more common in females and occurs in patients with syphilis, or following head trauma.
The diagnosis is made clinically.
It may resolve spontaneously and treatment today is empirical.
First line therapy consists of environmental and dietary modifications, including maintenance of a low-
salt diet (2 3 grams of sodium/day).
Triggers that increase the endolymphatic retention should be avoided, including alcohol, caffeine,
nicotine, and foods high in salt.
Symptoms other than hearing loss improve in the majority of patients without treatment.

Labyrinthitis
Sudden onset of severe vertigo
Frequently follows an upper respiratory tract infection.
It is treated symptomatically by meclizine and when symptoms are severe, diazepam.

Benign Paroxysmal Positional Vertigo (BPPV)


A common form of peripheral vertigo, occurs when calcium crystals within the inner ear shift position
(labyrinth dysfunction).
BPPV is not usually associated with hearing loss or tinnitus.
Transient, episodic vertigo (lasting < 1 minute) and nystagmus triggered by changes in head position
(classically while turning in bed or getting out of bed), together with nausea and vomiting.
Dix-Hallpike maneuverhave the patient go from a sitting to a supine position while quickly turning the
head to the side. If vertigo and/or nystagmus is reproduced, BPPV is the likely diagnosis.
Usually subsides spontaneously in weeks to months. Repositioning exercises or the Epley maneuver (the
reverse of Dix-Hallpike) may be beneficial.
BPPV is treated with positional maneuvers that attempt to move otolith that causes disturbances, out of
the circular canals.

Perilymphatic fistula
Vertigo related to head trauma.

406
Syncope
Generally, patients with syncope will not complain of post-ictal symptoms.
The most common pathophysiologic basis for syncope is an acute decrease in cerebral blood flow (with
resultant cerebral hypoxemia) secondary to decreased cardiac output; arrhythmias, including conduction
abnormalities, are the most frequent cause.
Clonic jerks may occur during any syncope if it is prolonged and are due to brain hypoxia.

Exertional (effort) syncope suggests cardiac outflow obstruction, mainly due to aortic stenosis.

Syncope of cardiac etiology typically begins and ends suddenly and spontaneously. It is most
commonly due to an arrhythmia.

Situational syncope should be considered in the differential diagnosis of syncopal episodes.


The typical scenario would include a middle age or older male, who loses his consciousness immediately
after urination, or a man who loses his consciousness during coughing fits
The pathophysiologic mechanism underlying the situational syncope includes autonomic dysregulation,
which can be partially explained by straining and rapid bladder emptying. Cardioinhibitory and/or
vasodepressor mechanisms may be involved.

Vasovagal syncope, a common fainting spell, is usually precipitated by emotional reaction and is
preceded by presyncopal dizziness, weakness and nausea. Usually occurs in the upright posture, and is
often preceded by vagal-mediated warning symptoms.

Syncope due to seizures is abrupt in onset and is associated with muscular jerking or convulsions,
incontinence, and tongue biting.

Syncope due to pulmonary embolism usually indicates massive pulmonary vascular obstruction
and is often associated with dyspnea, tachypnea, chest discomfort, cyanosis, and hypotension.

Syncope of gradual onset (with warning symptoms) and slow clearing suggests metabolic changes,
e.g. hypoglycemia or hypocapnia of hyperventilation.

407
Migraine headache
Prophylactic treatment include; beta blockers, calcium channel blockers, valporic acid
Drugs used to abort acute attacks of migraine: simple analgesics (e.g., acetaminophen), NSAIDs, ergot
derivatives and triptans.
Simple analgesics are always given first.
Serotonin agonists (triptans) to abort acute attacks of migraine when simple analgesics or NSAIDs fail.
Triptans are generally preferred over ergotamine because it has fewer side effects and greater efficacy
than ergotamine (pregnancy test should be performed in women of child-bearing age before starting
treatment with triptans).
Ergotamine derivatives are considered when acute attacks of migraine last for longer than 48 hours and
are frequently recurrent.
Metoclopramide (dopamine agonist) can be given acutely as oral formulations to aid in the absorption of
other abortive medications

Contraindications for serotonin agonists (e.g. sumatriptan):


1) Familial hemiplegic migraine
2) Basilar migraine
3) Uncontrolled hypertension
4) Ischemic stroke
5) Coronary artery disease
6) Prinzmetal angina
7) Pregnancy

NOTE: headache + nuchal rigidity + NO FEVER = intracranial hemorrhage

Cluster headache
It generally starts as retro-orbital pain and later spreads to the hemicranium and frequently awaken the
patient from sleep.
Associated with ipsilateral lacrimation, rhinorrhea, red eye, stuffy nose, pallor or even Horner's
syndrome.
Fundamental difference in treatment of cluster headache is the use of oxygen, steroids and lithium.
100% oxygen (treatment of choice) is an effective and rapid method to abort an acute attack of cluster
headache.

Tension headache
Band-like headaches that occur bilaterally.
Associated with tightness of the posterior neck muscles.
It may persist for several days.
Treatment with acetaminophen, NSAIDs and muscle relaxants.

408
TEMPORAL ARTERITIS GIANT CELL ARTERITIS
Giant cell arteritis should always be suspected in a patient older than 50 years who presents with
headache and pain in one or both temples, visual problems (decreased vision or blurry vision) and
polymyalgia rheumatica (proximal stiffness in neck, arms, hips).
Other common symptoms: scalp tenderness, and jaw claudication.
Giant cell arteritis chronic aortitis disruption of collagen and elastin Thoracic aortic aneurysm
(such patient need to have continuous monitoring).
Blood samples can be drawn for ESR before the start of glucocorticoid therapy.
Giant cell arteritis must be treated immediately with high dose glucocorticoids once the clinical diagnosis
is made.
One should not wait for temporal artery biopsy before starting glucocorticoid therapy as any delay in the
treatment may result in visual loss.
Temporal artery biopsy can be done within several days after treatment, for confirmation of the disease.

Steroid-induced myopathy is a well described result of long-term corticosteroid use. It is


characterized by painless proximal muscle weakness. Steroid myopathy will improve slowly once the
offending medication is discontinued.

Trigeminal Neuralgia
Carbamazepine is the standard care for treatment.

Benign/idiopathic intracranial hypertension (pseudotumor cerebri)


The pathology involves impaired absorption of CSF by the arachnoid villi.
Oral contraceptive pills have also been associated with this disorder.
Vitamin A and isotretinoin can also predispose.
Suspect pseudotumor cerebri in a young obese female presents with a headache, nausea, vomiting,
neck pain, double vision, visual loss and/or cranial nerve palsies (abducens CN VI palsy) ; that is
suggestive of a brain tumor.
Papilledema on fundoscopy.
When evaluating a patient suspicious for pseudotumor cerebri, do a lumbar puncture only after
completely ruling out a space-occupying brain lesion with neuroimaging (CT/MRI).
Normal neuroimaging. In fact, shrunken ventricles are seen on MRI.
Lumbar puncture shows an elevated CSF pressure.
Blindness is likely to develop if the patient is left untreated
The treatment includes weight reduction and acetazolamide (first line medical treatment).
Shunting or optic nerve sheath fenestration may be performed to prevent blindness.

409
NOTE: Acute ingestion of vitamin A causes anorexia, bulging fontanelle (pseudotumor cerebri),
hyperirritability and vomiting.

Acute closure glaucoma presents as unilateral eye pain, redness, and dilated pupil with poor light
response. It is important to distinguish it from migraine, cluster headache, temporal arteritis, and
keratoconjunctivitis because failure to correctly diagnose this disease can lead to blindness.

Intracranial hypertension
Intracranial hypertension is diagnosed when the intracranial pressure 20 mm Hg.
It may be due to trauma, space-occupying lesion, hydrocephalus, or impaired CNS venous outflow.
Typical symptoms are headache that tends to be worse in the morning, nausea and vomiting early in
the day, vision changes, papilledema, cranial nerve deficits, somnolence, confusion, unsteadiness and
Cushings reflex (hypertension and bradycardia).
Imaging via CT or MRI is important.

Guillain-Barr syndrome (acute idiopathic polyneuropathy)


Classic presentation includes ascending paralysis, areflexia and sensory changes 3-4 weeks after an
upper respiratory tract infection or gastroenteritis (antecedent history of infection).
Campylobacter jejuni is the most frequent precipitant of Guillain-Barre Syndrome (GBS)
CSF analysis reveals elevated protein level, normal glucose, normal WBC and normal RBC levels.
CSF finding of albumino-cytologic dissociation (elevated protein despite normal cell count) is a fairly
specific feature.
EMG is used to detect demyelination of the peripheral nerves (slowed nerve conduction velocities).
Treatment includes IV immunoglobulin and plasmapheresis.
It is extremely important to monitor the vital capacity in patients with GBS and initiate early respiratory
support to prevent death from respiratory failure.
The best way to monitor respiratory function in such cases is through serial measurements of bedside
vital capacity.

Tick-borne paralysis
Patients usually present with progressive ascending paralysis over hours to days.
Fever is typically not present; hence, a history of fever or prodromal illness makes the diagnosis unlikely.
Sensation is usually normal.
The CSF examination is typically normal.
Meticulous search and removal of the tick usually result in improvement within an hour and complete
recovery after several days.

410
Myasthenia gravis
The basic pathology lies at the neuromuscular junction (that why called myasthenia) and is mediated
by auto-antibodies against post-synaptic acetylcholine receptors.
Myasthenia gravis presents with easy fatigability and weakness that improves with rest.
Resolution of muscular weakness with rest is a hallmark feature of myasthenia gravis.
Bilateral Ptosis is typically seen in myasthenia gravis.
Extra-ocular muscles are most commonly affected.
Aminoglycosides may exacerbate myasthenia gravis.
CT of the chest should be done to look for a thymoma in all newly-diagnosed myasthenia gravis
patients.
EMG shows decrease in muscle fiber contraction on repetitive nerve stimulation.
Oral anticholinesterase (Pyridostigmine or neostigmine) is usually the initial treatment of choice for
myasthenia gravis.
Thymectomy may induce remission in patients with myasthenia gravis, and should be considered in all
patients who are between puberty and 60, and in those with disease that is not confined to the extraocular
muscles.
Treatment of Myasthenia crisis consists of endotracheal intubation and withdrawal of
anticholinesterases for several days. Plasmapheresis can also be used in myasthenia crisis

Lambert-Eaton or Myasthenic syndrome


Myasthenic or Lambert-Eaton syndrome can occur in association with small cell carcinoma of the lung.
Lambert-Eaton syndrome is caused by autoantibodies that are directed against the voltage-gated
calcium channels in the pre-synaptic motor nerve terminal. This leads to the defective release of
acetylcholine, thereby leading to proximal muscle weakness.
Electrophysiological studies confirm the diagnosis (the muscle response to motor nerve stimulation
should increase with repetitive stimulation muscular strength improve on repetitive tasks).
Treatment consists of plasmapheresis and immunosuppressive drug therapy.

Amyotrophic lateral sclerosis Lou Gehrigs disease


A fully mental alert patient loses nearly all motor control while still being able to think & perceive.
Almost always progresses to recurrent aspiration pneumonia, respiratory failure and death.
It is a chronic, asymmetric slowly progressive degenerative disease of unknown etiology characterized by
loss of upper and lower motor neurons.
UMNL: weakness with spasticity and hyperreflexia.
LMNL: weakness with muscle wasting and fasciculations.
Pronator drift is a relatively sensitive and specific physical exam finding of UMNL affecting upper
extremities. In particular, many patients with strokes will demonstrate Pronator drift.

411
EMG/nerve conduction studies is the most accurate confirmatory test. reveal widespread denervation
and fibrillation potentials.
Riluzole (glutamate inhibitor. Side effects are dizziness, nausea, weight loss, elevated liver enzymes and
skeletal weakness.)

Multiple sclerosis
Young patient (usually < 50 years) presents with multiple neurologic deficits separated by time and can
not be explained by the presence of CNS lesion.
Internuclear ophthalmoplegia is a pathognomonic finding of multiple sclerosis, and is due to
demyelination of the medial longitudinal fasciculus.
Suspect multiple sclerosis in a young female with bilateral trigeminal neuralgia.
Multiple sclerosis is best diagnosed with MRI of the brain and spine. The MRI shows asymmetric
periventricular white matter, which is very characteristic for multiple sclerosis and increased T2 and
decreased T1 intensity.
CSF findings include oligoclonal bands (in 85-90% of cases), pleocytosis, elevated IgG or myelin basic
protein and mild lymphocytosis.
Interferon-beta decreases the frequency of relapse and reduces disability in patients with the relapsing-
remitting form of MS.
Glatiramer acetate copolymer I is a long-term disease modifying treatment for MS, that works by
modulating T-cell-mediated autoimmunity to myelin basic protein.
Acute exacerbations of multiple sclerosis are generally treated with IV steroids.

412
Dementia
Impairment of daily functioning is essential in distinguishing between dementia and normal changes of
aging. Patients with dementia have functional impairments.
The course is typically chronic and progressive. The most common causes are Alzheimers disease
(50%) and multi-infarct dementia (25%).
Diagnostic criteria include memory impairment and 1 of the following:
Aphasia.
Apraxia.
Agnosia.
Impaired executive function in the presence of a clear sensorium.
Symptoms may worsen at night (sundowning).
Personality, mood, and behavior changes are common.
Insomnia and aggression are common caregiver complaints about the patient.
Mini-mental state examination (MMSE) score of 24 points is suggestive of dementia (total is 30).
Insomnia and aggression often necessitate nursing home placement. Provide environmental cues and a
rigid structure to the patients daily life.
Low-dose antipsychotics may be used for agitation. Avoid benzodiazepines, which may worsen
disinhibition and confusion. Adequate support for the caregiver is imperative.

Alzheimers dementia
Alzheimers disease is the most common cause of dementia in the western world.
A positive family history is the most important risk factor.
Alzheimer's dementia causes a gradually irreversible and progressive cognitive decline (memory
loss) that presents with memory impairment and one or more of the following: apraxia (difficulty in
carrying out activities), aphasia (language dysfunction), agnosia (difficulty in recognizing objects), or
disturbed executive functioning (organizing, planning, abstraction).
There is no impairment of consciousness.
Patient can not remember recent events but can remember past events.
CT findings of diffuse generalized atrophy (cortical & subcortical), flattened sulci, enlarged ventricles
are characteristic.
Histopathological findings: amyloid plaques, neurofibrillary tangles, and selective loss of cholinergic
neurons.
Supportive therapy for the patient and the family.
Cholinesterase inhibitors (donepezil, rivastigmine, tacrine, and galantamine, an NMDA receptor
antagonist) are first-line therapy, to raise the level of acetylcholine in CSF.
Vitamin E (-tocopherol) may also slow cognitive decline.
Memantine (disease-modifying drug) seems to be neuroprotective and reduces the rate of progression.
Death is usually 2 to aspiration pneumonia or other infections.

413
Vascular dementia

Alzheimer's dementia Vascular dementia


Women Men
Older age of onset Younger than Alzheimer
Chromosome 21 Hypertension
Linear or progressive deterioration Stepwise or patchy deterioration
No focal deficits Focal deficits
Supportive treatment Treat underlying condition

Picks disease
Pick's disease is a fronto-temporal dementia. It has a clinical presentation similar to that of Alzheimer's,
though behavioral abnormalities tend to precede the cognitive decline.
Patients may present in their 50s and are more likely to be female.
The typical features include personality change, (e.g., euphoria, disinhibition, apathy), compulsive
behaviors (e.g., dirty language, peculiar eating habits, hyperorality), and impaired memory.
Personality and language changes are often more prominent than cognitive symptoms. Visual-spatial
functions usually remain intact.
Furthermore, the disease progression is also different. There is initially less disorientation and memory
loss, but more personality changes and loss of social restraints. Eventually, patients exhibit profound
dementia, and become mute, immobile, and incontinent.
Pick's disease causes atrophy of the frontal lobes (resulting in altered personality) and atrophy of the
temporal lobes, changes that are clearly evident on head CT and MRI.

CJD dementia
Creutzfeldt-Jakob disease is a fatal neurodegenerative disease (spongiform encephalopathy) caused by
slow virus (prion).
Suspect it in an old patient (between 50- to 70-years-old) with rapidly progressive dementia, and
myoclonus.
Electroencephalogram (EEG) pattern of periodic synchronous bi or triphasic sharp wave complexes is
very typical.
Brain biopsy shows cortical spongiform changes.
There is no specific treatment. Death usually occurs within 12 months.

NOTE: CJD and Kuru are prion diseases caused by ingestion of human brain tissue. However, they are
not associated with cystic brain lesion, but rather have spongioform lesions.

414
Dementia with lewy bodies (DLB)
Dementia with Lewy body (DLB) is characterized by fluctuating cognitive impairment and bizarre,
visual hallucinations.
Parkinsonism is also seen, although there is a poor response to dopaminergic agonist therapy.
The central feature required for diagnosis is progressive, cognitive decline that interferes with normal
social or occupational functions.
Prominent or persistent memory impairment may not necessarily occur in the early stages, but is usually
evident with progression.
Deficits in attention, frontal-subcortical skills and visuospatial ability may be especially prominent.
Two of the following core features are essential for a diagnosis of probable DLB, and one is essential for
possible DLB:
1. Fluctuating cognition with pronounced variations in attention and alertness
2. Recurrent visual hallucinations that are typically well-formed and detailed
3. Spontaneous motor features of Parkinsonism
Features that can support the diagnosis are as follows:
1.Repeated falls
2.Syncope
3.Transient loss of consciousness
4.Neuroleptic sensitivity
5.Systematized delusions

Senile depression (pseudodementia)


Pseudodementia may be seen in the depressed elderly.
Senile depression is characterized by memory impairment, feelings of hopelessness, sadness and
decreased appetite.
The memory loss is moderate and does not interfere with everyday life, and patients usually complain
about their problem.
Patients with pseudodementia seem significantly concerned about their impaired memory; whereas
patients with Alzheimers dementia seem relatively unconcerned.
The CT scan is normal.
Dexamethasone suppression test (DST) is abnormal in up to 50% of patients with depression.
The memory loss reverses with treatment of the depression.
Antidepressants such as SSRIs are the treatment of choice.

415
Normal pressure hydrocephalus NPH
NPH is thought to result from decreased CSF absorption or transient increases in intracranial pressure
that causes permanent ventricular enlargement without chronically increasing intracranial pressure.
It is characterized by the triad of gait disturbance, dementia and urinary incontinence ("wet, wacky,
and wobbly").
Lumbar puncture reveals the normal CSF pressures, and MRI shows the enlarged ventricles.

Typical (e.g. haloperidol) and atypical (e.g. Risperidone) antipsychotics are good for treating acute
agitation in elderly and demented patients. Benzodiazepines are typically not recommended in this
setting.
Haloperidol is the treatment of choice for agitation in elderly.

Hypothyroidism can cause reversible changes in memory and mentation. Cognitive symptoms are
accompanied by systemic changes such as weight gain, fatigue, and constipation.

Delirium
Delirium is an acute confusional state due to organic illness which is commonly superimposed upon
dementia in elderly.
Urinary tract infection and metabolic disturbances are common precipitants in older patients.
The initial work-up of delirium should include urine analysis and measurement of serum electrolytes.
In the absence of any focal neurological signs, even if there is evidence of carotid bruit (Vascular
Dementia) Delirium is the most like Diagnosis.

DELIRIUM DEMENTIA
Level of attention Impaired (fluctuating). Usually alert.
Onset Acute. Gradual.
Course Fluctuating from hour to hour. Progressive deterioration.
Consciousness Clouded. Intact.
Hallucinations Present (often visual or tactile). Usually absent unless disease is very
advanced.
Prognosis Reversible. Irreversible.
Memory Global Remote memory spared
impairment
1
Chlordiazepoxide is the treatment of choice for delirium tremens that is characterized by
disorientation, hallucination, tachycardia, hypertension, and agitation.

416
Huntingtons Disease
It is caused by a defect in an autosomal-dominant gene on chromosome 4.
The pathology involves striatal neurodegeneration involving multiple abnormal CAG triplet repeats
(< 29 is normal). The number of repeats expands insubsequent generations earlier expression and
more severe disease (anticipation). Life expectancy is 20 years from time of diagnosis.
Presents at 3050 years of age with gradual onset of chorea, altered behavior, and dementia (begins as
irritability, moodiness, and antisocial behavior).
Chorea is characterized by sudden, jerky and irregular movements of the extremities. Impaired judgment,
executive function, awareness and attention occur at an early stage.
Memory impairment is a late finding. Depression, irritability and social withdrawal are also common.

CT /MRI: Cerebral atrophy (especially of the caudate nucleus and putamen).


There is no cure, and the disease cannot be halted. Symptomatic treatment only.
Haloperidol for psychosis; reserpine to minimize unwanted movements.
Genetic counseling should be offered to offspring.

WERNICKES ENCEPHALOPATHY VERSUS KORSAKOFF'S SYNDROME


Wernickes encephalopathy Korsakoffs dementia
Cause Thiamine (vitamin B1) deficiency Thiamine (vitamin B1) deficiency
Course Acute Chronic
Signs/Symptoms Classic triad: encephalopathy, Encephalopathy, ophthalmoplegia,
ophthalmoplegia (nystagmus, lateral ataxia, plus amnesia, horizontal
rectus palsy, conjugate-gaze palsy), nystagmus.
ataxia (polyneuropathy, cerebellar and
vestibular dysfunction).
Classic patients Alcoholics, hyperemesis, starvation, Alcoholics, hyperemesis, starvation,
renal dialysis, AIDS. Can be elicited renal dialysis, AIDS.
by large-dose glucose administration.
Reversibility Reversible, almost immediately, with Irreversible
thiamine administration
23
Parenteral thiamine needs to be given before glucose to decrease the risk of Wernicke's and Korsakoff's
syndrome.

417
Parkinson disease
Parkinsonism is caused by overactivity of cholinergic neurons and underactivity of dopaminergic
neurons in the substantia nigra basal ganglia.
Resting tremor is often the presenting symptom of Parkinsons disease.
The Parkinsons tetrad consists of the following:
Bradykinesia: Slowed movements and difficulty initiating movements. Festinating gait (wide leg
stance with short accelerating steps) without arm swing is also seen.
Rigidity: Cogwheeling is due to the combined effects of rigidity and tremor.
Instability Postural: Stooped posture, impaired righting reflexes, freezing, falls.
Tremor Resting (e.g., pill rolling, frequently first manifested in one hand).
The presence of at least two of the above signs on physical examination (diagnosis made clinically) is
ground for a clinical diagnosis of the disease.
A shuffling hypokinetic gait is characteristic of the disease (i.e. the patient appears as if he is chasing
his center of gravity).
Other manifestations include masked facies, memory loss, and micrographia.
Seborrheic dermatitis is characterized by dry scales with underlying erythema on the scalp, central face,
presternal region, interscapular areas, umbilicus and body folds. It may be seen in association with
Parkinsonism. Suspect a HIV infection in a young patient with seborrheic dermatitis.

The most effective symptomatic therapy of Parkinsonism is L-dopa.


Younger patients are usually initially treated with dopamine agonists (bromocriptine).
Older patients are given L-dopa.
L-dopa has beneficial effects on all the features of Parkinsonism, although it does not arrest the
progression of this disorder. Furthermore, L-dopa is associated with a greater risk of dyskinesia than
dopamine agonists.
On the other hand, dopamine agonists (bromocriptine) have slightly less efficacy than L-dopa. Nausea,
vomiting and hypotension are the most frequent early side effects. Other side effects include arrhythmias,
dyskinesia, confusion and restlessness.

Anticholinergics (e.g., benztropine or trihexyphenidyl) improve tremor and rigidity, but do not have
much effect on bradykinesia. Anticholinergics are reserved for patients whose predominant problem is
resting tremor.
Anticholinergics are most useful in patients younger than 70 years with disturbing tremor and minimal
bradykinesia (since these drugs do not have much effect on bradykinesia). These are also used in patients
with advanced disease and tremors that do not improve with L-dopa or dopamine agonist therapy.
When Parkinsonism develops as a side effect from antipsychotic use (within four days to four months
of treatment), the drug of choice is an anticholinergic such as benztropine.
Their use is not encouraged in patients who are elderly, have dementia or do not have tremor.
Anticholinergics are contraindicated with BPH and glaucoma.
Anticholinergics (eg, benztropine or trihexyphenidyl) used for Parkinsons disease can lead to
anticholinergic toxicity (dry skin, dry mouth, constipation, urinary retention, blurred vision, confusion
and may precipitate acute glaucoma headache and retro-orbital pain).

418
Selegiline (an MAO-B inhibitor) may be neuroprotective and may the need for levodopa.

Amantadine is an antiviral agent, and its mechanism of action is unknown. It has relatively lower
efficacy and is used for patients with mild disease who have no disability. It has the advantage of low
toxicity, and it improves all the features of Parkinsonism. Its side effects include confusion,
restlessness, skin rash, depression, edema, nausea, anorexia and postural hypotension; however, these
side effects are uncommon with the usual dose.

Clozapine is an atypical antipsychotic agent, and is used to treat hallucinations in patients of


Parkinsonism.

Parkinsonism + vertical gaze palsy = supranuclear palsy


Parkinsonism + prominent ataxia = olivopontocerebellar atrophy
Parkinsonism + orthostatic hypotension = Shy-Dragger syndrome
Always consider Shy-Dragger syndrome when a patient with Parkinsonism experiences orthostatic
hypotension, impotence, incontinence, abnormal sweating, abnormal salivation or lacrimation,
disturbance of bowel, gastroparesis, or or other autonomic symptoms.

Benign essential tremor


Essential tremor is a diagnosis of exclusion.
An autosomal dominant pattern of transmission may be present.
Tremors become especially apparent in the upper extremities when the arms are outstretched.
It is a persistent, progressive condition that usually begins in adulthood. Most patients are only mildly
affected and do not require treatment.
Patients who require medication are given either beta-blockers (Propranolol) or primidone.
Primidone is an anticonvulsant that converts into phenylethylmalonamide and phenobarbital. The
administration of primidone may precipitate acute intermittent porphyria, which manifests as
abdominal pain, neurologic and psychiatric abnormalities. Acute intermittent porphyria can be diagnosed
by checking for urine porphobilinogen.

Restless leg syndrome


Restless legs syndrome is a disorder characterized by an intense and unpleasant creeping sensation in the
lower extremities that is relieved by moving the legs.
The symptoms most often occur with the onset of sleep.
It can be treated with dopamine agonists (e.g. pramipexole and ropinerole) or levodopa.

419
Dystonia
Dystonia is defined as sustained muscle contraction resulting in twisting, repetitive movements, or
abnormal postures.
It may be focal (affecting only one muscle) or diffuse.
It can be congenital, idiopathic, secondary to trauma or local inflammation, or drug-induced.
Medications commonly responsible include typical antipsychotics, metoclopromide, and
prochlorperazine.
The condition is characterized by muscle spasms or stiffness, tongue protrusions or twisting,
opisthotonus, and oculogyric crisis (a forced, sustained elevation of the eyes in an upward position).
Treatment of acute dystonia is with antihistamines (eg, diphenhydramine) or anticholinergics (eg,
benztropine or trihexyphenidyl).
Patients who develop dystonia from the use of antipsychotics should be treated with benztropine or
diphenhydramine.

Torticollis
Torticollis is a common form of focal dystonia involving the sternocleidomastoid muscle.
It can be congenital, idiopathic, secondary to trauma or local inflammation, or drug-induced.
Discontinuation of the causative agent(s) may improve the symptoms

Intention tremor
It is due to cerebellar dysfunction.
Cerebellum helps co-ordinate movements.
Cerebellar damage leads to movement abnormalities (ipsilateral ataxia, patient tends to fall towards the
side of the lesion, broad-based gait, nystagmus, intention tremor, ipsilateral muscular hypotonia, and
marked difficulty in coordination and performing rapid, alternating movements).
Cerebellar damage may be secondary to tumor, chronic alcohol abuse
Patellar tendon elicits several to-and-fro leg movements.

Cerebellar tumor
Cerebellar tumors usually produce ipsilateral ataxia, nystagmus, intention tremors, ipsilateral muscular
hypotonia and loss of coordination.
Ipsilateral ataxia: patient tends to fall towards the side of the lesion and when asked to stand with feet
together, the patient also tends to sway to the affected side.
Obstruction of the CSF flow by the tumor results in increased intracranial pressure, which presents as
headache, nausea, vomiting, and papilledema.

420
Neurofibromatosis
Neurofibromatosis is a neurocutaneous syndrome which has a tendency to form tumors in the central
nervous system (CNS), peripheral nervous system, skin and viscera.

Neurofibromatosis type I
Characteristic features include cafe-au-lait spots, microcephaly, feeding problems, short stature,
learning disabilities, axillary freckles, Lisch nodules of the iris, and bony lesions.
Patients later develop fibromas, neurofibromas or different tumore.

Neurofibromatosiss type II
Multiple cafe-au-lait spots and relatively young age.
Gradually developing tinnitus and hearing loss is suggestive of an acoustic neuroma, which is best
diagnosed using MRI with gadolinium enhancement. Acoustic neuromas are frequently bilateral.

Craniopharyngioma
Craniopharyngiomas are benign suprasellar tumors arise in the sella turcica.
It usually present with signs of hypopituitarism (endocrinal symptoms), headaches and bitemporal
blindness.
Presence of a cystic calcified parasellar lesion on MRI is almost diagnostic of craniopharyngioma.
A young boy with symptoms of increased intracranial pressure (e.g., headaches, vomiting), bitemporal
hemianopsia, and a calcified lesion above the sella has a craniopharyngioma until proven otherwise.

Meningioma
Meningiomas are non-invasive tumors derived from arachnoid cap cells, and consist of concentric
whorls and calcified psammoma bodies.
These may indent the brain and cause hyperostosis.
These usually occur on the convex surfaces of the brain.

Glioblastoma multiforme
Symptoms: nausea, vomiting, headaches that worsen with changes in position, coughing and sneezing
and physical findings (e.g., papilledema) are indicative of increased intracranial pressure.
Furthermore, the presence of personality changes and strange behavior localizes the lesion to the
frontal lobe.
The typical CT/MRI findings in high-grade astrocytoma are heterogenous and serpiginous contrast
enhancement. Recognize the classic butterfly appearance of GBM

421
Empty sella syndrome
In empty sella syndrome, the sella turcica is often enlarged and contains no discernible pituitary gland.
This syndrome may be primarily due to an incompetent sellar diaphragm with compression of the
pituitary gland by the herniating arachnoid, or secondary to surgery or radiotherapy.

Paraneoplastic syndrome
Paraneoplastic syndromes can manifest as muscular weakness, but the pathologic process may be
localized at different levels (from the CNS to the muscles membranes).
In muscles involvement; proximal muscles are typically affected, and muscle strength is diminished
symmetrically. Reflexes are normal, and no sensation abnormality is present. Elevated CK levels and
myopathic electromyography help to confirm the diagnosis.

Heat stroke
Heat stroke is a life threatening catastrophic medical emergency.
It is due to failure of thermoregulatory center leading to severe hyperthermia with body temperature
generally > 40.5 C (105 F) with altered mental status.
Patients will have CNS symptoms.
Heat stroke generally has a sudden onset with altered mental status, however non specific prodromal
symptoms like weakness, dizziness, nausea, vomiting, anorexia, frontal headache, confusion, drowsiness,
disorientation, muscle twitching, ataxia and signs of cerebellar dysfunction, and psychiatric symptoms
ranging from anxiety and irritability to psychosis may also be seen.
Skin is hot and generally dry.
Rapid cooling of the patent is the cornerstone of management of heat stroke and should be done in
adjunction of stabilizing treatment. Evaporation cooling is the preferred modality.
If untreated it may lead to rhabdomyolisis, renal failure, ARDS, and coagulopathic bleeding.

Diabetic Mononeuropathy may be cranial (most commonly involving CN III, IV and VI) or peripheral
(most commonly involving the radial, peroneal and median nerves).
The etiology of mononeuropathy is mainly vascular, and recovery is usually seen in a few months time.
Nerve damage is ischemic, and only somatic nerve fibers are affected, and parasympathetic fibers
retain function.
Ptosis and down and out gaze in conjuction with normal light and accommodation reflexes indicate
diabetic CN III neuropathy.
For unknown reasons, patients with diabetes are predisposed to pressure palsies.

422
Hypokalemia
Hypokalemia is a common electrolyte abnormality that causes weakness, fatigue and muscle cramps.
When severe, it can lead to paralysis and arrhythmia.
The ECG may show U waves, flat & broad T waves, and premature ventricular beats.

Patients presenting to the ER with confusion or coma should receive empiric treatment with dextrose,
oxygen, naloxone, and thiamine.
Thiamine will help to prevent or reserve Wernickes encephalopathy in alcoholics and should be
administrated before dextrose.

Foot drop
Foot drop results from inability to dorsiflex the foot.
It leads to high-stepping or steppage gait.
The most common causes of foot drop are:
Peripheral neuropathy
Radiculopathy to any part of the spinal roots that contribute to the common peroneal nerve (L4 S2).
Traumatic damage to the common peroneal nerve

Brain death
Brain death refers to a total loss of brain function and is generally acceptable definition of death.
Criteria include: absent cranial nerve reflexes, fixed and dilated pupils, no spontaneous breaths, and
agreement of two physicians.
The spinal cord may still functioning; therefore, deep tendon reflexes may be present.
In a patient who is legally brain dead, there is no ethical issue.

423

Anda mungkin juga menyukai